RC Guide by Sandeep Gupta
RC Guide by Sandeep Gupta
RC Guide by Sandeep Gupta
Reading
Comprehension
Sandeep Gupta
Dear Friend,
I thank you for choosing this book. Right at the outset, I have a really bold
promise to make. If you are an extremely sincere and committed GMAT aspirant
(someone who not only wants to get into top 10 global b-schools such as
Harvard, Wharton, Stanford, Booth, MIT-Sloan, Kellogg, Columbia, LBS,
INSEAD, ISB, or IIMs, but also wants a significant scholarship, wherever
available), I assure you that you can’t find a better resource for your GMAT
preparation on the entire planet.
But I would also like to warn you at the very outset. If you are lazy, insincere, or
unambitious, or if you are someone who is looking for a quick-fix (“I want to prepare in just one month” | “I
want just a 700 kind of score” | “I just want to go to Canada or Germany, where schools are ready to take me
at 600-650”), you will not find this book or my content, my philosophies, and my approaches helpful.
Imagine addressing a thousand serious GMAT aspirants (in an auditorium) and still being able to convince
every single person, no matter which question / concept is being discussed from the entire GMAT universe!
Such is the level of depth I envision for you.
My goal for you: I don’t want you to somehow make it to ISB and be happy about the achievement; rather,
I want the ad-com members at Harvard to fight for your selection, 8 to 1.
Here are a few facts about me:
I assure you: my
goal for you is
wayyyyyyyyyy
bigger than your
own goal for
yourself.
Content: After having scored a perfect 800 score twice and after having taught students from 100+
nationalities for over two decades, I once again took the plunge to “STUDY” the GMAT as a student. I
subscribed to the most comprehensive contents from every major Global / Indian GMAT-Prep Company and
devoured everything they had to offer, howsoever useful / useless. I designed my contents after having gone
through the classes and contents of Manhattan, Kaplan, Princeton, E-GMAT, Magoosh, TTP, GMAT-Club,
Jamboree, CrackVerbal, Economist, Veritas, ExpertsGlobal, GMATWhiz, EmpowerGMAT, Wizako,
DominateTheGMAT, GMATCo, CJAnish, GMATwithCJ and 12 other companies. Every single useful bit from
every single GMAT-Prep company has been customized and incorporated in the content (even if I found just
ONE useful thing in a complete course, I have customized and incorporated that one thing in my books and
my courses).
In short, if YOU are an extremely sincere and ambitious student, I can get YOU a 99th percentile score.
This is an absolute PROMISE!
Reading
Comprehension
Let’s get started …
Reading Comprehension
Welcome to the
most DREADED
section on the
GMAT!
• RC: the single most important section on the GMAT that will decide whether you will
score above 700 or below 700. PERIOD!
• Almost every GMAT aspirant finds RC to be the make-or-break for a 700+ score.
• This is the section that checks your nerves to the hilt: concentration, pacing, stamina,
mental toughness, whether you go blank or can focus when nothing is registering in your
mind (or when your brain refuses to understand anything, when the clock is ticking, when
the stakes are as high as they are on the real GMAT).
• For whatever reason, if you can’t understand a passage during the exam, you are likely to
get 3 or 4 (consecutive) questions wrong. And, as you (most probably) already know, the
GMAT penalizes consecutive errors way more severely than spread-far-apart mistakes. If
you have 3-4 consecutive mistakes, your ‘current’ score drops by 60-80
points. Imagine losing 2 passages in this fashion! Your ‘current’ 750 may drop to ‘actual’
610 just because of two RC passages.
• Even at the cost of sounding dramatic, I will say this: one RC passage can mean the
difference between your being an MBA and your not being an MBA.
• Since 2008, I have personally interacted with more than 56,000 students who had to
drop their MBA plans only because they couldn’t crack JUST ONE RC PASSAGE!
• The biggest tragedy: no institute, trainer, or GMAT-Prep Company has any workable
method for RC at all—the method that actually WORKS on the test day! So, they devote
75-80% of their time teaching SC, 15-20% teaching CR, and a mere 5% to teaching RC.
RC is a formality that they think they have to cover, just for namesake.
• No one—absolutely no one—seems to have a workable strategy for RC. You are told all
kinds of things that don’t work on the test day. Here are some pieces of junk you may
have already heard / read about RC:
Read the questions first, then read the passage
Take brief notes—the worst advice of it all
Make a skeletal map of the passage
(Mentally) pause and summarize, after each para
Speed-read
Scan
Skip
Skim
Read, read, read … newspapers, novels, books, magazines, articles, NY Times, Economist …
Think of Topic, Purpose, and Scope of the passage while reading
Read the first and last lines of each para
Mainly read the first line and the last line of the passage
Rely on your notes for specific questions
To manage time well, leave one passage altogether (just blind-guess and move on)
Trust me! None of the above so-called “techniques” will work on the test day. NONE AT
ALL!
• Some trainers select a few passages on which their so-called ‘technique’ works; obviously
the ‘gimmick’ doesn’t work on the rest of the official passages / in the actual exam. But by
the time the student figures this out, the classes are over; these trainers cover just one or
two RC passages (as a formality) for one-half of a session at the very end.
• Truth be told! As a student and as a GMAT aspirant, I never struggled with Grammar,
Vocabulary, and Math. By the age of 14, I had developed enough Quant skills to be able
to clear International Math Olympiad, had read more than 250 books on grammar, and
had remembered the entire Oxford Advanced Learners’ Dictionary.
• One important thing: the two different types of questions asked in RC (Stated idea and
Inferred idea) test totally different skills, and your being good at one skill doesn’t guarantee your
being good at the other.
o In the same way, the two different types of questions asked in CR (Analyze / Critique and
Construction / Plan) test totally different skills, and your being good at one doesn’t
guarantee your being good at the other skill.
• Please refer to a few screenshots of the ESRs of a few students who contacted me after doing
badly in their earlier attempt(s) to understand the point I just made above.
O.C.C.T.A.A.V.E.
A guaranteed formula for 100% success in GMAT RC
Note: OCTAAVE is an application tool, not some concept that you can learn and hope
to improve your RC skills (with just the theory). To this effect, just the written word
can’t do justice to the application of OCTAAVE. You must watch me in action in my
live online classes or in pre-recorded videos to understand the application of the
technique thoroughly.
For this precise reason, the contents of this book are to be used as accompaniments
with my classes, not as standalone training materials.
An RC passage presents a formal piece of writing. Any formal piece of writing is very
different from a normal conversation, general chit-chat, YouTube comments,
WhatsApp shares, Facebook posts, Instagram updates, Twitter jargon and the like.
So, what distinguishes a formal piece of writing from all the other “junk”?
Simple: a formal piece of writing has a PURPOSE. No wonder almost every single RC
passage has a question called Primary Purpose of the passage. Otherwise, how many
times do you end up thinking what the primary purpose of a random WhatsApp group
message is?
So, what could be the purpose of a passage? Well, an RC passage can have the
following purposes (the list is indicative, not exhaustive):
1. To express an opinion, a point of view or a position
2. To argue for a conclusion by giving facts in support of the conclusion
3. To suggest something / to make a case for something / to recommend something
4. To prescribe something (the author wants the reader to accept a conclusion or accept his /
her rejection of a conclusion)
5. To predict something
6. To express a sentiment (optimism, pessimism etc.)
And so on …
• When the author expresses no personal opinion (50% of the passages, roughly)
• When the author expresses some personal opinion (50% of the passages,
roughly)
To this effect, your first task in RC is to find the personal opinion of the author or
of those people whom the author supports, implicitly or explicitly. So, the first
letter O in OCTAAVE stands for opinion. Let’s understand Opinion in some
detail:
Opinion:
• As the first step in RC, you must try to find the opinion (personal opinion) of the
author or of the people whom the author supports, implicitly or explicitly.
o The opinions that are just mentioned in the passage but are not supported by
the author don’t count in this scenario.
• The author of a passage will have either no opinion or some tangible opinion.
• If the author has an opinion, s/he may have a strong opinion or a mild opinion.
• BUT the author can’t have wavering | changing | uncertain | ambivalent opinions.
• Opinions can be challenged. Opinions can be supported, as well.
• On the contrary FACTS can’t be challenged. Facts can’t be supported, either.
• Prediction … (may | might | can | could | will | would | likely etc.) – FUTURE
• Suggestion … (should | propose | recommend | make a case for | ‘would need to
do’ | would be wise to follow etc. | must | ought to etc.) – FEELING / FUTURE
• Position | stand | stance | viewpoint | belief – FEELING / DEBATABILITY
• Criticism and praise | Agreement and disagreement – FEELING
• Conclusion drawn | Judgment reached | Hypothesis | Claim – DEBATABILITY
/ UNCERTAINTY
Please remember:
• The answer to the primary purpose and main idea of a passage is almost a
restatement of the opinion of the passage.
• The first verb in a primary purpose question belongs to the author.
• If the author has a personal opinion, it is bound to the primary purpose of the
passage. If not, the primary purpose is most likely going to be found at the Key
Contradiction Location.
The answer: the most likely location for the Primary purpose of a passage is:
Contradiction or the KEY Contradiction Location.
To this effect, the second letter (C) in the OCTAAVE formula stands for Contradiction
(key contradiction location).
Contradiction Words: Watch out for contradiction words: these are important
because the opinion might change or might be introduced at a contradiction location.
Also, a lot of times, there is a general | inference question from these locations. The
common contradiction words are:
Tone: Always try to predict the tone of the passage as positive | negative | neutral
| balanced. This is a big help.
o On the other hand, imagine this: “These are things that are right about the
US foreign policy, but these are things that are wrong about the US foreign
policy.” The author clearly has an opinion here. And hence, the tone is
balanced here: pros-and-cons analysis / pluses-and-minuses analysis always
means balanced tone.
o Even if there is one plus and ten minuses (or vice versa), the tone is still
balanced. We don’t need equal number of pluses and minuses.
o If the author doesn’t have a personal opinion, the tone is neutral (remember
the opinion must be author’s personal opinion, not anyone else’s)
o The tone of the author doesn’t change midway through the passage. For
example, if the author is pessimistic about something, s/he will remain
pessimistic throughout. Refer to the explanation of the Javan Rhino
passage to understand this further.
Some tricky words that can completely change the TONE of a passage:
Little: It means hardly any, no, or none … and hence it has a negative meaning. For
example:
• There is little hope of our team's victory now. Means NO hope or hardly any hope.
• He showed little concern for his niece. Means NO concern or hardly any concern.
• He showed little interest in Chemistry. Means NO interest or hardly any interest.
• Obama's presidency had little impact in countering the economic crisis in the
USA, which continued to worsen for a much longer duration. Means No impact or
hardly any impact.
A little: It means some or somewhat … and hence it has positive meaning. For
example:
• A little planning could have saved us from the disaster. Means some planning.
• There is a little hope of his recovery from the disease. Means some hope.
Few: It means hardly any or no one. Therefore, it has negative meaning. For example:
• Few people speak the truth today. Means NO ONE or hardly any
• Few people can keep a secret. Means NO ONE or hardly any
A Few: It means ‘some’ as opposed to ‘none’, and hence it has positive meaning. For
Example:
• A few words spoken politely will convince the boss. Means some words
• A few people have responded to the call for charity. Means some people
Critique: doesn’t mean criticize. It simply means to weigh the pros and cons of
something. A movie critic (n) who critiques (v) a movie can give a five-star or one-star
reviews both. Whatever the critic presents [a text / a video etc.] is called the critique (n)
Most of the GMAT RC passages deal with topics / contents that may be totally
unknown to non-American GMAT-takers. This proves to be an extraordinary hurdle
to quick and easy understanding. For example, if you don’t have any context about
Feminism in 19th Century America, property rights before and after the US Civil War,
the retentionist versus the abolitionist debate about Slavery in the US, prominent
Supreme Court cases in the US, various ethnic groups in the US and the difficulties
faced by them while assimilating with the American culture, Native Americans and
the treatment meted out to them by the European Whites, Capitalism (as a
philosophy) versus Socialism, challenges to religion in the US … the list is really long.
These topics are most likely to be unknown to non-Americans but fully known to
Americans, so, as a nonnative GMAT taker, unless you shore up your knowledge base
about these topics, you are bound to be at a disadvantage vis-à-vis your American
counterparts (American GMAT-takers).
To this effect, the next really important element in the OCTAAVE formula is
Awareness.
As mentioned above, many times (especially if you are a non-native GMAT aspirant),
understanding a passage can be extremely challenging as you may not be aware of the
background story / information / topic presented in the passage. Awareness stands
for background information. If you already know the background information related
to a new passage, your task becomes really simple. In my signature program
Before reading any further, please watch these five short videos …
these five videos can change your life … completely!
Video 1: https://youtu.be/h0VJHS4cTAU
Video 2: https://youtu.be/fMnrDgJ77BU
Video 3: www.youtube.com/watch?v=7HXIW7i7hmg
Video 4: https://youtu.be/hbNceVnRMlQ
Video 5: https://youtu.be/LbHMOPjNE0I
Predominant RC topics
• Biological Evolution | Human Behavior shaped by biological evolution
• Brain Science (evolution of the human brain and how it affects human behavior)
• Animal kingdom
• Challenges to religion
• Various groups in America: minorities, women, feminism, Blacks, Native Americans, Asians, Hispanics
*****
So far, all the components of OCTAAVE have been related to the passage. Opinion /
Contradiction / Tone / Awareness … all relate to the passage.
There are three components, however, that are related to the question and the options.
These three elements are WAYYYYYYYYYYYY more important than the first four.
Avoid: in the answer choices, you must be extremely cautious of (largely avoid):
• Superlatives (the third degrees of adjectives): best, latest, most, worst etc. Unless given verbatim, these
answers are almost always wrong.
• Specific to general (if the passage mentions something specific, the choices that generalize that
information are always wrong).
o Let’s say, the passage talks about lions, then it will be wrong to mark an answer that generalizes
lions to mammals or animals etc.
o If the passage is about a recent immigrant’s first experience of America, the exam will widen the
scope of an answer choice to include all immigrants.
o If the passage is about the existence of heavy metals on some planets, an incorrect answer choice
will talk about all planets.
• Extreme words (Always, Never, All, None, Only, Unique, Everyone, No one, Must, Impossible, The first,
Each, Every, Totally). Unless given verbatim, these answers are almost always wrong.
• Answers that make a prediction. Unless given verbatim, these answers are almost always wrong.
• Very strongly emotional language (dismiss, champion, radicalize, castigate, eulogize etc.). Unless given
verbatim, these answers are almost always wrong.
• Politically incorrect language (offensive to a group, region, religion, gender, race, nation etc.). Unless
given verbatim, these answers are almost always wrong.
In this argument, the first statement is the Fact and the second statement is the Conclusion.
So, we have: Fact: Every male above 18 can vote. Conclusion: So, Jack can vote.
If you notice carefully, each of the words – Jack, male, 18, and vote – is part of two of the three
sentences above, not all three. This is very helpful in solving assumption questions in RC.
Let’s now focus on the heart and soul of RC. OCTAAVE – Verify
It means ‘CAN YOU GO BACK AND VERIFY / JUSTIFY (as if putting your finger on it)?’
Can you spot why each of the choices given below will
automatically be wrong, no matter what the content of the
passage?
1. Women should not work in the public sphere because they are not as rational as men.
2. Prove that scientists are more intelligent than mathematicians.
3. Milton was the first English author to focus on society as a whole as well as on individual characters.
4. The public is not interested in increasing its awareness of the advantages / disadvantages of nuclear
fusion.
5. The public has been deliberately misinformed about the advantages and disadvantages of nuclear power.
6. James, more than any other novelist, was aware of the difficulties of novelistic construction.
7. The author is primarily concerned with disputing evidence a scholar uses to substantiate a claim about a
historical event
8. The emancipation of women would ultimately bring about the liberation of the entire Russian population
from political oppression
9. The United States will import more oil in the next 10 years.
10. The author believes the purpose of children’s literature should be to expose children to the cruelties of
life.
Explanations
1. Women should not work in the public sphere because they are not as rational as men. Politically
incorrect / sexist / can stir up a controversy
2. Prove that scientists are more intelligent than mathematicians. Politically incorrect / unprovable /
can stir up a controversy
3. Milton was the first English author to focus on society as a whole as well as on individual characters.
FIRST – unprovable / can stir up a controversy
4. The public is not interested in increasing its awareness of the advantages / disadvantages of nuclear
fusion. Generalized for all / can stir up a controversy
5. The public has been deliberately misinformed about the advantages and disadvantages of nuclear power.
Politically incorrect / can stir up a controversy
6. James, more than any other novelist, was aware of the difficulties of novelistic construction. Unprovable
/ can stir up a controversy
7. The author is primarily concerned with disputing evidence (means challenging a fact – not
allowed in any question in all of RC and CR) a scholar uses to substantiate a claim about a historical
event.
8. The emancipation of women would ultimately bring about (prediction – never choose
prediction answers) the liberation of the entire (extreme) Russian population from political
oppression
9. The United States will import more oil in the next 10 years. (Prediction – always wrong)
10. The author believes the purpose of children’s literature should be to expose children to the cruelties of
life. Politically incorrect / can stir up a controversy
Can you spot why these tone answers are automatically wrong?
The author’s attitude towards a particular artist’s approach can best be described as:
1. strong indignation
2. marked indifference
3. sharp derision
4. Overwhelming support
5. Unabashed admiration
6. Unqualified appreciation
7. Profound ambivalence
8. Deep-rooted hostility
9. Vehement condemnation
10. Unwavering denunciation
Explanations
SPECIAL NOTE:
CR questions in RC:
If any question in RC is about weaken, strengthen, assumption, evaluate, paradox etc.
and involves “IF TRUE”, the rules of OCTAVE will not be applicable. “If true” means
you can’t refer to the passage.
You are going to be so pressed for time on the test day that you won’t be able to make any notes. Why
learn and rely upon something that will not work on the test day?!
Even if you make notes, invariably our experience says that students never use what they write. Why
waste precious time?
Once your brain gets a subconscious signal that you can write, your understanding vanishes; you are
reduced to just taking notes passively. RC is about active thinking and application, not passive note-
taking.
• Must go back while solving the questions: A lot of times, students rely upon their memory and miss
out on the intentional distortion used so effectively by the test-makers (psychometricians). These people
are masters of making the wrong answers that seem wayyyy more attractive than the right answers. The
best way to avoid this is to go back to the passage to justify each word in the answer choices. PUT YOUR
FINGER ON IT!
• Don’t do more general reading; solve more RC exercises: The ubiquitous advice that people get
when they do badly on RC is: READ, READ, and READ. WRONG! Solve more and more RC passages.
RIGHT!
• Don’t bother about vocabulary: A lot of people bother about understanding each word in the passage
to an obsessional level. Indeed, a good vocabulary can help you, but there is no point in developing
superior vocabulary NOW. The efforts can be completely disproportionate compared to the returns. So,
the best bet is to guess the meaning of a new word by its context.
• Don’t solve anything else except what has officially appeared on Standardized American Tests such
as the GMAT, the GRE, or the LSAT. There are at least 500 genuine such passages available for you to
practice with.
• Ideally for a passage with 3 questions, you must not take more than 5 minutes and for a passage with 4
questions, you must not take more than 6.5 minutes.
• Avoid this blunder (not taking care of the SPEED aspect from the word go): this happens a lot in RC
and CR. I hear this grouse all the time: “in RC and CR, my accuracy is quite good if I don’t time myself.
But if I time myself, the accuracy drops to less than 50%.” It is equivalent to a marathoner saying: “I can
run 42 kilometers, but I need 4 days, not 4 hours.” Such people feel happy even if they manage to solve
only one RC passage from the official guide in 27 minutes and get 5/7 correct. Remember: on the test
day, you have less than 27 minutes to solve all the four RC passages. Invariably these students have 6-8
questions left in Quant and 8-10 questions left in Verbal to be done in the last 5-7 minutes (including the
last RC passage, which, according to them, always happens to be the longest and the most difficult passage
in the entire Verbal Section). The result: their mind goes blank, refusing to understand anything, no
matter how hard they try to focus. In addition, many students spend a disproportionately huge amount
of time in the beginning, only to realize around Q. 10 that “this isn’t my day”.
The solution: always solve using a countdown timer, one that compulsorily stops at the end of ___
minutes. Don’t use a simple clock; using the simple clock is disastrous because most students keep on
solving questions even when the time is up. For example, suppose you take four RC passages and set aside
25 minutes. Invariably, you will NOT stop at the end of 25 questions. Later you will go something like
this: “I took 41 minutes; so, I need to pace up.” This isn’t a strong message. The message is felt deeply and
strongly when, while solving the third passage, the alarm goes off and you are forced to stop. Cultivate the
habit of STOPPING when the alarm goes off; doing so may be very painful in the beginning, but is the
only effective strategy (keeping the exam in mind).
• A common grouse of many students: “I can’t sit for 3 hours straight. I need a break every 15 min.”
Moreover, they have all the distractions going on: WhatsApp messages, Instagram feeds, random calls—
as if the world can’t continue for even 15 minutes without these people. Such students commit the
cardinal GMAT sin: not building the most crucial aspect of the GMAT game: stamina.
• The most crucial: always practice on screen, not on paper. The biggest blunder people do while
preparing for the GMAT is that they study on paper. ABSOLUTELY DISASTROUS for the GMAT! Make
it a habit to never, ever study / practice anything on paper.
• Also, please don’t rely on take any shortcuts / gimmicks for RC. Solve at least 400 passages using
OCTAAVE to feel absolutely confident about this section.
The following pages contain 60 real-GMAT Official passages. Each of these questions will be discussed in the
classes (Live Online or Pre-recorded Videos) using OCTAAVE.
Session 1: 12 passages
Session 2: 20 passages
Session 3: 20 passages
Session 4: 8 passages
OCTAAVE Illustration
(Using 12 most important GMAT passages discussed with the help of a video – Video 1)
Passage 1
One undeniable fact about these frailties is that 2. The author is primarily concerned
some of these frailties are presently with:
maladaptive. Ironically, one clear example of A. presenting an overview of those human
this maladaptiveness is our ever-increasing frailties that impose constraints on
need for security. Yet beneath the overlay of human behavior
cultural detail, even these maladaptive frailties, B. summarizing a general position on
too, are said to be biological in direction, and are human frailties and presenting his own
therefore as natural to us as are our appendixes. stance
We would need to thoroughly comprehend the C. outlining a new claim about foundations
origin of these maladaptive frailties in order to of human behavior with a special focus
understand how badly they guide us now. And on maladaptive frailties and speculating
we might then begin to resist the pressure of on their origins
these maladaptive frailties in our distant future. D. pointing out those human frailties that
give us a sense of our constraints and
presenting the probable outcome of such
constricting behaviors
E. suggesting ways to deal with a problem
and predicting the eventual
consequences of not doing so
Passage 2
In 1965 a prestigious university in South Africa The Western model of personality development
started the first-ever African women’s studies does not explain major characteristics of the
curriculum that was modeled exclusively after African personality, which is social and group-
Western feminist values. The ensuing furore centered. The “self” is a social being defined by
was anything but pleasant. There was and acting in a group, and the well-being of both
widespread denunciation for what the program men and women is determined by the
propagated. Various groups and organizations equilibrium of the group, not by individual self-
protested tooth and nail, condemning the assertion.
program from start to finish. As expected, in all
the broadcast debates, critics dismissed the The founders also noted that, in such a context,
curriculum as a betrayal of national identity, an what is recognized as “dependency” in Western
imitation of Western ideas, and a distraction psychiatric terms is not, in African terms, an
from the real task of national unification and admission of weakness or failure. In the kinship-
economic development. Even supporters of the based society of Africa, four generations may
curriculum underestimated its potential; they live in the same house, which means that people
thought it would be merely another of the many can be sons and daughters all their lives,
Western ideas that had already proved useful in whereas in Western culture, the roles of
African culture, akin to airlines, electricity, and husband and son, wife and daughter, are often
the assembly line. Whatever the stance of the incompatible.
critics or of supporters may have been, one fact
stands undeniable: few academic programs had 3. It can be inferred from the passage
ever received such public attention in all of that the broadcast media in South
Africa. Africa considered the start of the
women’s studies curriculum
The founders of the program realized that both A. Praiseworthy
the critics and the supporters were wrong in B. Insignificant
their analyses. Their approach has thus far C. Newsworthy
yielded important critiques of Western theory, D. Imitative
aided by the special experiences of African E. Incomprehensible
women. For instance, in their analysis the
founders found that, like the Western feminist 4. Which of the following best
critique of the Freudian model of the human summarizes the content of the
psyche, the African critique finds Freudian passage?
theory culture-bound, but in ways different A. A critique of a particular women’s
from those cited by Western theorists. The studies program
African theorists claim that Freudian theory B. A report about a particular women’s
assumes the universality of the Western studies program
nuclear, male-headed family and focuses on the C. An assessment of the strengths and
personality formation of the individual, weaknesses of a particular women’s
independent of society. Such a concept projects studies program
the competitive model of Western society onto D. An analysis of the philosophy underlying
human personalities. But in the African concept women’s studies programs
of personality there is no ideal attached to E. An abbreviated history of African studies
individualism or to the independent self. programs
Passage 3
Rarely does a technology have such far reaching 5. The tone of the passage could best be
impact across the globe as has LCD technology described as
in the past decade. In recent times, video display A. disinterested explanation
technologies have undergone a sea change in B. supportive advocacy
their precision. This has been aided by the rapid C. cautious endorsement
rise in the demand for High Definition (HD) D. intellectual exploration
pictures and videos. As a result, the active- E. qualified support
matrix liquid crystal diode (LCD) display is the
dominant technology currently used in most 6. According to the passage, which of the
consumer display products. following may be considered
advantages of LCD displays relative to
LCDs that are capable of producing color other display technologies?
images, such as in televisions and computers, I. they are able to display continuous video
reproduce colors through a process of graphics images with no interruption
subtraction, blocking out particular color II. they reflect a widely adopted standard
wavelengths from the spectrum of white light III. they represent the latest, most advanced
until only the desired color remains. It is the technology
variation of the intensity of light permitted to
pass through the matrix of liquid crystals that Choose one answer from below:
enables LCD displays to present images full of A. I and III Only
gradations of different colors. B. II and III only
C. I and II only
The nature and functioning of LCD displays D. I, II and III
present many advantages relative to other E. III only
display technologies. The amount of power
required to untwist the crystals to display
images, even dark ones, is much lower than that
required for analogous processes using other
technologies, such as plasma. The dense array of
crystals displays images from computer or
other video graphics sources extremely well,
with full color detail, no flicker, and no screen
burn-in. Moreover, the number of pixels per
square inch on an LCD display is typically higher
than that for other display technologies, so LCD
monitors are particularly good at displaying
large amounts of data with exceptional clarity
and precision. As a result, LCD TVs are
considered the best display platform for video
games, high-definition television, movie special
effects, and other graphics-intensive uses.
Passage 4
Measuring more than five feet tall and ten feet 7. Which of the following statements can
long, the Javan rhinoceros is often called the be inferred from the passage?
rarest large mammal on earth. None exist in I. In the last sentence of the passage, the
zoos. While the Javan rhino habitat once author reiterates his pessimism about
extended across southern Asia, now there are the survival of the Javan rhino species
fewer than one hundred of the animals in II. In the last sentence of the passage, the
Indonesia and under a dozen in Vietnam. Very author expresses slight optimism about
little is known about Javan rhinos because they the fact that good actions on part of
lead secretive and solitary lives in remote humans can probably increase the
jungles. Till today, scientists debate whether chances of survival of the Javan rhino
females even have horns, and most scientific species
work has had to rely on DNA garnered from III. Genetic diversity is one of the most
dung. important factors for the survival of a
species
The near extinction of the Javan rhino is the
direct result of human actions. For centuries, Choose one answer from below:
farmers, who favored the same habitat, viewed A. I only
them as crop eating pests and shot them on sight. B. II and III only
During the colonial period, hunters slaughtered C. II only
them by thousands. Now, human efforts to save D. III only
them may well prove futile. The rhinos are E. I and III only
unprotected on the Ujung Kulon peninsula,
which is unsettled by humans, and don’t have 8. Which of the following can be inferred
sufficient genetic diversity to have a chance at from the passage?
survival. I. More is known about the genetics of the
Javan rhino than is known about its
But what would happen if humans were to mating patterns
completely stop interfering in the Javan rhino II. Most animal extinctions are the result of
habitats? Will there be a ray of hope for the human actions
survival of the Javan rhino species? The lack of III. Hunters killed more Javan rhinos in
human disturbance will allow mature forests to Vietnam than in Indonesia
replace the shrubby vegetation the animals
prefer. Thus, human benevolence (good actions Choose one answer from below:
on part of humans) may prove little better for A. I only
these rhinos than past human maltreatment. B. II and III only
C. III only
D. I, II, and III
E. II only
Passage 5
The function of capital markets is to facilitate an Also, it is assumed that each individual in the
exchange of funds among all participants, and community at large has the same access to the
yet in practice we find that certain participants market and the same opportunity to transact
are not on a par with others. Members of society and to express the preference appropriate to his
have varying degrees of market strength in or her individual interest. Moreover, it is
terms of information they bring to a transaction, assumed that transaction costs for various types
as well as of purchasing power and of financial instruments (stocks, bonds, etc.) are
creditworthiness, as defined by lenders. equally known and equally divided among all
community members.
For example, within minority communities,
capital markets do not properly fulfill their 9. The main point made by the passage is
functions; they do not provide access to the that
aggregate flow of funds in the United States. The A. financial markets provide for an optimum
financial system does not generate the credit or allocation of resources among all
investment vehicles needed for underwriting competing participants by balancing
economic development in minority areas. The supply and demand
problem underlying this dysfunction is found in B. the allocation of financial resources takes
a rationing mechanism affecting both the place among separate individual
available alternatives for investment and the participants, all of whom have access to
amount of financial resources. This creates a the market
C. the existence of certain factors adversely
distributive mechanism penalizing members of
affecting members of minority groups
minority groups because of their socioeconomic
shows that financial markets do not
differences from others. The existing system function as conventional theory says they
expresses definite socially based investment function
preferences that result from the previous D. investments in minority communities can
allocation of income and that influence the be made by the use of various alternative
allocation of resources for the present and financial instruments, such as stocks and
future. The system tends to increase the bonds
inequality of income distribution. And, in the E. since transaction costs for stocks, bonds,
United States economy, a greater inequality of and other financial instruments are not
income distribution leads to a greater equally apportioned among all minority-
concentration of capital in certain types of group members, the financial market is
investment. subject to criticism
Most traditional financial-market analysis 10. The author's main point is argued by
studies ignore financial markets’ deficiencies in A. giving examples that support a
allocation because of analysts’ inherent conventional generalization
preferences for the simple model of perfect B. showing that the view opposite to the
competition. Conventional financial analysis author's is self-contradictory
pays limited attention to issues of market C. criticizing the presuppositions of a
structure and dynamics, relative costs of proposed plan
information, and problems of income D. showing that omissions in a theoretical
distribution. Market participants are viewed as description make it inapplicable in certain
cases
acting as entirely independent and
E. demonstrating that an alternative
homogeneous individuals with perfect foresight
hypothesis more closely fits the data
about capital-market behavior.
Passage 6
Woodrow Wilson was referring to the liberal There is no pride in being an employee (Wilson
idea of the economic market when he said that asked for a return to the time when everyone
the free enterprise system is the most efficient was an employer). There has been no boasting
economic system. Fascination with this ideal about our social workers—they are merely
has made Americans defy the “Old World” signs of the system’s failure, of opportunity
categories of settled possessiveness versus denied or not taken, of things to be eliminated.
unsettling deprivation, the cupidity of retention We have no pride in our growing
versus the cupidity of seizure, a “status quo” interdependence, in the fact that our system can
defended or attacked. The United States, it was serve others, that we are able to help those in
believed, had no status quo ante. Our only need; empty boasts from the past make us
“station” was the turning of a stationary wheel, ashamed of our present achievements, make us
spinning faster and faster. We did not base our try to forget or deny them, move away from
system on property but opportunity—which them. There is no honor but, in the Wonderland,
meant we based it not on stability but on race we are all expected to all run, all trying to
mobility. The more things changed, that is, the win, none winning in the end (for there is no
more rapidly the wheel turned, the steadier we end).
would be. The conventional picture of class
politics is composed of the Haves, who want a 11. The primary purpose of the passage is
stability to keep what they have, and the Have- to
Nots, who want a touch of instability and change A. criticize the inflexibility of Americans’
in which to scramble for the things they have strong belief in the idea of the free
not. But Americans imagined a condition in enterprise economic system
which speculators, self-makers, runners are B. contrast “Old World” and “New World”
always using the new opportunities given by economic ideologies
our land. These economic leaders (front- C. question the integrity of a political leader
runners) would thus be mainly agents of D. support those Americans whom the
change. The nonstarters were considered the author deems to be neglected and
ones who wanted stability and a strong referee propose ways to improve their situation
to give them some position in the race, a E. suggest a substitute for the traditional
regulative hand to calm manic speculation; an metaphor of a race
authority that can call things to a halt, begin
things again from compensatorily staggered 12. Which of the following best expresses
“starting lines.” the author’s main point?
“Reform” in America has been sterile because it A. Americans’ pride in their jobs continues
can imagine no change except through the to give them stamina today.
extension of this metaphor of a race, wider B. The absence of a status quo ante has
inclusion of competitors, “a piece of the undermined United States economic
action,” as it were, for the disenfranchised. structure.
There is no attempt to call off the race. Since our C. The free enterprise system has been a
only stability is change, America seems not to very popular system in the United States.
honor the quiet work that achieves social D. The popular belief of Americans in the
interdependence and stability. The worst part is effectiveness of the free enterprise
that, in our legends, there is no heroism of the system is flawed.
office clerk or admiration for the stable E. Fascination with the ideal of “openness”
industrial workforce of the people who actually has made Americans sympathetic to the
make the system work. people who are not part of the economic
race.
Woodrow Wilson was referring to the liberal There has been no boasting about our social
idea of the economic market when he said that workers—they are merely signs of the system’s
the free enterprise system is the most efficient failure, of opportunity denied or not taken, of
economic system. Fascination with this ideal things to be eliminated. We have no pride in our
has made Americans defy the “Old World” growing interdependence, in the fact that our
categories of settled possessiveness versus system can serve others, that we are able to help
unsettling deprivation, the cupidity of retention those in need; empty boasts from the past make
versus the cupidity of seizure, a “status quo” us ashamed of our present achievements, make
defended or attacked. The United States, it was us try to forget or deny them, move away from
believed, had no status quo ante. Our only them. There is no honor but, in the Wonderland,
“station” was the turning of a stationary wheel, race we are all expected to all run, all trying to
spinning faster and faster. We did not base our win, none winning in the end (for there is no
system on property but opportunity—which end).
meant we based it not on stability but on
mobility. The more things changed, that is, the 13. It can be inferred from the passage that
more rapidly the wheel turned, the steadier we Woodrow Wilson’s ideas about the
would be. The conventional picture of class economic market
politics is composed of the Haves, who want a A. encouraged those who, according to the
stability to keep what they have, and the Have- author, make the system work
Nots, who want a touch of instability and change B. propagated and continued the traditional
in which to scramble for the things they have beliefs as to why Americans think they are so
not. But Americans imagined a condition in successful in the first place
which speculators, self-makers, runners are C. revealed the prejudices of a society that
always using the new opportunities given by values socialism over capitalism
our land. These economic leaders (front- D. foreshadowed the best possible economic
runners) would thus be mainly agents of system for America in times to come
change. The nonstarters were considered the E. began a tradition of the proclamation of the
ones who wanted stability and a strong referee most effective economic system in America
to give them some position in the race, a
regulative hand to calm manic speculation; an 14. The author sets off the word “Reform”
authority that can call things to a halt, begin with quotation marks in order to
things again from compensatorily staggered A. emphasize its departure from the concept of
“starting lines.” settled possessiveness
B. show his support for a legislative program
“Reform” in America has been sterile because it directed at massive changes in the American
can imagine no change except through the society
extension of this metaphor of a race, wider C. highlight the flexibility and even
inclusion of competitors, “a piece of the amorphousness of United States society
action,” as it were, for the disenfranchised. D. indicate that the term was one of Wilson’s
There is no attempt to call off the race. Since our favorites
only stability is change, America seems not to E. assert that reform in the United States has
honor the quiet work that achieves social not helped those who haven’t been able to
interdependence and stability. The worst part is exploit the full benefits of the economic race
that, in our legends, there is no heroism of the
office clerk or admiration for the stable
industrial workforce of the people who actually
make the system work. There is no pride in
being an employee (Wilson asked for a return to
the time when everyone was an employer).
Woodrow Wilson was referring to the liberal There has been no boasting about our social
idea of the economic market when he said that workers—they are merely signs of the system’s
the free enterprise system is the most efficient failure, of opportunity denied or not taken, of
economic system. Fascination with this ideal things to be eliminated. We have no pride in our
has made Americans defy the “Old World” growing interdependence, in the fact that our
categories of settled possessiveness versus system can serve others, that we are able to help
unsettling deprivation, the cupidity of retention those in need; empty boasts from the past make
versus the cupidity of seizure, a “status quo” us ashamed of our present achievements, make
defended or attacked. The United States, it was us try to forget or deny them, move away from
believed, had no status quo ante. Our only them. There is no honor but, in the Wonderland,
“station” was the turning of a stationary wheel, race we are all expected to all run, all trying to
spinning faster and faster. We did not base our win, none winning in the end (for there is no
system on property but opportunity—which end).
meant we based it not on stability but on
mobility. The more things changed, that is, the 15. It can be inferred from the passage that
more rapidly the wheel turned, the steadier we the author most probably thinks that
would be. The conventional picture of class giving the disenfranchised “a piece of the
politics is composed of the Haves, who want a action” is
stability to keep what they have, and the Have- A. a compassionate legislative measure
Nots, who want a touch of instability and change B. an example of Americans’ resistance to any
in which to scramble for the things they have major change
not. But Americans imagined a condition in C. an innovative program for genuine social
which speculators, self-makers, runners are reform
always using the new opportunities given by D. a monument to the efforts of industrial
our land. These economic leaders (front- reformers
runners) would thus be mainly agents of E. a surprisingly “Old World” remedy for social
change. The nonstarters were considered the ills
ones who wanted stability and a strong referee
to give them some position in the race, a
regulative hand to calm manic speculation; an
authority that can call things to a halt, begin
things again from compensatorily staggered
“starting lines.”
Passage 7
Seeking a competitive advantage, some 16. The primary function of the passage as a
professional service firms (for example, firms whole is to
providing advertising, accounting, or health A. account for the popularity of a practice
care services) have considered offering B. evaluate the utility of a practice
unconditional guarantees of satisfaction. Such C. demonstrate how to institute a practice
guarantees specify what clients can expect and D. denounce the utility of using a strategy
what the firm will do if it fails to fulfill these E. explain the reasons for pursuing a strategy
expectations. Particularly with first-time clients,
an unconditional guarantee can be an effective
marketing tool if the client is very cautious, the
firm’s fees are high, the negative consequences
of bad service are grave, or business is difficult
to obtain through referrals and word-of-mouth.
Passage 8
Planter-legislators of the post-Civil War 17. Which of the following best expresses the
southern United States enacted crop lien laws central idea of the passage?
stipulating that those who advanced cash or A. Planters in the post-Civil War southern
supplies necessary to plant a crop would United States sought to reinstate the
receive, as security, a claim, or lien, on the crop institution of slavery.
produced. In doing so, planters, most of whom B. Through their decisions regarding supply
were former slaveholders, sought access to credit, merchants controlled post-Civil War
credit from merchants and control over agriculture.
nominally free laborers—former slaves freed by C. Lien laws helped to defeat the purpose for
the victory of the northern Union over the which they were originally created.
southern Confederacy in the United States Civil D. Although slavery had ended, the economic
War. They hoped to reassure merchants that hierarchy changed little in the post-Civil War
despite the emancipation of the slaves, planters southern United States.
would produce crops and pay debts. Planters E. Newly freed workers enacted lien laws to
planned to use their supply credit to control hasten the downfall of the plantation
their workers, former slaves who were without economy.
money to rent land or buy supplies. Planters
imagined continuation of the pre-Civil War 18. The passage suggests which of the
economic hierarchy: merchants supplying following about merchants in the post-
landlords, landlords supplying laborers, and Civil War southern United States?
laborers producing crops from which their A. They sought to preserve pre-Civil War social
scant wages and planters' profits would come, conditions.
allowing planters to repay advances. Lien laws B. Their numbers in the legislatures had been
frequently had unintended consequences, diminished.
however, thwarting the planter fantasy of C. Their businesses had suffered from a loss of
mastery without slavery. The newly freed collateral.
workers, seeking to become self-employed D. They were willing to make business
tenant farmers rather than wage laborers, made arrangements with former slaves.
direct arrangements with merchants for E. Their profits had declined because planters
supplies. Lien laws, the centerpiece of a system defaulted on debts for supply advances.
designed to create a dependent labor force,
became the means for workers, with alternative
means of supply advances, to escape that
dependence.
Passage 9
Historians have identified two dominant 19. The passage is primarily concerned with
currents in the Russian women's movement of A. identifying points of agreement between two
the late tsarist period. "Bourgeois" feminism, so groups
B. advocating one approach to social reform over
called by its more radical opponents,
another
emphasized "individualist" feminist goals such C. contrasting two approaches to solving a political
as access to education, career opportunities, problem
and legal reform. "Socialist" feminists, by D. arguing that the views espoused by one political
contrast, emphasized class, rather than gender, group were more radical than those espoused by
as the principal source of women's inequality another group
and oppression, and socialist revolution, not E. criticizing historians for overlooking similarities
legal reform, as the only road to emancipation between the views espoused by two
and equality. superficially dissimilar groups
However, despite antagonism between 20. The passage suggests that socialists within
the Russian women's movement and most
bourgeois feminists and socialist feminists, the
bourgeois feminists believed that in Russia
two movements shared certain underlying
A. women would not achieve economic equality
beliefs. Both regarded paid labor as the until they had political representation within the
principal means by which women might attain government
emancipation: participation in the workplace B. the achievement of larger political aims should
and economic self-sufficiency, they believed, take precedence over the achievement of
would make women socially useful and women's rights
therefore deserving of equality with men. Both C. the emancipation of women would ultimately
groups also recognized the enormous bring about the liberation of the entire Russian
difficulties women faced when they combined population from political oppression
paid labor with motherhood. In fact, at the First D. women's oppression was more rooted in
economic inequality than was the case in other
All-Russian Women's Congress in 1908, most
countries
participants advocated maternity insurance and E. the women's movement was more ideologically
paid maternity leave, although the intense divided than were women's movements in other
hostility between some socialists and bourgeois countries
feminists at the Congress made it difficult for
them to recognize these areas of agreement. 21. According to the passage, Russian socialists
Finally, socialist feminists and most bourgeois within the women's movement and most
feminists concurred in subordinating women's bourgeois feminists disagreed about which
emancipation to what they considered the more of the following?
important goal of liberating the Russian A. Whether legal reform was central to the
population from political oppression, economic achievement of feminist goals
B. Whether paid employment was important for
backwardness, and social injustice.
the achievement of equality
C. Whether maternity insurance was desirable for
working mothers
D. Whether working mothers faced obstacles
E. Whether women's emancipation should be
subordinated to the goal of liberating the
Russian population
Passage 10
Colonial historian David Allen's intensive study of We are not told in what way, if at all, this
five communities in seventeenth-century discovery illuminates historical understanding.
Massachusetts is a model of meticulous Studies of local history have enormously
scholarship on the detailed microcosmic level, and expanded our horizons, but it is a mistake for their
is convincing up to a point. Allen suggests that authors to conclude that village institutions are all
much more coherence and direct continuity that mattered, simply because their functions are
existed between English and colonial agricultural all that the records of village institutions reveal.
practices and administrative organization than
other historians have suggested. However, he 22. The author of the passage is primarily
overstates his case with the declaration that he concerned with
has proved "the remarkable extent to which A. substantiating a claim about a historical event
diversity in New England local institutions was B. reconciling two opposing ideas about a
directly imitative of regional differences in the historical era
mother country." C. disputing evidence a scholar uses to
substantiate a claim about a historical event
Such an assertion ignores critical differences D. analyzing two approaches to scholarly
between seventeenth-century England and New research and evaluating their methodologies
England. First, England was overcrowded and E. criticizing a particular study and the approach
land-hungry; New England was sparsely to historical scholarship it represents
populated and labor-hungry. Second, England
suffered the normal European rate of mortality; 23. It can be inferred from the passage that the
New England, especially in the first generation of author of the passage considers Allen's
English colonists, was virtually free from “discovery” to be
infectious diseases. Third, England had an all- A. already known to earlier historians
embracing state church; in New England B. exclusive to Allen
membership in a church was restricted to the C. improbable but highly convincing
elect. Fourth, a high proportion of English D. an unexplained, isolated fact
villagers lived under paternalistic resident E. a new, insightful observation
squires; no such class existed in New England. By
narrowing his focus to village institutions and 24. The passage suggests that Professor Clive
ignoring these critical differences, which studies Holmes would most likely agree with
by Greven, Demos, and Lockridge have shown to which of the following statements?
be so important, Allen has created a somewhat A. It is important to understand broad national
distorted picture of reality. issues in order to understand seventeenth-
century English institutions thoroughly.
Allen's work is a rather extreme example of the B. The "country community" school of
"country community" school of seventeenth- seventeenth-century English history distorts
century English history whose intemperate historical evidence in order to establish
excesses in removing all national issues from the continuity between old and new institutions.
history of that period have been exposed by C. Most historians distort reality by focusing on
Professor Clive Holmes. What conclusion can be national concerns to the exclusion of local
drawn, for example, from Allen's discovery that concerns.
Puritan clergy who had come to the colonies from D. National issues are best understood from the
East Anglia were one-third to one-half as likely to perspective of those at the local level.
return to England by 1660 as were Puritan E. Isolated local pieces of information about
ministers from western and northern England? seventeenth-century English villages are the
most crucial pieces of evidence to understand
national life in that era.
Passage 11
For many years, historians thought that the 25. The primary purpose of the passage is to
development of capitalism had not faced serious A. reveal the underlying similarities of certain
arguments regarding the development of
challenges in the United States. Writing in the
capitalism in the United States
early twentieth century, progressive historians B. synthesize two competing arguments regarding
sympathized with the battles waged by farmers the development of capitalism in the United States
and small producers against large capitalists in C. defend an established argument regarding the
the late nineteenth century, but they did not development of capitalism in the United States
question the widespread acceptance of laissez- D. summarize a scholarly refutation of an argument
faire (unregulated) capitalism throughout regarding the development of capitalism in the
United States
American history. Similarly, Louis Hartz, who
E. discuss a new methodology for the study of the
sometimes disagreed with the Progressives, development of capitalism in the United States
argued that Americans accepted laissez-faire
capitalism without challenge because they 26. According to the passage, the Progressive
lacked a feudal, pre-capitalist past. Recently, historians and the scholars mentioned in the
however, some scholars have argued that even highlighted texts disagree with regard to which
though laissez-faire became the prevailing of the following?
A. Whether laissez-faire became the predominant
ethos in nineteen-century America, it was not
ethos in the nineteenth-century United States
accepted without struggle. Laissez-faire B. Whether moral restraints on acquisitiveness were
capitalism, they suggest, clashed with existing necessary in the nineteenth century United States
religious and communitarian norms that C. The economic utility of mercantilist notions of
imposed moral constraints on acquisitiveness government
to protect the weak from the predatory, the D. The nature of the historical conditions necessary
strong from corruption, and the entire culture for the development of laissez-faire capitalism in
the nineteen-century United States
from materialist excess. Buttressed by
E. The existence of significant opposition to the
mercantilist notions that government should be development of laissez-faire capitalism in the
both regulator and promoter of economic nineteen-century United States
activity, these norms persisted long after the
American Revolution helped unleash the 27. The passage suggests that the scholars
economic forces that produced capitalism. mentioned in the highlighted text would agree
These scholars argue that even in the late with which of the following statements
regarding the “norms” mentioned in the
nineteenth century, with the government’s role
subsequent highlighted text?
in the economy considerably diminished, A. They provided a source of opposition to the
laissez-faire had not triumphed completely. development of laissez-faire capitalism in the
Hard times continued to revive popular United States in the nineteenth century.
demands for regulating business and softening B. Their appeal was undermined by difficult
the harsh edges of laissez-faire capitalism. economic times in the United States at the end of
the nineteenth century.
C. They disappeared in the United States in the late
nineteenth century because of the triumph of
laissez-faire capitalism.
D. They facilitated the successful implementation of
mercantilist notions of government in the United
States in the nineteenth-century.
E. They are now recognized by historians as having
been an important part of the ideology of the
American Revolution.
Passage 12
According to a theory advanced by researcher 28. Which of the following is true about
Paul Martin, the wave of species extinctions that Martin’s theory, as that theory is
occurred in North America about 11,000 years described in the passage?
ago, at the end of the Pleistocene era, can be A. It assumes that the Paleo-Indians were
directly attributed to the arrival of humans, i.e., dependent only on hunting for survival.
the Paleoindians, who were ancestors of B. It denies that the Pleistocene species
modern Native Americans. However, extinctions were caused by climate change.
anthropologist Shepard Krech points out that C. It uses as evidence the fact that humans have
large animal species vanished even in areas produced local extinctions in other
where there is no evidence to demonstrate that situations.
Paleoindians hunted them. Nor were extinctions D. It attempts to address the controversy over
confined to large animals: small animals, plants, the date of human arrival in North America.
and insects disappeared, presumably not all E. It admits the possibility that factors other
through human consumption. Krech also than the arrival of humans played a role in
contradicts Martin’s exclusion of climatic the Pleistocene extinctions.
change as an explanation by asserting that
widespread climatic change did indeed occur at 29. The author refers to “recent
the end of the Pleistocene. Still, Krech attributes archaeological discoveries” most
secondary if not primary responsibility for the probably in order to
extinctions to the Paleoindians, arguing that A. refute White’s suggestion that neither
humans have produced local extinctions Martin nor Krech adequately account for
elsewhere. But, according to historian Richard Paleoindians’ contributions to the
White, even the attribution of secondary Pleistocene extinctions
responsibility may not be supported by the B. challenge on the possibility that a more
evidence. White observes that Martin’s thesis definitive theory regarding the causes of the
depends on coinciding dates for the arrival of Pleistocene extinctions may be forthcoming
humans and the decline of large animal species, C. suggest that Martin’s, Krech’s, and White’s
and Krech, though aware that the dates are theories regarding the Pleistocene
controversial, does not challenge them; yet extinctions are all open to question
recent archaeological discoveries are D. call attention to the most controversial
providing evidence that the date of human aspect of all the current theories regarding
arrival was much earlier than 11,000 years ago. the Pleistocene extinctions
Thus, White’s hypothesis stands vindicated. E. provide support for White’s questioning of
both Martin’s and Krech’s positions
regarding the role of Paleoindians in the
Pleistocene extinctions
Passage 1
Roger Rosenblatt’s book Black Fiction, in In addition, the style of some Black novels, like
attempting to apply literary rather than Jean Toomer’s Cane, verges on expressionism or
sociopolitical criteria to its subject, successfully surrealism; does this technique provide a
alters the approach taken by most previous counterpoint to the prevalent theme that
studies. As Rosenblatt notes, criticism of Black portrays the fate against which Black heroes are
writing has often served as a pretext for pitted, a theme usually conveyed by more
expounding on Black history. Addison Gayle’s naturalistic modes of expression?
recent work, for example, judges the value of
Black fiction by overtly political standards, rating In spite of such omissions, what Rosenblatt does
each work according to the notions of Black include in his discussion makes for an astute and
identity which it propounds. worthwhile study. Black Fiction surveys a wide
variety of novels, bringing to our attention in the
Although fiction assuredly springs from political process some fascinating and little-known works
circumstances, its authors react to those like James Weldon Johnson’s Autobiography of an
circumstances in ways other than ideological, and Ex-Colored Man. Its argument is tightly
talking about novels and stories primarily as constructed, and its forthright, lucid style
instruments of ideology circumvents much of the exemplifies levelheaded and penetrating
fictional enterprise. Rosenblatt’s literary analysis criticism.
discloses affinities and connections among works
of Black fiction which solely political studies have A. Tone of the passage
overlooked or ignored.
____________________________________________
Writing acceptable criticism of Black fiction,
however, presupposes giving satisfactory
answers to a number of questions. First of all, is B. Contradiction words: highlight the key
there a sufficient reason, other than the racial contradiction word(s).
identity of the authors, to group together works
by Black authors? Second, how does Black fiction C. Highlight all Opinions
make itself distinct from other modern fiction
with which it is largely contemporaneous? • Does the author have a personal
Rosenblatt shows that Black fiction constitutes a opinion? YES / NO
distinct body of writing that has an identifiable, • Is the author implicitly or explicitly
coherent literary tradition. Looking at novels supporting anyone?
written by Blacks over the last eighty years, he
discovers recurring concerns and designs
independent of chronology. These structures are
thematic, and they spring, not surprisingly, from
the central fact that the Black characters in these
novels exist in a predominantly White culture,
whether they try to conform to that culture or
rebel against it.
Roger Rosenblatt’s book Black Fiction, in In spite of such omissions, what Rosenblatt does
attempting to apply literary rather than include in his discussion makes for an astute and
sociopolitical criteria to its subject, successfully worthwhile study. Black Fiction surveys a wide
alters the approach taken by most previous variety of novels, bringing to our attention in the
studies. As Rosenblatt notes, criticism of Black process some fascinating and little-known works
writing has often served as a pretext for like James Weldon Johnson’s Autobiography of an
expounding on Black history. Addison Gayle’s Ex-Colored Man. Its argument is tightly
recent work, for example, judges the value of constructed, and its forthright, lucid style
Black fiction by overtly political standards, rating exemplifies levelheaded and penetrating
each work according to the notions of Black criticism.
identity which it propounds.
The author of the passage is primarily
Although fiction assuredly springs from political concerned with
circumstances, its authors react to those A. evaluating the soundness of a work of
circumstances in ways other than ideological, and criticism
talking about novels and stories primarily as B. comparing various critical approaches to
instruments of ideology circumvents much of the work of criticism
fictional enterprise. Rosenblatt’s literary analysis C. discussing the limitations of a particular work
discloses affinities and connections among works of criticism
of Black fiction which solely political studies have D. summarizing the major points made in a work
overlooked or ignored. of criticism
E. explaining the theoretical background of a
Writing acceptable criticism of Black fiction, certain work of criticism
however, presupposes giving satisfactory
answers to a number of questions. First of all, is The author of the passage believes that Black
there a sufficient reason, other than the racial Fiction would have been improved had
identity of the authors, to group together works Rosenblatt
by Black authors? Second, how does Black fiction A. praised more carefully the ideological and
make itself distinct from other modern fiction historical aspects of Black fiction
with which it is largely contemporaneous? B. attempted to be more objective in his
Rosenblatt shows that Black fiction constitutes a approach to novels and stories by Black
distinct body of writing that has an identifiable, authors
coherent literary tradition. Looking at novels C. explored in greater detail the recurrent
written by Blacks over the last eighty years, he thematic concerns of Black fiction throughout
discovers recurring concerns and designs its history
independent of chronology. These structures are D. established a basis for placing Black fiction
thematic, and they spring, not surprisingly, from within its own unique literary tradition
the central fact that the Black characters in these E. assessed the relative literary merit of the
novels exist in a predominantly White culture, novels he analyzes thematically
whether they try to conform to that culture or
rebel against it.
Passage 2
Most economists in the United States seem Socialist industry also works within a framework
captivated by the spell of the free market. of controlled prices. In the early 1970’s, the Soviet
Consequently, nothing seems good or normal that Union began to give firms and industries some of
does not accord with the requirements of the free the flexibility in adjusting prices that a more
market. A price that is determined by the seller or, informal evolution has accorded the capitalist
for that matter, established by anyone other than system. Economists in the United States have
the aggregate of consumers seems pernicious. hailed the change as a return to the free market.
Accordingly, it requires a major act of will to think But Soviet firms are no more subject to prices
of price-fixing (the determination of prices by the established by a free market over which they
seller) as both “normal” and having a valuable exercise little influence than are capitalist firms;
economic function. In fact, price-fixing is normal rather, Soviet firms have been given the power to
in all industrialized societies because the fix prices.
industrial system itself provides, as an effortless
consequence of its own development, the price-
fixing that it requires. Modern industrial planning A. Tone of the passage
requires and rewards great size. Hence, a
comparatively small number of large firms will be ____________________________________________
competing for the same group of consumers. That
each large firm will act with consideration of its
own needs and thus avoid selling its products for B. Contradiction words: highlight the key
more than its competitors charge is commonly contradiction word(s).
recognized by advocates of free-market economic
theories. But each large firm will also act with full C. Highlight all Opinions
consideration of the needs that it has in common
with the other large firms competing for the same • Does the author have a personal
customers. Each large firm will thus avoid opinion? YES / NO
significant price-cutting, because price-cutting • Is the author implicitly or explicitly
would be prejudicial to the common interest in a supporting anyone?
stable demand for products. Most economists do
not see price-fixing when it occurs because they
expect it to be brought about by a number of
explicit agreements among large firms; it is not.
Most economists in the United States seem Socialist industry also works within a framework
captivated by the spell of the free market. of controlled prices. In the early 1970’s, the Soviet
Consequently, nothing seems good or normal that Union began to give firms and industries some of
does not accord with the requirements of the free the flexibility in adjusting prices that a more
market. A price that is determined by the seller or, informal evolution has accorded the capitalist
for that matter, established by anyone other than system. Economists in the United States have
the aggregate of consumers seems pernicious. hailed the change as a return to the free market.
Accordingly, it requires a major act of will to think But Soviet firms are no more subject to prices
of price-fixing (the determination of prices by the established by a free market over which they
seller) as both “normal” and having a valuable exercise little influence than are capitalist firms;
economic function. In fact, price-fixing is normal rather, Soviet firms have been given the power to
in all industrialized societies because the fix prices.
industrial system itself provides, as an effortless
consequence of its own development, the price- In the passage, the author is primarily
fixing that it requires. Modern industrial planning concerned with
requires and rewards great size. Hence, a A. predicting the consequences of a practice
comparatively small number of large firms will be B. criticizing a point of view
competing for the same group of consumers. That C. calling attention to recent discoveries
each large firm will act with consideration of its D. proposing a topic for research
own needs and thus avoid selling its products for E. summarizing conflicting opinions
more than its competitors charge is commonly
recognized by advocates of free-market economic The author’s attitude toward “Most
theories. But each large firm will also act with full economists in the United States” in the first
consideration of the needs that it has in common statement of the passage can best be
with the other large firms competing for the same described as
customers. Each large firm will thus avoid A. spiteful
significant price-cutting, because price-cutting B. scornful
would be prejudicial to the common interest in a C. critical
stable demand for products. Most economists do D. ambivalent
not see price-fixing when it occurs because they E. deferential
expect it to be brought about by a number of
explicit agreements among large firms; it is not.
Passage 3
In the two decades between 1910 and 1930, over After the boll weevil infestation, urban Black
ten percent of the Black population of the United workers faced competition from the continuing
States left the South, where the preponderance of influx of both Black and White rural workers, who
the Black population had been located, and were driven to undercut the wages formerly paid
migrated to northern states, with the largest for industrial jobs. Thus, a move north would be
number moving, it is claimed, between 1916 and seen as advantageous to a group that was already
1918. It has been frequently assumed, but not urbanized and steadily employed, and the easy
proved, that the majority of the migrants in what conclusion tying their subsequent economic
has come to be called the Great Migration came problems in the North to their rural background
from rural areas and were motivated by two comes into question.
concurrent factors: the collapse of the cotton
industry following the boll weevil infestation, A. Tone of the passage
which began in 1898, and increased demand in
the North for labor following the cessation of ____________________________________________
European immigration caused by the outbreak of
the First World War in 1914. This assumption has
led to the conclusion that the migrants’ B. Contradiction words: highlight the key
subsequent lack of economic mobility in the contradiction word(s).
North is tied to rural background, a background
that implies unfamiliarity with urban living and a C. Highlight all Opinions
lack of industrial skills.
• Does the author have a personal
But the question of who actually left the South has opinion? YES / NO
never been rigorously investigated. Although • Is the author implicitly or explicitly
numerous investigations document an exodus supporting anyone?
from rural southern areas to southern cities prior
to the Great Migration, no one has considered
whether the same migrants then moved on to
northern cities. In 1910 over 600,000 Black
workers, or ten percent of the Black work force,
reported themselves to be engaged in
“manufacturing and mechanical pursuits,” the
federal census category roughly encompassing
the entire industrial sector. The Great Migration
could easily have been made up entirely of this
group and their families. It is perhaps surprising
to argue that an employed population could be
enticed to move, but an explanation lies in the
labor conditions then prevalent in the South.
In the two decades between 1910 and 1930, over After the boll weevil infestation, urban Black
ten percent of the Black population of the United workers faced competition from the continuing
States left the South, where the preponderance of influx of both Black and White rural workers, who
the Black population had been located, and were driven to undercut the wages formerly paid
migrated to northern states, with the largest for industrial jobs. Thus, a move north would be
number moving, it is claimed, between 1916 and seen as advantageous to a group that was already
1918. It has been frequently assumed, but not urbanized and steadily employed, and the easy
proved, that the majority of the migrants in what conclusion tying their subsequent economic
has come to be called the Great Migration came problems in the North to their rural background
from rural areas and were motivated by two comes into question.
concurrent factors: the collapse of the cotton
industry following the boll weevil infestation, The primary purpose of the passage is to
which began in 1898, and increased demand in A. support an alternative to an accepted
the North for labor following the cessation of methodology
European immigration caused by the outbreak of B. present evidence that resolves a contradiction
the First World War in 1914. This assumption has C. introduce a recently discovered source of
led to the conclusion that the migrants’ information
subsequent lack of economic mobility in the D. challenge a widely accepted explanation
North is tied to rural background, a background E. argue that a discarded theory deserves new
that implies unfamiliarity with urban living and a attention
lack of industrial skills.
Passage 4
At the end of the nineteenth century, a rising Despite all of this, autobiography remains a useful
interest in Native American customs and an tool for ethnological research: such personal
increasing desire to understand Native American reminiscences and impressions, incomplete as
culture prompted ethnologists to begin recording they may be, are likely to throw more light on the
the life stories of Native American. Ethnologists working of the mind and emotions than any
had a distinct reason for wanting to hear the amount of speculation from an ethnologist or
stories: they were after linguistic or ethnological theorist from another culture.
anthropological data that would supplement their
own field observations, and they believed that the A. Tone of the passage
personal stories, even of a single individual, could
increase their understanding of the cultures that ____________________________________________
they had been observing from without. In
addition, many ethnologists at the turn of the
century believed that Native American manners B. Contradiction words: highlight the key
and customs were rapidly disappearing, and that contradiction word(s).
it was important to preserve for posterity as
much information as could be adequately C. Highlight all Opinions
recorded before the cultures disappeared forever.
• Does the author have a personal
There were, however, arguments against this opinion? YES / NO
method as a way of acquiring accurate and • Is the author implicitly or explicitly
complete information. Franz Boas, for example, supporting anyone?
described autobiographies as being “of limited
value, and useful chiefly for the study of the
perversion of truth by memory,” while Paul Radin
contended that investigators rarely spent enough
time with the tribes they were observing, and
inevitably derived results too tinged by the
investigator’s own emotional tone to be reliable.
At the end of the nineteenth century, a rising Despite all of this, autobiography remains a useful
interest in Native American customs and an tool for ethnological research: such personal
increasing desire to understand Native American reminiscences and impressions, incomplete as
culture prompted ethnologists to begin recording they may be, are likely to throw more light on the
the life stories of Native American. Ethnologists working of the mind and emotions than any
had a distinct reason for wanting to hear the amount of speculation from an ethnologist or
stories: they were after linguistic or ethnological theorist from another culture.
anthropological data that would supplement their
own field observations, and they believed that the The primary purpose of the passage as a
personal stories, even of a single individual, could whole is to
increase their understanding of the cultures that A. question an explanation
they had been observing from without. In B. correct a misconception
addition, many ethnologists at the turn of the C. critique an approach
century believed that Native American manners D. discredit an idea
and customs were rapidly disappearing, and that E. clarify an ambiguity
it was important to preserve for posterity as
much information as could be adequately
recorded before the cultures disappeared forever.
Passage 5
Since the late 1970’s, in the face of a severe loss of Every company I know that has freed itself from the
market share in dozens of industries, paradox has done so, in part, by developing and
manufacturers in the United States have been trying implementing a manufacturing strategy. Such a
to improve productivity—and therefore enhance strategy focuses on the manufacturing structure
their international competitiveness—through cost- and on equipment and process technology and must
cutting programs. (Cost-cutting here is defined as be implemented by all those companies that want to
raising labor output while holding the amount of improve long-term productivity. In one company a
labor constant.) However, from 1978 through 1982, manufacturing strategy that allowed different areas
productivity—the value of goods manufactured of the factory to specialize in different markets
divided by the amount of labor input—did not replaced the conventional cost-cutting approach;
improve; and while the results were better in the within three years the company regained its
business upturn of the three years following, they competitive advantage. Together with such
ran 25 percent lower than productivity strategies, successful companies are also
improvements during earlier, post-1945 upturns. encouraging managers to focus on a wider set of
At the same time, it became clear that the harder objectives besides cutting costs. This is an example
manufactures worked to implement cost-cutting, worth emulating. If we are to have a hope for
the more they lost their competitive edge. manufacturing, the companies must change to a
With this paradox in mind, I recently visited 25 different way of managing.
companies; it became clear to me that the cost-
cutting approach to increasing productivity is A. Tone of the passage
fundamentally flawed. Manufacturing regularly
observes a “40, 40, 20” rule. Roughly 40 percent of ____________________________________________
any manufacturing-based competitive advantage
derives from long-term changes in manufacturing
structure (decisions about the number, size, B. Contradiction words: highlight the key
location, and capacity of facilities) and in contradiction word(s).
approaches to materials. Another 40 percent comes
from major changes in equipment and process
C. Highlight all Opinions
technology. The final 20 percent rests on
implementing conventional cost-cutting. This rule
does not imply that cost-cutting should not be tried.
• Does the author have a personal
The well-known tools of this approach—including opinion? YES / NO
simplifying jobs and retraining employees to work • Is the author implicitly or explicitly
smarter, not harder—do produce results. But the supporting anyone?
tools quickly reach the limits of what they can
contribute.
Another problem is that the cost-cutting approach
hinders innovation and discourages creative
people. As Abernathy’s study of automobile
manufacturers has shown, an industry can easily
become prisoner of its own investments in cost-
cutting techniques, reducing its ability to develop
new products. And managers under pressure to
maximize cost-cutting will resist innovation
because they know that more fundamental changes
in processes or systems will wreak havoc with the
results on which they are measured. Production
managers have always seen their job as one of
minimizing costs and maximizing output. This
dimension of performance has until recently
sufficed as a basis of evaluation, but it has created a
penny-pinching, mechanistic culture in most
factories that has kept away creative managers.
Since the late 1970’s, in the face of a severe loss of Every company I know that has freed itself from the
market share in dozens of industries, paradox has done so, in part, by developing and
manufacturers in the United States have been trying implementing a manufacturing strategy. Such a
to improve productivity—and therefore enhance strategy focuses on the manufacturing structure
their international competitiveness—through cost- and on equipment and process technology and must
cutting programs. (Cost-cutting here is defined as be implemented by all those companies that want to
raising labor output while holding the amount of improve long-term productivity. In one company a
labor constant.) However, from 1978 through 1982, manufacturing strategy that allowed different areas
productivity—the value of goods manufactured of the factory to specialize in different markets
divided by the amount of labor input—did not replaced the conventional cost-cutting approach;
improve; and while the results were better in the within three years the company regained its
business upturn of the three years following, they competitive advantage. Together with such
ran 25 percent lower than productivity strategies, successful companies are also
improvements during earlier, post-1945 upturns. encouraging managers to focus on a wider set of
At the same time, it became clear that the harder objectives besides cutting costs. This is an example
manufactures worked to implement cost-cutting, worth emulating. If we are to have a hope for
the more they lost their competitive edge. manufacturing, the companies must change to a
With this paradox in mind, I recently visited 25 different way of managing.
companies; it became clear to me that the cost-
cutting approach to increasing productivity is The author of the passage is primarily
fundamentally flawed. Manufacturing regularly concerned with
observes a “40, 40, 20” rule. Roughly 40 percent of A. summarizing a thesis
any manufacturing-based competitive advantage B. recommending a different approach
derives from long-term changes in manufacturing C. comparing points of view
structure (decisions about the number, size, D. making a series of predictions
location, and capacity of facilities) and in E. describing a number of paradoxes
approaches to materials. Another 40 percent comes
from major changes in equipment and process The author’s attitude toward the culture in most
technology. The final 20 percent rests on factories is best described as
implementing conventional cost-cutting. This rule A. cautious
does not imply that cost-cutting should not be tried. B. critical
The well-known tools of this approach—including C. disinterested
simplifying jobs and retraining employees to work D. respectful
smarter, not harder—do produce results. But the E. adulatory
tools quickly reach the limits of what they can
contribute. The author suggests that implementing
Another problem is that the cost-cutting approach conventional cost-cutting as a way of increasing
hinders innovation and discourages creative manufacturing competitiveness is a strategy
people. As Abernathy’s study of automobile that is
manufacturers has shown, an industry can easily A. appealing but ruinous
become prisoner of its own investments in cost- B. self-destructive but important for growth
cutting techniques, reducing its ability to develop C. popular but not easily accomplished
new products. And managers under pressure to D. somewhat useful in the short term but
maximize cost-cutting will resist innovation inadequate for massive growth in the long term
because they know that more fundamental changes E. misunderstood but promising
in processes or systems will wreak havoc with the
results on which they are measured. Production
managers have always seen their job as one of
minimizing costs and maximizing output. This
dimension of performance has until recently
sufficed as a basis of evaluation, but it has created a
penny-pinching, mechanistic culture in most
factories that has kept away creative managers.
Passage 6
Since the early 1970’s, historians have begun to A. Tone of the passage
devote serious attention to the working class in
the United States. Yet while we now have studies ____________________________________________
of working-class communities and culture, we
know remarkably little of worklessness. When
historians have paid any attention at all to B. Contradiction words: highlight the key
unemployment, they have focused on the Great contradiction word(s).
Depression of the 1930’s. The narrowness of this
perspective ignores the pervasive recessions and C. Highlight all Opinions
joblessness of the previous decades, as Alexander
Keyssar shows in his recent book. Examining the • Does the author have a personal
period 1870-1920, Keyssar concentrates on opinion? YES / NO
Massachusetts, where the historical materials are • Is the author implicitly or explicitly
particularly rich, and the findings applicable to supporting anyone?
other industrial areas.
Since the early 1970’s, historians have begun to The passage is primarily concerned with
devote serious attention to the working class in A. recommending a new course of investigation
the United States. Yet while we now have studies B. summarizing and assessing a study
of working-class communities and culture, we C. making distinctions among categories
know remarkably little of worklessness. When D. criticizing the current state of a field
historians have paid any attention at all to E. comparing and contrasting two methods for
unemployment, they have focused on the Great calculating data
Depression of the 1930’s. The narrowness of this
perspective ignores the pervasive recessions and The author views Keyssar’s study with
joblessness of the previous decades, as Alexander A. impatient disapproval
Keyssar shows in his recent book. Examining the B. wary concern
period 1870-1920, Keyssar concentrates on C. polite skepticism
Massachusetts, where the historical materials are D. scrupulous neutrality
particularly rich, and the findings applicable to E. qualified admiration
other industrial areas.
Passage 7
Increasingly, historians are blaming diseases Unfortunately, the documentation of these and
imported from the Old World for the staggering other epidemics is slight and frequently
disparity between the indigenous population of unreliable, and it is necessary to supplement
America in 1492—new estimates of which soar as whatever little we do know with evidence from
high as 100 million, or approximately one-sixth of recent epidemics among Native Americans. For
the human race at that time—and the few million example, in 1952 an outbreak of measles among
full-blooded Native Americans alive at the end of the Native American inhabitants of Ungava Bay,
the nineteenth century. There is no doubt that Quebec, affected 99 percent of the population and
chronic disease was an important factor in the killed 7 percent, even though some had the
precipitous decline, and it is highly probable that benefit of modern medicine. Cases such as this
the greatest killer was epidemic disease, demonstrate that even diseases that are not
especially as manifested in virgin-soil epidemics. normally fatal can have devastating
consequences when they strike an
Virgin-soil epidemics are those in which the immunologically defenseless community.
populations at risk have had no previous contact
with the diseases that strike them and are A. Tone of the passage
therefore immunologically almost defenseless.
That virgin-soil epidemics were important in ____________________________________________
American history is strongly indicated by
evidence that a number of dangerous maladies—
smallpox, measles, malaria, yellow fever, and B. Contradiction words: highlight the key
undoubtedly several more—were unknown in contradiction word(s).
the pre-Columbian New World. The effects of
their sudden introduction are demonstrated in C. Highlight all Opinions
the early chronicles of America, which contain
reports of horrendous epidemics and steep • Does the author have a personal
population declines, confirmed in many cases by opinion? YES / NO
recent quantitative analyses of Spanish tribute • Is the author implicitly or explicitly
records and other sources. The evidence provided supporting anyone?
by the documents of British and French colonies
is not as definitive because the conquerors of
those areas did not establish permanent
settlements and begin to keep continuous records
until the seventeenth century, by which time the
worst epidemics had probably already taken
place. Furthermore, the British tended to drive
the native populations away, rather than
enslaving them as the Spaniards did, so that the
epidemics of British America occurred beyond
the range of colonists’ direct observation.
Increasingly, historians are blaming diseases Unfortunately, the documentation of these and
imported from the Old World for the staggering other epidemics is slight and frequently
disparity between the indigenous population of unreliable, and it is necessary to supplement
America in 1492—new estimates of which soar as whatever little we do know with evidence from
high as 100 million, or approximately one-sixth of recent epidemics among Native Americans. For
the human race at that time—and the few million example, in 1952 an outbreak of measles among
full-blooded Native Americans alive at the end of the Native American inhabitants of Ungava Bay,
the nineteenth century. There is no doubt that Quebec, affected 99 percent of the population and
chronic disease was an important factor in the killed 7 percent, even though some had the
precipitous decline, and it is highly probable that benefit of modern medicine. Cases such as this
the greatest killer was epidemic disease, demonstrate that even diseases that are not
especially as manifested in virgin-soil epidemics. normally fatal can have devastating
consequences when they strike an
Virgin-soil epidemics are those in which the immunologically defenseless community.
populations at risk have had no previous contact
with the diseases that strike them and are The primary purpose of the passage is to
therefore immunologically almost defenseless. A. refute a common misconception
That virgin-soil epidemics were important in B. provide support for a hypothesis
American history is strongly indicated by C. analyze an argument
evidence that a number of dangerous maladies— D. suggest a solution to a dilemma
smallpox, measles, malaria, yellow fever, and E. reconcile opposing viewpoints
undoubtedly several more—were unknown in
the pre-Columbian New World. The effects of
their sudden introduction are demonstrated in
the early chronicles of America, which contain
reports of horrendous epidemics and steep
population declines, confirmed in many cases by
recent quantitative analyses of Spanish tribute
records and other sources. The evidence provided
by the documents of British and French colonies
is not as definitive because the conquerors of
those areas did not establish permanent
settlements and begin to keep continuous records
until the seventeenth century, by which time the
worst epidemics had probably already taken
place. Furthermore, the British tended to drive
the native populations away, rather than
enslaving them as the Spaniards did, so that the
epidemics of British America occurred beyond
the range of colonists’ direct observation.
Passage 8
Until recently most astronomers believed that the A. Tone of the passage
space between the galaxies in our universe was a
near-perfect vacuum. This orthodox view of the ____________________________________________
universe is now being challenged by astronomers
who believe that a heavy “rain” of gas is falling into
many galaxies from the supposedly empty space B. Contradiction words: highlight the key
around them. The gas apparently condenses into a contradiction word(s).
collection of small stars, each a little larger than the
planet Jupiter. These stars vastly outnumber the C. Highlight all Opinions
other stars in a given galaxy. The amount of
“intergalactic rainfall” into some of these galaxies
• Does the author have a personal
has been enough to double their mass in the time
since they formed. Scientists have begun to suspect
opinion? YES / NO
that this intergalactic gas is probably a mixture of • Is the author implicitly or explicitly
gases left over from the “big bang” when the supporting anyone?
galaxies were formed and gas was forced out of
galaxies by supernova explosions.
Until recently most astronomers believed that the The primary purpose of the passage is to
space between the galaxies in our universe was a A. illustrate a hypothesis about the origin of
near-perfect vacuum. This orthodox view of the galaxies
universe is now being challenged by astronomers B. introduce a hypothesis that challenges an
who believe that a heavy “rain” of gas is falling accepted theory about the evolution of
into many galaxies from the supposedly empty galaxies and present some evidence to
space around them. The gas apparently support the hypothesis
condenses into a collection of small stars, each a C. summarize the state of and prospects for
little larger than the planet Jupiter. These stars research in intergalactic astronomy
vastly outnumber the other stars in a given D. report new data on the origins of intergalactic
galaxy. The amount of “intergalactic rainfall” into gas
some of these galaxies has been enough to double E. reconcile opposing views on the formation of
their mass in the time since they formed. intergalactic gas
Scientists have begun to suspect that this
intergalactic gas is probably a mixture of gases
left over from the “big bang” when the galaxies
were formed and gas was forced out of galaxies by
supernova explosions.
Passage 9
Japanese firms have achieved the highest levels of A. Tone of the passage
manufacturing efficiency in the world automobile
industry. Some observers of Japan have assumed ____________________________________________
that Japanese firms use the same manufacturing
equipment and techniques as United States firms
but have benefited from the unique B. Contradiction words: highlight the key
characteristics of Japanese employees and the contradiction word(s).
Japanese culture. However, if this were true, then
one would expect Japanese auto plants in the C. Highlight all Opinions
United States to perform no better than factories
run by United States companies. This is not the • Does the author have a personal
case; Japanese-run automobile plants located in opinion? YES / NO
the United States and staffed by local workers • Is the author implicitly or explicitly
have demonstrated higher levels of productivity supporting anyone?
when compared with factories owned by United
States companies.
Japanese firms have achieved the highest levels of The primary purpose of the passage is to
manufacturing efficiency in the world automobile A. present the major steps of a process
industry. Some observers of Japan have assumed B. clarify an ambiguity
that Japanese firms use the same manufacturing C. chronicle a dispute
equipment and techniques as United States firms D. correct misconceptions
but have benefited from the unique E. defend an accepted approach
characteristics of Japanese employees and the
Japanese culture. However, if this were true, then
one would expect Japanese auto plants in the
United States to perform no better than factories
run by United States companies. This is not the
case; Japanese-run automobile plants located in
the United States and staffed by local workers
have demonstrated higher levels of productivity
when compared with factories owned by United
States companies.
Passage 10
Historians sometimes forget that history is The attitude of the author of the passage
continually being made and experienced before it toward the work of C. Vann Woodward is best
is studied, interpreted, and read. These latter described as one of
activities have their own history, of course, which A. respectful regard
may impinge in unexpected ways on public B. qualified approbation
events. It is difficult to predict when “new pasts” C. implied skepticism
will overturn established historical D. pointed criticism
interpretations and change the course of history. E. fervent advocacy
Passage 11
Joseph Glatthaar’s Forged in Battle is not the first A. Tone of the passage
excellent study of Black soldiers and their White
officers in the Civil War, but it uses more soldiers’ ____________________________________________
letters and diaries—including rare material from
Black soldiers—and concentrates more intensely
on Black-White relations in Black regiments than B. Contradiction words: highlight the key
do any of its predecessors. Glatthaar’s title contradiction word(s).
expresses his thesis: loyalty, friendship, and
respect among White officers and Black soldiers C. Highlight all Opinions
were fostered by the mutual dangers they faced in
combat. • Does the author have a personal
opinion? YES / NO
Glatthaar accurately describes the government’s • Is the author implicitly or explicitly
discriminatory treatment of Black soldiers in pay, supporting anyone?
promotion, medical care, and job assignments,
appropriately emphasizing the campaign by
Black soldiers and their officers to get the
opportunity to fight. That chance remained
limited throughout the war by army policies that
kept most Black units serving in rear-echelon
assignments and working in labor battalions.
Thus, while their combat death rate was only one-
third that of White units, their mortality rate from
disease, a major killer in his war, was twice as
great. Despite these obstacles, the courage and
effectiveness of several Black units in combat won
increasing respect from initially skeptical or
hostile White soldiers. As one White officer put it,
“they have fought their way into the respect of all
the army.”
Joseph Glatthaar’s Forged in Battle is not the first The passage as a whole can best be
excellent study of Black soldiers and their White characterized as which of the following?
officers in the Civil War, but it uses more soldiers’ A. An evaluation of a scholarly study
letters and diaries—including rare material from B. A description of an attitudinal change
Black soldiers—and concentrates more intensely C. A discussion of an analytical defect
on Black-White relations in Black regiments than D. An analysis of the causes of a phenomenon
do any of its predecessors. Glatthaar’s title E. An argument in favor of revising a view
expresses his thesis: loyalty, friendship, and
respect among White officers and Black soldiers
were fostered by the mutual dangers they faced in
combat.
Passage 12
In most earthquakes the Earth’s crust cracks like A. Tone of the passage
porcelain. Stress builds up until a fracture forms
at a depth of a few kilometers and the crust slips ____________________________________________
to relieve the stress. Some earthquakes, however,
take place hundreds of kilometers down in the
Earth’s mantle, where high pressure makes rock B. Contradiction words: highlight the key
so ductile that it flows instead of cracking, even contradiction word(s).
under stress severe enough to deform it like
putty. How can there be earthquakes at such C. Highlight all Opinions
depths?
• Does the author have a personal
That such deep events do occur has been accepted opinion? YES / NO
only since 1927, when the seismologist Kiyoo • Is the author implicitly or explicitly
Wadati convincingly demonstrated their supporting anyone?
existence. Instead of comparing the arrival times
of seismic waves at different locations, as earlier
researchers had done. Wadati relied on a time
difference between the arrival of primary (P)
waves and the slower secondary (S) waves.
Because P and S waves travel at different but
fairly constant speeds, the interval between their
arrivals increases in proportion to the distance
from the earthquake focus, or rupture point.
In most earthquakes the Earth’s crust cracks like The passage is primarily concerned with
porcelain. Stress builds up until a fracture forms A. demonstrating why the methods of early
at a depth of a few kilometers and the crust slips seismologists were flawed
to relieve the stress. Some earthquakes, however, B. arguing that deep events are poorly
take place hundreds of kilometers down in the understood and deserve further study
Earth’s mantle, where high pressure makes rock C. providing support for a theory about a type of
so ductile that it flows instead of cracking, even earthquake that was previously not well
under stress severe enough to deform it like understood
putty. How can there be earthquakes at such D. discussing evidence for the existence of deep
depths? events and the conditions that allow them to
occur
That such deep events do occur has been accepted E. comparing the effects of shallow events with
only since 1927, when the seismologist Kiyoo those of deep events
Wadati convincingly demonstrated their
existence. Instead of comparing the arrival times
of seismic waves at different locations, as earlier
researchers had done. Wadati relied on a time
difference between the arrival of primary (P)
waves and the slower secondary (S) waves.
Because P and S waves travel at different but
fairly constant speeds, the interval between their
arrivals increases in proportion to the distance
from the earthquake focus, or rupture point.
Passage 13
Most large corporations in the United States were A. Tone of the passage
once run by individual capitalists who owned
enough stock to dominate the board of directors ____________________________________________
and dictate company policy. Because putting such
large amounts of stock on the market would only
depress its value, they could not sell out for a B. Contradiction words: highlight the key
quick profit and instead had to concentrate on contradiction word(s).
improving the long-term productivity of their
companies. Today, with few exceptions, the stock C. Highlight all Opinions
of large United States corporations is held by
large institutions—pension funds, for example— • Does the author have a personal
and because these institutions are prohibited by opinion? YES / NO
antitrust laws from owning a majority of a • Is the author implicitly or explicitly
company’s stock and from actively influencing a supporting anyone?
company’s decision-making, they can enhance
their wealth only by buying and selling stock in
anticipation of fluctuations in its value. A minority
shareholder is necessarily a short-term trader. As
a result, United States productivity is unlikely to
improve unless shareholders and the managers of
the companies in which they invest are
encouraged to enhance long-term productivity
(and hence long-term profitability), rather than
simply to maximize short-term profits.
Most large corporations in the United States were In the passage, the author is primarily
once run by individual capitalists who owned concerned with doing which of the following?
enough stock to dominate the board of directors A. Comparing two different approaches to a
and dictate company policy. Because putting such problem
large amounts of stock on the market would only B. Describing a problem and proposing a
depress its value, they could not sell out for a solution
quick profit and instead had to concentrate on C. Defending an established method
improving the long-term productivity of their D. Presenting data and drawing conclusions
companies. Today, with few exceptions, the stock from the data
of large United States corporations is held by E. Comparing two different analyses of a current
large institutions—pension funds, for example— situation
and because these institutions are prohibited by
antitrust laws from owning a majority of a
company’s stock and from actively influencing a
company’s decision-making, they can enhance
their wealth only by buying and selling stock in
anticipation of fluctuations in its value. A minority
shareholder is necessarily a short-term trader. As
a result, United States productivity is unlikely to
improve unless shareholders and the managers of
the companies in which they invest are
encouraged to enhance long-term productivity
(and hence long-term profitability), rather than
simply to maximize short-term profits.
Passage 14
For over 300 years, one of the most enduring If further studies of country-house ownership
beliefs among historians of England has been that attest to the representativeness and accuracy of
the character of English society has been shaped their data, then the Stones’ conclusion that the
by the unique openness of its ruling elite to entry open elite thesis cannot be maintained may,
by self-made entrepreneurs (especially newly indeed, prove true.
wealthy merchants) able to buy their way into the
ranks of elite society. This upward mobility, A. Tone of the passage
historians have argued, allowed England to
escape the clash between those with social / ____________________________________________
political power and those with economic power, a
conflict that beset the rest of Europe during the
eighteenth and nineteenth centuries. Upward B. Contradiction words: highlight the key
mobility was also used to explain England’s contradiction word(s).
exceptional stability since the late seventeenth
century (no revolutions, for example), as well as C. Highlight all Opinions
such major events as the development of the most
efficient agricultural system in Europe, the • Does the author have a personal
making of the first industrial revolution, and the opinion? YES / NO
onset of severe economic decline. • Is the author implicitly or explicitly
supporting anyone?
But is the thesis true? Recent work on the
supposed consequences of an open elite has
already produced some doubts. Little credence,
for example, is now accorded the idea that
England’s late nineteenth-century economic
decline resulted from absentee business owners
too distracted by the demands of elite life to
manage their firms properly. But, although the
importance of an open elite to other major events
has been severely questioned, it is only with a
new work by Lawrence and Jeanne Stone that the
openness itself has been confronted. Eschewing
the tack of tracing the careers of successful
entrepreneurs to gauge the openness of the elite,
the Stones chose the alternative approach of
analyzing the elite itself, and proceeded via the
ingenious route of investigating country-house
ownership.
For over 300 years, one of the most enduring beliefs The tone of the passage suggests that the author
among historians of England has been that the regards the Stones’ methodological approach as
character of English society has been shaped by the A. problematic
unique openness of its ruling elite to entry by self- B. difficult
made entrepreneurs (especially newly wealthy C. controversial
merchants) able to buy their way into the ranks of D. rigorous
elite society. This upward mobility, historians have E. clever
argued, allowed England to escape the clash
between those with social / political power and The primary purpose of the passage is to
those with economic power, a conflict that beset the A. resolve a debate between two schools of
rest of Europe during the eighteenth and nineteenth thought.
centuries. Upward mobility was also used to explain B. Present research that questions an established
England’s exceptional stability since the late view.
seventeenth century (no revolutions, for example), C. Describe and criticize a new approach.
as well as such major events as the development of D. Defend a traditional interpretation against
the most efficient agricultural system in Europe, the recent criticisms.
making of the first industrial revolution, and the E. Analyze possible approaches to resolving a
onset of severe economic decline. long-standing controversy.
But is the thesis true? Recent work on the supposed Which of the following best states the main idea of
consequences of an open elite has already produced the passage?
some doubts. Little credence, for example, is now A. Assumptions about the nature of England’s ruling
accorded the idea that England’s late nineteenth- elite can no longer be used with certitude to
century economic decline resulted from absentee explain many major economic developments.
business owners too distracted by the demands of B. The concept of the open elite is of paramount
elite life to manage their firms properly. But, importance in explaining major English political,
although the importance of an open elite to other social, and economic events.
major events has been severely questioned, it is C. The long-standing belief that England possessed a
remarkably open ruling elite has recently been
only with a new work by Lawrence and Jeanne
subjected to lethal criticism.
Stone that the openness itself has been confronted.
D. Although many possibilities are available, the
Eschewing the tack of tracing the careers of
most reliable means of testing the truth of the
successful entrepreneurs to gauge the openness of ‘open elite’ hypothesis is to analyze changes in the
the elite, the Stones chose the alternative approach composition of the elite.
of analyzing the elite itself, and proceeded via the E. An analysis of English country-house ownership in
ingenious route of investigating country-house England indicates that there were far too many
ownership. opportunities for merchants to buy the estates of
old members of the landed elite.
Arguing that ownership of a country house was seen
as essential for membership in the ruling elite, the
Stones analyze the nature of country-house
ownership in three counties for the period 1540-
1880. Their critical findings are provocative: there
was strikingly little change in the ownership of such
houses throughout the period. Instead, even in the
face of a demographic crisis (fewer marriages,
declining fertility, rising infant mortality), the old
elite was able to maintain itself, and its estates,
intact for centuries through recourse to various
marriage and inheritance strategies. The popular
picture of venerable elite families overcome by debt
and selling out to merchants is simply not borne out
by the Stones’ findings. Rather, the opportunities for
entrepreneurs to buy their way into the elite, the
Stones show, were extremely limited.
Passage 15
A majority taken collectively may be regarded as When an individual or a party is wronged in the
a being whose opinions and, most frequently, United States, to whom can he apply for redress?
whose interests are opposed to those of another If to the public opinion, public opinion constitutes
being, which is styled a minority. If it is admitted the majority; if to the legislature, it represents the
that a man possessing absolute power may majority and implicitly obeys its injunctions; if to
misuse that power by wronging his adversaries, the executive power, it is appointed by the
why should a majority not be liable to the same majority and remains a passive tool in its hands;
reproach? Men are not apt to change their the public troops consist of the majority under
characters by agglomeration; nor does their arms; the jury is the majority invested with the
patience in the presence of obstacles increase right of hearing judicial cases, and in certain
with the consciousness of their strength. For states even the judges are elected by the majority.
these reasons we should not willingly invest any However iniquitous or absurd the evil
group of our fellows with that unlimited authority complained about, no sure barrier is established
which we should refuse to any individual. to defend against it.
One social power must always predominate over A. Tone of the passage
others, but liberty is endangered when this power
is checked by no obstacles which may retard its ____________________________________________
course and force it to moderate its own
vehemence. Unlimited power is in itself a bad and
dangerous thing, and no power on earth is so B. Contradiction words: highlight the key
worthy of honor for itself or of reverential contradiction word(s).
obedience to the rights which it represents that
we should admit its uncontrolled and all- C. Highlight all Opinions
predominant authority. When the right and
means of absolute command are conferred on a • Does the author have a personal
people or a king, on an aristocracy or a opinion? YES / NO
democracy, a monarchy or a republic, there has • Is the author implicitly or explicitly
been implanted the germ of tyranny. supporting anyone?
A majority taken collectively may be regarded as When an individual or a party is wronged in the
a being whose opinions and, most frequently, United States, to whom can he apply for redress?
whose interests are opposed to those of another If to the public opinion, public opinion constitutes
being, which is styled a minority. If it is admitted the majority; if to the legislature, it represents the
that a man possessing absolute power may majority and implicitly obeys its injunctions; if to
misuse that power by wronging his adversaries, the executive power, it is appointed by the
why should a majority not be liable to the same majority and remains a passive tool in its hands;
reproach? Men are not apt to change their the public troops consist of the majority under
characters by agglomeration; nor does their arms; the jury is the majority invested with the
patience in the presence of obstacles increase right of hearing judicial cases, and in certain
with the consciousness of their strength. For states even the judges are elected by the majority.
these reasons we should not willingly invest any However iniquitous or absurd the evil
group of our fellows with that unlimited authority complained about, no sure barrier is established
which we should refuse to any individual. to defend against it.
One social power must always predominate over In the passage, the author is primarily
others, but liberty is endangered when this power concerned with
is checked by no obstacles which may retard its A. presenting the dangers inherent in a popular
course and force it to moderate its own idea
vehemence. Unlimited power is in itself a bad and B. contrasting two opposing views
dangerous thing, and no power on earth is so C. advocating a specific course of action to solve
worthy of honor for itself or of reverential a problem
obedience to the rights which it represents that D. reconciling an apparent conflict
we should admit its uncontrolled and all- E. proposing a solution to an unrecognized
predominant authority. When the right and problem
means of absolute command are conferred on a
people or a king, on an aristocracy or a
democracy, a monarchy or a republic, there has
been implanted the germ of tyranny.
Passage 16
Many readers assume that, as a neoclassical From those sounds which we hear on small or on
literary critic, Samuel Johnson would normally coarse occasions, we do not easily receive strong
prefer the abstract, the formal, and the regulated impressions, or delightful images; and words to
to the concrete, the natural, and the spontaneous which we are nearly strangers, whenever they
in a work of literature. Yet any close reading of occur, draw that attention on themselves which
Johnson’s criticism shows that Johnson is not they should transmit to things.” If the poetic
blind to the importance of the immediate, vivid, diction of the neoclassical poets, at its worst,
specific detail in literature; rather, he would erects needless barriers between reader and
underscore the need for the telling rather than the meaning, that envisioned by Johnson would do
merely accidental detail. just the opposite: it would put the reader in closer
In other ways, too, Johnson’s critical method had contact with the “things” that are the poem’s
much in common with that of the Romantics, with subject.
whom Johnson and, indeed, the entire
neoclassical tradition, are generally supposed to A. Tone of the passage
be in conflict. Johnson was well aware, for
example, of the sterility of literary criticism that is ____________________________________________
legalistic or pedantic, as was the case with the
worst products of the neoclassical school. His
famous argument against the slavish following of B. Contradiction words: highlight the key
the “three unities” of classical drama is a good contradiction word(s).
example, as is his defense of the supposedly
illegitimate “tragicomic” mode of Shakespeare’s C. Highlight all Opinions
latest plays. Note, in particular, the basis of that
defense: “That this is a practice contrary to the • Does the author have a personal
rules of criticism,” Johnson wrote, “will be readily opinion? YES / NO
allowed; but there is always an appeal from • Is the author implicitly or explicitly
criticism to nature.” supporting anyone?
The sentiment thus expressed could easily be
endorsed by any of the Romantics; the empiricism
it exemplifies is vital quality of Johnson’s
criticism, as is the willingness to jettison “laws” of
criticism when to do so makes possible a more
direct appeal to the emotions of the reader.
Addison’s Cato, highly praised in Johnson’s day
for its “correctness,” is damned by Johnson: “Cato
affords a splendid exhibition of artificial and
fictitious manners, and delivers just and noble
sentiments, its diction is easy, elevated, and
harmonious, but its hopes and fears communicate
no vibration to the heart.” Wordsworth could
hardly demur.
Even on the question of poetic diction, which,
according to the usual interpretation of
Wordsworth’s 1800 preface to the Lyrical Ballads,
was the central area of conflict between Romantic
and Augustan, Johnson’s views are surprisingly
“modern.” In his Life of Dryden, it is true that he
defends the use of a special diction in poetry; but
his reasons are all-important. For Johnson, poetic
diction should serve the ends of direct emotional
impact and ease of comprehension, not those of
false profundity or grandiosity. “Words too
familiar,” he wrote, “or too remote, defeat the
purpose of a poet.
Many readers assume that, as a neoclassical literary From those sounds which we hear on small or on
critic, Samuel Johnson would normally prefer the coarse occasions, we do not easily receive strong
abstract, the formal, and the regulated to the impressions, or delightful images; and words to
concrete, the natural, and the spontaneous in a which we are nearly strangers, whenever they
work of literature. Yet any close reading of occur, draw that attention on themselves which
Johnson’s criticism shows that Johnson is not blind they should transmit to things.” If the poetic diction
to the importance of the immediate, vivid, specific of the neoclassical poets, at its worst, erects
detail in literature; rather, he would underscore the needless barriers between reader and meaning,
need for the telling rather than the merely that envisioned by Johnson would do just the
accidental detail. opposite: it would put the reader in closer contact
In other ways, too, Johnson’s critical method had with the “things” that are the poem’s subject.
much in common with that of the Romantics, with
whom Johnson and, indeed, the entire neoclassical The author of the passage is primarily
tradition, are generally supposed to be in conflict. concerned with
Johnson was well aware, for example, of the sterility A. defending a reputation
of literary criticism that is legalistic or pedantic, as B. reconciling conflicting views
was the case with the worst products of the C. comparing two schools of thought
neoclassical school. His famous argument against D. challenging an assumption
the slavish following of the “three unities” of E. presenting new evidence in support of an
classical drama is a good example, as is his defense established theory
of the supposedly illegitimate “tragicomic” mode of
Shakespeare’s latest plays. Note, in particular, the According to the passage, Johnson’s opinion of
basis of that defense: “That this is a practice Addison’s Cato was
contrary to the rules of criticism,” Johnson wrote, A. Inflammatory
“will be readily allowed; but there is always an B. Self-contradictory
appeal from criticism to nature.” C. Negative
The sentiment thus expressed could easily be D. Adulatory
endorsed by any of the Romantics; the empiricism it E. Bold
exemplifies is vital quality of Johnson’s criticism, as
is the willingness to jettison “laws” of criticism Which one of the following statements best
when to do so makes possible a more direct appeal summarizes the main point of the passage?
to the emotions of the reader. Addison’s Cato, highly A. Although some readers feel that Johnson’s
praised in Johnson’s day for its “correctness,” is critical opinions at times resemble those of the
damned by Johnson: “Cato affords a splendid neoclassical critics, his basic concerns are closer
exhibition of artificial and fictitious manners, and to those of the Romantics.
delivers just and noble sentiments, its diction is B. The usual classification of Johnson as a member
easy, elevated, and harmonious, but its hopes and of the Romantic school of criticism is based on
fears communicate no vibration to the heart.” an inaccurate evaluation of his critical theories
Wordsworth could hardly demur. and ideals.
Even on the question of poetic diction, which, C. The Romantic critics were mistaken in their
according to the usual interpretation of belief that the critical ideas they formulated
Wordsworth’s 1800 preface to the Lyrical Ballads, represented a departure from those
was the central area of conflict between Romantic propounded by Johnson.
and Augustan, Johnson’s views are surprisingly D. Although many of Johnson’s critical opinions
“modern.” In his Life of Dryden, it is true that he resemble those of the Romantic critics, his basic
defends the use of a special diction in poetry; but his concerns are closer to those of the neoclassical
reasons are all-important. For Johnson, poetic critics.
diction should serve the ends of direct emotional E. Johnson’s literary criticism represents an
impact and ease of comprehension, not those of attempt to unify the best elements of the
false profundity or grandiosity. “Words too neoclassical and the Romantic schools of
familiar,” he wrote, “or too remote, defeat the criticism.
purpose of a poet.
Passage 17
The new school of political history that emerged A. Tone of the passage
in the 1960’s and 1970’s sought to go beyond the
traditional focus of political historians on leaders ____________________________________________
and government institutions by examining
directly the political practices of ordinary
citizens. Like the old approach, however, this new B. Contradiction words: highlight the key
approach excluded women. The very techniques contradiction word(s).
these historians used to uncover mass political
behavior in the nineteenth-century United C. Highlight all Opinions
States—quantitative analyses of election returns,
for example—were useless in analyzing the • Does the author have a personal
political activities of women, who were denied opinion? YES / NO
the vote until 1920. • Is the author implicitly or explicitly
supporting anyone?
By redefining “political activity,” historian Paula
Baker has developed a political history that
includes women. She concludes that among
ordinary citizens, political activism by women in
the nineteenth century prefigured trends in
twentieth-century politics. Defining “politics” as
“any action taken to affect the course of behavior
of government or of the community,” Baker
concludes that, while voting and holding office
were restricted to men, women in the nineteenth
century organized themselves into societies
committed to social issues such as temperance
and poverty. In other words, Baker contends,
women activists were early practitioners of
nonpartisan, issue-oriented politics and thus
were more interested in enlisting lawmakers,
regardless of their party affiliation, on behalf of
certain issues than in ensuring that one party or
another won an election. In the twentieth century,
more men drew closer to women’s ideas about
politics and took up modes of issue-oriented
politics that Baker sees women as having
pioneered.
The new school of political history that emerged The primary purpose of the passage is to
in the 1960’s and 1970’s sought to go beyond the A. enumerate reasons why both traditional
traditional focus of political historians on leaders scholarly methods and newer scholarly
and government institutions by examining methods have limitations
directly the political practices of ordinary B. identify a shortcoming in a scholarly approach
citizens. Like the old approach, however, this new and describe an alternative approach
approach excluded women. The very techniques C. provide empirical data to support a long-held
these historians used to uncover mass political scholarly assumption
behavior in the nineteenth-century United D. compare two scholarly publications on the
States—quantitative analyses of election returns, basis of their authors’ backgrounds
for example—were useless in analyzing the E. attempt to provide a partial answer to a long-
political activities of women, who were denied standing scholarly dilemma
the vote until 1920.
Passage 18
Passage 19
Arboria is floundering in the global marketplace, Both assumptions are wrong. The 40-year-old
incurring devastating losses in market position GATT now covers less than 7 percent of global
and profits. The problem is not Arboria's commerce. World trade is no longer dominated
products, but Arboria's trade policy, which must by the free-trade economies; nearly 75 percent is
change. Arboria faces the prospect of continuing conducted by economic systems operating with
economic loss until Arborian business and principles at odds with those of Arboria. Forging
political leaders recognize the fundamental a multilateral trade policy consensus among so
differences between Arborian and foreign many diverse economic systems has become
economic systems. There is still chance to salvage virtually impossible. And while multilateral talks
the situation and the leaders must come forward drag on, Arboria misses opportunities for trade
to actively resolve every single issue that plagues expansion.
the economy. Today the key trade issue is not free
trade versus protectionism but diminishing trade A. Tone of the passage
versus expanding trade.
____________________________________________
Arboria is operating with an obsolete trade policy,
an artifact of the mid-1940s when Arboria and
Whorfland dominated the global economy, tariffs B. Contradiction words: highlight the key
were the principal obstacle to trade, and Arborian contradiction word(s).
supremacy was uncontested in virtually all
industries. In the intervening decades, economic C. Highlight all Opinions
circumstances have shifted radically. Arborian
trade policy has not. • Does the author have a personal
opinion? YES / NO
Today, Arboria's trade policy seems paralyzed by • Is the author implicitly or explicitly
the relentless conflict between proponents of supporting anyone?
“free” and “fair” trade. The free traders argue that
Arborian markets should be open, and the
movement of goods and services across national
borders unrestrained. The fair traders assert that
access to Arborian markets should be restricted
until Arborian businesses are granted equal
access to foreign markets. They contend that free
trade is impossible while other nations erect
barriers to Arborian exports.
Arboria is floundering in the global marketplace, The passage is primarily concerned with
incurring devastating losses in market position A. illustrating the erosion of Arboria's position in
and profits. The problem is not Arboria's the world marketplace
products, but Arboria's trade policy, which must B. examining the differences between “free” and
change. Arboria faces the prospect of continuing “fair” traders
economic loss until Arborian business and C. advocating a reassessment of Arboria's trade
political leaders recognize the fundamental policy
differences between Arborian and foreign D. criticizing the terms of the General Agreement
economic systems. There is still chance to salvage on Tariffs and Trade (GATT)
the situation and the leaders must come forward E. comparing the different economic
to actively resolve every single issue that plagues circumstances of Arboria's trade partners
the economy. Today the key trade issue is not free
trade versus protectionism but diminishing trade
versus expanding trade.
Passage 20
In the Sonoran Desert of northwestern Mexico The primary purpose of the passage is to
and southern Arizona, the flowers of several A. compare the adaptive responses of several
species of columnar cacti—cardon, saguaro, and species of columnar cacti in the Sonoran
organ pipe—were once exclusively pollinated at Desert with those in the arid tropical regions
night by nectar-feeding bats, as their close of southern Mexico
relatives in arid tropical regions of southern B. discuss some of the possible consequences of
Mexico still are. In these tropical regions, diurnal the relatively low abundance of migratory
(daytime) visitors to columnar cactus flowers are nectar-feeding bats in the Sonoran Desert
ineffective pollinators because, by sunrise, the C. provide a possible explanation for a particular
flowers' stigmas become unreceptive or the evolutionary change in certain species of
flowers close. Yet the flowers of the Sonoran columnar cacti in the Sonoran Desert
Desert cacti have evolved to remain open after D. present recent findings that challenge a
sunrise, allowing pollination by such diurnal particular theory as to why several species of
visitors as bees and birds. Why have these cacti columnar cacti in the Sonoran Desert have
expanded their range of pollinators by remaining expanded their range of pollinators
open and receptive in daylight? E. compare the effectiveness of nocturnal and
diurnal pollination for several different
This development at the northernmost range of species of columnar cacti in the Sonoran
columnar cacti may be due to a yearly variation in Desert
the abundance—and hence the reliability—of
migratory nectar-feeding bats. Pollinators can be
unreliable for several reasons. They can be
dietary generalists whose fidelity to a particular
species depends on the availability of alternative
food sources. Or, they can be dietary specialists,
but their abundance may vary widely from year to
year, resulting in variable pollination of their
preferred food species. Finally, they may be
dietary specialists, but their abundance may be
chronically low relative to the availability of
flowers.
Passage 21
A small number of the forest species of The primary purpose of the passage is to
lepidoptera (moths and butterflies, which exist as A. describe the development of new techniques
caterpillars during most of their life cycle) exhibit that may help to determine the driving force
regularly recurring patterns of population growth behind population cycles in lepidoptera
and decline—such fluctuations in population are B. present evidence that refutes a particular
known as population cycles. Although many theory about the driving force behind
different variables influence population levels, a population cycles in lepidoptera
regular pattern such as a population cycle seems C. present a hypothesis about the driving force
to imply a dominant, driving force. Identification behind population cycles in lepidoptera
of that driving force, however, has proved D. describe the fluctuating patterns of
surprisingly elusive despite considerable population cycles in lepidoptera
research. The common approach of studying E. question the idea that a single driving force is
causes of population cycles by measuring the behind population cycles in lepidoptera
mortality caused by different agents, such as
predatory birds or parasites, has been
unproductive in the case of lepidoptera.
Moreover, population ecologists' attempts to
alter cycles by changing the caterpillars' habitat
and by reducing caterpillar populations have not
succeeded. In short, the evidence implies that
these insect populations, if not self-regulating,
may at least be regulated by an agent more
intimately connected with the insect than are
predatory birds or parasites.
Passage 22
During the 1980s, many economic historians The primary purpose of the passage is to
studying Latin America focused on the impact of A. compare the impact of the Great Depression
the Great Depression of the 1930s. Most of these on Latin America with its impact on the United
historians argued that although the Depression States
began earlier in Latin America than in the United B. criticize a school of economic historians for
States, it was less severe in Latin America and did failing to analyze the Great Depression in
not significantly impede industrial growth there. Latin America within a global context
The historians' argument was grounded in C. illustrate the risks inherent in comparing
national government records concerning tax different types of economic enterprises to
revenues and exports and in government- explain economic phenomena
sponsored industrial censuses, from which D. call into question certain scholars' views
historians have drawn conclusions about total concerning the severity of the Great
manufacturing output and profit levels across Depression in Latin America
Latin America. However, economic statistics E. demonstrate that the Great Depression had a
published by Latin American governments in the more severe impact on industry in Latin
early twentieth century are neither reliable nor America than in certain other regions
consistent; this is especially true of
manufacturing data, which were gathered from
factory owners for taxation purposes and which
therefore may well be distorted. Moreover, one
cannot assume a direct correlation between the
output level and the profit level of a given
industry as these variables often move in
opposite directions. Finally, national and regional
economies are composed of individual firms and
industries, and relying on general, sweeping
economic indicators may mask substantial
variations among these different enterprises. For
example, recent analyses of previously
unexamined data on textile manufacturing in
Brazil and Mexico suggest that the Great
Depression had a more severe impact on this
Latin American industry than scholars had
recognized.
Passage 23
Among the myths taken as fact by the The primary purpose of the passage is to
environmental managers of most corporations is A. address a widespread environmental
the belief that environmental regulations affect management problem and suggest possible
all competitors in a given industry uniformly. In solutions
reality, regulatory costs—and therefore B. illustrate varying levels of compliance with
compliance—fall unevenly, economically environmental regulation among different
disadvantaging some companies and benefiting corporations
others. For example, a plant situated near a C. describe the various alternatives to
number of larger noncompliant competitors is traditional methods of environmental
less likely to attract the attention of local management
regulators than is an isolated plant, and less D. advocate increased corporate compliance
attention means lower costs. with environmental regulation
E. correct a common misconception about the
Additionally, large plants can spread compliance impact of environmental regulations
costs such as waste treatment across a larger
revenue base; on the other hand, some smaller
plants may not even be subject to certain
provisions such as permit or reporting
requirements by virtue of their size. Finally, older
production technologies often continue to
generate toxic wastes that were not regulated
when the technology was first adopted. New
regulations have imposed extensive compliance
costs on companies still using older industrial
coal-fired burners that generate high sulfur
dioxide and nitrogen oxide outputs, for example,
whereas new facilities generally avoid processes
that would create such waste products. By
realizing that they have discretion and that not all
industries are affected equally by environmental
regulation, environmental managers can help
their companies to achieve a competitive edge by
anticipating regulatory pressure and exploring all
possibilities for addressing how changing
regulations will affect their companies
specifically.
Passage 24
It is an odd but indisputable fact that the The primary purpose of the passage is to
seventeenth-century English women who are A. trace the historical roots of a modern
generally regarded as among the forerunners of sociopolitical movement
modern feminism are almost all identified with B. present one scholar's explanation for a
the Royalist side in the conflict between Royalists puzzling historical phenomenon
and Parliamentarians known as the English Civil C. contrast two interpretations of the ideological
Wars. Since Royalist ideology is often associated origins of a political conflict
with the radical patriarchalism of seventeenth- D. establish a link between the ideology of an
century political theorist Robert Filmer—a influential political theorist and that of a
patriarchalism that equates family and kingdom notoriously eccentric writer
and asserts the divinely ordained absolute power E. call attention to some points of agreement
of the king and, by analogy, of the male head of the between opposing sides in an ideological
household—historians have been debate
understandably puzzled by the fact that Royalist
women wrote the earliest extended criticisms of
the absolute subordination of women in marriage
and the earliest systematic assertions of women's
rational and moral equality with men. Some
historians have questioned the facile equation of
Royalist ideology with Filmerian patriarchalism;
and indeed, there may have been no consistent
differences between Royalists and
Parliamentarians on issues of family organization
and women's political rights, but in that case one
would expect early feminists to be equally divided
between the two sides.
Passage 25
There are recent reports of apparently drastic It is indisputably true that there is simply not
declines in amphibian populations and of enough long-term scientific data on amphibian
extinctions of a number of the world's populations to enable researchers to identify real
endangered amphibian species. These declines, if declines in amphibian populations. Many fairly
real, may be signs of a general trend toward common amphibian species declared all but
extinction, and many environmentalists have extinct after severe declines in the 1950s and
claimed that immediate environmental action is 1960s have subsequently recovered, and so might
necessary to remedy this “amphibian crisis,” the apparently declining populations that have
which, in their view, is an indicator of general and generated the current appearance of an
catastrophic environmental degradation due to amphibian crisis. Unfortunately, long-term data
human activity. will not soon be forthcoming, and postponing
environmental action while we wait for it may
To evaluate these claims, it is useful to make a doom species and whole ecosystems to
preliminary distinction that is far too often extinction.
ignored. A declining population should not be
confused with an endangered one. An endangered A. Tone of the passage
population is always rare, almost always small,
and, by definition, under constant threat of ____________________________________________
extinction even without a proximate cause in
human activities. Its disappearance, however
unfortunate, should come as no great surprise. B. Contradiction words: highlight the key
Moreover, chance events—which may indicate contradiction word(s).
nothing about the direction of trends in
population size—may lead to its extinction. The C. Highlight all Opinions
probability of extinction due to such random
factors depends on the population size and is • Does the author have a personal
independent of the prevailing direction of change opinion? YES / NO
in that size. • Is the author implicitly or explicitly
supporting anyone?
For biologists, population declines are potentially
more worrisome than extinctions. Persistent
declines, especially in large populations, indicate
a changed ecological context. Even here,
distinctions must again be made among declines
that are only apparent (in the sense that they are
part of habitual cycles or of normal fluctuations),
declines that take a population to some lower but
still acceptable level, and those that threaten
extinction (e.g., by taking the number of
individuals below the minimum viable
population). Anecdotal reports of population
decreases cannot distinguish among these
possibilities, and some amphibian populations
have shown strong fluctuations in the past.
There are recent reports of apparently drastic The primary purpose of the passage is to
declines in amphibian populations and of A. assess the validity of a certain view
extinctions of a number of the world's B. distinguish between two phenomena
endangered amphibian species. These declines, if C. identify the causes of a problem
real, may be signs of a general trend toward D. describe a disturbing trend
extinction, and many environmentalists have E. allay concern about a particular phenomenon
claimed that immediate environmental action is
necessary to remedy this “amphibian crisis,”
which, in their view, is an indicator of general and
catastrophic environmental degradation due to
human activity.
Passage 26
Human beings, born with a drive to explore and A. Tone of the passage
experiment, thrive on learning. Unfortunately,
corporations are oriented predominantly toward ____________________________________________
controlling employees, not fostering their
learning. Ironically, this orientation creates the
very conditions that predestine employees to B. Contradiction words: highlight the key
mediocre performances. Over time, superior contradiction word(s).
performance requires superior learning, because
long-term corporate survival depends on C. Highlight all Opinions
continually exploring new business and
organizational opportunities that can create new • Does the author have a personal
sources of growth. opinion? YES / NO
• Is the author implicitly or explicitly
To survive in the future, corporations must supporting anyone?
become “learning organizations,” enterprises that
are constantly able to adapt and expand their
capabilities. To accomplish this, corporations
must change how they view employees. The
traditional view that a single charismatic leader
should set the corporation's direction and make
key decisions is rooted in an individualistic
worldview. In an increasingly interdependent
world, such a view is no longer viable. In learning
organizations, thinking and acting are integrated
at all job levels. Corporate leadership is shared,
and leaders become designers, teachers, and
stewards, roles requiring new skills: the ability to
build shared vision, to reveal and challenge
prevailing mental models, and to foster broader,
more integrated patterns of thinking. In short,
leaders in learning organizations are responsible
for building organizations in which employees
are continually learning new skills and expanding
their capabilities to shape their future.
Human beings, born with a drive to explore and The primary purpose of the passage is to
experiment, thrive on learning. Unfortunately, A. endorse a traditional corporate structure
corporations are oriented predominantly toward B. introduce a new approach to corporate
controlling employees, not fostering their leadership and evaluate criticisms of it
learning. Ironically, this orientation creates the C. explain competing theories about
very conditions that predestine employees to management practices and reconcile them
mediocre performances. Over time, superior D. contrast two typical corporate organizational
performance requires superior learning, because structures
long-term corporate survival depends on E. propose an alternative to a common corporate
continually exploring new business and approach
organizational opportunities that can create new
sources of growth.
Passage 27
In 1971 researchers hoping to predict Evidence against the kind of regular earthquake
earthquakes in the short term by identifying cycles that Lindh and Baker tried to establish has
precursory phenomena (those that occur a few come from a relatively new field,
days before large quakes but not otherwise) paleoseismology. Paleoseismologists have
turned their attention to changes in seismic unearthed and dated geological features such as
waves that had been detected prior to fault scarps that were caused by earthquakes
earthquakes. An explanation for such changes thousands of years ago. They have determined
was offered by “dilatancy theory,” based on a that the average interval between ten
well-known phenomenon observed in rocks in earthquakes that took place at one site along the
the laboratory: as stress builds, microfractures in San Andreas Fault in the past two millennia was
rock close, decreasing the rock's volume. But as 132 years, but individual intervals ranged greatly,
stress continues to increase, the rock begins to from 44 to 332 years.
crack and expand in volume, allowing
groundwater to seep in, weakening the rock.
According to this theory, such effects could lead to A. Tone of the passage
several precursory phenomena in the field,
including a change in the velocity of seismic ____________________________________________
waves, and an increase in small, nearby tremors.
In 1971 researchers hoping to predict Evidence against the kind of regular earthquake
earthquakes in the short term by identifying cycles that Lindh and Baker tried to establish has
precursory phenomena (those that occur a few come from a relatively new field,
days before large quakes but not otherwise) paleoseismology. Paleoseismologists have
turned their attention to changes in seismic unearthed and dated geological features such as
waves that had been detected prior to fault scarps that were caused by earthquakes
earthquakes. An explanation for such changes thousands of years ago. They have determined
was offered by “dilatancy theory,” based on a that the average interval between ten
well-known phenomenon observed in rocks in earthquakes that took place at one site along the
the laboratory: as stress builds, microfractures in San Andreas Fault in the past two millennia was
rock close, decreasing the rock's volume. But as 132 years, but individual intervals ranged greatly,
stress continues to increase, the rock begins to from 44 to 332 years.
crack and expand in volume, allowing
groundwater to seep in, weakening the rock. The passage is primarily concerned with
According to this theory, such effects could lead to A. explaining why one method of earthquake
several precursory phenomena in the field, prediction has proven more practicable than
including a change in the velocity of seismic an alternative method
waves, and an increase in small, nearby tremors. B. suggesting that accurate earthquake
forecasting must combine elements of long-
Researchers initially reported success in term and short-term prediction
identifying these possible precursors, but C. challenging the usefulness of dilatancy theory
subsequent analyses of their data proved for explaining the occurrence of precursory
disheartening. Seismic waves with unusual phenomena
velocities were recorded before some D. discussing the deficiency of two methods by
earthquakes, but while the historical record which researchers have attempted to predict
confirms that most large earthquakes are the occurrence of earthquakes
preceded by minor tremors, these foreshocks E. describing the development of methods for
indicate nothing about the magnitude of an establishing patterns in the occurrence of past
impending quake and are indistinguishable from earthquakes
other minor tremors that occur without large
earthquakes.
Passage 28
A key decision required of advertising managers The primary purpose of the passage is to
is whether a “hard-sell” or “soft-sell” strategy is A. point out the risks involved in the use of a
appropriate for a specific target market. The particular advertising strategy
hard- sell approach involves the use of direct, B. make a case for the superiority of one
forceful claims regarding the benefits of the advertising strategy over another
advertised brand over competitors' offerings. In C. illustrate the ways in which two advertising
contrast, the soft-sell approach involves the use of strategies may be implemented
advertising claims that imply superiority more D. present the advantages and disadvantages of
subtly. two advertising strategies
E. contrast the types of target markets for which
One positive aspect of the hard-sell approach is its two advertising strategies are appropriate
use of very simple and straightforward product
claims presented as explicit conclusions, with
little room for confusion regarding the
advertiser's message. However, some consumers
may resent being told what to believe and some
may distrust the message. Resentment and
distrust often lead to counter-argumentation and
to boomerang effects where consumers come to
believe conclusions diametrically opposed to
conclusions endorsed in advertising claims. By
contrast, the risk of boomerang effects is greatly
reduced with soft-sell approaches. One way to
implement the soft-sell approach is to provide
information that implies the main conclusions the
advertiser wants the consumer to draw, but leave
the conclusions themselves unstated. Because
consumers are invited to make up their own
minds, implicit conclusions reduce the risk of
resentment, distrust, and counter-argumentation.
Passage 29
Most pre-1990 literature on businesses' use of The passage is primarily concerned with
information technology (IT)—defined as any A. describing a resource and indicating various
form of computer-based information system— methods used to study it
focused on spectacular IT successes and reflected B. presenting a theory and offering an opposing
a general optimism concerning IT's potential as a point of view
resource for creating competitive advantage. But C. providing an explanation for unexpected
toward the end of the 1980s, some economists findings
spoke of a “productivity paradox”: despite huge IT D. demonstrating why a particular theory is
investments, most notably in the service sectors, unfounded
productivity stagnated. In the retail industry, for E. resolving a disagreement regarding the uses
example, in which IT had been widely adopted of a technology
during the 1980s, productivity (average output
per hour) rose at an average annual rate of 1.1
percent between 1973 and 1989, compared with
2.4 percent in the preceding 25-year period.
Proponents of IT argued that it takes both time
and a critical mass of investment for IT to yield
benefits, and some suggested that growth figures
for the 1990s proved these benefits were finally
being realized. They also argued that measures of
productivity ignore what would have happened
without investments in IT—productivity gains
might have been even lower. There were even
claims that IT had improved the performance of
the service sector significantly, although
macroeconomic measures of productivity did not
reflect the improvement.
Passage 30
The dry mountain ranges of the western United A. Tone of the passage
States contain rocks dating back 440 to 510
million years, to the Ordovician period, and ____________________________________________
teeming with evidence of tropical marine life. This
rock record provides clues about one of the most
significant radiations (periods when existing life- B. Contradiction words: highlight the key
forms gave rise to variations that would contradiction word(s).
eventually evolve into entirely new species) in the
history of marine invertebrates. During this C. Highlight all Opinions
radiation the number of marine biological
families increased greatly, and these families • Does the author have a personal
included species that would dominate the marine opinion? YES / NO
ecosystems of the area for the next 215 million • Is the author implicitly or explicitly
years. Although the radiation spanned tens of supporting anyone?
millions of years, major changes in many species
occurred during a geologically short time span
within the radiation and, furthermore, appear to
have occurred worldwide, suggesting that
external events were major factors in the
radiation. In fact, there is evidence of major
ecological and geological changes during this
period: the sea level dropped drastically and
mountain ranges were formed. In this instance,
rather than leading to large-scale extinctions,
these kinds of environmental changes may have
resulted in an enriched pattern of habitats and
nutrients, which in turn gave rise to the
Ordovician radiation. However, the actual
relationship between these environmental
factors and the diversification of life-forms is not
yet fully understood.
The dry mountain ranges of the western United The passage is primarily concerned with
States contain rocks dating back 440 to 510 A. evaluating the evidence of a major geologic
million years, to the Ordovician period, and period and determining its duration
teeming with evidence of tropical marine life. This B. describing an evolutionary phenomenon and
rock record provides clues about one of the most speculating about its cause
significant radiations (periods when existing life- C. explaining the mechanisms through which
forms gave rise to variations that would marine life-forms evolved during a particular
eventually evolve into entirely new species) in the period
history of marine invertebrates. During this D. analyzing the impact on later life-forms of an
radiation the number of marine biological important evolutionary development
families increased greatly, and these families E. contrasting a period of evolutionary change
included species that would dominate the marine with other such periods
ecosystems of the area for the next 215 million
years. Although the radiation spanned tens of
millions of years, major changes in many species
occurred during a geologically short time span
within the radiation and, furthermore, appear to
have occurred worldwide, suggesting that
external events were major factors in the
radiation. In fact, there is evidence of major
ecological and geological changes during this
period: the sea level dropped drastically and
mountain ranges were formed. In this instance,
rather than leading to large-scale extinctions,
these kinds of environmental changes may have
resulted in an enriched pattern of habitats and
nutrients, which in turn gave rise to the
Ordovician radiation. However, the actual
relationship between these environmental
factors and the diversification of life-forms is not
yet fully understood.
Passage 31
Passage 32
For most species of animals, the number of Titi monkeys routinely live within territories of 6
individuals in the species is inversely to 8 hectares, and night monkeys seldom defend
proportional to the average body size for more than 10 hectares, but tamarin groups
members of the species: the smaller the body size, routinely occupy areas of 30 to 120 hectares.
the larger the number of individual animals. The Contrast this with the 1 to 2 hectares needed by
tamarin, a small South American monkey, breaks the common North American gray squirrel, a
this rule. Of the ten primate species studied in nonterritorial mammal of about the same size. A
Peru's Manu National Park, for example, the two group of tamarins uses about as much space as a
species of tamarins, saddle-backed and emperor, troop of brown capuchins, though the latter
are the eighth and ninth least abundant, weighs 15 times as much. Thus, in addition to
respectively. Only the pygmy marmoset, which is being rare, tamarins require an amount of space
even smaller, is less abundant. The tamarin's that seems completely out of proportion to their
scarcity is not easily explained; it cannot be size.
dismissed as a consequence of diet, because
tamarins feed on the same mixture of fruit, nectar, A. Tone of the passage
and small prey as do several of their more
numerous larger counterparts, including the two ____________________________________________
capuchins known as the squirrel monkey and the
night monkey. Although the relative proportions
of fruits consumed varies somewhat among B. Contradiction words: highlight the key
species, it is hard to imagine that such subtle contradiction word(s).
differences are crucial to understanding the
relative rarity of tamarins. C. Highlight all Opinions
To emphasize just how anomalously rare • Does the author have a personal
tamarins are, we can compare them to the other opinion? YES / NO
omnivorous primates in the community. In terms • Is the author implicitly or explicitly
of numbers of individuals per square kilometer, supporting anyone?
they rank well below the two capuchins, the
squirrel monkey and the night monkey. And in
terms of biomass, or the total weight of the
individuals that occupy a unit area of habitat, each
tamarin species is present at only one-twentieth
the mass of brown capuchins or one-tenth that of
squirrel monkeys. To gain another perspective,
consider the spatial requirements of tamarins.
Tamarins are rigidly territorial, vigorously
expelling any intruders that may stray within the
sharply defined boundaries of their domains.
Groups invest an appreciable part of their time
and energy in patrolling their territorial
boundaries, announcing their presence to their
neighbors with shrill, sweeping cries. Such
concerted territoriality is rather exceptional
among primates, though the gibbons and
siamangs of Asia show it, as do a few other New
World species such as the titi and night monkeys.
What is most surprising about tamarin territories
is their size.
For most species of animals, the number of Titi monkeys routinely live within territories of 6
individuals in the species is inversely to 8 hectares, and night monkeys seldom defend
proportional to the average body size for more than 10 hectares, but tamarin groups
members of the species: the smaller the body size, routinely occupy areas of 30 to 120 hectares.
the larger the number of individual animals. The Contrast this with the 1 to 2 hectares needed by
tamarin, a small South American monkey, breaks the common North American gray squirrel, a
this rule. Of the ten primate species studied in nonterritorial mammal of about the same size. A
Peru's Manu National Park, for example, the two group of tamarins uses about as much space as a
species of tamarins, saddle-backed and emperor, troop of brown capuchins, though the latter
are the eighth and ninth least abundant, weighs 15 times as much. Thus, in addition to
respectively. Only the pygmy marmoset, which is being rare, tamarins require an amount of space
even smaller, is less abundant. The tamarin's that seems completely out of proportion to their
scarcity is not easily explained; it cannot be size.
dismissed as a consequence of diet, because
tamarins feed on the same mixture of fruit, nectar, The primary concern of the passage is to
and small prey as do several of their more A. recommend a policy
numerous larger counterparts, including the two B. evaluate a theory
capuchins known as the squirrel monkey and the C. describe an unusual condition
night monkey. Although the relative proportions D. explain the development of a hypothesis
of fruits consumed varies somewhat among E. support one of several competing hypotheses
species, it is hard to imagine that such subtle
differences are crucial to understanding the
relative rarity of tamarins.
Passage 33
In addition to conventional galaxies, the universe The primary purpose of the passage is to
contains very dim galaxies that until recently A. describe a phenomenon and consider its
went unnoticed by astronomers. Possibly as scientific significance
numerous as conventional galaxies, these galaxies B. contrast two phenomena and discuss a
have the same general shape and even the same puzzling difference between them
approximate number of stars as a common type C. identify a newly discovered phenomenon and
of conventional galaxy, the spiral, but tend to be explain its origins
much larger. Because these galaxies' mass is D. compare two classes of objects and discuss the
spread out over larger areas, they have far fewer physical properties of each
stars per unit volume than do conventional E. discuss a discovery and point out its
galaxies. Apparently these low-surface- inconsistency with existing theory
brightness galaxies, as they are called, take much
longer than conventional galaxies to condense
their primordial gas and convert it to stars—that
is, they evolve much more slowly.
Passage 34
The fact that superior service can generate a A. Tone of the passage
competitive advantage for a company does not
mean that every attempt at improving service will ____________________________________________
create such an advantage. Investments in service,
like those in production and distribution, must be
balanced against other types of investments on B. Contradiction words: highlight the key
the basis of direct, tangible benefits such as cost contradiction word(s).
reduction and increased revenues. If a company is
already effectively on a par with its competitors C. Highlight all Opinions
because it provides service that avoids a
damaging reputation and keeps customers from • Does the author have a personal
leaving at an unacceptable rate, then investment opinion? YES / NO
in higher service levels may be wasted, since • Is the author implicitly or explicitly
service is a deciding factor for customers only in supporting anyone?
extreme situations.
The fact that superior service can generate a The primary purpose of the passage is to
competitive advantage for a company does not A. contrast possible outcomes of a type of
mean that every attempt at improving service will business investment
create such an advantage. Investments in service, B. suggest more careful evaluation of a type of
like those in production and distribution, must be business investment
balanced against other types of investments on C. illustrate various ways in which a type of
the basis of direct, tangible benefits such as cost business investment could fail to enhance
reduction and increased revenues. If a company is revenues
already effectively on a par with its competitors D. trace the general problems of a company to a
because it provides service that avoids a certain type of business investment
damaging reputation and keeps customers from E. criticize the way in which managers tend to
leaving at an unacceptable rate, then investment analyze the costs and benefits of business
in higher service levels may be wasted, since investments
service is a deciding factor for customers only in
extreme situations.
Passage 35
Antonia Castañeda has utilized scholarship from The primary purpose of the passage is to
women's studies and Mexican-American history A. trace historical influences on the depiction of
to examine nineteenth-century literary Mexican Americans in the nineteenth century
portrayals of Mexican women. As Castañeda B. explain how research in history has been
notes, scholars of women's history observe that in affected by scholarship in women's studies
the United States, male novelists of the period— C. describe the historical origins of a literary
during which, according to these scholars, stereotype
women's traditional economic role in home- D. discuss ways in which minority writers have
based agriculture was threatened by the sought to critique a dominant culture through
transition to a factory-based industrial their writing
economy—define women solely in their domestic E. evaluate both sides in a scholarly debate
roles of wife and mother. Castañeda finds that about a prominent literary stereotype
during the same period that saw non-Hispanic
women being economically displaced by
industrialization, Hispanic law in territorial
California protected the economic position of
“Californianas” (the Mexican women of the
territory) by ensuring them property rights and
inheritance rights equal to those of males.
Passage 36
Micro-wear patterns found on the teeth of long- The passage is primarily concerned with
extinct specimens of the primate species A. comparing two research methods for
australopithecine may provide evidence about determining a species' dietary habits
their diets. For example, on the basis of tooth B. describing and evaluating conjectures about a
micro-wear patterns, Walker dismisses Jolly's species' diet
hypothesis that australopithecines ate hard C. contrasting several explanations for a species'
seeds. He also disputes Szalay's suggestion that dietary habits
the heavy enamel of australopithecine teeth is an D. discussing a new approach and advocating its
adaptation to bone crunching, since both seed use in particular situations
cracking and bone crunching produce distinctive E. arguing that a particular research
micro-wear characteristics on teeth. His methodology does not contribute useful data
conclusion that australopithecines were
frugivores (fruit eaters) is based upon his
observation that the tooth micro-wear
characteristics of east African australopithecine
specimens are indistinguishable from those of
chimpanzees and orangutans, which are
commonly assumed to be frugivorous primates.
Passage 37
Is there a massive black hole at the center of our A. Tone of the passage
galaxy, the Milky Way? The evidence is
inconclusive. Just as the Sun's mass can be ____________________________________________
determined, given knowledge of other variables,
by the velocity at which its planets orbit, the mass
at the center of the Milky Way can be revealed by B. Contradiction words: highlight the key
the velocities of stars and gas orbiting the galactic contradiction word(s).
center. This dynamical evidence, based on
recently confirmed assumptions about the stars' C. Highlight all Opinions
velocities, argues for an extremely compact object
with a mass two to three million times the mass • Does the author have a personal
of our Sun. Although according to current theory opinion? YES / NO
this makes the mass at the center of the galaxy too • Is the author implicitly or explicitly
dense to be anything but a black hole, the relative supporting anyone?
lack of energy radiating from the galactic center
presents a serious problem. A black hole's
gravity attracts surrounding matter, which swirls
around the black hole, emitting some energy as it
is engulfed. Scientists believe that the amount of
energy that escapes the black hole should be
about 10 percent of the matter's rest energy (the
energy equivalent of its mass according to the
equation E=mc2). But when the energy coming
from the galactic center is compared to widely
held predictions based on how much matter
should be falling into a theoretical central black
hole, there is a discrepancy by a factor of a few
thousand.
Is there a massive black hole at the center of our The primary purpose of the passage is to
galaxy, the Milky Way? The evidence is A. present several theories that could account for a
inconclusive. Just as the Sun's mass can be particular phenomenon
determined, given knowledge of other variables, B. argue that a certain question needs to be
by the velocity at which its planets orbit, the mass reframed in light of new evidence
at the center of the Milky Way can be revealed by C. resolve an apparent inconsistency between two
the velocities of stars and gas orbiting the galactic lines of evidence
center. This dynamical evidence, based on D. explain why a certain issue remains unresolved
recently confirmed assumptions about the stars' E. present evidence that calls into question certain
velocities, argues for an extremely compact object assumptions of a current theory
with a mass two to three million times the mass
of our Sun. Although according to current theory
this makes the mass at the center of the galaxy too
dense to be anything but a black hole, the relative
lack of energy radiating from the galactic center
presents a serious problem. A black hole's
gravity attracts surrounding matter, which swirls
around the black hole, emitting some energy as it
is engulfed. Scientists believe that the amount of
energy that escapes the black hole should be
about 10 percent of the matter's rest energy (the
energy equivalent of its mass according to the
equation E=mc2). But when the energy coming
from the galactic center is compared to widely
held predictions based on how much matter
should be falling into a theoretical central black
hole, there is a discrepancy by a factor of a few
thousand.
Passage 38
Critics maintain that the fiction of Herman The primary purpose of the passage is to
Melville (1819–1891) has limitations, such as its A. make a case for the importance of skillful
lack of inventive plots after Moby-Dick (1851) and psychological motivation in well-written novels
its occasionally inscrutable style. A more serious, and romances
yet problematic, charge is that Melville is a B. contrast the romantic and novelistic traditions
deficient writer because he is not a practitioner of and assert the aesthetic superiority of the
the “art of fiction,” as critics have conceived of this romantic tradition
art since the late nineteenth-century essays and C. survey some of the responses to Melville's fiction
novels of Henry James. Indeed, most twentieth- put forward by James and twentieth-century
century commentators regard Melville not as a literary critics
novelist but as a writer of romance, since they D. argue that the charges made against Melville's
believe that Melville's fiction lacks the continuity fiction by literary critics are suspect and
that James viewed as essential to a novel: the misleading
continuity between what characters feel or think E. note several accusations made against Melville's
and what they do, and the continuity between fiction by literary critics and refute one of these
characters' fates and their pasts or original social accusations
classes. Critics argue that only Pierre (1852),
because of its subject and its characters, is close
to being a novel in the Jamesian sense.
Passage 39
There are two theories that have been used to A. Tone of the passage
explain ancient and modern tragedy. Neither
quite explains the complexity of the tragic process ____________________________________________
or the tragic hero, but each explains important
elements of tragedy, and, because their
conclusions are contradictory, they represent B. Contradiction words: highlight the key
extreme views. The first theory states that all contradiction word(s).
tragedy exhibits the workings of external fate. Of
course, the overwhelming majority of tragedies C. Highlight all Opinions
do leave us with a sense of the supremacy of
impersonal power and of the limitation of human • Does the author have a personal
effort. But this theory of tragedy is an opinion? YES / NO
oversimplification, primarily because it confuses • Is the author implicitly or explicitly
the tragic condition with the tragic process: the supporting anyone?
theory does not acknowledge that fate, in a
tragedy, normally becomes external to the hero
only after the tragic process has been set in
motion. Fate, as conceived in ancient Greek
tragedy, is the internal balancing condition of life.
It appears as external only after it has been
violated, just as justice is an internal quality of an
honest person, but the external antagonist of the
criminal. Secondarily, this theory of tragedy does
not distinguish tragedy from irony. Irony does not
need an exceptional central figure: as a rule, the
more ignoble the hero the sharper the irony,
when irony alone is the objective. It is heroism
that creates the splendor and exhilaration that is
unique to tragedy. The tragic hero normally has
an extraordinary, often a nearly divine, destiny
almost within grasp, and the glory of that original
destiny never quite fades out of the tragedy.
There are two theories that have been used to The primary purpose of the passage is to
explain ancient and modern tragedy. Neither A. compare and criticize two theories of tragedy
quite explains the complexity of the tragic process B. develop a new theory of tragedy
or the tragic hero, but each explains important C. summarize the thematic content of tragedy
elements of tragedy, and, because their D. reject one theory of tragedy and offer another
conclusions are contradictory, they represent theory in its place
extreme views. The first theory states that all E. distinguish between tragedy and irony
tragedy exhibits the workings of external fate. Of
course, the overwhelming majority of tragedies
do leave us with a sense of the supremacy of
impersonal power and of the limitation of human
effort. But this theory of tragedy is an
oversimplification, primarily because it confuses
the tragic condition with the tragic process: the
theory does not acknowledge that fate, in a
tragedy, normally becomes external to the hero
only after the tragic process has been set in
motion. Fate, as conceived in ancient Greek
tragedy, is the internal balancing condition of life.
It appears as external only after it has been
violated, just as justice is an internal quality of an
honest person, but the external antagonist of the
criminal. Secondarily, this theory of tragedy does
not distinguish tragedy from irony. Irony does not
need an exceptional central figure: as a rule, the
more ignoble the hero the sharper the irony,
when irony alone is the objective. It is heroism
that creates the splendor and exhilaration that is
unique to tragedy. The tragic hero normally has
an extraordinary, often a nearly divine, destiny
almost within grasp, and the glory of that original
destiny never quite fades out of the tragedy.
Passage 40
Conventional wisdom has it that large deficits in Which of the following best summarizes the
the United States budget cause interest rates to central idea of the passage?
rise. Two main arguments are given for this A. A decrease in nongovernment borrowing or
claim. According to the first, as the deficit an increase in the availability of credit can
increases, the government will borrow more to eliminate or lessen the ill effects of increased
make up for the ensuing shortage of funds. borrowing by the government.
Consequently, it is argued, if both the total supply B. Educating financiers about the true
of credit (money available for borrowing) and the relationship between large federal deficits
amount of credit sought by nongovernment and high interest rates will make financiers
borrowers remain relatively stable, as is often less prone to raise interest rates in response
supposed, then the price of credit (the interest to deficits.
rate) will increase. That this is so is suggested by C. There is little support for the widely held
the basic economic principle that if supplies of a belief that large federal deficits will create
commodity (here, credit) remain fixed and higher interest rates, as the main arguments
demand for that commodity increases, its price given to defend this claim are flawed.
will also increase. The second argument D. When the government borrows money,
supposes that the government will tend to finance demand for credit increases, typically creating
its deficits by increasing the money supply with higher interest rates unless special conditions
insufficient regard for whether there is enough such as decreased consumer spending arise.
room for economic growth to enable such an E. Given that most financiers believe in a cause-
increase to occur without causing inflation. It is and-effect relationship between large deficits
then argued that financiers will expect the deficit and high interest rates, it should be expected
to cause inflation and will raise interest rates, that financiers will raise interest rates.
anticipating that because of inflation the money
they lend will be worth less when paid back. The author uses the term "admittedly" (see
highlighted text) in order to indicate that
Unfortunately for the first argument, it is A. the second argument has some truth to it,
unreasonable to assume that nongovernment though not for the reasons usually supposed
borrowing and the supply of credit will remain B. the author has not been successful in
relatively stable. Nongovernment borrowing attempting to point out inadequacies in the
sometimes decreases. When it does, increased two arguments
government borrowing will not necessarily push C. the thesis that large deficits directly cause
up the total demand for credit. Alternatively, interest rates to rise has strong support after
when credit availability increases, for example all
through greater foreign lending to the United D. financiers should admit that they were wrong
States, then interest rates need not rise, even if in thinking that large deficits will cause higher
both private and government borrowing increase. inflation rates
E. financiers generally do not think that the
The second argument is also problematic. author's criticisms of the second argument are
Financing the deficit by increasing the money worthy of consideration
supply should cause inflation only when there is
not enough room for economic growth.
Currently, there is no reason to expect deficits to
cause inflation. However, since many financiers
believe that deficits ordinarily create inflation,
then admittedly they will be inclined to raise
interest rates to offset mistakenly anticipated
inflation. This effect, however, is due to
ignorance, not to the deficit itself, and could be
lessened by educating financiers on this issue.
Answer-keys: Passage 22
Passage 1 D
AE Passage 23
Passage 2 E
BC Passage 24
Passage 3 B
D Passage 25
Passage 4 A
C Passage 26
Passage 5 E
BBD Passage 27
Passage 6 D
BE Passage 28
Passage 7 D
B Passage 29
Passage 8 C
B Passage 30
Passage 9 B
D Passage 31
Passage 10 C
A Passage 32
Passage 11 C
A Passage 33
Passage 12 A
C Passage 34
Passage 13 B
B Passage 35
Passage 14 C
EBA Passage 36
Passage 15 B
A Passage 37
Passage 16 D
DCA Passage 38
Passage 17 E
B Passage 39
Passage 18 A
B Passage 40
Passage 19 CA
C Detailed OCTAAVE based video solutions (to
each of these passages) are shared as part of
Passage 20
Top-One-Percent GMAT courses (for
C registered courses only).
Passage 21
C
RC Inference Questions
Note: Of the 36 questions in the Verbal Section, about 12 questions (both RC and
CR combined) are based on Inference – about one-third of the Verbal Section.
Inference questions are always challenging for most students because of extremely
close option choices.
Approach:
1. Inferences are absolutely (100%) correct and can be safely and logically deduced / derived from
the passage without any ambiguity.
2. Inference questions play out like “treasure hunt”; the answer lies right within the passage but is
masked / hidden / difficult to spot. You have to uncover inferences with some effort.
3. There is a difference between “inference” and “assumption”. Either inferences are given in the
passage (indirectly) or they are deducible / derivable from the information given. Essentially, any
inference is a logical conclusion that can be derived from the information presented in the
passage. So, inferences are definitely present in the passage but may be difficult to find.
Assumptions, on the other hand, aren’t present in the passage. They are unstated. They are
required by the argument for the conclusion of the argument to be valid.
4. The best approach to solve any inference question is OCTAAVE, especially the AAVE part of
OCTAAVE.
5. Eliminate wrong answers. NEVER, EVER, try to justify why the right answer is right. OCTAAVE
will help you identify all the wrong answers using Awareness, Avoid, Verify, and Eliminate, BUT
please don’t be fixated on understanding WHY the right answer is right. Your entire focus should
be on why the wrong answers are wrong.
6. No matter what happens, don’t mark the answer without verifying. PUT YOUR FINGER ON IT.
7. Use the Avoid part of OCTAVE extensively. You can easily eliminate many wrong answers.
Passage 1
Since the early 1970’s, historians have begun to While Keyssar could have spent more time
devote serious attention to the working class in developing the implications of his findings on
the United States. Yet while we now have joblessness for contemporary public policy, his
extensive studies of working-class communities study, in its thorough research and creative use
and culture, we know remarkably little of of quantitative and qualitative evidence, is a
worklessness. When historians have paid any model of historical analysis.
attention at all to unemployment, they have
focused on the Great Depression of the 1930’s. 1. The passage suggests that before the
The narrowness of this perspective ignores the early 1970’s, which of the following was
pervasive recessions and joblessness of the true of the study by historians of
previous decades, as Alexander Keyssar shows unemployment in the United States?
in his recent book. Examining the period 1870- A. The study was infrequent or superficial,
1920, Keyssar concentrates on Massachusetts, or both.
where the historical materials are particularly B. The study was repeatedly criticized by
rich, and the findings applicable to other many people for its allegedly narrow
industrial areas. focus.
C. The study relied more on qualitative
The unemployment rates that Keyssar than quantitative evidence.
calculates appear to be relatively modest, at D. The study focused more on the working-
least by Great Depression standards: during the class community than on working-class
worst years, in the 1870’s and 1890’s, culture.
unemployment was around 15 percent. Yet E. The study ignored working-class
Keyssar rightly understands that a better way to joblessness during the Great Depression.
measure the impact of unemployment is to
calculate unemployment frequencies— 2. Which of the following statements about
measuring the percentage of workers who the unemployment rate during the Great
experience any unemployment in the course of Depression can be inferred from the
a year. Given this perspective, joblessness looms passage?
much larger. A. It was sometimes higher than 15 percent.
B. It has been analyzed seriously only since
Keyssar also scrutinizes unemployment the early 1970’s.
patterns according to skill level, ethnicity, race, C. It can be calculated more easily than can
age, class, and gender. He finds that rates of unemployment frequency.
joblessness differed according to class: those in D. It was never as high as the rate during
middle-class and white-collar occupations were the 1870’s.
far less likely to be unemployed. Yet the impact E. It has been shown by Keyssar to be lower
of unemployment on a specific class was not than previously thought.
always the same. Even when dependent on the
same trade, adjoining communities and places
could have dramatically different
unemployment rates. Keyssar uses these
differential rates to help explain a phenomenon
that has puzzled historians—the startlingly high
rate of geographical mobility in the nineteenth-
century United States.
THE MOST AUTHORITATIVE GUIDE EVER WRITTEN ON GMAT READING COMPREHENSION
Since the early 1970’s, historians have begun to While Keyssar could have spent more time
devote serious attention to the working class in developing the implications of his findings on
the United States. Yet while we now have joblessness for contemporary public policy, his
extensive studies of working-class communities study, in its thorough research and creative use
and culture, we know remarkably little of of quantitative and qualitative evidence, is a
worklessness. When historians have paid any model of historical analysis.
attention at all to unemployment, they have
focused on the Great Depression of the 1930’s. 3. According to the passage, which of the
The narrowness of this perspective ignores the following is true of Keyssar’s findings
pervasive recessions and joblessness of the concerning unemployment in
previous decades, as Alexander Keyssar shows Massachusetts?
in his recent book. Examining the period 1870- A. They tend to contradict earlier findings
1920, Keyssar concentrates on Massachusetts, about such unemployment.
where the historical materials are particularly B. They are possible because Massachusetts
rich, and the findings applicable to other has the most easily accessible historical
industrial areas. records.
C. They are the first to mention the existence of
The unemployment rates that Keyssar high rates of geographical mobility in the
calculates appear to be relatively modest, at nineteenth century.
least by Great Depression standards: during the D. They are relevant to a historical
worst years, in the 1870’s and 1890’s, understanding of the nature of
unemployment was around 15 percent. Yet unemployment in other states.
Keyssar rightly understands that a better way to E. They have caused historians to reconsider
measure the impact of unemployment is to the role of the working class during the
calculate unemployment frequencies— Great Depression.
measuring the percentage of workers who
experience any unemployment in the course of 4. According to the passage, which of the
a year. Given this perspective, joblessness looms following is true of the unemployment
much larger. rates mentioned in the passage?
A. They hovered, on average, around 15
Keyssar also scrutinizes unemployment percent during the period 1870-1920.
patterns according to skill level, ethnicity, race, B. They give less than a full sense of the impact
age, class, and gender. He finds that rates of of unemployment on working-class people.
joblessness differed according to class: those in C. They overestimate the importance of middle
middle-class and white-collar occupations were class and white-collar unemployment.
far less likely to be unemployed. Yet the impact D. They have been considered by many
of unemployment on a specific class was not historians to underestimate the extent of
always the same. Even when dependent on the working-class unemployment.
same trade, adjoining communities and places E. They are more open to question when
could have dramatically different calculated for years other than those of peak
unemployment rates. Keyssar uses these recession.
differential rates to help explain a phenomenon
that has puzzled historians—the startlingly high
rate of geographical mobility in the nineteenth-
century United States.
THE MOST AUTHORITATIVE GUIDE EVER WRITTEN ON GMAT READING COMPREHENSION
Since the early 1970’s, historians have begun to While Keyssar could have spent more time
devote serious attention to the working class in developing the implications of his findings on
the United States. Yet while we now have joblessness for contemporary public policy, his
extensive studies of working-class communities study, in its thorough research and creative use
and culture, we know remarkably little of of quantitative and qualitative evidence, is a
worklessness. When historians have paid any model of historical analysis.
attention at all to unemployment, they have
focused on the Great Depression of the 1930’s. 5. According to the passage, Keyssar
The narrowness of this perspective ignores the considers which of the following to be
pervasive recessions and joblessness of the among the important predictors of the
previous decades, as Alexander Keyssar shows likelihood that a particular person would
in his recent book. Examining the period 1870- be unemployed in late nineteenth-
1920, Keyssar concentrates on Massachusetts, century Massachusetts?
where the historical materials are particularly I. The person’s class
rich, and the findings applicable to other II. Where the person lived or worked
industrial areas. III. The person’s age
Passage 2
Kazuko Nakane’s history of the early Japanese 7. It can be inferred that the passage is trying
immigrants to central California’s Pajaro Valley to:
focuses on the development of farming communities A. defend a controversial hypothesis
there from 1890 to 1940. The Issei (first-generation presented in a history of early Japanese
immigrants) were brought into the Pajaro Valley to immigrants to California
raise sugar beets. Like Issei laborers in American B. dismiss a history of an early Japanese
cities, Japanese men in rural areas sought settlement in California as narrow and ill
employment via the “boss” system. The system constructed
comprised three elements: immigrant wage C. summarize and critique a history of an early
laborers; Issei boardinghouses where laborers Japanese settlement in California
stayed; and labor contractors, who gathered D. compare a history of one Japanese American
workers for a particular job and then negotiated a community with studies of Japanese
contract between workers and employer. This same settlements throughout California
system was originally utilized by the Chinese E. examine the differences between Japanese
laborers who had preceded the Japanese. A related and Chinese immigrants to central
institution was the “labor club,” which provided job California in the 1890’s
information and negotiated employment contracts
and other legal matters, such as the rental of land, for 8. The author of the passage would most likely
Issei who chose to belong and paid an annual fee to agree that which of the following, if it had
the cooperative for membership. been included in Nakane's study, would best
remedy the particularistic nature of that
When the local sugar beet industry collapsed in study?
1902, the Issei began to lease land from the valley’s A. A statistical table comparing per capita
strawberry farmers. The Japanese provided the income of Issei wage laborers and
labor and the crop was divided between laborers sharecroppers in the Pajaro Valley
and landowners. The Issei thus moved quickly from B. A statistical table showing per capita income
wage-labor employment to sharecropping of Issei in the Pajaro Valley from 1890 to
agreements. A limited amount of economic progress 1940
was made as some Issei were able to rent or buy C. A statistical table showing rates of farm
farmland directly, while others joined together to ownership by Japanese Americans in four
form farming corporations. As the Issei began to central California counties from 1890 to
operate farms, they began to marry and start 1940
families, forming an established Japanese American D. A discussion of original company documents
community. Unfortunately, the Issei’s efforts to dealing with the Pajaro Valley sugar beet
attain agricultural independence were hampered by industry at the turn of the century
government restrictions, such as the Alien Land Law E. Transcripts of interviews conducted with
of 1913. But immigrants could circumvent such members of the Pajaro Valley Japanese
exclusionary laws by leasing or purchasing land in American community who were born in the
their American-born children’s names. 1920's and 1930's.
Nakane’s case study of one rural Japanese American 9. It can be inferred from the passage that,
community provides valuable information about the when the Issei began to lease land from the
lives and experiences of the Issei. It is, however, too Valley's strawberry farmers, the Issei most
particularistic. Nakane’s methodology cannot probably did which of the following?
substitute for a broader theoretical or comparative A. They used profits made from selling the
perspective. Future research might well consider strawberry crop to hire other Issei.
two issues raised by her study: were the Issei of the B. They negotiated such agricultural contracts
Pajaro Valley similar to or different from Issei in using the "boss" system.
urban settings, and what variations existed between C. They paid for the use of the land with a share
rural Japanese American communities? of the strawberry crop.
D. They earned higher wages than when they
raised sugar beets.
E. They violated the Alien Land Law.
THE MOST AUTHORITATIVE GUIDE EVER WRITTEN ON GMAT READING COMPREHENSION
Kazuko Nakane’s history of the early Japanese 10. Which of the following best describes a
immigrants to central California’s Pajaro Valley “labor club,” as defined in the passage?
focuses on the development of farming communities A. An organization to which Issei were compelled
there from 1890 to 1940. The Issei (first-generation to belong if they sought employment in the
immigrants) were brought into the Pajaro Valley to Pajaro Valley
raise sugar beets. Like Issei laborers in American B. An association whose members included labor
cities, Japanese men in rural areas sought contractors and landowning “bosses”
employment via the “boss” system. The system C. A type of farming corporation set up by Issei who
comprised three elements: immigrant wage had resided in the Pajaro Valley for some time
laborers; Issei boardinghouses where laborers D. A cooperative association whose members were
stayed; and labor contractors, who gathered dues-paying Japanese laborers
workers for a particular job and then negotiated a E. A social organization to which Japanese laborers
contract between workers and employer. This same and their families belonged
system was originally utilized by the Chinese
laborers who had preceded the Japanese. A related 11. Based on information in the passage, which
institution was the “labor club,” which provided job of the following statements concerning the
information and negotiated employment contracts Alien Land Law of 1913 is most accurate?
and other legal matters, such as the rental of land, for A. It excluded American-born citizens of Japanese
Issei who chose to belong and paid an annual fee to ancestry from landownership.
the cooperative for membership. B. It sought to restrict the number of foreign
immigrants to California.
When the local sugar beet industry collapsed in C. It successfully prevented Issei from ever
1902, the Issei began to lease land from the valley’s purchasing farmland.
strawberry farmers. The Japanese provided the D. It was applicable to first-generation immigrants
labor and the crop was divided between laborers but not to their American-born children.
and landowners. The Issei thus moved quickly from E. It was passed under pressure from the Pajaro
wage-labor employment to sharecropping Valley’s strawberry farmers.
agreements. A limited amount of economic progress
was made as some Issei were able to rent or buy
farmland directly, while others joined together to
form farming corporations. As the Issei began to
operate farms, they began to marry and start
families, forming an established Japanese American
community. Unfortunately, the Issei’s efforts to
attain agricultural independence were hampered by
government restrictions, such as the Alien Land Law
of 1913. But immigrants could circumvent such
exclusionary laws by leasing or purchasing land in
their American-born children’s names.
Kazuko Nakane’s history of the early Japanese 12. Several Issei families join together to
immigrants to central California’s Pajaro Valley purchase a strawberry field and the
focuses on the development of farming communities necessary farming equipment. Such a
there from 1890 to 1940. The Issei (first-generation situation best exemplifies which of the
immigrants) were brought into the Pajaro Valley to following, as it is described in the passage?
raise sugar beets. Like Issei laborers in American A. A typical sharecropping agreement
cities, Japanese men in rural areas sought B. A farming corporation
employment via the “boss” system. The system C. A “labor club”
comprised three elements: immigrant wage D. The “boss” system
laborers; Issei boardinghouses where laborers E. Circumvention of the Alien Land Law
stayed; and labor contractors, who gathered
workers for a particular job and then negotiated a 13. The passage suggests that which of the
contract between workers and employer. This same following was an indirect consequence of the
system was originally utilized by the Chinese collapse of the sugar beet industry in the
laborers who had preceded the Japanese. A related Pajaro Valley?
institution was the “labor club,” which provided job A. The Issei formed a permanent, family-based
information and negotiated employment contracts community.
and other legal matters, such as the rental of land, for B. Boardinghouses were built to accommodate the
Issei who chose to belong and paid an annual fee to Issei.
the cooperative for membership. C. The Issei couldn’t lease land in their children’s
names.
When the local sugar beet industry collapsed in D. The Issei adopted a labor contract system
1902, the Issei began to lease land from the valley’s similar to that used by Chinese immigrants.
strawberry farmers. The Japanese provided the E. The Issei suffered a massive dislocation caused
labor and the crop was divided between laborers by unemployment.
and landowners. The Issei thus moved quickly from
wage-labor employment to sharecropping
agreements. A limited amount of economic progress
was made as some Issei were able to rent or buy
farmland directly, while others joined together to
form farming corporations. As the Issei began to
operate farms, they began to marry and start
families, forming an established Japanese American
community. Unfortunately, the Issei’s efforts to
attain agricultural independence were hampered by
government restrictions, such as the Alien Land Law
of 1913. But immigrants could circumvent such
exclusionary laws by leasing or purchasing land in
their American-born children’s names.
Passage 3
The single-celled parasite known as These oocysts are shed in the cats’ droppings
Toxoplasma gondii infects more than half of the and contaminate ground water and soil,
world’s human population without creating any eventually finding their way into the food chain.
noticeable symptoms. Once inside the human Because Toxoplasma must somehow find its
body, Toxoplasma rapidly spreads to the heart way into a new host cat in order to reproduce, it
and other organs. It can even penetrate the tight cannot kill its current host. Instead, it waits for
barrier that normally protects the brain from the host, usually a small rodent, to be eaten by a
most pathogens. Yet, the blood of infected cat, thus providing Toxoplasma the opportunity
persons carries very few free-floating to reproduce.
Toxoplasma cells. Scientists have long been
puzzled by this ability of Toxoplasma to 14. It can be inferred from the passage that
parasitize the human body without triggering which of the following statements is true of
an immune response and without an dendritic cells in the human body?
appreciable presence in the bloodstream. A. They are produced by the lymph nodes.
Recent research, however, has shed light on the B. They are more numerous in the digestive tract
ways in which Toxoplasma achieves its than in any other part of the human body.
remarkable infiltration of the human body. C. Most dendritic cells of persons infected with
Toxoplasma carry the parasite.
Though there are few individual Toxoplasma D. They are the only cells capable of being
cells coursing freely in the blood of an infected infected by Toxoplasma.
E. They are able to penetrate the tight barrier
person, scientists have discovered that the
that normally protects the brain from most
parasite is quite common in certain cells, known
pathogens.
as dendritic cells, involved in the human
immune system. Dendritic cells are found in the 15. According to the passage, all of the
digestive tract and frequently come into contact following are true of Toxoplasma gondii
with the various pathogens that enter the EXCEPT
human body through food and water. When the A. it can contaminate ground water
dendritic cells encounter pathogens, they travel B. it enters the human body through the food
to lymph nodes and relay this information to chain
other immune cells that then take action against C. it can alter the usual behavior of dendritic cells
the reported pathogen. Scientists have found, D. the human body is capable of detecting,
however, that Toxoplasma is capable of fighting, and eliminating it completely without
hijacking dendritic cells, forcing them from their any devastating consequence
usual activity and using them as a form of E. it must find a host cat in order to reproduce
transportation to infect the human body
quickly. Without this transport mechanism,
Toxoplasma could not reach the better-
protected areas of the body.
The single-celled parasite known as These oocysts are shed in the cats’ droppings
Toxoplasma gondii infects more than half of the and contaminate ground water and soil,
world’s human population without creating any eventually finding their way into the food chain.
noticeable symptoms. Once inside the human Because Toxoplasma must somehow find its
body, Toxoplasma rapidly spreads to the heart way into a new host cat in order to reproduce, it
and other organs. It can even penetrate the tight cannot kill its current host. Instead, it waits for
barrier that normally protects the brain from the host, usually a small rodent, to be eaten by a
most pathogens. Yet, the blood of infected cat, thus providing Toxoplasma the opportunity
persons carries very few free-floating to reproduce.
Toxoplasma cells. Scientists have long been
puzzled by this ability of Toxoplasma to 16. The second paragraph performs which of
parasitize the human body without triggering the following functions in the passage?
an immune response and without an A. It summarizes the research that remains to be
appreciable presence in the bloodstream. done regarding Toxoplasma.
Recent research, however, has shed light on the B. It presents a recommendation based on the
ways in which Toxoplasma achieves its new understanding of Toxoplasma.
remarkable infiltration of the human body. C. It describes the mechanism by which
Toxoplasma is able to parasitize the human
Though there are few individual Toxoplasma body.
D. It introduces information that is essential to
cells coursing freely in the blood of an infected
understanding the role of Toxoplasma in
person, scientists have discovered that the
human development.
parasite is quite common in certain cells, known
E. It discusses an outdated scientific model that
as dendritic cells, involved in the human has been discredited and offers a new model
immune system. Dendritic cells are found in the in its place.
digestive tract and frequently come into contact
with the various pathogens that enter the 17. Which of the following is the
human body through food and water. When the incontrovertible outcome for Toxoplasma
dendritic cells encounter pathogens, they travel cells that invade the human body?
to lymph nodes and relay this information to A. They will be destroyed by the immune system.
other immune cells that then take action against B. They will collect in the lymphatic system.
the reported pathogen. Scientists have found, C. They will not reproduce.
however, that Toxoplasma is capable of D. They will be detected after several weeks.
hijacking dendritic cells, forcing them from their E. They will be destroyed by other pathogens in
usual activity and using them as a form of the bloodstream.
transportation to infect the human body
quickly. Without this transport mechanism, 18. The author mentions “pigs and chickens” in
Toxoplasma could not reach the better- the final paragraph in order to
protected areas of the body. A. provide specific examples of animals that can
carry Toxoplasma
Toxoplasma invades the human body through B. provide specific examples of animals that are
consumption of the undercooked meat of often eaten by cats
infected animals, primarily pigs and chickens. C. provide specific examples of other animals
Other animals, such as cats, can become infected whose dendritic cells are exploited by
Toxoplasma
as well. In fact, cats are a necessary component
D. provide specific examples of animals in which
in the reproductive cycle of Toxoplasma, since
Toxoplasma can breed
the animal’s intestines are the parasite’s sole E. provide specific examples of animals that are
breeding ground. Toxoplasma creates egg-like immune to Toxoplasma
cysts, known as oocysts, in the cats’ intestines.
THE MOST AUTHORITATIVE GUIDE EVER WRITTEN ON GMAT READING COMPREHENSION
Passage 4
The Pan-American land bridge, or isthmus, 19. Which of the following statements is most
connecting North and South America was readily inferable from the information in
formed volcanically long after dinosaurs the passage?
became extinct. The isthmus cleaved A. Species of marine organisms in the Atlantic
populations of marine organisms, creating Ocean number fewer today than before the
sister species. These twin species, called formation of the Pan-American isthmus.
“geminates,” then evolved independently. B. The number of terrestrial animal species in
Scientists observe, for example, that Pacific South America today exceeds the number
pistol shrimp no longer mate with those from prior to the formation of the Pan-American
the Atlantic Ocean. Yet the two oceans had isthmus.
already begun to form their distinctive C. Of the indigenous North American species
personalities long before the isthmus was fully that migrated south across the Pan-
formed. As the seabed rose, Pacific waters grew American isthmus, more than three survive
cooler, their up-swelling currents carrying rich to this day.
nutrients, while the Atlantic side grew D. Since the formation of the Pan-American
shallower, warmer, and nutrient poor. In fact, it isthmus, fewer terrestrial animals have
was these new conditions, and not so much the traveled north across the isthmus than
fully-formed isthmus, that spawned changes in south.
the shrimp population. E. As the Pan-American isthmus began to form,
most pistol shrimp migrated west to what is
For terrestrial life, the impact of the isthmus now the Pacific Ocean.
was more immediate. Animals traversed the
newly formed bridge in both directions, 20. Which of the following statements finds
although North American creatures proved the LEAST support in the passage?
better colonizers—more than half of South A. Population divergences resulting from the
America’s mammals trace direct lineage to this formation of the Pan-American isthmus
so-called Great American Biotic Exchange. Only were more a process than an event.
three animals—the armadillo, opossum, and B. The divergence in ocean temperature during
hedgehog—survive as transplants in the North the formation of the Pan-American isthmus
today. resulted in a divergence in the ocean’s
nutrient value.
C. Genetic differences among pistol shrimp
have grown to the point that there are now
at least two distinct species of these shrimp.
D. The part of ocean which is now the Pacific
grew deeper due to the geologic forces that
created the Pan-American isthmus.
E. After the formation of the Pan-American
isthmus, geminate marine organisms began
to develop in that area of the ocean.
THE MOST AUTHORITATIVE GUIDE EVER WRITTEN ON GMAT READING COMPREHENSION
The Pan-American land bridge, or isthmus, 21. The author mentions the mating habits of
connecting North and South America was pistol shrimp in order to show that
formed volcanically long after dinosaurs A. Some species of marine organism inhabiting
became extinct. The isthmus cleaved the Pacific Ocean are now entirely distinct
populations of marine organisms, creating from those in the Atlantic Ocean.
sister species. These twin species, called B. Twin species of marine organisms can’t
“geminates,” then evolved independently. survive if one species can no longer mate
Scientists observe, for example, that Pacific with the other.
pistol shrimp no longer mate with those from C. Since the formation of the Pan-American
the Atlantic Ocean. Yet the two oceans had isthmus, marine geminates in general no
already begun to form their distinctive longer mate with their sister species.
personalities long before the isthmus was fully D. Geminate species that mate with one
formed. As the seabed rose, Pacific waters grew another can also be considered separate
cooler, their up-swelling currents carrying rich species.
nutrients, while the Atlantic side grew E. The evolutionary impact of the Pan-
shallower, warmer, and nutrient poor. In fact, it American isthmus was greater for land
was these new conditions, and not so much the animals than for marine organisms.
fully-formed isthmus, that spawned changes in
the shrimp population.
Passage 5
For years, many U.S. employers have employed a 22. Which of the following statements can
steady flow of laborers from Mexico willing to accept NOT be inferred from the passage?
low-skilled, low paying jobs. These workers, many of A. Due to the government’s family planning
whom leave economically depressed villages in the campaign, Mexico’s population is currently
Mexican interior, are often more than willing to
diminishing.
work for wages well below both the “U.S. Minimum
B. On average, Mexican women are having one-
Wage” and the poverty line, a reality duly exploited
by many U.S. employers. However, thanks to a third the number of children that they had in
dramatic demographic shift currently taking place in 1968.
Mexico, the seemingly inexhaustible supply of C. Many Mexicans still migrate to the United
workers migrating from Mexico to the United States States in search of work.
might one day greatly diminish if not cease. D. As a result of declining birth rates, Mexico’s
population is aging.
Predictions of such a drastic decrease in the number E. A healthy middle class in Mexico has not yet
of Mexican immigrants, both legal and illegal, are fully developed.
driven by Mexico’s rapidly diminishing population
growth. As a result of a decades-long family planning
23. Which of the following can be inferred
campaign, most Mexicans are having far fewer
about U.S. employers of Mexican
children than was the norm a generation ago. The
campaign, organized around the slogan that “the immigrants?
small family lives better,” saw the Mexican A. Most of these employers pay Mexican
government establish family-planning clinics and immigrants less money than they pay
offer free contraception. For nearly three decades, American citizens.
the government’s message concerning population B. Some of these employers violate wage laws.
hasn’t wavered. In fact, the Mexican Senate recently C. Many of these employers work in the
voted to extend public school sex education agricultural industry.
programs to kindergarten. D. Without Mexican immigrants, some of these
employers would be forced to close their
The result of Mexico’s efforts to stem population
businesses.
growth is nothing short of stunning. In 1968, the
E. Most of these employers show no concern
average Mexican woman had six children; today, the
figure is two. For two primary reasons, Mexico’s new for the welfare of their workers.
demographics could greatly impact the number of
Mexicans seeking work in the U.S. First, smaller 24. With which of the following statements
families by their nature limit the pool of potential would the author of the passage MOST
migrants. Second, the slowing of Mexico’s likely agree?
population growth has fostered hope that Mexico A. The United States will soon have to replace
will develop a healthy middle class of people content lost Mexican labor with labor provided by
to make their livelihoods in their home country. other immigrant groups.
B. It is difficult for a country with a large
Though the former of these factors is all but assured,
population to develop a healthy middle
the growth of a healthy middle class is far from any
class.
signs of reality. The critical challenge for Mexico is
what it does with the next 20 years. Mexico must C. Many Mexican immigrants who work in the
invest in education, job training, and infrastructure, United States believe that they are taken
as well as a social-security system to protect its advantage of by American employers.
aging population, which is a direct result of the D. Most rapidly growing countries should
family planning programs run over many decades. If institute a family planning campaign to limit
Mexico is willing to step forward and meet this population growth.
challenge, America may one day wake up to find that, E. Mexico does not currently have a healthy
like cheap gasoline, cheap Mexican labor has become middle class.
a thing of the past.
THE MOST AUTHORITATIVE GUIDE EVER WRITTEN ON GMAT READING COMPREHENSION
For years, many U.S. employers have employed a 25. One function of the final paragraph of the
steady flow of laborers from Mexico willing to accept passage is to
low-skilled, low paying jobs. These workers, many of A. explain why the number of Mexican
whom leave economically depressed villages in the immigrants seeking work in the United
Mexican interior, are often more than willing to
States is bound to decline.
work for wages well below both the “U.S. Minimum
B. detail the successes of Mexico’s family
Wage” and the poverty line, a reality duly exploited
by many U.S. employers. However, thanks to a planning campaign.
dramatic demographic shift currently taking place in C. explain why the number of Mexican
Mexico, the seemingly inexhaustible supply of immigrants seeking work in the United
workers migrating from Mexico to the United States States may not dramatically decrease.
might one day greatly diminish if not cease. D. specify the types of infrastructure in which
Mexico must invest.
Predictions of such a drastic decrease in the number E. warn American employers that they will
of Mexican immigrants, both legal and illegal, are soon need to find alternative sources of
driven by Mexico’s rapidly diminishing population labor.
growth. As a result of a decades-long family planning
campaign, most Mexicans are having far fewer
children than was the norm a generation ago. The
campaign, organized around the slogan that “the
small family lives better,” saw the Mexican
government establish family-planning clinics and
offer free contraception. For nearly three decades,
the government’s message concerning population
hasn’t wavered. In fact, the Mexican Senate recently
voted to extend public school sex education
programs to kindergarten.
Passage 6
In a 1984 book, Claire C. Robertson argued that, 27. The passage suggests that after conducting
before colonialism, age was a more important the research mentioned in the highlighted
text, but not before, Robertson would have
indicator of status and authority than gender in
agreed with which of the following about
Ghana and in Africa generally. British
women’s status and authority in Ghana?
colonialism imposed European-style male- A. Greater land rights and greater involvement in
dominant notions upon more egalitarian local trade made women in precolonial Ghana less
situations to the detriment of women generally, dependent on men than were European women
and gender became a defining characteristic at that time.
that weakened women’s power and authority. B. Colonialism had a greater impact on the status
and authority of Ghanaian women than on
Subsequent research in Kenya convinced Kenyan women.
Robertson that she had overgeneralized about C. Colonialism had less of an impact on the status
Africa. Before colonialism, gender was more and authority of Ghanaian women that it had on
the status and authority of other African women.
salient in central Kenya than it was in Ghana,
D. The relative independence of Ghanaian women
although age was still crucial in determining
prior to colonialism was unique in Africa.
authority. In contrast with Ghana, where E. Before colonialism, the status and authority of
women had traded for hundreds of years and Ghanaian women was similar to that of Kenyan
achieved legal majority (not unrelated women.
phenomena), the evidence regarding central
Kenya indicated that women were legal minors 28. The author of the passage mentions the
and were sometimes treated as male property, status of age as a principle of social
as were European women at that time. Factors organization in precolonial central Kenya in
like strong patrilinearity and patrilocality, as the highlighted text most likely in order to
well as women’s inferior land rights and lesser A. indicate that women’s dependence on men in
precolonial Kenya was not absolute
involvement in trade, made women more
B. contrast the situation of senior women to that of
dependent on men than was generally the case less senior women in precolonial Ghana
in Ghana. However, since age apparently C. differentiate between the status and authority of
remained the overriding principle of social precolonial Kenyan women and that of
organization in central Kenya, some senior precolonial African women
women had much authority. D. explain why age superseded gender to a greater
extent in precolonial Kenya than it did elsewhere
26. The passage indicates that Robertson’s in Africa
subsequent research in Kenya caused her to E. identify a factor that led Robertson to revise her
change her mind regarding which of the hypothesis about precolonial Ghana
following?
A. Whether age was the prevailing principle of
social organization in Africa before colonialism
B. Whether gender was the primary determinant of
social authority in Africa generally before
colonialism
C. Whether it was after colonialism that gender
became a significant determinant of authority in
Kenyan society
D. Whether age was a crucial factor determining
authority in Africa after colonialism
E. Whether British colonialism imposed European-
style male-dominant notions upon local
situations in Ghana
THE MOST AUTHORITATIVE GUIDE EVER WRITTEN ON GMAT READING COMPREHENSION
Passage 7
Australian researchers have discovered 29. Which of the following can be inferred
electroreceptors (sensory organs designed to about the experiment described in the
respond to electrical fields) clustered at the tip first paragraph?
of the spiny anteater’s snout. The researchers A. Researchers had difficulty verifying the
made this discovery by exposing small areas of existence of electroreceptors in the anteater
the snout to extremely weak electrical fields and because electroreceptors respond to such a
recording the transmission of resulting nervous narrow range of electrical field strengths.
activity to the brain. While it is true that tactile B. Researchers found that the level of nervous
receptors, another kind of sensory organ on the activity in the anteater’s brain increased
anteater’s snout, can also respond to electrical dramatically as the strength of the electrical
stimuli, such receptors do so only in response to stimulus was increased.
electrical field strengths about 1,000 times C. Researchers found that some areas of the
greater than those known to excite anteater’s snout were not sensitive to a
electroreceptors. weak electrical stimulus.
D. Researchers found that the anteater’s tactile
Having discovered the electroreceptors, receptors were more easily excited by a
researchers are now investigating how strong electrical stimulus than were the
anteaters utilize such a sophisticated sensory electro receptors.
system. In one behavioral experiment, E. Researchers tested small areas of the
researchers successfully trained an anteater to anteater’s snout in order to ensure that only
distinguish between two troughs of water, one electroreceptors were responding to the
with a weak electrical field and the other with stimulus.
none. Such evidence is consistent with
researchers’ hypothesis that anteaters use 30. Which of the following, if true, would
electroreceptors to detect electrical signals most strengthen the hypothesis
given off by prey; however, researchers as yet mentioned in the highlighted text?
have been unable to detect electrical signals A. Researchers are able to train anteaters to
emanating from termite mounds, where the break into an underground chamber that is
favorite food of anteaters live. Still, researchers emitting a strong electrical signal.
have observed anteaters breaking into a nest of B. Researchers are able to detect a weak
ants at an oblique angle and quickly locating electrical signal emanating from the nesting
nesting chambers. This ability quickly to locate chamber of an ant colony.
unseen prey suggests, according to the C. Anteaters are observed taking increasingly
researchers, that the anteaters were using their longer amounts of time to locate the nesting
electroreceptors to locate the nesting chambers. chambers of ants.
D. Anteaters are observed using various angles
to break into nests of ants.
E. Anteaters are observed using the same angle
used with nests of ants to break into the
nests of other types of prey.
THE MOST AUTHORITATIVE GUIDE EVER WRITTEN ON GMAT READING COMPREHENSION
Australian researchers have discovered 31. The passage suggests that the
electroreceptors (sensory organs designed to researchers mentioned in the second
respond to electrical fields) clustered at the tip paragraph who observed anteaters
of the spiny anteater’s snout. The researchers break into a nest of ants would most
made this discovery by exposing small areas of likely agree with which of the following
the snout to extremely weak electrical fields and statements?
recording the transmission of resulting nervous A. The event they observed provides
activity to the brain. While it is true that tactile conclusive evidence that anteaters use their
receptors, another kind of sensory organ on the electroreceptors to locate unseen prey.
anteater’s snout, can also respond to electrical B. The event they observed was unusual and
stimuli, such receptors do so only in response to may not reflect the usual hunting practices
electrical field strengths about 1,000 times of anteaters.
greater than those known to excite C. It is likely that the anteaters located the ants’
electroreceptors. nesting chambers without the assistance of
electroreceptors.
Having discovered the electroreceptors, D. Anteaters possess a very simple sensory
researchers are now investigating how system for use in locating prey.
anteaters utilize such a sophisticated sensory E. The speed with which the anteaters located
system. In one behavioral experiment, their prey is greater than what might be
researchers successfully trained an anteater to expected in the absence of any external
distinguish between two troughs of water, one stimulus.
with a weak electrical field and the other with
none. Such evidence is consistent with 32. Which of the following can be inferred
researchers’ hypothesis that anteaters use about anteaters from the behavioral
electroreceptors to detect electrical signals experiment mentioned in the second
given off by prey; however, researchers as yet paragraph?
have been unable to detect electrical signals A. They are unable to distinguish between
emanating from termite mounds, where the stimuli detected by their electroreceptors
favorite food of anteaters live. Still, researchers and stimuli detected by their tactile
have observed anteaters breaking into a nest of receptors.
ants at an oblique angle and quickly locating B. They are unable to distinguish between the
nesting chambers. This ability quickly to locate electrical signals emanating from termite
unseen prey suggests, according to the mounds and those emanating from ant
researchers, that the anteaters were using their nests.
electroreceptors to locate the nesting chambers. C. They can be trained to recognize the
presence of a particular electrical stimulus.
D. They react more readily to strong than to
weak stimuli.
E. They are more efficient at detecting stimuli
in a controlled environment than in a natural
environment.
THE MOST AUTHORITATIVE GUIDE EVER WRITTEN ON GMAT READING COMPREHENSION
Australian researchers have discovered 33. The author of the passage most probably
electroreceptors (sensory organs designed to discusses the function of tactile receptors
respond to electrical fields) clustered at the tip in order to
of the spiny anteater’s snout. The researchers A. eliminate an alternative explanation of
made this discovery by exposing small areas of anteaters’ response to electrical stimuli
the snout to extremely weak electrical fields and B. highlight a type of sensory organ that has a
recording the transmission of resulting nervous function identical to that of electroreceptors
activity to the brain. While it is true that tactile C. point out a serious complication in the
receptors, another kind of sensory organ on the research on electroreceptors in anteaters.
anteater’s snout, can also respond to electrical D. suggest that tactile receptors assist
stimuli, such receptors do so only in response to electroreceptors in the detection of
electrical field strengths about 1,000 times electrical signals.
greater than those known to excite E. introduce a factor that was not addressed in
electroreceptors. the research on electroreceptors in
anteaters.
Having discovered the electroreceptors,
researchers are now investigating how 34. According to the passage, which of the
anteaters utilize such a sophisticated sensory following is a characteristic that
system. In one behavioral experiment, distinguishes electroreceptors from
researchers successfully trained an anteater to tactile receptors?
distinguish between two troughs of water, one A. The manner in which electroreceptors
with a weak electrical field and the other with respond to electrical stimuli
none. Such evidence is consistent with B. The tendency of electroreceptors to be
researchers’ hypothesis that anteaters use found in clusters
electroreceptors to detect electrical signals C. The unusual locations in which
given off by prey; however, researchers as yet electroreceptors are found in most species.
have been unable to detect electrical signals D. The amount of electrical stimulation
emanating from termite mounds, where the required to excite electroreceptors
favorite food of anteaters live. Still, researchers E. The amount of nervous activity transmitted
have observed anteaters breaking into a nest of to the brain by electroreceptors when they
ants at an oblique angle and quickly locating are excited
nesting chambers. This ability quickly to locate
unseen prey suggests, according to the
researchers, that the anteaters were using their
electroreceptors to locate the nesting chambers.
THE MOST AUTHORITATIVE GUIDE EVER WRITTEN ON GMAT READING COMPREHENSION
Passage 8
Insect behavior generally appears to be 35. The author mentions the work of Darwin
explicable in terms of unconscious, inflexible and Fabre in order to
stimulus-response mechanisms. For instance, a A. provide experimental evidence of the
female sphex wasp leaves her egg sealed in a inflexibility of one kind of insect behavior
burrow alongside a paralyzed grasshopper, B. contradict the conventional wisdom about
which her larvae can eat upon hatching. Before “typical” wasp behavior
she deposits the grasshopper in the burrow, she C. illustrate the strength of the wasp's
inspects the burrow; if the inspection reveals no maternal affection
problems, she drags the grasshopper inside by D. explore the logical implications of the thesis
its antennae. As thoughtful as this behavior articulated earlier
appears, it reveals its mechanistic character E. highlight historical changes in the
upon interference. Darwin discovered that prior conduction of scientific research
removal of the grasshopper's antennae
prevents the wasp from depositing the 36. Which of the following hypothetical
grasshopper, even though the legs or ovipositor variations in the experiments described
could also serve as handles. Likewise, Fabre in the passage would most weaken the
moved the grasshopper a few centimeters away primary claim of the passage?
from the burrow's mouth while the wasp was A. Darwin removes the ovipositor, a small
inside inspecting. The wasp returned the appendage, instead of the antennae; the
grasshopper to the edge of the burrow and then wasp fails to deposit the grasshopper in the
began a new inspection. Fabre performed this burrow.
disruptive maneuver forty times; the wasp's B. Darwin restrains the grasshopper while the
response never changed. wasp attempts to drag it by its antennae,
which subsequently break off; although
Darwin then releases the grasshopper, the
wasp ignores it.
C. Fabre moves the grasshopper several
meters away during the wasp's inspection;
the wasp takes significant time to retrieve
the grasshopper, then re-inspects the
burrow.
D. Fabre repeatedly varies the exact position
near the burrow to which he moves the
grasshopper, causing the wasp to adjust its
retrieval path slightly before re-inspecting
the burrow.
E. Fabre replaces the grasshopper with a
paralyzed praying mantis, a rather different
insect that the wasp inspects and then
deposits in the burrow.
THE MOST AUTHORITATIVE GUIDE EVER WRITTEN ON GMAT READING COMPREHENSION
Insect behavior generally appears to be 37. The passage supports which of the
explicable in terms of unconscious, inflexible following statements about insect
stimulus-response mechanisms. For instance, a behavior?
female sphex wasp leaves her egg sealed in a A. Reptiles such as snakes behave more flexibly
burrow alongside a paralyzed grasshopper, than do insects.
which her larvae can eat upon hatching. Before B. Insects such as honeybees can always be
she deposits the grasshopper in the burrow, she expected to behave inflexibly.
inspects the burrow; if the inspection reveals no C. Many species of insects leave eggs alongside
problems, she drags the grasshopper inside by living but paralyzed food sources.
its antennae. As thoughtful as this behavior D. Stimulus-response mechanisms in insects
appears, it reveals its mechanistic character have evolved because, under ordinary
upon interference. Darwin discovered that prior circumstances, they help insects to survive.
removal of the grasshopper's antennae E. More than one species of insect displays
prevents the wasp from depositing the inflexible, routine behaviors.
grasshopper, even though the legs or ovipositor
could also serve as handles. Likewise, Fabre 38. Based on the passage, which of the
moved the grasshopper a few centimeters away following would prove a similar point to
from the burrow's mouth while the wasp was that promoted by the author?
inside inspecting. The wasp returned the I. In a similar experiment, the paralyzed
grasshopper to the edge of the burrow and then grasshopper was replaced with another,
began a new inspection. Fabre performed this equally nutritive insect, and the wasp did
disruptive maneuver forty times; the wasp's not drag it into the burrow.
response never changed. II. In a similar experiment with a bird, the bird
was shown to act in the exact same manner
as the wasp.
III. In a similar experiment with a different
wasp, the wasp dragged the grasshopper
into the burrow by its ovipositor.
A. I only
B. I and II
C. II only
D. II and III
E. I, II and III
THE MOST AUTHORITATIVE GUIDE EVER WRITTEN ON GMAT READING COMPREHENSION
RC Primary Purpose
Refresher
(Discussed in RC video 5)
Passage 1
Archaeology as a profession faces two major The basements of museums are simply not large
problems. First, it is the poorest of the poor. enough to store the artifacts that are likely to be
Only paltry sums are available for excavating discovered in the future. There is not enough
and even less is available for publishing the money even to catalog the finds; as a result, they
results and preserving the sites once excavated. cannot be found again and become as
Yet archaeologists deal with priceless objects inaccessible as if they had never been
every day. Second, there is the problem of illegal discovered. Indeed, with the help of a computer,
excavation, resulting in museum-quality pieces sold artifacts could be more accessible than are
being sold to the highest bidder. the pieces stored in bulging museum
basements. Prior to sale, each could be
I would like to make an outrageous suggestion photographed and the list of the purchasers
that would at one stroke provide funds for could be maintained on the computer. A
archaeology and reduce the amount of illegal purchaser could even be required to agree to
digging. I would propose that scientific return the piece if it should become needed for
archaeological expeditions and governmental scientific purposes.
authorities sell excavated artifacts on the open
market. Such sales would provide substantial It would be unrealistic to suggest that illegal
funds for the excavation and preservation of digging would stop if artifacts were sold on the
archaeological sites and the publication of open market. But the demand for the
results. At the same time, they would break the clandestine product would be substantially
illegal excavator’s grip on the market, thereby reduced. Who would want an unmarked pot
decreasing the inducement to engage in illegal when another was available whose provenance
activities. was known, and that was dated
stratigraphically by the professional
You might object that professionals excavate to archaeologist who excavated it?
acquire knowledge, not money. Moreover,
ancient artifacts are part of our global cultural The primary purpose of the passage is to
heritage, which should be available for all to propose
appreciate, not sold to the highest bidder. I A. an alternative to museum display of artifacts
agree. Sell nothing that has unique artistic merit B. a way to curb illegal digging while benefiting
or scientific value. But, you might reply, the archaeological profession
everything that comes out of the ground has C. a way to distinguish artifacts with scientific
scientific value. Here we part company. value from those that have no such value
Theoretically, you may be correct in claiming D. the governmental regulation of
that every artifact has potential scientific value. archaeological sites
Practically, you are wrong. E. a new system for cataloging duplicate
artifacts
I refer to the thousands of pottery vessels and
ancient lamps that are essentially duplicates of
one another. In one small excavation in Cyprus,
archaeologists recently uncovered 2,000
virtually indistinguishable small jugs in a single
courtyard. Even precious royal seal impressions
known as l’melekh handles have been found in
abundance—more than 4,000 examples so far.
THE MOST AUTHORITATIVE GUIDE EVER WRITTEN ON GMAT READING COMPREHENSION
Passage 2
Women’s grassroots activism and their vision of The primary purpose of the passage is to
a new civic consciousness lay at the heart of A. explain why women reformers of the
social reform in the United States throughout Progressive Era failed to achieve their goals
the Progressive Era, the period between the B. discuss the origins of child labor laws in the
depression of 1893 and America’s entry into the late nineteenth and early twentieth
Second World War. Though largely centuries
disenfranchised except for school elections, C. compare the living conditions of working-
white middle-class women reformers won a class and middle-class women in the
variety of victories, notably in the improvement Progressive Era
of working conditions, especially for women D. discuss an oversight on the part of women
and children. Ironically, though, child labor reformers of the Progressive Era
legislation pitted women of different classes E. revise a traditional view of the role played
against one another. To the reformers, child by women reformers in enacting
labor and industrial home work were equally Progressive Era reforms
inhumane practices that should be outlawed,
but, as a number of women historians have
recently observed, working-class mothers did
not always share this view. Given the precarious
finances of working-class families and the
necessity of pooling the wages of as many family
members as possible, working-class families
viewed the passage and enforcement of
stringent child labor statutes as a personal
economic disaster and made strenuous efforts
to circumvent child labor laws. Yet reformers
rarely understood this resistance in terms of the
desperate economic situation of working-class
families, interpreting it instead as evidence of
poor parenting. This is not to dispute women
reformers’ perception of child labor as a terribly
exploitative practice, but their understanding of
child labor and their legislative solutions for
ending it failed to take account of the economic
needs of working-class families.
THE MOST AUTHORITATIVE GUIDE EVER WRITTEN ON GMAT READING COMPREHENSION
Passage 3
Passage 4
In 1955 Maurice Duverger published The The primary purpose of the passage is to
Political Role of Women, the first behavioralist, A. evaluate a research study
multinational comparison of women’s electoral B. summarize the history of a research area
participation ever to use election data and C. report new research findings
survey data together. His study analyzed D. reinterpret old research findings
women’s patterns of voting, political candidacy, E. reconcile conflicting research findings
and political activism in four European
countries during the first half of the twentieth
century. Duverger’s research findings were that
women voted somewhat less frequently than
men (the difference narrowing the longer
women had the vote) and were slightly more
conservative.
Passage 5
Frazier and Mosteller assert that medical The passage is primarily concerned with
research could be improved by a move toward A. identifying two practices in medical
larger, simpler clinical trials of medical research that may affect the accuracy of
treatments. Currently, researchers collect far clinical trials
more background information on patients than B. describing aspects of medical research that
is strictly required for their trials— tend to drive up costs
substantially more than hospitals collect— C. evaluating an analysis of certain
thereby escalating costs of data collection, shortcomings of current medical research
storage, and analysis. Although limiting practices
information collection could increase the risk D. describing proposed changes to the ways in
that researchers will overlook facts relevant to which clinical trials are conducted
a study, Frazier and Mosteller contend that such E. explaining how medical researchers have
risk, never entirely eliminable from research, traditionally conducted clinical trials and
would still be small in most studies. Only in how such trials are likely to change
research on entirely new treatments are new
and unexpected variables likely to arise.
Passage 6
Current feminist theory, in validating women’s The passage is primarily concerned with
own stories of their experience, has encouraged A. contrasting the benefits of one methodology
scholars of women’s history to view the use of with the benefits of another
women’s oral narratives as the methodology, B. describing the historical origins and
next to the use of women’s written inherent drawbacks of a particular
autobiography, that brings historians closest to methodology
the “reality” of women’s lives. Such narratives, C. discussing the appeal of a particular
unlike most standard histories, represent methodology and some concerns about its
experience from the perspective of women, use
affirm the importance of women’s D. showing that some historians’ adoption of a
contributions, and furnish present-day women particular methodology has led to criticism
with historical continuity that is essential to of recent historical scholarship
their identity, individually and collectively. E. analyzing the influence of current feminist
views on women’s interpretations of their
Scholars of women’s history should, however, experience
be as cautious about accepting oral narratives at
face value as they already are about written
memories. Oral narratives are no more likely
than are written narratives to provide a
disinterested commentary on events or people.
Moreover, the stories people tell to explain
themselves are shaped by narrative devices and
storytelling conventions, as well as by other
cultural and historical factors, in ways that the
storytellers may be unaware of. The political
rhetoric of a particular era, for example, may
influence women’s interpretations of the
significance of their experience. Thus, a woman
who views the Second World War as pivotal in
increasing the social acceptance of women’s
paid work outside the home may reach that
conclusion partly and unwittingly because of
wartime rhetoric encouraging a positive view of
women’s participation in such work.
THE MOST AUTHORITATIVE GUIDE EVER WRITTEN ON GMAT READING COMPREHENSION
Passage 7
During the 1960s and 1970s, the primary The primary purpose of the passage is to
economic development strategy of local A. advocate more effective strategies for
governments in the United States was to attract encouraging the development of high
manufacturing industries. Unfortunately, this technology enterprises in the United States
strategy was usually implemented at another B. contrast the incentives for economic
community’s expense: many manufacturing development offered by local governments
facilities were lured away from their moorings with those offered by the private sector
elsewhere through tax incentives and slick C. acknowledge and counter adverse criticism
promotional efforts. Through the transfer of of programs being used to stimulate local
jobs and related revenues that resulted from economic development
this practice, one town’s triumph could become D. define and explore promotional efforts used
another town’s tragedy. by local governments to attract new
industry
In the 1980s the strategy shifted from this zero- E. review and evaluate strategies and
sum game to one called “high-technology programs that have been used to stimulate
development,” in which local governments economic development
competed to attract newly formed high-
technology manufacturing firms. Although this
approach was preferable to victimizing other
geographical areas by taking their jobs, it also
had its shortcomings: high-tech manufacturing
firms employ only a specially trained fraction of
the manufacturing workforce, and there simply
are not enough high-tech firms to satisfy all
geographic areas.
Passage 8
The Gross Domestic Product (GDP), which The primary purpose of the passage is to
measures the dollar value of finished goods and A. identify ways in which the GDP could be
services produced by an economy during a modified so that it would serve as a more
given period, serves as the chief indicator of the accurate indicator of the economic well-
economic well-being of the United States. The being of the United States
GDP assumes that the economic significance of B. suggest that the GDP, in spite of certain
goods and services lies solely in their price, and shortcomings, is still the most reliable
that these goods and services add to the national indicator of the economic wellbeing of the
well-being, not because of any intrinsic value United States
they may possess, but simply because they were C. examine crucial shortcomings of the GDP as
produced and bought. Additionally, only those an indicator of the economic well-being of
goods and services involved in monetary the United States
transactions are included in the GDP. Thus, the D. argue that the growth of the United States
GDP ignores the economic utility of such things economy in recent decades has diminished
as a clean environment and cohesive families the effectiveness of the GDP as an indicator
and communities. It is therefore not merely of the nation’s economic well-being
coincidental, since national policies in capitalist E. discuss how the GDP came to be used as the
and non-capitalist countries alike are primary indicator of the economic well-
dependent on indicators such as the GDP, that being of the United States
both the environment and the social structure
have been eroded in recent decades. Not only
does the GDP mask this erosion, it can actually
portray it as an economic gain: an oil spill off a
coastal region “adds” to the GDP because it
generates commercial activity. In short, the
nation’s central measure of economic well-
being works like a calculating machine that adds
but cannot subtract.
THE MOST AUTHORITATIVE GUIDE EVER WRITTEN ON GMAT READING COMPREHENSION
Learning objectives: This exercise has been designed to help you strengthen your grip
on Conclusion / Opinion for Primary Purpose / Main Idea Questions in RC and to help
you with the foundational building blocks for CR.
Note: this question-type is NOT tested on the GMAT. This is part of the concept-building
pre-work for a great performance in RC and CR.
Premise Cues: Since, The reason is, Because, For (when it means ‘because’), For example, For the
reason that, In that, Given that, As indicated by, Due to, Owing to, This can be seen from, We know this
by
Contradiction Cues: But, However, Nonetheless, Nevertheless, Notwithstanding, Even so, Despite,
Rather, Yet, On the other hand, Admittedly, In contrast, By contrast, Contrary to, Although, Even though,
Still, Whereas, In spite of, After all, Alternatively, Apart from, Conversely, Regardless, Then again,
Unfortunately, Ironically etc.
THE MOST AUTHORITATIVE GUIDE EVER WRITTEN ON GMAT READING COMPREHENSION
A special cue: Some people believe: One of the most frequently used constructions is to raise a viewpoint
at the beginning of the stimulus and then disagree with it immediately thereafter. This efficiently raises
two opposing views in a very short paragraph. These stimuli are recognizable because they often begin
with the phrase, “Some people claim...” or one of the many variations on this theme, including but not
limited to the following:
All these imply that, in the passage, there will be challenge to whatever is said by others.
For example, every time the word thus may not signal a conclusion. The test maker knows this: the
moment you see the word “thus”, you will automatically be thinking "conclusion!" So, sometimes the
GMAT will attempt to fool you.
Countries that invest heavily in technology and thus become highly advanced automatically start seeing
themselves as global superpowers.
1. The term “pit bull” does not designate a breed of dog, as do the terms “German shepherd” and
“poodle.” It is like the terms “Seeing-Eye dog” and “police dog,” which designate dogs according to
what they do. If you take two German shepherds and place them side by side, you cannot tell by
appearance alone which is the police dog and which is the Seeing-Eye dog.
Which one of the following is the main point of the passage?
A. German shepherds can be pit bulls.
B. Pit bulls can be distinguished from other kinds of dogs by appearance alone.
C. A dog is a pit bull because of what it does, not because of its breed.
D. German shepherds can function both as police dogs and as Seeing-Eye dogs.
E. Some breeds of dogs cannot be distinguished from other breeds of dogs by appearance alone.
2. Can any research be found to validate the contention that those who spend time plucking out their
gray hairs have more negative attitudes toward the elderly than those who shrug their shoulders
about their gray hairs? Unless a person’s psychopathology leads him or her to overgeneralize, there
is no necessary connection. Certainly, it is reasonable to like the elderly yet dislike the idea of
impaired eyesight and hearing. Furthermore, holding negative attitudes toward older people merely
because they are old is immoral, according to nearly universally accepted ethical standards. But
there is nothing immoral about disliking some concomitants of the aging process.
Which one of the following best expresses the main point of the passage?
A. It cannot be assumed that people who dislike some of the physical concomitants of growing old
necessarily have negative feelings toward the elderly.
B. To dislike some of the physical concomitants growing old is reasonable, while to dislike the elderly
is immoral.
C. Since no one likes the physical concomitants of growing old, it is wrong to dislike the elderly merely
because of their physical characteristics.
D. Being elderly is fine, but the process of becoming elderly is not; and people need to understand the
distinction between the two.
E. To dislike the elderly is immoral, and to do so just because one dislikes some of the physical
concomitants of growing old is unreasonable.
3. As symbols of the freedom of the wilderness, bald eagles have the unique capacity to inspire people
and foster in them a sympathetic attitude toward the needs of other threatened species. Clearly,
without that sympathy and the political will it engenders, the needs of more obscure species will go
unmet. The conservation needs of many obscure species can only be met by beginning with the
conservation of this symbolic species, the bald eagle.
Which one of the following is the main point of the passage as a whole?
A. Because bald eagles symbolize freedom, conservation efforts should be concentrated on them rather
than on other, more obscure species.
B. The conservation of bald eagles is the first necessary step in conserving other endangered species.
C. Without increased public sympathy for conservation, the needs of many symbolic species will go
unmet.
D. People’s love of the wilderness can be used to engender political support for conservation efforts.
E. Other threatened species do not inspire people or foster sympathy as much as do bald eagles.
THE MOST AUTHORITATIVE GUIDE EVER WRITTEN ON GMAT READING COMPREHENSION
4. Like a number of other articles, Ian Raghnall’s article relied on a recent survey in which over half
the couples applying for divorces listed “money” as a major problem in their marriages. Raghnall’s
conclusion from the survey data is that financial problems are the major problem in marriages and
an important factor contributing to the high divorce rate. Yet couples often express other types of
marital frustrations in financial terms. Despite appearances, the survey data do not establish that
financial problems are the major problem in contemporary marriages.
Which one of the following sentences best expresses the main point of the passage?
A. Financial problems are not an important factor contributing to the divorce rate.
B. Marital problems are more easily solved by marriage counselors than by married couples on their
own.
C. The conclusion drawn in Raghnall’s article is inadequately justified.
D. Over half the couples applying for divorces listed money as a major problem in their marriages.
E. Many articles wrongly claim that financial problems are the major factor contributing to the divorce
rate.
5. The United States government generally tries to protect valuable natural resources. But one
resource has been ignored for too long. In the United States, each bushel of corn produced might
result in the loss of as much as two bushels of topsoil. Moreover, in the last 100 years, the topsoil in
many states, which once was about fourteen inches thick, has been eroded to only six or eight inches.
Nonetheless, federal expenditures for nationwide soil conservation programs have remained at
ridiculously low levels. Total federal expenditures for nationwide soil conservation programs have
been less than the allocations of some individual states.
Which one of the following best expresses the main point of the argument?
A. Corn is not a cost-effective product and substitutes should be found where possible.
B. A layer of topsoil only six to eight inches thick cannot support the continued cultivation of corn.
C. Soil conservation is a responsibility of the federal government, not the states.
D. The federal government’s expenditures for soil conservation in the various states have been
inequitable.
E. The federal government should spend much more on soil conservation than it has been spending.
6. A law that is not consistently enforced does not serve its purpose. Law without enforcement is not
law; it is merely statute—a promise of law. To institute real law is not merely to declare that such
and such behavior is forbidden; it is also to punish those who violate that edict. Furthermore, those
who enforce law must punish without favor for their friends or malice for their enemies. To punish
only those one dislikes while forgiving others is not to enforce law but to engage in the arbitrary and
unjust exercise of power.
The main point of the passage is that instituting real law consists in
A. the exercise of power
B. authorizing the enforcement of punishments
C. the unbiased punishment of prohibited behavior
D. understanding the purpose of law
E. clearly defining unacceptable behavior
THE MOST AUTHORITATIVE GUIDE EVER WRITTEN ON GMAT READING COMPREHENSION
7. There is no mystery as to why figurative painting revived in the late 1970s. People want to look at
recognizable images. Sorting out art theories reflected in abstract paintings is no substitute for the
sense of empathy that comes from looking at a realistic painting of a figure in a landscape. Perhaps
members of the art-viewing public resented abstract art because they felt that its lack of realistic
subject matter was a rejection of the viewers and their world.
Which one of the following most accurately expresses the main point of the passage?
A. Abstract paintings often include shapes or forms that are suggestive of real objects or emotions.
B. The art-viewing public wished to see traditional subjects treated in a nontraditional manner.
C. Paintings that depict a recognizable physical world rather than the emotional world of the artist’s
life require more artistic talent to create.
D. The general public is unable to understand the theories on which abstract painting is based.
E. The artistic preferences of the art-viewing public stimulated the revival.
8. Arguing that there was no trade between Europe and East Asia in the early Middle Ages because
there are no written records of such trade is like arguing that the yeti, an apelike creature
supposedly existing in the Himalayas, does not exist because there have been no scientifically
confirmed sightings. A verifiable sighting of the yeti would prove that the creature does exist, but
the absence of sightings cannot prove that it does not.
Which one of the following best expresses the point of the argument?
A. Evidence for the existence of trade between Europe and East Asia in the early Middle Ages is, like
evidence for the existence of the yeti, not scientifically confirmed.
B. In order to prove that in the early Middle Ages there was trade between Europe and East Asia it is
necessary to find both Asian and European evidence that such trade existed.
C. That trade between Europe and East Asia did not exist in the early Middle Ages cannot be
established simply by the absence of a certain sort of evidence that this trade existed.
D. The view that there was trade between Europe and East Asia in the early Middle Ages can only be
disproved by showing that no references to this trade exist in surviving records.
E. There is no more evidence that trade between Europe and East Asia existed in the early Middle Ages
than there is that the yeti exists.
THE MOST AUTHORITATIVE GUIDE EVER WRITTEN ON GMAT READING COMPREHENSION
9. Some legislators refuse to commit public funds for new scientific research if they cannot be assured
that the research will contribute to the public welfare. Such a position ignores the lessons of
experience. Many important contributions to the public welfare that resulted from scientific
research were never predicted as potential outcomes of that research. Suppose that a scientist in
the early twentieth century had applied for public funds to study molds: who would have predicted
that such research would lead to the discovery of antibiotics—one of the greatest contributions ever
made to the public welfare?
Which one of the following most accurately expresses the main point of the argument?
A. The committal of public funds for new scientific research will ensure that the public welfare will be
enhanced.
B. If it were possible to predict the general outcome of a new scientific research effort, then legislators
would not refuse to commit public funds for that effort.
C. Scientific discoveries that have contributed to the public welfare would have occurred sooner if
public funds had been committed to the research that generated those discoveries.
D. In order to ensure that scientific research is directed toward contributing to the public welfare,
legislators must commit public funds to new scientific research.
E. Lack of guarantees that new scientific research will contribute to the public welfare is not sufficient
reason for legislators to refuse to commit public funds to new scientific research.
10. Balance is particularly important when reporting the background of civil wars and conflicts. Facts
must not be deliberately manipulated to show one party in a favorable light, and the views of each
side should be fairly represented. This concept of balance, however, does not justify concealing or
glossing over basic injustices in an effort to be even-handed. If all the media were to adopt such a
perverse interpretation of balanced reporting, the public would be given a picture of a world where
each party in every conflict had an equal measure of justice on its side, contrary to our experience
of life and, indeed, our common sense.
Which one of the following best expresses the main point of the argument?
A. Balanced reporting presents the public with a picture of the world in which all sides to a conflict
have equal justification.
B. Balanced reporting requires impartially revealing injustices where they occur no less than fairly
presenting the views of each party in a conflict.
C. Our experience of life shows that there are indeed cases in which conflicts arise because of an
injustice, with one party clearly in the wrong.
D. Common sense tells us that balance is especially needed when reporting the background of civil
wars and conflicts.
E. Balanced reporting is an ideal that cannot be realized, because judgments of balance are necessarily
subjective.
THE MOST AUTHORITATIVE GUIDE EVER WRITTEN ON GMAT READING COMPREHENSION
11. The fire that destroyed the Municipal Building started before dawn this morning, and the last fire
fighters did not leave until late this afternoon. No one could have been anywhere in the vicinity of a
fire like that one and fail to notice it. Thomas must have seen it, whatever he now says to the
contrary. He admits that, as usual, he went from his apartment to the library this morning, and there
is no way for him to get from his apartment to the library without going past the Municipal Building.
The main conclusion of the argument is that
A. Thomas was in the vicinity of the fire this morning
B. Thomas claimed not to have seen the fire
C. Thomas saw the fire this morning
D. Thomas went directly from his apartment to the library this morning
E. Thomas went by the Municipal Building this morning
12. That long-term cigarette smoking can lead to health problems including cancer and lung disease is
a scientifically well-established fact. Contrary to what many people seem to believe, however, it is
not necessary to deny this fact in order to reject the view that tobacco companies should be held
either morally or legally responsible for the poor health of smokers. After all, excessive consumption
of candy undeniably leads to such health problems as tooth decay, but no one seriously believes that
candy eaters who get cavities should be able to sue candy manufacturers.
The main point of the argument is that
A. no one should feel it necessary to deny the scientifically well-established fact that long-term
cigarette smoking can lead to health problems
B. people who get cavities should not be able to sue candy manufacturers
C. the fact that smokers’ health problems can be caused by their smoking is not enough to justify
holding tobacco companies either legally or morally responsible for those problems
D. excessive consumption of candy will lead to health problems just as surely as long-term cigarette
smoking will
E. if candy manufacturers were held responsible for tooth decay among candy eaters then tobacco
companies should also be held responsible for health problems suffered by smokers
13. It is probably within the reach of human technology to make the climate of Mars inhabitable. It might
be several centuries before people could live there, even with breathing apparatuses, but some of
the world’s great temples and cathedrals took centuries to build. Research efforts now are justified
if there is even a chance of making another planet inhabitable. Besides, the intellectual exercise of
understanding how the Martian atmosphere might be changed could help in understanding
atmospheric changes inadvertently triggered by human activity on Earth.
The main point of the argument is that
A. it is probably technologically possible for humankind to alter the climate of Mars
B. it would take several centuries to make Mars even marginally inhabitable
C. making Mars inhabitable is an effort comparable to building a great temple or cathedral
D. research efforts aimed at discovering how to change the climate of Mars are justified
E. efforts to change the climate of Mars could facilitate understanding of the Earth’s climate
THE MOST AUTHORITATIVE GUIDE EVER WRITTEN ON GMAT READING COMPREHENSION
14. Since multinational grain companies operate so as to maximize profits, they cannot be relied to
initiate economic changes that would reform the world’s food-distribution system. Although it is
true that the actions of multinational companies sometimes do result in such economic change, this
result is incidental, arising not from the desire for reform but from the desire to maximize profits.
The maximization of profits normally depends on a stable economic environment, one that
discourages change.
The main point of the argument is that
A. the maximization of profits depends on a stable economic environment
B. when economic change accompanies business activity, that change is initiated by concern for the
profit motive
C. multinational grain companies operate so as to maximize profits
D. the world’s current food-distribution system is not in need of reform
E. multinational grain companies cannot be relied on to initiate reform of the world’s food-distribution
system
15. Kim: Some people claim that the battery-powered electric car represents a potential solution to the
problem of air pollution. But they forget that it takes electricity to recharge batteries and that most
of our electricity is generated by burning polluting fossil fuels. Increasing the number of electric cars
on the road would require building more generating facilities since current facilities are operating
at maximum capacity. So even if all of the gasoline-powered cars on the roads today were replaced
by electric cars, it would at best be an exchange of one source of fossil-fuel pollution for another.
The main point made in Kim’s argument is that
A. replacing gasoline-powered cars with battery-powered electric cars will require building more
generating facilities
B. a significant reduction in air pollution cannot be achieved unless people drive less
C. all forms of automobile transportation are equally harmful to the environment in terms of the air
pollution they produce
D. battery-powered electric cars are not a viable solution to the air-pollution problem
E. gasoline-powered cars will probably remain a common means of transportation for the foreseeable
future
THE MOST AUTHORITATIVE GUIDE EVER WRITTEN ON GMAT READING COMPREHENSION
16. A report on the likely effects of current levels of air pollutions on forest growth in North America
concluded that, since nitrogen is necessary nutrient for optimal plant growth, the nitrogen deposited
on forest soil as result of air pollution probably benefits eastern forests. However, European soil
scientists have found that in forests saturated with sulfate and nitrate, tress begin to die when the
nitrogen deposited exceeds the amount of nitrogen absorbed by the forest system. Since this finding
is likely to apply to forests everywhere, large areas of eastern forests of North America are,
undoubtedly, already being affected adversely.
Which one of the following most accurately expresses the main point of the passage?
A. The implication of the report cited is that the amount of nitrogen reaching eastern forests by way of
polluted air is approximately what those forests need for optimal growth.
B. If large areas of eastern forests were increasingly saturated with sulfate and nitrate, the capacity of
those forest systems for absorbing nitrogen would also increase.
C. The type of analysis used by European soil scientists does not necessarily apply to eastern forests
of North America.
D. The eastern forests are the only forests of North America currently affected by polluted air.
E. Contrary to the report cited, the nitrogen pollution now in the air is more likely to cause trees to die
in eastern forests than to benefit them.
17. Those who support the continued reading and performance of Shakespeare’s plays maintain that in
England appreciation for his work has always extended beyond educated elites and that ever since
Shakespeare’s own time his plays have always been known and loved by comparatively uneducated
people. Skepticism about this claim is borne out by examining early eighteen-century editions of the
plays. These books, with their fine paper and good bindings, must have been far beyond the reach
of people of ordinary means.
The main point of the argument is to
A. suggest that knowledge of Shakespeare’s play is a suitable criterion for distinguishing the educated
elite from other members of English society
B. provide evidence that at some time in the past appreciation for Shakespeare’s play was confined to
educated elites
C. prove that early eighteenth-century appreciation for Shakespeare’s works rested on aspects of the
works that are less appreciated today
D. demonstrate that since Shakespeare’s time the people who have known and loved his work have all
been members of educated elites
E. confirm the skepticism of the educated elite concerning the worth of Shakespeare’s plays
THE MOST AUTHORITATIVE GUIDE EVER WRITTEN ON GMAT READING COMPREHENSION
18. The frequently expressed view that written constitutions are inherently more liberal than unwritten
ones is false. No written constitution is more than a paper with words on it until those words are
both interpreted and applied. Properly understood, then, a constitution is the sum of those
procedures through which the power of the state is legitimately exercised and limited. Therefore,
even a written constitution becomes a liberal constitution only when it is interpreted and applied in
a liberal way.
The main point of the argument above is that
A. written constitutions are no more inherently liberal than are unwritten constitutions
B. the idea of a written constitution, properly understood, is inherently self-contradictory
C. unwritten constitutions are less subject to misinterpretation than are constitutions that have been
written down
D. liberal constitutions are extremely difficult to preserve
E. there are criteria for evaluating the interpretation and application of a constitution
19. Engineer: Some people argue that the world’s energy problems could be solved by mining the Moon
for helium-3, which could be used for fuel in fusion reactors. But this is nonsense. Even if it were
possible to mine the Moon for helium-3, the technology needed to build viable fusion reactors that
could use such fuel is at least 50 years away. If the world’s energy problems are not solved before
then, it will be too late to solve those problems.
The main point of the argument is that
A. mining the Moon for helium-3 is currently not feasible
B. fusion reactors that are now being planned are not designed to use helium-3 as fuel
C. people who advocate mining the Moon for helium-3 do not realize that fusion reactors could be
designed to use fuels other than helium-3
D. mining the Moon for helium-3 is not a possible solution to the world’s energy problems
E. if the world’s energy problems are not solved within the next 50 years, it will be too late to solve
those problems
20. Most people are indignant at the suggestion that they are not reliable authorities about their real
wants. Such self-knowledge, however, is not the easiest kind of knowledge to acquire. Indeed,
acquiring it often requires hard and even potentially risky work. To avoid such effort, people
unconsciously convince themselves that they want what society says they should want.
The main point of the argument is that
A. acquiring self-knowledge can be risky
B. knowledge of what one really wants is not as desirable as it is usually thought to be
C. people cannot really want what they should want
D. people usually avoid making difficult decisions
E. people are not necessarily reliable authorities about what they really want
THE MOST AUTHORITATIVE GUIDE EVER WRITTEN ON GMAT READING COMPREHENSION
21. When politicians resort to personal attacks, many editorialists criticize these attacks but most voters
pay them scant attention. Everyone knows such attacks will end after election day, and politicians
can be excused for mudslinging. Political commentators, however, cannot be. Political
commentators should be engaged in sustained and serious debate about ideas and policies. In such
a context personal attacks on opponents serve not to beat those opponents but to cut off the debate.
Which of the following most accurately states the main point of the argument?
A. Personal attacks on opponents serve a useful purpose for politicians.
B. Political commentators should not resort to personal attacks on their opponents.
C. Editorialists are right to criticize politicians who resort to personal attacks on their opponents.
D. The purpose of serious debate about ideas and policies is to counteract the effect of personal attacks
by politicians.
E. Voters should be concerned about the personal attacks politicians make on each other.
22. Most people who ride bicycles for pleasure do not ride until the warm weather of spring and
summer arrives. Yet it is probably more effective to advertise bicycles earlier in the year. Most
bicycles are purchased in the spring, but once shoppers are ready to shop for a bicycle, they usually
have already decided which brand and model of bicycle they will purchase. By then it is generally
too late to induce them to change their minds.
The main point of the argument is that
A. bicycle advertisements are probably more effective if they appear before the arrival of warm spring
weather
B. most bicycle purchasers decide on the brand and model of bicycle that they will buy before
beginning to shop for a bicycle
C. more bicycles are purchased in the spring than at any other time of year
D. in general, once a bicycle purchaser has decided which bicycle he or she intends to purchase, it is
difficult to bring about a change in that decision
E. spring and summer are the time of year in which bicycle riding as a leisure activity is most popular
23. The terms “sex” and “gender” are often used interchangeably. But “sex” more properly refers to
biological differences of male and female, while “gender” refers to society’s construction of a system
that identifies what is masculine and feminine. Unlike the set of characteristics defining biological
sex, the set of traits that are associated with gender does not sort people into two nonoverlapping
groups. The traits characterize people in a complex way, so that a person may have both “masculine”
and “feminine” traits.
Which one of the following statements best expresses a main point of the argument?
A. Distinctions based on gender are frequently arbitrary.
B. Gender traits are not determined at birth.
C. Masculine gender traits are highly correlated with maleness.
D. The terms “sex” and “gender” are not properly interchangeable.
E. Society rather than the individual decides what is considered proper behavior.
THE MOST AUTHORITATIVE GUIDE EVER WRITTEN ON GMAT READING COMPREHENSION
24. For years scientists have been scanning the skies in the hope of finding life on other planets. But in
spite of the ever-increasing sophistication of the equipment they employ, some of it costing
hundreds of millions of dollars, not the first shred of evidence of such life has been forthcoming. And
there is no reason to think that these scientists will be any more successful in the future, no matter
how much money is invested in the search. The dream of finding extraterrestrial life is destined to
remain a dream as science’s experience up to this point should indicate.
Which one of the following most accurately states the main point of the argument?
A. There is no reason to believe that life exists on other planets.
B. The equipment that scientists employ is not as sophisticated as it should be.
C. Scientists searching for extraterrestrial life will not find it.
D. Only if scientists had already found evidence of life on other planets would any more continued
search be justified.
E. We should not spend money on sophisticated equipment to aid in the search for extraterrestrial life.
25. The authors of a recent article examined warnings of an impending wave of extinctions of animal
species within the next 100 years. These authors say that no evidence exists to support the idea that
the rate of extinction of animal species is now accelerating. They are wrong, however. Consider only
the data on fishes: 40 species and subspecies of North American fishes have vanished in the
twentieth century, 13 between 1900 and 1950, and 27 since 1950.
Which one of the following is the main point of the argument?
A. There is evidence that the rate of extinction of animal species is accelerating.
B. The future rate of extinction of animal species cannot be determined from available evidence.
C. The rate of extinction of North American fishes is parallel to the rate of extinction of all animal
species taken together.
D. Forty species and subspecies of North American fishes have vanished in the twentieth century.
E. A substantial number of fish species are in danger of imminent extinction.
THE MOST AUTHORITATIVE GUIDE EVER WRITTEN ON GMAT READING COMPREHENSION
26. Generations of European-history students have been taught that a political assassination caused the
First World War. Without some qualification, however, this teaching is bound to mislead, since the
war would not have happened without the treaties and alliances that were already in effect and the
military force that was already amassed. These were the deeper causes of the war, whereas the
assassination was a cause only in a trivial sense. It was like the individual spark that happens to
ignite a conflagration that was, in the prevailing conditions, inevitable.
Which one of the following most accurately restates the main point of the passage?
A. The assassination did not cause the war, since the assassination was only the last in a chain of events
leading up to the war, each of which had equal claim to being called its “cause.”
B. The war was destined to happen, since the course of history up to that point could not have been
altered.
C. Though the statement that the assassination caused the war is true, the term “cause” more
fundamentally applies to the conditions that made it possible for that event to start the war.
D. If the assassination had occurred when it did but less military force had at that time been amassed,
then the war’s outbreak might have been considerably delayed or the war might not have occurred
at all.
E. Although the conditions prevailing at the time the war started made war inevitable, if the war had
not been triggered by the assassination it would not have taken the course with which students of
history are familiar.
27. Taxpayer: For the last ten years, Metro City’s bridge-maintenance budget of $1 million annually has
been a prime example of fiscal irresponsibility. In a well-run bridge program, the city would spend
$15 million a year on maintenance, which would prevent severe deterioration, thus limiting capital
expenses for needed bridge reconstruction to $10 million. However, as a result of its attempt to
economize, the city is now faced with spending $400 million over two years on emergency
reconstruction of its bridges.
The main point of the taxpayer’s argument is that Metro City
A. should have budgeted substantially more money for maintenance of its bridges
B. would have had a well-run bridge program of it had spent more money for reconstruction of its
bridges
C. is spending more than it needs to on maintenance of its bridges
D. is economizing on its bridge program to save money in case of emergencies
E. has bridges that are more expensive to maintain than they were to build
THE MOST AUTHORITATIVE GUIDE EVER WRITTEN ON GMAT READING COMPREHENSION
28. It is well known that many species adapt to their environment, but it is usually assumed that only
the most highly evolved species alter their environment in ways that aid their own survival.
However, this characteristic is actually quite common. Certain species of plankton, for example,
generate a gas that is converted in the atmosphere into particles of sulfate. These particles cause
water vapor to condense, thus forming clouds. Indeed, the formation of clouds over the ocean largely
depends on the presence of these particles. More cloud cover means more sunlight is reflected, and
so the Earth absorbs less heat. Thus, plankton cause the surface of the Earth to be cooler and this
benefits the plankton.
Of the following, which one most accurately expresses the main point of the argument?
A. The Earth would be far warmer than it is now if certain species of plankton became extinct.
B. By altering their environment in ways that improve their chances of survival, certain species of
plankton benefit the Earth as a whole.
C. Improving their own chances of survival by altering the environment is not limited to the most
highly evolved species.
D. The extent of the cloud cover over the oceans is largely determined by the quantity of plankton in
those oceans.
E. Species such as plankton alter the environment in ways that are less detrimental to the wellbeing of
other species than are the alterations to the environment made by more highly evolved species.
29. Some judges complain about statutes that specify mandatory minimum sentences for criminal
offenses. These legal restrictions, they complain, are too mechanical and prevent judges from
deciding when a given individual can or cannot be rehabilitated. But that is precisely why mandatory
minimum sentences are necessary. History amply demonstrates that when people are free to use
their own judgment, they invariably believe themselves to act wisely when in fact they are often
arbitrary and irrational. There is no reason to think that judges are an exception to this rule.
Which one of the following sentences most accurately expresses the main point of the
passage?
A. People believe that they have good judgment but never do.
B. Mandatory minimum sentences are too mechanical and reduce judicial discretion.
C. Judges should be free to exercise their own judgment.
D. Judges are often arbitrary and irrational.
E. Mandatory minimum sentences are needed to help prevent judicial arbitrariness.
THE MOST AUTHORITATIVE GUIDE EVER WRITTEN ON GMAT READING COMPREHENSION
30. The current theory about earthquakes holds that they are caused by adjoining plates of rock sliding
past each other; the plates are pressed together until powerful forces overcome the resistance. As
plausible as this may sound, at least one thing remains mysterious on this theory. The overcoming
of such resistance should create enormous amounts of heat. But so far, no increases in temperature
unrelated to weather have been detected following earthquakes.
Which one of the following most accurately expresses the main point of the argument?
A. No increases in temperature have been detected following earthquakes.
B. The current theory does not fully explain earthquake data.
C. No one will ever be sure what the true cause of earthquakes is.
D. Earthquakes produce enormous amounts of heat that have so far gone undetected.
E. Contrary to the current theory, earthquakes are not caused by adjoining plates of rock sliding past
one another.
31. Doctor: The practice of using this therapy to treat the illness cannot be adequately supported by the
claim that any therapy for treating the illness is more effective than no therapy at all. What must
also be taken into account is that this therapy is expensive and complicated.
Which one of the following most accurately expresses the main point of the doctor’s
argument?
A. The therapy is more effective than no treatment at all for the illness.
B. The therapy is more effective than other forms of treatment for the illness.
C. The therapy is more expensive and complicated than other forms of treatment for the illness.
D. The therapy should not be used to treat the illness unless it is either effective or inexpensive.
E. The therapy’s possible effectiveness in treating the illness is not sufficient justification for using it.
32. Camera manufacturers typically advertise their products by citing the resolution of their cameras’
lenses, the resolution of a lens being the degree of detail the lens is capable of reproducing in the
image it projects onto the film. Differences between cameras in this respect are irrelevant for
practical photography, however, since all modern lenses are so excellent that they project far more
detail onto the film than any photographic film is capable of reproducing in a developed image.
Which one of the following most accurately states the main point of the argument?
A. Camera manufacturers ought to concentrate on building other desirable qualities into their
cameras’ lenses, rather than concentrating only on the lenses’ resolution.
B. Apart from differences in resolution, there is no practical difference among modern cameras in the
quality of the images that they produce.
C. Advertised differences among cameras in the resolution of their lenses have no practical bearing on
the cameras’ relative quality as photographic tools.
D. In concentrating their advertising on the issue of image quality, manufacturers are making a mistake
about the interests of potential purchasers of cameras.
E. Differences among photographic films in the amount of detail they reproduce have a more
significant effect on the quality of the developed image than do differences in the resolution of
camera lenses.
THE MOST AUTHORITATIVE GUIDE EVER WRITTEN ON GMAT READING COMPREHENSION
33. It is widely believed that eating chocolate can cause acne. Indeed, many people who are susceptible
to acne report that, in their own experience, eating large amounts of chocolate is invariably followed
by an outbreak of that skin condition. However, it is likely that common wisdom has mistaken an
effect for a cause. Several recent scientific studies indicate that hormonal changes associated with
stress can cause acne and there is good evidence that people who are fond of chocolate tend to eat
more chocolate when they are under stress.
Of the following, which one most accurately expresses the main point of the argument?
A. People are mistaken who insist that whenever they eat large amounts of chocolate they invariably
suffer from an outbreak of acne.
B. The more chocolate a person eats, the more likely that person is to experience the hormonal changes
associated with stress.
C. Eating large amounts of chocolate is more likely to cause stress than it is to cause outbreaks of acne.
D. It is less likely that eating large amounts of chocolate causes acne than that both the chocolate eating
and the acne are caused by stress.
E. The more stress a person experiences, the more likely that person is to crave chocolate.
34. Last month OCF, Inc., announced what it described as a unique new product: an adjustable computer
workstation. Three days later ErgoTech unveiled an almost identical product. The two companies
claim that the similarities are coincidental and occurred because the designers independently
reached the same solution to the same problem. The similarities are too fundamental to be mere
coincidence, however. The two products not only look alike, but they also work alike. Both are oddly
shaped with identically placed control panels with the same types of controls. Both allow the same
types of adjustments and the same types of optional enhancements.
The main point of the argument is that
A. the two products have many characteristics in common
B. ErgoTech must have copied the design of its new product from OCF’s design
C. the similarities between the two products are not coincidental
D. product designers sometimes reach the same solution to a given problem without consulting each
other
E. new products that at first appear to be unique are sometimes simply variations of other products
35. Musicologist: Many critics complain of the disproportion between text and music in Handel’s da
capo arias. These texts are generally quite short and often repeated well beyond what is needed for
literal understanding. Yet such criticism is refuted by noting that repetition serves a vital function:
it frees the audience to focus on the music itself, which can speak to audiences whatever their
language.
Which one of the following sentences best expresses the main point of the musicologist’s
reasoning?
A. Handel’s da capo arias contain a disproportionate amount of music.
B. Handel’s da capo arias are superior to most in their accessibility to diverse audiences.
C. At least one frequent criticism of Handel’s da capo arias is undeserved.
D. At least some of Handel’s da capo arias contain unnecessary repetitions.
E. Most criticism of Handel’s da capo arias is unwarranted.
THE MOST AUTHORITATIVE GUIDE EVER WRITTEN ON GMAT READING COMPREHENSION
36. A famous artist once claimed that all great art imitates nature. If this claim is correct, then any music
that is great art would imitate nature. But while some music may imitate ocean waves or the
galloping of horses, for example, most great music imitates nothing at all.
Which one of the following most accurately expresses the main point of the argument?
A. Music is inferior to the other arts.
B. Either the artist's claim is incorrect, or most great music is not great art.
C. Like some great music, some great painting and sculpture may fail to imitate nature.
D. Some elements of nature cannot be represented adequately by great art.
E. Sounds that do not imitate nature are not great music.
37. Economist: Prosperity is a driving force behind increases in the release of carbon dioxide, the
main cause of global warming. As incomes rise, more people spend money on energy-consuming
devices such as cars, thereby producing more carbon dioxide. Also, in countries that experienced
deep economic recessions, there were steep drops in carbon dioxide emissions.
Which one of the following most accurately states the overall conclusion drawn in the
economist's argument?
A. Carbon dioxide is the main cause of global warming.
B. Prosperity is an important cause of increases in the release of carbon dioxide.
C. When incomes rise, more people spend money on energy-consuming devices.
D. Countries that experienced deep economic recessions also experienced steep drops in carbon
dioxide emissions.
E. When people spend money on energy-consuming devices, more carbon dioxide is produced as a
result.
38. Brianna: It would have been better to buy a tree last summer rather than this summer. The one we
bought this summer is struggling to survive this summer's drought. If we had bought one last
summer, it would have been able to survive this summer's drought, because last summer's
normal rainfall would have enabled it to develop established roots. Trees with established roots can
better withstand droughts.
Which one of the following most accurately expresses the overall conclusion drawn in
Brianna's argument?
A. It would have been better to buy a tree last summer rather than this summer.
B. The tree purchased this summer is struggling to survive this summer's drought.
C. If a tree had been purchased last summer, it would be better able to survive this summer's drought.
D. A tree purchased last summer would have established roots.
E. Trees with established roots can better withstand droughts.
THE MOST AUTHORITATIVE GUIDE EVER WRITTEN ON GMAT READING COMPREHENSION
39. Research into artificial intelligence will fail to produce truly intelligent machines unless the focus of
the discipline is radically changed. Progress has been made in creating devices of tremendous
computational sophistication, but the present focus on computational ability to the exclusion of other
abilities will produce devices only as capable of displaying true intelligence as a human being would
be who was completely devoid of emotional and other noncognitive responses.
Which one of the following most accurately expresses the main conclusion argued for above?
A. The current focus of research into artificial intelligence will produce devices no more capable of
displaying true intelligence than a person would be who lacked emotions and other noncognitive
responses.
B. If the current focus of research into artificial intelligence is not radically changed, this research will
not be able to produce machines capable of true intelligence.
C. Despite progress in creating machines of great computational sophistication, current research into
artificial intelligence has failed to fulfill its objectives.
D. The capacity to express noncognitive responses such as emotion is at least as important for true
intelligence as is computational sophistication.
E. If a machine is not capable of producing humanlike noncognitive responses, then it cannot be
regarded as truly intelligent.
40. Some claim that migratory birds have an innate homing sense that allows them to return to the same
areas year after year. However, there is little evidence to support this belief, since the studies testing
whether the accuracy of birds' migratory patterns is due to such an innate ability are inconclusive.
After all, birds may simply navigate using landmarks, just as humans do, and we do not say that
humans have an innate sense of direction simply because they find their way home time after time.
Which one of the following statements most accurately expresses the main conclusion drawn in
the argument?
A. Neither migratory birds nor humans have an innate homing sense.
B. There is as yet little reason to accept that birds have an innate homing sense.
C. Studies testing whether the accuracy of birds' migratory patterns is due to an innate homing sense
are inconclusive.
D. The ability to use landmarks to find one's way home is probably not an innate ability in birds.
E. It is as false to claim that humans have an innate sense of direction as it is to claim that birds have an
innate homing sense.
THE MOST AUTHORITATIVE GUIDE EVER WRITTEN ON GMAT READING COMPREHENSION
41. Counselor: Many people assume that personal conflicts are inevitable, but that assumption is just
not so. Personal conflicts arise primarily because people are being irrational. For instance, people
often find it easier to ascribe bad qualities to a person than good ones—even when there is more
evidence of the latter. If someone suspects that a friend is unreliable, for example, a single instance
may turn this suspicion into a feeling of certainty, whereas a belief that someone is reliable is normally
built up only after many years of personal interaction.
Which one of the following most accurately expresses the main conclusion drawn in the
argument?
A. Many people assume that personal conflicts are inevitable.
B. Even when there is more evidence of good qualities than of bad ones, people find it easier to ascribe
bad qualities than good ones.
C. It is irrational to allow a single instance to turn one's suspicion that a friend is unreliable into a
feeling of certainty.
D. Personal conflicts are not inevitable.
E. Unlike a suspicion that a friend is unreliable, a belief that someone is reliable is normally built up
only after many years of personal interaction.
42. Doctor: It would benefit public health if junk food were taxed. Not only in this country but in many
other countries as well, the excessive proportion of junk food in people's diets contributes to
many common and serious health problems. If junk food were much more expensive than
healthful food, people would be encouraged to make dietary changes that would reduce these
problems.
Which one of the following most accurately expresses the conclusion drawn in the doctor's
argument?
A. Taxing junk food would benefit public health.
B. In many countries, the excessive proportion of junk food in people's diets contributes to many
common and serious health problems.
C. If junk food were much more expensive than healthful food, people would be encouraged to make
dietary changes that would reduce many common and serious health problems.
D. Taxing junk food would encourage people to reduce the proportion of junk food in their diets.
E. Junk food should be taxed if doing so would benefit public health.
43. Scientists generally believe that no deep-sea creature can detect red light, but they need to
reassess that view. Researchers recently discovered a foot-long deep-sea creature of the genus
Erenna with bioluminescent red lights on some of its tentacles. These red lights, which are shaped
like a common food source for small, deep-sea fish, probably function as lures to attract prey.
Which one of the following most accurately expresses the overall conclusion drawn in the
argument?
A. Red lights on the tentacles of a newly discovered deep-sea creature probably function as lures.
B. Red lights on the tentacles of a newly discovered deep-sea creature are shaped like a common food
source for small, deep-sea fish.
C. A foot-long deep-sea creature of the genus Erenna has been discovered recently.
D. Scientists generally believe that deep-sea creatures cannot detect red light.
E. Scientists need to reconsider the belief that deep-sea creatures cannot detect red light.
THE MOST AUTHORITATIVE GUIDE EVER WRITTEN ON GMAT READING COMPREHENSION
44. Parents who consistently laud their children for every attempt to accomplish something, whether
successful or not, actually erode the youngsters' sense of self-esteem. Children require
commendation for their achievements, but if uniformly praised for both what they have
accomplished and what they have merely attempted, they will eventually discount all words of
commendation. In effect, such children never hear any praise at all.
Which one of the following most accurately expresses the overall conclusion of the
argument?
A. Parents should praise their children for their achievements.
B. Children whose actions are praised undeservedly eventually learn to discount all words of praise.
C. Parents need to distinguish between their own expectations for their children and what their
children are actually capable of accomplishing.
D. Children's self-esteem will suffer if their parents uniformly praise their attempts to accomplish
things regardless of their success or failure.
E. Children will develop low self-esteem if their parents do not praise them when they succeed.
45. Field studies, which have long been a staple of anthropological research, involve the researcher
living within the community being studied. However, the usefulness of field studies tends to be
overrated by anthropologists. Although most anthropologists do realize that living within the
community one is studying affects that community, they generally underestimate the extent of
such effects.
Which one of the following most accurately expresses the conclusion drawn in the
argument?
A. Anthropologists tend to overestimate the value of field studies.
B. In a field study, the researcher lives within the community being studied.
C. Field studies have been a central feature of anthropological research for a long time.
D. Most anthropologists know that when they live within a community being studied, the community
is affected at least somewhat.
E. Most anthropologists underestimate how much of an effect the researcher's presence has on a
community being studied.
THE MOST AUTHORITATIVE GUIDE EVER WRITTEN ON GMAT READING COMPREHENSION
46. Some heartburn-medication advertisements imply that unrelieved heartburn is likely to cause
esophageal cancer. This is simply false. The fact is that only about 5 percent of people with severe
heartburn have a condition called Barrett's esophagus, in which cells similar to those in the stomach's
lining develop in the lower esophagus. Only these people have an increased risk of developing cancer
because of heartburn.
Which one of the following most accurately expresses the overall conclusion drawn in the
argument?
A. Only those people with Barrett's esophagus can suffer an increased risk of developing cancer from
heartburn.
B. An increase in the risk of esophageal cancer arises from cells similar to those in the stomach's lining
developing in the lower esophagus.
C. Unrelieved heartburn is not likely to cause esophageal cancer.
D. Some heartburn-medication advertisements imply that unrelieved heartburn is likely to cause
esophageal cancer.
E. The dangers touted by heartburn-medication advertisements will affect relatively few of the people
who see those advertisements.
47. Essayist: If Earth's population continues to grow geometrically, then in a few centuries there will be ten
people for every square meter (approximately one person per square foot) of Earth's surface. Some
people have claimed that this will probably not be a problem, since humans will have learned by then
how to colonize other planets. This would, however, be a temporary solution at best: if the population
continues to double every 30 years, and if in the year 2500 half of Earth's population emigrated to Mars,
then by the year 2530 Earth would be just as crowded as it had been before the emigration.
Which one of the following most accurately expresses the conclusion drawn in the essayist's
argument?
A. If Earth's population continues to grow geometrically, then in a few centuries the population density
of Earth's surface will be ten people per square meter.
B. Due to the continuing geometric growth of Earth's population, the problem of overpopulation of
Earth will probably persist.
C. If Earth's population continues to double every 30 years, and if at some point half of the population
of Earth emigrated elsewhere, then after 30 years Earth would be just as crowded as it had been
before the emigration.
D. The population of Earth's surface will probably continue to grow geometrically even if temporary
solutions to population growth, such as colonizing other planets, are adopted.
E. Learning how to colonize other planets would, at best, be a temporary solution to the overcrowding of
Earth.
THE MOST AUTHORITATIVE GUIDE EVER WRITTEN ON GMAT READING COMPREHENSION
48. Ethicist: Robert Gillette has argued that because a thorough knowledge of genetics would enable
us to cure the over 3,000 inherited disorders that affect humanity, deciphering the human genetic code
will certainly benefit humanity despite its enormous cost. Gillette's argument is not persuasive,
however, because he fails to consider that such knowledge might ultimately harm human beings
more than it would benefit them.
Which one of the following most accurately expresses the conclusion of the ethicist's
argument?
A. Gillette's argument wrongly assumes that deciphering the genetic code will lead to cures for genetic
disorders.
B. Deciphering the genetic code might ultimately harm human beings more than benefit them.
C. Because of its possible negative consequences, genetic research should not be conducted.
D. Gillette's claim that a thorough knowledge of genetics would enable us to cure over 3,000 disorders
is overstated.
E. Gillette's argument is unconvincing because it ignores certain possible consequences of genetic
research.
49. Consumer advocate: Even if one can of fruit or vegetables weighs more than another, the heavier can
does not necessarily contain more food. Canned fruits and vegetables are typically packed in water,
which can make up more than half the total weight of the can's contents. And nothing stops
unscrupulous canning companies from including more water per can than others include.
Which one of the following most accurately expresses the conclusion drawn in the consumer
advocate's argument?
A. The heavier of two cans of fruit or vegetables does not necessarily contain more food than the lighter
of the two cans contains.
B. The weight of the water in a can of fruit or vegetables can be more than half the total weight of the
can's contents.
C. Nothing stops unscrupulous canning companies from including more water per can than others
include.
D. Some canning companies include less food in cans of a given weight than others include.
E. The heavier of two cans of fruits or vegetables may include more water than the lighter of the two
cans contains.
50. Electric stovetop burners would cause fewer fires if their highest temperature were limited to
350°C (662°F), which provides more than enough heat for efficient and effective cooking. The
lowest temperature at which cooking oil and most common fibers ignite is 387°C, and electric
burners on high go well above 700°C.
Which one of the following most accurately expresses the conclusion drawn in the argument?
A. Electric stovetop burners would cause fewer fires if their highest temperature were limited to
350°C.
B. A maximum temperature of 350°C provides more than enough heat for efficient and effective
cooking.
C. The lowest ignition temperature for cooking oil and most common fibers is 387°C.
D. Electric burners on high go well above 700°C.
E. Electric stovetop burners cause fires because they go well above 700°C when set on high.
THE MOST AUTHORITATIVE GUIDE EVER WRITTEN ON GMAT READING COMPREHENSION
51. Restaurant owner: The newspaper reporter who panned my restaurant acknowledges having no
special expertise about food and its preparation. His previous job was as a political reporter. He is
a good writer, but he is not a true restaurant critic. A newspaper would never call someone a drama
critic who had no special training in theater.
Which one of the following most accurately expresses the conclusion drawn in the restaurant
owner's argument?
A. The newspaper reporter who panned the restaurant acknowledges having no special expertise
about food and its preparation.
B. The previous job of the newspaper reporter who panned the restaurant was as a political reporter.
C. The newspaper reporter who panned the restaurant is a good writer.
D. The newspaper reporter who panned the restaurant is not a true restaurant critic.
E. A newspaper would never call someone a drama critic who had no special training in theater.
52. Politician: Some proponents of unilateral nuclear arms reduction argue that it would encourage
other countries to reduce their own nuclear arsenals, eventually leading to an international
agreement on nuclear arms reduction. Our acting on the basis of this argument would be dangerous,
because the argument ignores the countries presently on the verge of civil wars. These countries, many
of which have nuclear capability, cannot be relied upon to conform to any international military
policy.
Which one of the following most accurately expresses the conclusion of the politician's
argument?
A. Countries that are on the verge of civil wars are unlikely to agree to reduce either their nuclear arms
or their conventional weapons.
B. Unilateral nuclear arms reduction by the politician's country would encourage all countries to
reduce their nuclear arsenals.
C. Many countries cannot be relied upon to disclose the extent of their nuclear capability.
D. It is unlikely that an international agreement on nuclear disarmament will ever be achieved.
E. It is risky for the politician's country to unilaterally reduce nuclear arms in hopes of achieving an
international agreement on arms reduction.
THE MOST AUTHORITATIVE GUIDE EVER WRITTEN ON GMAT READING COMPREHENSION
53. Municipal legislator: The mayor proposes that the city accept a lighting company's gift of several
high-tech streetlights. Surely there would be no problem in accepting these despite some people's
fear that the company wants to influence the city's decision regarding park lighting contracts. The
only ulterior motive I can find is the company's desire to have its products seen by mayors who will
visit the city for an upcoming convention. In any case, favoritism in city contracts is prevented by
our competitive-bidding procedure.
Which one of the following most accurately expresses the main conclusion of the municipal
legislator's argument?
A. Some people's fear that the company wants to influence the city's decision regarding park lighting
contracts is unfounded.
B. The mayor's proposal to accept the gift of streetlights should not be considered problematic.
C. It is not appropriate that any company should have the unique opportunity to display its products
to mayors attending the upcoming convention.
D. The city's competitive-bidding procedure prevents favoritism in the dispensing of city contracts.
E. The lighting company's desire to display its products to visiting mayors is the real motivation behind
the suggested gift of streetlights.
54. While biodiversity is indispensable to the survival of life on Earth, biodiversity does not require
the survival of every currently existing species. For there to be life on Earth, various ecological
niches must be filled; many niches, however, can be filled by more than one species.
Which one of the following statements most accurately expresses the conclusion drawn in the
argument?
A. Biodiversity does not require that all existing species continue to exist.
B. There are various ecological niches that must be filled if there is to be life on Earth.
C. The survival of life on Earth depends upon biodiversity.
D. There are many ecological niches that can be filled by more than one species.
E. The species most indispensable for biodiversity fill more than one ecological niche.
THE MOST AUTHORITATIVE GUIDE EVER WRITTEN ON GMAT READING COMPREHENSION
55. Manager: This company's supply chain will develop significant weaknesses unless we make changes
to our vendor contracts now. Some will argue that this problem is so far in the future that there is no
need to address it today. But that is an irresponsible approach. Just imagine if a financial planner
offered the same counsel to a 30-year-old client: "Don't worry, Jane, retirement is 35 years away; you
don't need to save anything now." That planner would be guilty of gross malpractice.
Which one of the following most accurately expresses the overall conclusion drawn in the
manager's argument?
A. Some people argue that the supply-chain problem is so far in the future that there is no need to
address it now.
B. It would be irresponsible to postpone changes to the vendor contracts just because the supply chain
will not develop weaknesses for a long time.
C. If no changes are made to the vendor contracts, the supply chain will eventually develop significant
weaknesses.
D. In planning to meet its future obligations, a company should follow the same practices that are
appropriate for an individual who is planning for retirement.
E. Financial planners should advise their clients to save money for retirement only if retirement is
many years away.
56. Community organizations wanting to enhance support for higher education programs need to
convince the public that such programs benefit society as a whole. Taking this approach makes the
public more receptive. It is much easier, for example, to get the public to support road building, which
is seen as benefiting everyone, than it is to get them to support programs that are seen as benefiting
only a relatively small segment of society.
Which one of the following most accurately expresses the overall conclusion drawn in the
argument?
A. Community organizations seeking to encourage higher education programs must persuade the
public that these programs benefit society as a whole.
B. It is easier to get the public to support programs that are seen as benefiting everyone than it is to
get them to support programs that are seen as benefiting only a small segment of society.
C. It is easy to get the public to support road building, because road building is seen as benefiting
society as a whole.
D. Convincing the public that higher education programs will benefit society as a whole makes the
public more receptive to those programs.
E. Higher education is similar to road building in that both are beneficial to society as a whole.
THE MOST AUTHORITATIVE GUIDE EVER WRITTEN ON GMAT READING COMPREHENSION
57. Historian: During the Industrial Revolution, for the first time in history, the productivity of the
economy grew at a faster rate than the population and thus dramatically improved living standards.
An economist theorizes that this growth was made possible by the spread of values such as hard work
and thrift. But successful explanations need to be based on facts, so no one should accept this
explanation until historical evidence demonstrates that a change in values occurred prior to the
Industrial Revolution.
The overall conclusion of the historian's argument is that
A. during the Industrial Revolution the productivity of the economy grew at a faster rate than the
population
B. the fact that the productivity of the economy grew at a faster rate than the population during the
Industrial Revolution led to a dramatic improvement in living standards
C. no one should accept the economist's explanation until historical evidence demonstrates that a
change in values occurred prior to the Industrial Revolution
D. the improvement in living standards that occurred during the Industrial Revolution was not due to
the spread of a change in values
E. values such as hard work and thrift did not become widespread prior to the Industrial Revolution
58. Client: The owners of the catering company we use decided to raise their rates. They argued that the
increase was necessary to allow them to hire and train new staff to accommodate their expanding
client base. They should reconsider that decision and not raise their rates. After all, the mission of
the company is to provide low-cost gourmet catering, and this mission will be jeopardized if they
raise rates.
Which one of the following most accurately expresses the main conclusion of the client's
argument?
A. The owners of the catering company decided to raise their rates.
B. The catering company needs to increase its rates to accommodate its expanding client base.
C. The catering company's rates should not be raised.
D. The catering company's mission is to provide low-cost gourmet catering.
E. The catering company's mission will be jeopardized if its rates are increased.
59. The local news media have long heralded Clemens as an honest politician. They were proven
wrong when Clemens was caught up in a corruption scandal. This demonstrates how the local
media show too much deference toward public figures. Even the editor of the local newspaper
admitted that her reporters neglected to follow leads that might have exposed the scandal far
earlier.
Which one of the following most accurately expresses the overall conclusion drawn in the
argument?
A. Clemens has long been portrayed as an honest politician by the local news media.
B. The local news media were wrong to herald Clemens as an honest politician.
C. The local news media show too much deference toward public figures.
D. Reporters from the local newspaper neglected to follow leads that might have exposed the scandal
much earlier.
E. The local newspaper's treatment of Clemens is indicative of its treatment of public figures in general.
THE MOST AUTHORITATIVE GUIDE EVER WRITTEN ON GMAT READING COMPREHENSION
60. Johnson is on firm ground when he asserts that the early editors of Dickinson’s poetry often
distorted her intentions. Yet Johnson’s own, more faithful, text is still guilty of its own forms of
distortion. To standardize Dickinson’s often indecipherable handwritten punctuation by the use of
the dash is to render permanent a casual mode of poetic phrasing that Dickinson surely never
expected to see in print. It implies that Dickinson chose the dash as her typical mark of punctuation
when, in fact, she apparently never made any definitive choice at all.
Which of the following best summarizes the author’s main point?
A. Although Johnson is right in criticizing Dickinson’s early editors for their distortion of her work, his
own text is guilty of equally serious distortions.
B. Johnson’s use of the dash in his text of Dickinson’s poetry misleads readers about the poet’s
intentions.
C. Because Dickinson never expected her poetry to be published, virtually any attempt at editing it
must run counter to her intentions.
D. Although Johnson’s attempt to produce a more faithful text of Dickinson’s poetry is well-meaning,
his study of the material lacks sufficient thoroughness.
E. Dickinson’s editors, including Johnson, have failed to deal adequately with the problem of
deciphering Dickinson’s handwritten manuscripts.
1. (C)
You can tell that the point of the argument comes in the first clause—The pit bull is not a breed—because
everything that follows it is signaled by “as do...” and “It is like...,” both of which are reliable signals of
evidence, examples, and analogies. Both police dogs and Seeing-Eye dogs—terms that are “like” the
term “pit bull,” remember—are of the German shepherd breed, we’re told; what differentiates them is
what they do. Inferably, then, a “pit bull” could be a dog of any number of breeds, because it’s a
designation of what the dog does. (C) picks up on this analogy: If “Seeing-eye dog” and “police dog”
designate dogs according to what they do, and “pit bull” is like those terms in this respect, then a dog
can be designated a “pit bull” based only on what the dog does, irrespective of its breed.
(A) is quite possible, logically speaking, since “pit bull” is a term for a function not a breed. But it’s hardly
the point the author is leading toward. The role of the German shepherd here is to illustrate the
difference between breed and function.
(B) 180°, a pit bull is distinguished by what it does and not by appearance alone.
(D) repeats evidence. The phrase “It is like” introduces an analogy, and the last sentence (which is (D),
after all) is there to explain what that analogy is all about.
(E) Distinguishing between breeds is totally outside the author’s scope.
2. (A)
We’re asked for the main point of the passage, and so you’ll want to key in whenever the author
expresses her opinion. The first sentence merely asks if there is a necessary relationship between
negative feelings toward the elderly and plucking out gray hairs. The second sentence begins with the
Keyword “unless,” signaling an exception, followed by the strong statement of opinion “there is no
necessary connection.” The third sentence makes a distinction between attitudes that supports the
notion that there is no necessary connection. The fourth sentence begins with the Keyword
“furthermore,” signaling additional supporting evidence, and the fifth sentence begins with the
Keyword “but,” signaling a contrast to the supporting evidence in the previous sentence. Neither of
these sentences can be the main point. So where are the expressions of opinion? The second sentence
is the only qualifier. It claims that it would be premature to conclude that there is a necessary
connection between hair-plucking and having negative attitudes about the elderly, which is
paraphrased by choice (A).
(B) is a statement with which the author would agree, but the differing evaluations of these attitudes is
used to support the author’s argument that there is no necessary connection between them. (B) is a step
along the way, but not the final destination.
(C) The two claims mentioned in (C) are not linked to each other by the passage. They are both used to
support a further point, the view that there is no necessary connection between disliking aging and
disliking the elderly.
(D) brings in a new term, “fine,” without support from the stimulus. Furthermore, “being elderly” is not
part of the distinction drawn in the stimulus—that distinction is between people’s attitude toward the
elderly and their feelings about the process of getting old themselves.
(E), like (B), is a view with which the author would agree, but as discussed above, the author’s moral
arguments are used to support the main point that there is no necessary connection between these two
attitudes.
THE MOST AUTHORITATIVE GUIDE EVER WRITTEN ON GMAT READING COMPREHENSION
3. (B)
This author pounds home the main point with each sentence. First: bald eagles have the unique capacity
to foster a sympathetic attitude toward other threatened species. Second: without that sympathy, the
needs of those other species will be unmet. Third: the needs of other species can only be met by
beginning with the conservation of the bald eagle. Clearly, the author’s point is that it is first necessary
to see to the conservation of the bald eagle if we hope to be able to conserve any other threatened
species; or, as (B) has it, the conservation of the bald eagle is the necessary first step toward conserving
other threatened species.
(A) The author doesn’t say conservation efforts should concentrate on bald eagles instead of other
species; she says they should concentrate on bald eagles because that’s the only way other obscure
endangered species can be saved.
(C) is a distortion of the author’s point; it makes only a vague reference to the bald eagle (an important
symbolic species), although never mentions it by name, and also ignores entirely the conservation
needs of the more obscure endangered species.
(D) is too vague—like (C), it doesn’t even mention the bald eagle, which was clearly the focus of the
argument.
(E) While (E) is technically correct in saying that the author gives special importance to the bald eagle,
this is not the author’s main point. It is, in fact evidence for her main point: that because of the unique
importance of the bald eagle, conservation of the bald eagle is a necessary first step toward conserving
other species.
4. (C)
The author’s whole point is that Raghnall’s conclusion is based on inadequate evidence. The author’s
evidence is the alternative explanation he provides for the survey’s results; namely, that couples may
blame finances for their marriage problems when finances aren’t the real problem. He uses this
alternative explanation to make the point that Raghnall has jumped to conclusions—that she has failed
to consider other possible explanations for the survey’s results. Thus, the author believes that
Raghnall’s conclusion is inadequately justified.
(A) distorts the argument. The author’s point isn’t so much that financial problems are not a big factor
in the breakup of marriages, but rather that Raghnall cannot reasonably conclude that they are without
additional evidence.
(B) is outside the scope, a sure sign it’s not the main point. Marriage counselors have never even been
mentioned.
(D) simply restates the evidence, and not even the author’s evidence, but Raghnall’s.
(E) is a subtle misreading. The author does allude to “a number of other articles,’’ but all we know is
that these articles relied on the same survey that Raghnall’s did; we don’t know that they necessarily
drew the same conclusion that Raghnall did.
THE MOST AUTHORITATIVE GUIDE EVER WRITTEN ON GMAT READING COMPREHENSION
5. (E)
The author has found an exception to the general government policy of conserving valuable natural
resources—topsoil, which is eroding at an alarming rate. In many states, 100 years of farming has led
to the loss of half the topsoil. The federal government’s response has been inadequate; federal funding
is described as “ridiculously low,” and we’re told that some states actually devote more money to
conserving topsoil than the federal government does. The point of all this is expressed in (E): The
federal government is spending far too little on soil conservation and it should spend more.
(A) Come on, you know this thing isn’t about corn—in fact, corn is only mentioned as an example of
topsoil erosion. This choice fails to even mention the main topic, topsoil.
(B) may or may not be inferable, but it’s certainly not the main point. Like (A), this choice over-relies on
the corn element, and totally ignores the issue in the last two sentences: federal expenditures.
(C) misses the author’s criticism; instead of merely arguing that the federal government is responsible,
she says that the feds aren’t doing enough monetarily and should do more. Moreover, she never says
conservation isn’t the responsibility of the states.
(D) too is a distortion. The author doesn’t accuse the feds of making an unfair distribution between
states; her point is that the federal government is shortchanging all the states.
If you tried to summarize the whole argument in a single sentence, would it be (A), “corn isn’t cost
effective,” or (B), “six inches of topsoil isn’t enough?” Clearly not. These choices don’t even mention the
federal government. No, a summary of the argument must include the author’s criticism of the feds;
something like “the federal government isn’t spending enough on soil conservation.”
There are two main elements to this argument: topsoil and federal expenditures relating to topsoil
conservation. Any choice that omits either one of these elements must be wrong.
6. (C)
“Law without enforcement is not law.” So, to the author, a necessary condition for real law must be
“enforcement.” That’s not one of the answer choices per se, but the author goes on to define
“enforcement” not just as announcing forbidden acts, but as punishing those acts, and “without favor.”
This is what (C) is getting at: the author’s deeper definition of enforcement—that without which “real
law” doesn’t exist.
(A) lacks context. Our author would consider impartial and just use of power as a part of real law, but
not its “arbitrary and unjust” use. So (A) leaves matters too vague.
(B) Law that merely authorizes enforcement, without its being carried through, would fall short of the
author’s definition of real law.
(D) distorts the stimulus. The author sets out to specify when law is and is not “real,” and that’s
independent of whether someone “understands law’s purpose.”
(E) Sentence three makes it clear that real law “is not merely” (E)’s definition, but involves the
punishment of violators as well.
THE MOST AUTHORITATIVE GUIDE EVER WRITTEN ON GMAT READING COMPREHENSION
7. (E)
Previewing the stem, we see that we need to discern the author’s main point. When the author begins
by saying that something “is no mystery,” it’s a good bet that his point will be the mystery’s solution,
and so it is here. Those unfamiliar with the term “figurative painting” must figure out that it means
“recognizable images,” and you can do so by combining the first two sentences. The author figures that
in the 1970s, people wanted to empathize with paintings, and puzzling over abstract art didn’t afford
this opportunity; he equates “lack of realism” with abstraction in the final sentence, and asserts that
abstract art left people alienated. So, what explains the revival of figurative art in the 70’s? Evidently,
the author believes the public’s taste is the solution to the mystery, choice (E).
(A) Quite the contrary: if abstract art did reflect reality, by the author’s own logic people might have
been drawn to it. His whole point is that abstract art is divorced from reality—which may help explain
why the 70’s public resented it.
(B) is a little vague—technically, the idea of “traditionalism” is not mentioned at all. But even if we
interpret “traditional” to mean the recognizable realistic paintings the author believes that people
prefer to abstract paintings, one would have to conclude from the argument that people wanted to see
such subjects portrayed in traditional ways, not nontraditional ways as (B) would have it.
(C) The argument is about what the art-viewing public wants to see, and why; not which form of art is
more difficult to create. Talent plays no part in the argument, which is why (C) is far from the author’s
main point.
(D) The passage doesn’t lead us to believe that the public can’t understand the theoretical basis of
abstract art. It’s just that, in the 1970s, they didn’t want to. They wanted an art closer to home.
8. (C)
Determining the point of the argument hinges on one tiny phrase: “so-and-so is like arguing that the yeti
etc.” To be sure, all the rest of the evidence concerns the Himalayan yeti, but that’s all in the service of
“so-and-so,” the analogy that precedes the yeti material. The point is that just as the absence of
documentation for the yeti doesn’t prove that it doesn’t exist, the absence of documentation for
European/East Asian trade in a certain time frame doesn’t prove that it didn’t happen. It’s the historical
trade issue that the author is interested in; the yeti is just used as analogous evidence. (C) is right on.
(A) The author applies the term “scientific confirmation” to the yeti example only, and anyway it doesn’t
really apply to a situation in which one is trying to prove whether trade existed between two world
regions.
(B) The author never indicates that we need evidence from both sides; we can’t even be sure that the
author agrees with (B), and so (B) cannot be the main point of the argument.
(D) focuses on disproving the idea that the author seems interested in proving or, at least, keeping alive
(that these two areas traded during the Middle Ages). The author, by use of the analogy, tries to say the
opposite: Just because there are no records, that alone doesn’t disprove the existence of the trade.
(E) merely compares the relative amounts of evidence for the two analogous alleged phenomena, when
what the author’s trying to do is keep alive the possibility of trade between Europe and East Asia in the
face of the absence of evidence. (E) gives too much prominence to the place of the yeti example in the
argument; as discussed above, it’s there to shed light on the trade issue, and is thus not part of the main
point itself.
THE MOST AUTHORITATIVE GUIDE EVER WRITTEN ON GMAT READING COMPREHENSION
9. (E)
The author’s point of view jumps forcefully off the page from the second sentence: “Such a position
ignores the lessons of experience.” So, we know right away that the author is arguing against the
“position” in the first sentence: the refusal of some legislators to commit public money to research
lacking a clearly defined public benefit. Predictably, the author’s rationale for arguing against this
position comes next: Many unexpected, yet socially beneficial discoveries have emerged from just such
research. Antibiotics is offered as an example of a hugely beneficial discovery that was accidentally born
out of the seemingly unrelated study of molds. Where is the author going with all this? You may have
been able to pre-phrase, at least in spirit, if not word for word, the point the author is leading toward:
The legislators are wrong—specifically, they are wrong to refuse to fund research solely because that
research isn’t guaranteed to benefit the public welfare. (E) is a close paraphrase of this idea.
(A) is overly optimistic: The author’s notion that you can’t know for sure whether a given research
program will benefit society is a far cry from arguing that any public funding for scientific research will
necessarily enhance the public good.
(B) is too broad, for it necessarily leads to an absurd result: Suppose the predicted “general outcome”
of a new research effort was utter failure, something with no public use, or something that would even
harm the public? Would legislators really be willing to fund such research? According to (B), simply
knowing the outcome would result in funding, but this is far from the point the author is trying to make.
(C) While the author may possibly agree with (C), still, this isn’t the main point she’s trying to make. The
author’s evidence is not presented in order to lament what could have been the case for past discoveries,
but rather to show what the unexpected results of past research can tell us about the short-sightedness
of the legislators’ current position.
(D) The author certainly believes that legislators should fund research, but not, as (D) has it, because
this would ensure that the research is directed toward the public good. Rather, she promotes funding
because we can’t be sure what research, regardless of its direction, will ultimately increase the public
good.
Don’t confuse a statement with which the author might agree, like (C) here, with the author’s main point.
It’s very possible for a statement to be consistent with the author’s argument without it being the main
point s/he’s trying to make.
THE MOST AUTHORITATIVE GUIDE EVER WRITTEN ON GMAT READING COMPREHENSION
10. (B)
It’s especially important to have balance when reporting on civil conflicts, eschewing bias and fact
manipulation, but — this idea doesn’t justify concealing injustices in order to be fair to both sides, since
if everyone were to bend over backwards to seem impartial while ignoring real problems, the resulting
picture would portray everyone equally, in violation of reality or common sense.
So, the main point is that balanced reporting requires that one not only fairly report the views of each
party, but also be sure not to ignore basic injustices in order to seem impartial; and that’s what correct
choice (B) says. Choice (A) erroneously picks up on the conclusion — if reporters ignored injustices,
then balanced reporting would show that everyone has equal justification. This is a detail, the author’s
hypothesis of what could happen — not the main point. (C) is outside the scope, since the author doesn’t
mention the idea of life experience making us aware of specific cases of conflict involving injustice. (D)
just repeats the topic, while adding the idea of “common sense” gleaned from the last line, which gives
us no idea of the point of the argument. And (E) might have been tempting, but it goes too far beyond
the scope, since the author doesn’t mention the problem of subjectivity in reaching the ideal of balanced
reporting.
11. (C)
After stating that anyone in the vicinity of the fire would have seen it, the speaker places Thomas in the
area and states that “he must have seen it.” All statements regarding Thomas build up to this point;
nothing more, nothing less. Therefore, “he must have seen [the fire]” is this argument’s conclusion.
(A), (D), and (E) represent the logical link between the fact that Thomas went to the library and the
conclusion that Thomas must have seen the fire. As such, they’re part of the logical progression that
eventually culminates in the conclusion just stated, not the conclusion itself.
(B) Thomas’ claim that he did not see the fire is specifically what the author is trying to refute in order
to form the real conclusion: that he did.
Remember that an argument’s conclusion needn’t come at the end of the stimulus; it’s often in the
beginning, and sometimes will be found in the middle. The conclusion is the terminal point of the logical
chain. For every statement made in an argument, determine whether it represents the “final step” or
whether other statements are logical consequences of it. Statements of fact introduced into an argument
are usually pieces of evidence, rather than conclusions.
12. (C)
The author doesn’t deny that too much smoking causes health problems. The point of his argument is
that those problems are not the legal or moral responsibility of the tobacco industry. To support this
point, the author reasons by analogy that too, much candy also causes health problems, and this, too, is
public, undisputed information; yet no one looks to sue the candy makers.
(A) The author does seem to believe in open acknowledgment of the hazards of smoking, but this is only
a springboard into the main thrust of the argument.
(B), (D) The references to the candy industry are evidence, present only to support the author’s main
point, that the tobacco companies are blameless.
(E) turns the argument inside out. The purpose of the argument is to convince the reader that the
tobacco companies shouldn’t be held responsible for the health problems of their customers. Why not?
Because the candymakers aren’t. Why worry about a Never-Never Land where candy companies are
held responsible for cavities?
THE MOST AUTHORITATIVE GUIDE EVER WRITTEN ON GMAT READING COMPREHENSION
Remember, the conclusion isn’t always at the beginning or end of the passage. Here, it’s in the middle.
Recognize when an issue or idea is brought up specifically to support a larger overriding point. Choices
that focus too heavily on a side issue, such as (B), (D), and (E) here, will never be the answer to a main
point or conclusion question.
13. (D)
Research efforts into making the Martian climate inhabitable are justified. This is the main point—
everything else (the possibility of successful climate change, the success of other long-term efforts, the
benefits of climate research) is support.
(A) merely restates part of the evidence. Very well, the change is possible, but the author is trying to
persuade us that research should be begun now.
(B) states a drawback to the author's plan, a drawback the author deals with by saying that other
worthwhile projects have taken centuries to complete.
(C) concentrates on an illustration, instead of the main point, and furthermore misses the meaning of
the comparison between cathedral building and changing the Martian climate. The comparison had to
do with time, not effort.
(E) Again, is evidence supporting the main point, giving one of the reasons for beginning the research
the author recommends.
A common problem with wrong choices on “main point” questions is that they restate part of the
argument, usually some of the supporting evidence. Don't be satisfied with evidence; you want a choice
that expresses the purpose of the whole argument.
14. (E)
We get the crux of the argument in the first sentence. The conclusion, or main point, is that one cannot
depend on multinational grain companies to start the process “that would reform the world's food-
distribution system.” Why not? (that’s the question that the first sentence should have prompted in your
mind)—Because such companies operate in order to maximize profits. The rest of the stimulus simply
explains what it means for a company to be interested in maximizing profits (change is incidental,
stability is preferred).
(A) explains why the evidence leads to the conclusion (interest in profits discourages interest in change)
but is not itself the main point, or conclusion.
(B) and (C) are both evidences. (B) goes back to the second sentence, restating the author's reason for
saying that economic change caused by multinational companies is merely incidental. (C) simply
repeats the main piece of evidence from the first sentence's capsule statement of the argument:
multinational companies are unreliable because they operate to
maximize profits.
(D) introduces a scope shift, bringing up an irrelevant issue. The author says nothing about whether or
not reform of the food-distribution system is needed, only about who can be relied upon to initiate such
reform.
On main point questions, reject any choice that rehashes material that is prefaced in the stimulus by a
Keyword such as “although” or “because” or “since” (like (C) above). Such Keywords signal evidence,
not conclusion, so they never introduce the main point. The conclusion can appear in the beginning or
middle or end of the stimulus.
THE MOST AUTHORITATIVE GUIDE EVER WRITTEN ON GMAT READING COMPREHENSION
15. (D)
Kim’s conclusion or main point is nicely highlighted in the last sentence by the Keyword “So.” Replacing
gas-powered cars with electric cars would be, at best, an even trade in terms of air pollution. And
remember, the argument is framed around the claim that the battery-powered electric car would solve
the air pollution problem. Based on the last sentence, then, we can rest assured that the author disagrees
with that claim and that (D) is the main point.
(A) simply restates part of Kim’s evidence. It isn’t the conclusion.
(B) Kim doesn’t call for a reduction in driving, for one thing; for another, maybe there are ways of
reducing air pollution besides “cleaner” cars; e.g., switching from coal to hydroelectric power plants. So
(B) is way off in scope and point of view.
(C) is too extreme. Kim deals only with gasoline-powered cars and electric cars, concluding that the
latter isn’t better for air pollution than the former. To go one step further and say that Kim’s point is
that all cars create an equal amount of air pollution involves a major distortion.
(E) Maybe Kim believes that gasoline-powered cars are here to stay, maybe not. We just don’t know; in
any case, there’s no way that this can be the point she set out to prove in her argument.
All Kim says about battery-powered cars is that they aren’t an improvement over gasoline-powered
cars in terms of air pollution. The argument remains silent on other issues such as noise or fuel
economy.
16. (E)
The author takes issue with the report’s conclusion that nitrogen deposited by air pollution is good for
North America’s eastern forests. He points out that European scientists have found that when too much
nitrogen is deposited, trees begin to die. He says that this probably applies to North America’s eastern
forests also, which are already suffering from excess nitrogen. The point is stated in (E): The report’s
conclusion is wrong, and the nitrogen in air pollution, far from benefiting trees in North America’s
eastern forests, will probably cause them to die.
(A) The author disagrees with the report’s conclusion, so his “main point” can hardly be simply to state
that conclusion.
(B) The author never says anything to suggest that the capacity of forests to absorb nitrogen increases
as the forest becomes nitrate-saturated—quite the opposite!
(C) 180°—the author states that “this finding is likely to apply to forests everywhere.”
(D) makes a false comparison not based on the argument; the author never contrasts eastern North
American forests to other North American forests.
Grab those keywords! The whole passage hinges on the word “however”. Once you see that the author’s
disagreeing with the report, the answer becomes obvious. Also, note the elegance of the word
“however,” a contrast keyword, leading towards a correct answer choice that begins with the phrase
“contrary to.”
THE MOST AUTHORITATIVE GUIDE EVER WRITTEN ON GMAT READING COMPREHENSION
17.
The stimulus begins with a claim made by lovers of Shakespeare’s works, who feel that in England, at
least, Shakespeare’s work has been known and loved by all classes, not just the elite. The key phrase
comes midway: “Skepticism about this claim is borne out by. …” This indicates that the author’s main
point is to take issue with that claim. Meanwhile, the counterevidence comes from only one period and
source, the bound copies of Shakespeare from the early 1800’s that the author feels could not have been
available to ordinary folk. So (B) is correct in taking issue with the original claim, and has an
appropriately narrow scope (“at some time in the past”).
(A) The issue here is ‘Who historically has revered Shakespeare (the elite or all classes?),’ and not ‘How
can you tell the elite apart from everyone else?’
(C) What aspects? What lack of appreciation today?
(D) goes too far. The evidence, as noted above, comes only from the 1800’s. The author cannot be
leading to a blanket statement that only the educated upper class has ever known and loved
Shakespeare.
(E) It’s not the elite who are skeptical but the author, and that skepticism is of the claim made in
sentence 1, not of whether Shakespeare in fact wrote worthy plays.
18. (A)
If that “frequently expressed view … is false,” it must mean that, to the author, written constitutions are
inherently no more liberal than, and may be even less liberal than, unwritten ones. Which is all (A) is
saying. Notice that the actual “therefore” sentence, the one you expect to be the conclusion, does in fact
simply echo sentence 1: It asserts a condition (liberal interpretation and application) that’s necessary
for a written constitution to become liberal, in contrast to the view that a written constitution is per se
liberal.
(B) screws up the phrase “properly understood.” The author uses it in sentence 3 to introduce her view
of what a constitution should be properly understood to be—namely, a sum of procedures. But (B)
crazily implies that one cannot properly understand a written constitution. Huh?
(C), besides making no sense (how could it be easier to misinterpret a written document than a set of
concepts that are mental or verbal only?), falls into much the same trap as (B), by implying that the issue
is how a constitution can be understood or interpreted, when the issue is, in fact, how liberal per se, is
a written constitution?
(D) The preservation of constitutions is never mentioned or alluded to.
(E) There probably are criteria for evaluating how a constitution is to be interpreted and applied, but
the author never gets into that. What she’s presenting, in the last sentence, are criteria for deeming a
constitution as “liberal.”
Don’t be surprised when the author seems to state her conclusion twice—once at the outset and once
at the end, albeit in slightly different words. This is a common tactic in argument and debate.
19. (D)
“Some people argue blah-blah-blah.... But this is nonsense.” The sentence demands the response: How
so? What’s your evidence? This alerts us that that which “is nonsense,” that which demands evidence, is
almost certainly the argument’s main point. (D) has it just right: The argument that using the Moon’s
helium-3 will solve Earth’s energy problems is nuts because, as sentences 3 and 4 explain, we couldn’t
build the reactors that the helium-3 would fuel until too late.
THE MOST AUTHORITATIVE GUIDE EVER WRITTEN ON GMAT READING COMPREHENSION
(A) The feasibility of mining the Moon is taken off the table by the engineer’s “Even if.” He bypasses that
whole question, in order to take up his real issue, which is the feasibility of using helium-3 in time to do
the world any good.
(B) Current reactors are beside the point. The engineer is concerned with the reactors 50 years hence
which could, conceivably, be designed to use helium-3.
(C) implies that the people mentioned in line 1 are the engineer’s target, but they’re not; it’s their point
of view that he objects to. Besides, fuels other than helium-3 are never mentioned, so they could hardly
be part of the “main point.”
(E) is the gist of the engineer’s last sentence, which is evidence for the main point, not the point itself.
(If you chose (E), ask yourself: Where is the evidence supporting it? None is provided, so (E) can’t be
the main point the author is trying to make.)
Remember that the author’s conclusion can appear anywhere—at the beginning, middle, or end.
20. (E)
The author begins by telling us how “most people” feel; they’re insulted by the suggestion that they may
not understand what they really want. The rest of the stimulus is devoted to explaining why these
people are misguided to feel this way—self-knowledge is hard to come by, it’s easier just to go with the
flow and accept what society says, etc. So, the point of the argument is to show that it’s true that people
aren’t necessarily the best authorities as to what their real wants are, despite the fact that most people
don’t want to hear it.
(A) The author does say acquiring self-knowledge can be risky, but that’s not her main point; she makes
that point in order to show why many people never bother to acquire self-knowledge.
(B) The author doesn’t argue that self-knowledge is undesirable, only that it’s not as easy to acquire or
as common as people think.
(C) is an overstatement and a distortion; the author says that many people don’t know what they do
want, not that all people don’t want what they should want.
(D) is far too vague. The author never speaks of “difficult decisions” in general; her point is limited to
the fact that people often avoid doing the work necessary to gain self-knowledge.
When a stimulus begins by telling you what “most people” think, or what “accepted theory” holds, or
what “used to be believed,” usually the purpose of the stimulus is to attack that belief.
21. (B)
Campaign mudslinging—who should do it?
To draw a distinction between those who should sling mud and those who should not. The evidence
provided speaks to the latter— “Who is it that should not sling mud?”—and the answer is
“commentators,” for reasons spelled out in the last two sentences.
(A) focuses only on the broad topic, and ignores the crucial distinction drawn. In addition, the author
only says that politicians can be excused for mudslinging, not that mudslinging serves any useful
purpose.
(C) distorts the author’s reference (which is a side note anyway) to editorialists. The author takes no
stance on the right or wrong of their criticism of mudslinging.
(D) This “counteraction” business is simply an enormous distortion. To the author, serious debate is an
end in itself and not, as (D) says, mainly designed to counteract mudslinging.
THE MOST AUTHORITATIVE GUIDE EVER WRITTEN ON GMAT READING COMPREHENSION
(E) While the author feels that voters aren’t concerned about political mud, we cannot be sure that she
believes they should be. Besides which, if this is the main point, where’s the evidence for it? None
provided.
22. (A)
On the issue of when bikes should be advertised for purchase, the author takes the position that the
likely best time is prior to peak riding season, for the reasons laid out in the last two sentences: You
want to influence the consumer’s purchase decision when his/her mind is still open. (A) is practically
verbatim.
(B), (C), and (D) all cite evidence provided en route to the conclusion. The pre-shopping decision made
by consumers (B) is a prime reason as to why bike advertising ought to be done before the warm
weather and shopping season begin. Once people’s minds are made up it’s hard to change them (D)—
another good reason to get the ads in early. And of course, since most bikes are sold in the spring (C),
the smart bike seller will try to get a piece of that action by advertising when ads will do the most
amount of good—i.e., before the weather warms up.
(E) distorts sentence one, which deals with when bike riding starts, not when it’s most popular. For (E)
to be the conclusion, all the evidence would have to concern numbers of bike riders at different times
during the year. But as we know, all of the evidence concerns one central question—When is the best
time to advertise … that (A) answers.
23. (D)
People use the words “sex” and “gender” interchangeably, but they shouldn’t —there are the two missing
words—and the rest of the stimulus explains why: because the two words mean very different things.
(D) expresses the main point here.
(A) and (B) are both discardable because the author gives equal weight to the concepts of sex and
gender; he’s interested in a comparison, but these two choices mention gender only. If you need more
reasons to toss them, note that (A) distorts the author’s view—the fact that the nature of gender
definition is complex doesn’t mean that it’s “arbitrary”; while (B) offers an unwarranted inference (for
all we know, gender traits are determined at birth).
(C) tries to lump together “maleness” (a sex trait) and “gender,” but the whole point of the evidence is
that these two are different, however much the average person may think they’re one and the same.
(E) goes way off into another topic altogether, i.e., who decides whether behavior is proper or not.
24. (C)
“But” in the second sentence signals that a contrast to the first sentence is forthcoming, and that’s
exactly what we get: Despite the fact that scientists have been looking for life on other planets, they
haven’t found any evidence of it. The continuation Keyword “And” indicates that the author is
continuing this line of reasoning—there’s no reason to believe that evidence of extraterrestrial life will
be found in the future. Finally, the author expresses her opinion in the last sentence: The dream will
remain a dream. All this points to (C)—scientists will not find life on other planets.
(A) shifts the scope ever so slightly—saying that we’ll never find extraterrestrial life is not the same as
saying that such life does not exist. There may be many reasons for believing in the existence of life on
other planets, but the author here is concerned only with the likelihood of discovering it.
THE MOST AUTHORITATIVE GUIDE EVER WRITTEN ON GMAT READING COMPREHENSION
(B) Attempts to find extraterrestrial life have failed despite the sophistication of the equipment; the
problem evidently lies elsewhere. At no point does the author claim or imply that the equipment needs
to be more sophisticated, so (B) cannot be the main point here.
For (D) to be the correct answer, the author would have to explicitly lobby for calling off the search for
life on other planets. However, she never advocates ending the search; instead, she simply states that
the dream “is destined to remain a dream.”
(E) seems like a practical sentiment, but is one step beyond the scope of the argument. The cost of the
equipment is mentioned in passing, and the author does suggest via the rest of her argument that such
spending will be in vain, but she never explicitly says that the money should not be spent. “The dream
will remain a dream”—but the author leaves it to the space people to decide whether or not to continue
springing for costly equipment.
Many students have trouble eliminating choice (E), probably because (E) seems like such a practical
and logical extension of the argument. However, we’re looking for the main point, not a likely
implication of the argument. (E) contains a recommendation (“We should not”), and the wording of the
argument is simply not strong enough for us to infer that the main point of the argument is to suggest
what should or should not be done. (C) fits in better with the tone of the passage.
25. (A)
Some authors believe that the rate of extinctions is not accelerating. Moreover, they make the stronger
claim that no evidence exists which supports the opposing view. The author disagrees with this extreme
claim (“They are wrong, however”), meaning the author believes that there is at least some evidence
that the rate of extinction is accelerating. The final sentence provides such evidence, in the form of data
on North American fishes. So, the author’s main point is, as choice (A) has it, that there is some evidence
that animal species are becoming extinct at an increasing rate.
(B) undermines the author’s argument, since the author is trying to make a prediction.
(C) goes too far. The statement “They are wrong, however” is a reaction to what immediately preceded
it: the claim that there is no evidence at all. The fish data just shows that there is at least some evidence.
It isn’t supposed to be a model of the entire animal population.
(D) This data is used to support a further point about the rate of extinctions; it isn’t the main point itself.
(E) We don’t know anything about species that are in danger of becoming extinct, so (E) isn’t part of the
argument, much less the author’s main point. The author disagrees with the article writers, since she
says “They are wrong,” but picking up on the way in which the author disagrees is key to picking up the
point.
26. (C)
The topic here is the cause of World War I, and the passage quickly establishes the author’s view that
the commonly accepted idea (that an assassination caused the war) “is bound to mislead.” Mislead?
How so? We read on to learn the evidence: The war had deeper political and military causes, so the
assassination was the cause in a limited sense only; it lit the spark that set off the “gunpowder”
represented by the treaties and alliances and armies. So, the right answer, the main point of the passage,
has to involve a correction of the “misleading” view that the main cause of the war was the assassination.
Consider each choice in turn.
THE MOST AUTHORITATIVE GUIDE EVER WRITTEN ON GMAT READING COMPREHENSION
(A) denies the assassination as a cause, which contradicts the third sentence which states that it was a
cause, if only a trivial one. Also, the passage’s list of political and military causes doesn’t amount to “a
chain of events.” (A) is way off.
(B)’s fatalistic assessment distorts the text. The author believes that the outbreak of war was
“inevitable” once the various “prevailing conditions” were in place. But (B) implies that it was the
establishment of those conditions that was inevitable, unalterable; and for that we have no evidence.
(C) has it right, granting the assassination as a cause per se but arguing (as the passage does) that the
true causes run deeper. (C) essentially paraphrases the spark/conflagration metaphor with which the
passage ends.
(D)’s hypothesis is in no way supported by the passage’s information. Logically speaking, the statement
that “These factors made war inevitable” does not imply that “An absence of these factors would’ve
made war avoidable.” Another problem with (D) is that it unreasonably singles out military force for
special attention, whereas the author merely lumps in military force as one of the “prevailing
conditions” before the war.
(E), by acknowledging the huge impact of the assassination on the war, echoes the misleading teaching
with which the passage opens. The author’s whole purpose is to assign that single event a back seat in
the face of the many overwhelming and complex other causes, but (E) reverses that emphasis.
27. (A)
According to the first sentence, the city’s annual bridge maintenance budget has demonstrated “fiscal
irresponsibility” for a decade. That value judgment has to be supported by evidence, and it is—that’s
what the rest of the passage is all about. Maintenance, says the author, ideally should cost $15 million a
year, which would require another $10 million for repairs. However, in the upcoming two years alone,
the repair bill will hit a staggering $400 million. And why? Because the city was so cheap as to budget
not $15 million, but $1 million for maintenance originally. They cut corners up front and are paying
through the nose now. The “irresponsibility,” then, is that the original maintenance budget was
suicidally low; and that’s (A).
(B), for one thing, confuses necessity and sufficiency: A necessary trait of a well-run program is good
budgeting, but (B) seems to believe that fiscal responsibility would ensure a well-run program. For
another thing, (B) commits a scope shift: The author is attacking the city’s maintenance budgeting, not
its repairs spending.
(C) 180°: It’s not that the city is spending too much; it’s that they’re not budgeting enough in the first
place.
(D) is one possible explanation of why the budgeting was so chintzy. But we already know that it was
an “attempt to economize,” so (D) just repeats evidence without focusing on the author’s fundamental
point.
(E) We have no idea how much these bridges cost to build; that issue is outside the scope, so the
comparison in (E) certainly doesn’t express the main point here.
THE MOST AUTHORITATIVE GUIDE EVER WRITTEN ON GMAT READING COMPREHENSION
28. (C)
It’s well-known that species adapt to their environment, but most assume that only highly evolved
species actually change their environment to aid in survival. But we find out, this behavior is actually
quite common, as the plankton example is presented to demonstrate. The process that follows is
actually relatively unimportant, given that we’re looking for general stuff like the main point and the
argumentative technique. (See bullet point below.) The Keyword “However” indicates that the example
of plankton is meant to show that not only the highly evolved species act in the manner described. (C)
captures the main gist of the argument.
(A) and (D) deal with specifics relating to plankton behavior (namely, its effect on earth temperature
and cloud cover) and ignore the function of the plankton example altogether. The main point here must
somehow relate to the common assumption stated in the first sentence.
(B) Far from the main point, (B) is not even inferable. The plankton’s looking out for number one—the
chain it sets in motion benefits itself. Nowhere do we find out that these reactions benefit “the Earth as
a whole.” And again, even if we could infer this, (B) wouldn’t suffice as the main point. Nothing that
ignores the common assumption presented in the second half of the first sentence will do.
(E) posits an unwarranted and unsupported comparison between the effects of plankton’s alteration of
the environment and those of more highly-evolved species. But all we know is that these species, highly
developed or not, alter their environments to suit themselves. The relative well-being of other species
as influenced by these alterations is beyond the scope.
29. (E)
Some judges think that minimum sentences are too restrictive, as they prevent judges from exercising
discretion in certain matters. “But,” (note the Keyword) the author says this is precisely the problem.
So, the author must believe that judges shouldn’t have the power to make certain kinds of decisions.
The rest of the stimulus backs up the notion that judges act arbitrarily when left on their own. So
minimum sentences, which remove this discretion, are a good thing after all. (E) captures this point.
(A) is a bit extreme, since some people might have good judgment at least some of the time, even if
people don’t use their judgment in an objective way. In any case, (A) doesn’t mention sentencing, and
so cannot be the main point here.
(B) and (C) go against the author’s view. The author likes minimum sentencing rules and doesn’t trust
judges.
(D) is a view the author would agree with, but isn’t the main point. The point about arbitrary decisions
is designed to prove the author’s case regarding sentencing rules.
When you’re asked for the author’s main point, follow the chain of statements in the argument and see
where they end up. A point that supports another point cannot be the author’s main conclusion. Words
like “but” help you predict where the passage will go and keep you involved, making you less likely to
lose your concentration.
30. (B)
An earthquake theory is plausible save one mystery: There should be lots of heat given off during
earthquakes, but such increases in heat have not been detected. What’s the main point? Simply that—
the current theory doesn’t fully explain earthquake data; specifically, the absence of enormous amounts
of heat. That’s (B).
THE MOST AUTHORITATIVE GUIDE EVER WRITTEN ON GMAT READING COMPREHENSION
(A) distorts the text: There may be weather-related increases in heat following earthquakes. More
importantly, however, even if we overlook this distinction, the lack of large heat increases following
earthquakes isn’t the overall point of the argument but rather the mystery suggesting that the current
theory isn’t fully plausible.
(C) A little fatalistic, eh? (C) is way too extreme. Nothing in the passage suggests that this theory won’t
be amended to plausibly explain the causes of earthquakes one day, or that some other theory won’t do
the same. The author’s not this negative; just a little puzzled, perhaps.
(D) Because the author finds the theory “plausible,” we might infer that he believes the theory is right
and the heat evidence is simply lacking. But we can’t rule out the possibility that he feels that the lack
of heat evidence points to a flaw in the theory. Even if we go with the former, we would only be able to
say that the author would agree with (D), but not that the argument is structured to lead to this
conclusion. The main point must include something about the theory.
(E) No, the author doesn’t rule out the possibility that the theory may be correct. He simply points to
one aspect of it that to this point remains mysterious.
Beware of extreme answer choices that blow the author’s ideas way out of proportion. (C) is a perfect
example.
31. (E)
This argument requires careful paraphrasing. “Don’t use this therapy merely because doing something
is better than doing nothing. Also consider that the therapy is expensive and complicated.” In other
words, the possible benefit of the therapy must be weighed against the fact that it’s costly and
complicated. That, in a nutshell, is answer choice (E). Note the similarities between the wording of the
stimulus and (E): “…cannot be adequately supported…” in the stimulus is perfectly in line with (E)’s
“…is not sufficient justification…”
(A) First of all, the doctor is making reference to a claim that any therapy for the illness is better than
nothing at all; this fact is not itself the doctor’s idea. Furthermore, even if we attribute this notion to the
doctor, what about the whole thing about the expense and complication of the therapy? We can’t just
ignore this in formulating the doctor’s main point, so (A) is not only a distortion but also too narrow to
serve as the main point here.
(B), (C) Other forms of treatment aren’t mentioned, so nothing regarding them can be part of the main
point here.
(D) The passage gives no conditions under which the treatment should definitely be used, so this choice
is out.
32. (C)
The structure here is: factual claim; conclusion (the value judgment about the irrelevancy of
resolution differences); and evidence (signaled by “since”). That value judgment is paraphrased by (C),
with “have no practical bearing” translating into “irrelevant for practical photography.”
(A) asserts that which camera makers “ought to” do, but no recommendation is made in this argument.
By saying that the cameras have no differences apart from lens resolution, (B) is a 180.
(D) focuses (pardon the pun) on the tastes of camera consumers, but for all intents and purposes they
are outside the scope. (E) constructs an unwarranted distinction between resolution differences and
film differences; the former are the sole topic of the conclusion.
THE MOST AUTHORITATIVE GUIDE EVER WRITTEN ON GMAT READING COMPREHENSION
33. (E)
“It is widely believed that.…However,” is a sure sign that the author takes issue with the common view.
The author says that those who believe that chocolate-eating can cause acne “[mistake] an effect for a
cause,” but that doesn’t mean that those people reverse the two. She goes on to specify that both are
effects, that it’s stress that causes both chocolate-eating (which sentence 1 says people believe is a
cause) and acne. All of that is summed up by (D). To the author, chocolate-eating and acne are a
correlation, and both phenomena share an independent cause.
The author doesn’t contradict those who see acne appear after a big chocolate binge (A); indeed, since
both are caused by stress, she’d be surprised if they didn’t occur together. (B) reverses the stress
chocolate-eating relationship posited by the author. (C) creates an unwarranted comparison between
two phenomena—stress and acne—that are in a cause + effect relationship, not an opposed one. Causes
of chocolate-craving (E) are outside the scope of chocolate-eating. One can crave a foodstuff without
eating it, and vice versa. The author never mentions craving.
34. (C)
The conclusion is summed up in a single sentence: “The similarities are too fundamental to be mere
coincidence, however.” The information that comes before is factual background, and the information
that follows provides evidence for the conclusion. Choice (C) effectively restates the conclusion, and
thus describes the argument’s main point.
(A) is true, but doesn’t go far enough. The author is not just saying that the products are similar; they
are so similar that (in his or her opinion) it can’t be coincidence.
(B)—The author seems to imply that one of the companies copied the other, but there is no evidence
about which company did the copying.
(D) is a 180 wrong answer choice. The author believes that believes that it was not simply parallel
evolution, but something more which caused the similarities.
(E)—The products don’t appear to be at all unique, even at first glance.
35. C
This author first introduces a commonly held position and then refutes it. The initial viewpoint is that
Handel’s de capo arias needlessly repeat texts. The author’s response is that the repetition serves an
important purpose and allows audiences to focus on the music itself. In such a stimulus, the main point
is always the author’s response.
Answer choice (A): This does not accurately describe either the initial viewpoint (that the arias contain
a disproportionate amount of text) or the author’s response (that the amount of text is vital to
appreciating the arias).
Answer choice (B): Neither the critics nor the musicologist address other arias. Thus, we cannot infer if
the musicologist believes Handel’s de capo arias to be superior to most in any way, much less in their
accessibility to diverse audiences.
Answer choice (C): This is the correct answer choice. This is a relatively accurate restatement of the
musicologists claim that “such criticism is refuted by noting that repetition serves a vital function.”
Answer choice (D): This may be an appropriate paraphrase of the critics’ complaint regarding the
disproportion between text and music in Handel’s arias, but the question stem asks us to express the
musicologist’s main point.
THE MOST AUTHORITATIVE GUIDE EVER WRITTEN ON GMAT READING COMPREHENSION
Answer choice (E): This answer is too broad. The musicologist rejects a particular criticism, but makes
no claim regarding most criticism of the arias.
36. B
The author introduces a famous artist’s claim that all great art imitates nature:
Great art Imitate nature
According to the author, the logical implication of this claim is that any music that is great art would
also imitate nature:
Great artmusic Imitate naturemusic
CONCLUSION: The great artist was wrong to say that all great art imitates nature, OR great music is not
always great art.
We can conclude that most great music is not great art—assuming, of course, that the famous artist’s
claim is correct. It need not be: the artist’s claim is presented as an opinion, whereas the claims in the
second and the third sentences are presented as fact. Like all opinions on the test, the artist’s assertion
should be approached with caution. If it were correct, then most great music is indeed not great art. But
if it were incorrect, then great music can still be great art even if it imitates nothing. This prephrase is
consistent with answer choice (B).
Answer choice (A): This answer choice is incorrect because the author never argued that music is
inferior to other forms of art. The only logical implication of the artist’s claim is that most great music
is not great art, not that all music is inferior to all other art forms. Furthermore, this answer choice does
not acknowledge the possibility that the artist’s claim is incorrect.
Answer choice (B): This is the correct answer choice. Either the artist’s claim is incorrect, and great
music need not imitate anything to be considered great art, or his claim is correct—in which case most
great music is not great art.
Artist’s Claim: Great art ➞ Imitates Nature
Author’s evidence: Some great music does not imitate nature
It’s possible that great music is not always great art. If true, then it could be that the artist is correct, and
any great music that doesn’t imitate nature is not great art.
If great music is great art, then the artist is wrong, because great music doesn’t always imitate nature.
If the artist is right, then great music is sometimes not great art.
Answer choice (C): The author never compares music to other art forms such as painting or sculpture.
This answer choice falls entirely outside the scope of the argument.
Answer choice (D): Whether great art can represent all elements of nature cannot be determined with
the information provided in the stimulus.
Answer choice (E): This is an attractive answer choice because it can easily be concluded by the
contrapositive of the artist’s claim. Indeed, if all great art imitates nature, then sounds that
do not imitate nature cannot be great music. The question stem, however, never asked us to identify
the artist’s hypothetical conclusion given the facts presented by the author. Instead, our goal is to
express the author’s main point, who does not necessarily agree with the artist (note the conditional
qualifier “if this claim is correct…” at the beginning of the second sentence).
1. The author never mentioned great paintings or sculpture.
2. The author never said whether or not it’s possible to imitate all of nature.
3. This would be true if the artist was correct, and if great music is necessarily correct.
THE MOST AUTHORITATIVE GUIDE EVER WRITTEN ON GMAT READING COMPREHENSION
37. B
Conclusion: Prosperity is a driving force behind increases in the release of CO2 (main cause of global
warming).
Evidence: As incomes rise, more things like cars are purchased, producing more CO2. As incomes decline
due to recessions, steep drops in CO2 emissions follow.
(A) That is an aside in the first sentence, not the main claim about the connection between prosperity
and CO2 emissions.
(B) YES! First sentence.
(C) Premise
(D) Premise
(E) Premise
The economist’s first premise is that as incomes rise, more people spend money on energy-consuming
devices, such as cars, thereby producing more carbon dioxide. This premise establishes a correlation
between prosperity (i.e., when incomes rise) and carbon dioxide: as prosperity increases, carbon
dioxide increases.
Next, the economist provides another correlation between prosperity and carbon dioxide. In countries
that experienced deep economic recessions, there were steep drops in carbon dioxide emissions. So, as
prosperity decreases, carbon dioxide emissions decrease.
Based on these bookend correlations between prosperity and carbon dioxide, the economist reaches
the causal conclusion that prosperity is a driving force behind increases in the release of carbon dioxide.
This conclusion is flawed, because the evidence of correlation does not by itself establish causation.
However, the task in this Main Point question is merely to identify the conclusion, which is presented
in the first sentence of the stimulus.
Answer choice (A): This choice is not a restatement of the conclusion, because it fails to mention
prosperity and its relationship to increases in carbon dioxide. While it is true that the phrase “a main
cause of global warming” followed the conclusion, it plays the role of an appositive, further explaining
what carbon dioxide is.
Answer choice (B): This is the correct answer choice. This choice restates the conclusion, essentially
replacing the phrase “driving force” with “important cause.”
Answer choice (C): This choice restates a premise of the argument.
Answer choice (D): This choice also restates a premise of the argument.
Answer choice (E): As with choices (C) and (D), this choice restates a premise of the argument.
38. A
Conclusion: It would've been better to buy a tree last summer rather than this summer.
Evidence: This summer's tree is struggling to survive the drought. Had we bought one last year, it
would've stood a better chance against the drought because its roots would've been better established.
Here, we see we are front loaded with an opinion, and once the author proceeds to start supporting that
opinion, we can confirm our suspicion that the first sentence was the conclusion.
(A) Yes, first sentence. (B) Premise (C) Premise (D) Premise
(E) Premise
THE MOST AUTHORITATIVE GUIDE EVER WRITTEN ON GMAT READING COMPREHENSION
39. B
This author discusses the prospect for future production of truly intelligent machines, saying that
artificial intelligence research will not be able to produce such machines without making major changes
to the focus of the discipline. The author says the current focus on computational ability, without focus
on other types of abilities, has limited potential—limited in the same way that a person would be
without any emotional or any other non-cognitive response. In short, while progress has been made in
some areas, without a radical shift in focus, potential is limited.
The argument’s logical components break down as follows:
Premise: Progress has been made in creating computationally complex devices.
Premise: The present focus will create devices that are limited in the true intelligence they are capable
of.
Conclusion: Thus, research will be incapable of creating truly intelligent machines without major
changes to the focus of their discipline.
The question stem asks for the main conclusion of the author’s argument. As discussed, based on the
limits of the discipline’s current focus, the author concludes that research will be unable to produce
machines of true intelligence without a major shift in focus.
Answer choice (A): This point, that a machine with mere computational sophistication is limited in the
same way as a person without emotional or other non-cognitive responses, is a premise that the author
provides in support of the main point, it is not the main conclusion itself.
Answer choice (B): This is the correct answer choice. As prephrased above, the author’s main
conclusion is that without a major shift in the discipline’s focus, the research into artificial intelligence
will be incapable of creating truly intelligent machines.
Answer choice (C): The author does not specify the objectives of current research. If the objective was
to create devices of sophisticated computational ability, that objective has been met. If the objective was
to create “truly intelligent machines,” the objective has not been met. Regardless, this is certainly not
the conclusion of the author’s argument, so it should be ruled out of contention in response to this Main
Point question.
Answer choice (D): The author does not specify whether noncognitive responses are more or less
important to true intelligence than computational sophistication; the author’s point is that
computational complexity is not enough to achieve true intelligence. Since this point cannot be
confirmed by the information in the stimulus, it fails the Fact Test and cannot possibly be the main point
of the stimulus.
Answer choice (E): The author makes a point of saying that a machine with just computational ability
is only as close to true intelligence as a person lacking noncognitive responses, but this is not the
author’s main point, which is that unless there is a shift in focus the current research will not be able to
produce truly intelligent machines.
THE MOST AUTHORITATIVE GUIDE EVER WRITTEN ON GMAT READING COMPREHENSION
40. B
The author opens this stimulus by introducing the claim that migrating birds may have some sort of
innate directional sense that allows them to find their way back to the same areas each year. The author
refutes this assertion, based on the lack of conclusive evidence that birds have such an innate homing
ability. The author goes on to provide the counter example of people who are able to find their way
home based on the use of landmarks; despite this ability, the author points out, no one thinks that
humans have a similar innate sense of direction. Birds, the author suggests, could be using the same
method.
Re-ordered and simplified, the argument breaks down as follows:
Premise: Some claim that migratory birds have an innate homing sense.
Premise: But they may just use landmarks, much like humans.
Conclusion: Thus, there is not much support for the belief that migratory birds have a homing sense.
The question that follows asks for the author’s main conclusion. In this case, based on the lack of
conclusive evidence, the author concludes that there is little support for the argument that birds have
an innate sense of direction.
Answer choice (A): The author does not go so far as to claim that migratory birds have no innate homing
sense; rather, the author’s conclusion is that there is insufficient evidence to support this assertion.
Answer choice (B): This is the correct answer choice. As discussed, the author's main conclusion is that
there is not much support for the claim that migratory birds have an innate homing sense.
Answer choice (C): The author does not mention inconclusive studies of migratory birds; while this
choice would support the author's conclusion, it is not the main point of the argument and should thus
be eliminated from contention.
Answer choice (D): The author introduces the use of landmarks as an approach the birds might take if
they do not actually have an innate homing sense. The author does not mention whether or not such
landmark use is an innate ability itself, so this cannot be the main point of the argument.
Answer choice (E): The author does not go so far as to say that birds definitely lack an innate
homing ability; rather, the conclusion is merely that there is insufficient evidence for the claim that the
birds have such an ability.
41. D
The author introduces a common belief, only then to immediately refute it: Many people believe this to
be the case, but they are mistaken. In this case, the author is discussing the common assumption that
personal conflicts are inevitable, an assumption, the author asserts, which is mistaken, since personal
conflicts are generally just the result of human irrationality. The author gives the example of a common
rush to negative judgment, in spite of what the evidence might otherwise suggest:
people more quickly perceive someone as unreliable (often after a single instance) than as reliable (a
perception that can take years to foster). The argument breaks down more simply as follows:
Premise: Personal conflicts are primarily the result of irrationality.
Conclusion: Thus, despite common perception, personal conflicts are not actually inevitable.
The question stem asks for the author’s main conclusion, which has already been prephrased above:
Personal conflicts are not inevitable.
THE MOST AUTHORITATIVE GUIDE EVER WRITTEN ON GMAT READING COMPREHENSION
42. A
Conclusion: It would benefit public health if junk food were taxed.
Evidence: Eating too much junk food contributes to many health problems. If it were more expensive
(taxed) than healthy food, people would switch to healthier food, which would reduce these problems.
(A) Yes! First sentence.
(B) Premise
(C) Premise
(D) Never said, but kind of implied, not the first sentence.
(E) Tricky! But the conclusion was "if junk food taxed, it would ben public health", and this says the
conclusion was "if taxing would benefit public health, we should do it."
This comes down to (A) vs. (E), as long as we correctly recognized the 1st sentence as the conclusion.
At that point, we can't be duped by the "word match" in (E). We need to look for the "meaning match",
which we get with (A).
43. E
Premise: A recently discovered deep-sea creature has red lights on some of its tentacles that probably
serve to attract prey.
Conclusion: Scientists need to reassess their belief that no deep-sea creature can detect red light.
(A) This is a rewording of the premise.
(B) This is a rewording of the premise.
(C) This is part of the premise. You could consider it background information: the fact that the red lights
are shaped like a common food source for deep-sea fish, and probably serve as lures, is really the
support for the conclusion. The genus of the creature, its size, and even the fact that it has only been
discovered recently aren't the parts that provide support. In any case, this isn't the conclusion.
(D) This is what we call an opposing point. It describes the scientists' belief, which is refuted by the
conclusion of the argument.
(E) Correct. This is the conclusion.
44. D
Premise: If children receive as much praise for attempting something as for accomplishing something,
they eventually disregard all praise.
Intermediate Conclusion:
This effectively means that the children never hear any praise.
Conclusion: Parents who consistently praise children for accomplishments regardless of whether or not
they succeed will erode the children's self-esteem.
OR
Parents should not necessarily raise their children the way that experts recommend.
(A) This is background information. It's similar to the statement at the beginning of the second sentence
in the stimulus, but that isn't the conclusion.
(B) This is the premise—sort of. We might say that "undeservedly" is a slight change from what's in the
stimulus. But regardless of that, choice (B) doesn't describe the conclusion.
(C) Out of scope. Nothing in the stimulus is about parents' expectations or children's capabilities.
(D) This is correct. It is a rewording of the conclusion, but retains the same meaning.
THE MOST AUTHORITATIVE GUIDE EVER WRITTEN ON GMAT READING COMPREHENSION
(E) Out of scope. Nothing in the stimulus tells us exactly what happens if children aren't praised when
they succeed.
45. A
This author begins by outlining what is involved in anthropological “field studies,” but quickly interjects
by claiming that their usefulness is overrated. Note the use of the adverb “however,” which is commonly
used to change the direction of the argument. The last sentence provides support for the second: field
studies are not terribly useful because anthropologists underestimate the impact of living within the
communities they study. The argument is structured as follows:
Premise: Anthropologists underestimate the impact of living within the communities they study.
Conclusion: Anthropologists overestimate the usefulness of field studies.
The question that follows asks for the author’s overall conclusion. Answer choice (A) agrees with our
prephrase, and is correct. Note that the first sentence is irrelevant to the author’s conclusion, as it
merely defines field studies and concedes that they have played an important role in research. Another
concession is made in the beginning of the last sentence: most anthropologists know about the impact
they are having on the communities they study. Neither claim plays any structural role in the argument.
To identify the conclusion of this argument it's helpful to recognize that the word "however" represents
a pivot in the argument away from an opposing point and towards the author's point. At the pivot the
author states the argument's conclusion and then goes on to offer qualified evidence. The word
"although" qualifies the support for the argument's main point.
Answer choice (A): This is the correct answer choice, because the second sentence of the stimulus is
directly supported by the third. Do not be misled by the fact that certain words, such as “usefulness”
and “value,” or “overrate” and “overestimate,” are being used interchangeably. Correct answers to Main
Point questions frequently avoid restating the conclusion verbatim.
Answer choice (B): The author does say that in a field study, the researcher lives within the community
being studied. However, as discussed, this is not the conclusion of the argument, so this choice should
be ruled out of contention.
Answer choice (C): It is true that field studies have long been a staple of anthropological research, but
the argument as a whole is not intended to support this claim.
Answer choice (D): It is true that most anthropologists know about the impact they are having on the
communities they study, but the argument is not intended to support this observation, and no evidence
is given to substantiate it.
Answer choice (E): The author clearly states that anthropologists underestimate the impact they are
having on the communities they study, but this is not the conclusion of the argument. Rather, it is the
only premise for that conclusion, because it provides support for the claim that the value of field studies
is overrated.
46. C
In this case, the author began, “Some heartburn-medication advertisements imply...” This introduction
foreshadows that the author’s conclusion will disagree with the implication made by the
advertisements, and the author concludes that “This (i.e., the implication by some heartburn-
medication advertisements) is simply false.”
THE MOST AUTHORITATIVE GUIDE EVER WRITTEN ON GMAT READING COMPREHENSION
“This is simply false” does not provide support to any other piece of the argument, but is itself supported
by the third and fourth sentences of the stimulus. Because it is supported by other parts of the stimulus
and does not support any other part, the statement that “this is simply false” is the conclusion.
Answer choice (A): This statement was a premise offered in support of the conclusion.
Answer choice (B): While this causal statement makes sense in the context of the stimulus, the stimulus
did not include a causal statement. Rather than causality, the stimulus presented a conditional
relationship, in which only the people who have cells similar to those in the stomach’s lining developing
in their lower esophagus have an increased risk of developing cancer because of heartburn.
Answer choice (C): This is the correct answer choice. While the information in this answer choice is
worded from a negative perspective, such that “unrelieved heartburn is not likely to cause esophageal
cancer,” it is a restatement of the conclusion, that it is false to imply that unrelieved heartburn is likely
to cause esophageal cancer.
Answer choice (D): This statement was a claim presented by the argument to introduce the topic and
concerning which the author disagreed.
Answer choice (E): While being close to a statement presented in the stimulus, that “only 5 percent of
people with severe heartburn have a condition called Barrett’s esophagus,” it is not identical to that
statement. The difference between the statements is that the information in this answer choice refers
to a small percentage of the people who see the advertisement, as opposed to a small percentage of
people with severe heartburn.
The argument follows a common pattern: opposing point, conclusion, premises. The conclusion here is
that the claim made in the first sentence is false; in other words, unrelieved heartburn is not likely to
cause esophageal cancer.
47. E
In the second sentence, the author uses the “some people say...” technique, providing the viewpoint of
someone else, which the author will conclude is incorrect. Some people say that by a few centuries from
now, humans will have learned to colonize other planets and, because of this ability, the geometric
growth of the population will not be a problem.
As we anticipated, the author disagrees with this position, and concludes that colonizing other planets
would be a temporary solution at best. The author points out that if the population doubles every 30
years, then if half the Earth’s population emigrated to Mars in 2500, a date within the “few centuries”
timeline provided in the first sentence, the population would be back to the 2500 level by 2530. Because
of this basic math, the author concludes colonization of other planets would only be a temporary
solution.
Answer choice (A): This statement was a claim made to provide the factual background for the
argument. It was neither a premise nor a conclusion.
Answer choice (B): The author does not make a prediction about whether overpopulation will persist.
Instead, the author concludes the planetary emigration idea will not be a permanent solution.
Answer choice (C): This statement was a premise offered in support of the conclusion.
Answer choice (D): As with answer choice (B), this choice is incorrect, because the author does not make
a prediction about what will occur.
Answer choice (E): This is the correct answer choice, because it is a restatement of the author’s
conclusion.
THE MOST AUTHORITATIVE GUIDE EVER WRITTEN ON GMAT READING COMPREHENSION
48. E
The ethicist's argument starts with an opposing point (Gillette's argument), followed by the ethicist's
conclusion (Gillett's argument is not persuasive), followed by support for the ethicist's conclusion.
(A) This misrepresents the ethicist's conclusion. The ethicist doesn't disagree with Gillette's claim about
curing genetic disorders.
(B) This is the ethicist's premise. It provides support for the ethicist's conclusion that Gillette's
argument is not persuasive.
(C) This also misrepresents the ethicists conclusion. It's definitely appealing at first glance, but the
ethicist's actual conclusion is not that genetic research should not be conducted. The ethicist's
conclusion is specifically about Gillette's argument for deciphering the human genome, and specifically
that this one argument is not persuasive.
(D) This misrepresents the ethicist's conclusion in a similar way to (A). The ethicist doesn't disagree
with Gillette's claim about curing genetic disorders.
(E) This is the correct answer. It properly states the ethicist's precise conclusion, which is that Gillette's
argument is unconvincing.
49. A
The author concludes that “even if one can of fruit or vegetables weighs more than another, the heavier
can does not necessarily contain more food.” Without conclusion indicators to rely on, you must identify
this statement as the conclusion based on its relationship to the other statements in the stimulus. Why
is it that the heavier can does not necessarily contain more food? Because canned fruits and vegetables
are packed in water. More than half of the total can weight can be comprised of water, rather than the
fruits or vegetables. Moreover, it is possible for “unscrupulous” canning companies to include even
more water per can.
“canned fruits and vegetables are typically packed in water, which can make up more than half the total
weight of the can’s contents”
because
“even if one can of fruit or vegetables weighs more than another, the heavier can does not necessarily
contain more food”
Do you notice how this configuration does not sound right? Hopefully, you do. The fact that the heavier
can does not necessarily contain more food does not provide evidence for a conclusion that canned
fruits and vegetables are typically packed in water.
Answer choice (A): This is the correct answer choice, because it provides a restatement of the first
sentence in the stimulus, which, as discussed above, was the conclusion.
Answer choice (B): This statement is true according to the stimulus. However, it is incorrect because it
was a premise in the advocate’s argument.
Answer choice (C): As with answer choice (B), this statement, while true according to the stimulus, was
a premise of the argument, not the main point.
Answer choice (D): While this statement is possible given the advocate’s argument, it is not something
that must be true. And, even if it were provable from the stimulus, this statement does not capture the
main point.
THE MOST AUTHORITATIVE GUIDE EVER WRITTEN ON GMAT READING COMPREHENSION
(D) seems close, but doesn't exactly match the conclusion. The consumer advocate's conclusion
specifically compares one can of food that weighs more than another, while "cans of a given weight" in
choice (D) implies a comparison between cans that weigh the same. Also, choice (D) specifically
compares cans from different companies, while the consumer advocate's conclusion doesn't necessarily
involve cans from different companies.
Answer choice (E): This statement is a valid inference drawn from the stimulus. However, it is incorrect
because it does not reflect the advocate’s main point.
(E) This is also not an exact match with the conclusion. We have to be careful, because it seems close,
but the advocate's conclusion is specifically about the amount of food in the heavier can, not the amount
of water. Wouldn't a giant, restaurant-sized can of peaches usually contain more water than a small,
snack-sized one? The consumer advocate might not be concerned about that as long as the giant can
also contains more peaches. The problem is that the larger can could be almost all water, and not contain
more peaches. That's the consumer advocate's point.
50. A
Here, the stimulus begins with the statement that if the temperature of electric stovetop burners was
capped at 350° C, then the burners would cause fewer fires. The reason given in support of this
statement, which is the argument’s conclusion, is that cooking oil and most common fibers do not ignite
at less than 387° C. This is a simply structured one-premise, one-conclusion argument.
In addition to the premise and conclusion, the argument contains two other pieces of information: a
temperature of 350° C provides enough heat for efficient and effective cooking; and electric burners can
reach temperatures well above 700° C. These facts provide context, but are neither premises nor
conclusions of the argument. Respectively, they tell us that the suggested change is practical and
necessary.
Conclusion: “electric stovetop burners would cause fewer fires if their highest temperature was limited
to 350° C (662° F).
Answer choice (A): This is the correct answer choice, and is a nearly perfect restatement of the
conclusion. All that is missing is the parenthetical statement of the proposed temperature limit in
Fahrenheit.
Answer choice (B): This choice restates one of the context-providing statements from the stimulus, and
does not restate the conclusion. This is evidence that supports the conclusion indirectly. If 350 degrees
provides enough heat, then it’s realistic to limit electric stove burners to that temperature. This helps
show the conclusion is not describing a ridiculous situation.
Answer choice (C): This answer choice describes a premise of the argument. This is evidence that
supports the conclusion that a 350-degree limit would lead to fewer fires.
Answer choice (D): As with answer choice (A), this choice restates a fact offered in the stimulus to
provide context to the argument, rather than restating the conclusion. This is evidence that supports
the conclusion. Current stove max temperatures are more than high enough to start fires.
Answer choice (E): While this statement of causality was implied by the argument, it was not
explicitly stated anywhere. The argument actually doesn’t say this answer. It’s just implied that stoves
cause fires because they go above 700 degrees Celsius. That is not the main conclusion. That’s
something that supports the idea that we’d have fewer fires if stove heat was limited.
THE MOST AUTHORITATIVE GUIDE EVER WRITTEN ON GMAT READING COMPREHENSION
51. D
The restaurant owner in this stimulus takes issue with the qualifications of a critic who wrote a negative
review of his restaurant in the newspaper. Apparently, the critic, whose last job was as a political
reporter, acknowledges that he is not an expert in the fields of food or food preparation. So, while
conceding that the critic is a good writer, the owner concludes that the “critic” is not a true restaurant
critic. By analogy, the owner points out that a newspaper would not label someone with no special
training in theater a “drama critic.”
CONCLUSION: The newspaper reporter is not a true restaurant critic.
REASONING: The reporter has said he has no expertise about food or food preparation. His past
experience was as a political reporter.
ANALYSIS: The restaurant owner is annoyed at the negative review. So, she’s trying to show that the
review shouldn’t be listened to. She wants to show us that the newspaper reporter isn’t really a good
critic. Notice the word “but”. Words like “but”, “however” etc. almost always introduce a conclusion.
The real point of the argument, of course, is that we should not pay attention to the negative review.
But that main point is merely implied. The fact about the reporter not being a true critic is the explicitly
stated conclusion.
You may have had some difficulty isolating the conclusion because the stimulus contains neither a
premise nor a conclusion indicator. In cases such as this, focus on the relationship between the
statements in the stimulus and the word choice. Here, the phrase “true restaurant critic” implies that
the owner is making a judgment regarding the critic’s qualifications. The other statements in the
stimulus provide facts concerning the critic’s qualifications. By taking that context into consideration,
we can understand that the owner’s conclusion is that the critic is not a true restaurant critic.
Answer choice (A): This answer choice restates one of the argument’s premises.
Answer choice (B): This too is a restatement of a premise supporting the owner’s conclusion.
Answer choice (C): Here, the answer choice describes the concession made by the owner regarding the
general writing ability of the critic. The fact that the reporter is a good writer is evidence in favor of the
reporter. The restaurant owner hopes to sound more persuasive by acknowledging a positive (but
mostly irrelevant) fact about the reporter.
Answer choice (D): This is the correct answer choice. The word “but” indicates that this is the
author’s opinion. And this answers the question “why is the owner telling us this?”. The entire argument
was aimed at convincing us that the reporter is not a true food critic (and therefore we shouldn’t pay
attention to the negative review).
Answer choice (E): This answer choice contains the author’s analogy invoking the case of a
purported drama critic with no expertise in theater. This is an analogy that supports the conclusion that
someone without food training is not a food critic.
THE MOST AUTHORITATIVE GUIDE EVER WRITTEN ON GMAT READING COMPREHENSION
52. E
“some people say…”
The politician disagrees with this view, concluding that unilateral nuclear arms reduction would be
dangerous. In support of this view, the politician says that it does not consider countries that are about
to enter into civil wars, many of which have nuclear capability, and which cannot be trusted to abide by
an international arms reduction agreement.
ANALYSIS: The argument has the following structure:
1. Opposing opinion (“some proponents”)
2. Conclusion (“this would be dangerous”)
3. First premise (“because….”)
4. Second premise (“These countries cannot be relied upon….”)
The word “because” indicates the conclusion. In an argument, the words before “because” are usually
the conclusion, and what comes after “because” is evidence.
Also, usually when an answer says “some proponents argue”, the author’s conclusion will be “those
proponents are wrong”.
Conclusion: reducing the country’s nuclear arsenal unilaterally would be dangerous.
Answer choice (A): The politician did reference the problem of nations on the verge of civil war, but that
statement was a premise offered in support of the conclusion, not the conclusion itself. This is evidence.
The conclusion is that, because of this, we shouldn’t get rid of our nuclear weapons.
Answer choice (B): This answer choice restates the view of unilateral nuclear arms reduction
proponents, with which the politician disagrees. The author disagrees with this idea. Countries in civil
war wouldn’t be influenced by international agreements.
Answer choice (C): While this seems to be a rational real-world consideration, the politician did not
raise it. The author doesn’t say this. This answer is playing on your outside knowledge. In the real world,
many countries hide the extent of their nuclear programs. But while this is true, it has nothing to do
with the argument!
Answer choice (D): It appears that the politician would agree with this sentiment. However, this concern
did not appear in the stimulus.
The author didn’t say we couldn’t make an agreement. We might succeed in making an international
agreement. But it would be dangerous to rely on it, since countries in civil war can’t be expected to obey
the agreement.
Answer choice (E): This is the correct answer choice, because it restates the politician’s
conclusion, as described above. This is it. It would be risky because many countries are near civil war
and couldn’t be relied upon.
THE MOST AUTHORITATIVE GUIDE EVER WRITTEN ON GMAT READING COMPREHENSION
53. B
The municipal legislator discusses the mayor’s proposal to accept a gift of several high-tech streetlights
from a lighting company. The legislator does not think that there is any problem with accepting the
streetlights as a gift, even though there are those who are afraid that the reason the company is giving
the gift is to influence the city’s decision concerning park lighting contracts. In support of this view, the
legislator offers two reasons. First, the legislator considers the company’s only potential ulterior motive
to be gaining visibility for its products. Second, even if there is some other ulterior motive, the city’s
competitive-bidding procedure precludes favoritism from influencing city contracts. For these two
reasons, the legislator concludes that there is no problem with the city accepting the lighting company’s
gift.
Conclusion: there is no problem with the city accepting the lighting company’s gift.
Answer choice (A): This answer choice is incorrect because the legislator does not conclude that the
company has no desire to influence the city’s decision regarding lighting contracts. Rather, the legislator
thinks that there is no problem with accepting the gift, regardless of the company’s motives in offering
it.
Answer choice (B): This is the correct answer choice because it contains a restatement of the
legislator’s main point, as discussed above.
Answer choice (C): The legislator did not discuss the propriety of a company having a unique
opportunity to display its products, so this answer choice is inconsistent with the stimulus.
Answer choice (D): Here, the answer choice restates a premise of the argument.
Answer choice (E): As with answer choice (D), this answer choice restates a premise of the legislator’s
argument.
54. A
The "While" at the beginning suggests a concession being made before the author shifts, and we see that
in the second clause. That's a statement that needs back up, so it seems a good candidate for conclusion.
The following information seems to cut against it, but the author shifts back again with a "however".
Focus on the second half of the first statement and the "however" statement, using the "Therefore" test
to see which supports which. In this case, the final statement (multiple species fill the same role)
supports the second half of the first sentence (that some species can die off while maintaining B). The
answer should reflect the statement that we don't need all these species.
CONCLUSION: Biodiversity doesn’t require the survival of all current species.
REASONING: Biodiversity requires that all niches be filled, but often a niche can be filled by more than
one species.
ANALYSIS: The word “while” is a conclusion indicator. Any statement of opinion, uncertainty or
qualification tends to be the conclusion.
Another way of finding the conclusion is asking yourself: why are they telling me this? In this case, the
author is trying to convince us that not all species are essential to biodiversity.
A third way to find the conclusion is to ask which statements support other statements, and which are
supported. In this case, the second sentence is clearly supporting a claim about biodiversity in general.
In particular, the fact that many niches can be filled by multiple species is support for the conclusion
that biodiversity does not require that all species survive.
A. CORRECT. The correct answer is almost word-for-word the statement we identified.
B. This is context that provides support for the conclusion.
THE MOST AUTHORITATIVE GUIDE EVER WRITTEN ON GMAT READING COMPREHENSION
55. B
In this stimulus, the manager’s argument concerning problems with the company’s supply chain is
structurally complicated. It begins with a conditional statement that if the company does not change its
vendor contracts now, its supply chain will develop significant weaknesses. This statement is offered as
a fact that sets the stage for the argument, and is not part of the reasoning that leads to the manager’s
conclusion.
“some people say...” … the manager immediately concludes that their position is wrong and labels their
position “irresponsible.” In support of this conclusion, the manager argues by analogy, pointing out that
it would be negligent for a financial planner to tell a 30-year-old client not to worry about retirement
investing since the client would not retire for another 35 years.
Conclusion: it is irresponsible not to address the vendor contract problems now just because the supply
chain problems will not emerge until later.
Answer choice (A): This answer choice restates the anticipated view of others that the manager
concludes is irresponsible.
Answer choice (B): This is the correct answer choice because it restates the manager’s conclusion, as
described above.
It brings "irresponsible" in from the text of the argument, which should have led you to spend more
time with this answer. While it is much more verbose than the actual conclusion, the stimulus refers
back to the initial approach ("that is an irresponsible approach"), and this answer just rephrases "that
approach" to what it actually is.
Answer choice (C): Here, the answer choice restates the factual assertion made at the beginning of the
stimulus to establish that there is a problem with the vendor supply contracts.
Answer choice (D): This answer choice is inconsistent with the stimulus. Although the manager made
an analogy between the company’s responsibilities and those of a financial planner, the manager did
not say that the company should follow the same practices as the financial planner.
Answer choice (E): As with answer choice (D), this answer is inconsistent with the stimulus, which did
not say that financial planners should advise their clients to save for retirement only if retirement is far
in the future. Rather, the manager’s analogy indicated that financial planners should advise retirement
savings even when retirement is far in the future. Even if this answer choice were an accurate
restatement of what occurred in the stimulus it would still be incorrect, because it relates to the
manager’s support for the conclusion, rather than to the conclusion itself.
56. A
If a community organization wants to accomplish this goal, they need to convince the public of the
benefit. Why? That approach will convince more people. This is then backed up with an example.
The final sentence is explicitly stated to be an example of the preceding sentence, so it's a premise
backing up a conclusion (that conclusion being the second sentence). At this point, it's a matter of
determining if the first sentence is a premise of that conclusion, or if the second sentence is an
intermediate conclusion.
THE MOST AUTHORITATIVE GUIDE EVER WRITTEN ON GMAT READING COMPREHENSION
In this case, the second sentence is the reason why the author believes the first sentence (Why must
community orgs do this? Because it'll be more effective.). As the second sentence is the reason to do
something, it is support for that action. Therefore, the second sentence is an i. conclusion, with the first
sentence being the main conclusion.
What this author wants us to believe, and what they set out to prove, is the first sentence, about what
community organizations need to do. The rest of the stimulus is designed to support that claim.
Rearranging the stimulus can help clarify this, as follows:
Premise: Convincing the public that they will benefit from a program makes them more receptive to it.
Premise/Example: It's easier to get support for roads, which benefit everyone, than for things that do
not.
Conclusion: If you want support for education, you need to convince the public it is beneficial to
everyone. OR Community organizations need to convince the public that higher education is good for
all of society.
REASONING: It’s easier to get the public to support-programs which benefit everyone, like road
building.
ANALYSIS: In this case, the author is giving advice to community groups that want to get funding for
higher education. The author is telling them how to make their case. The author says that community
groups should argue that higher education benefits everyone.
The second sentence, that the public is receptive to widely beneficial programs, is not the conclusion.
This is evidence supporting the idea that the approach in the first sentence will be effective.
The words “for example” are a structural indicator. They show that what comes after “for example” is a
premise supporting the conclusion.
Note: You may think that higher education obviously benefits everybody. Why would anyone have to
argue for that? The main reason you believe university benefits everyone is because you went to
university. The American university system asserts that education benefits everyone in order to justify
its existence. That claim is not a universal truth.
(A) Bingo. This is a paraphrase of the first sentence.
(B) Wrong function (premise). This answer is the example, stripped of the specific application (roads).
(C) Wrong function (premise/example).
This answer misstates the premise about roads. Nobody said it was easy to get support for roads - they
just said it was easier, relative to programs that do not appear to benefit everyone.
(D) Wrong function. This is the intermediate conclusion with the topic of the argument written in.
This answer is essentially an implied premise of the argument, using much of the same language as
found in the conclusion, but it misses the mark by not expressing the idea that community organizations
need to take this approach if they want to achieve their goal. This answer is about what will happen,
rather than what needs to happen.
(E) Unstated in the argument. This answer is actually an assumption of the argument.
Answer choice (E): The author never suggested that education IS beneficial to everyone, but only that
community organizations supporting education need to convince people of this idea. Again, the
conclusion is not about what will happen or what is true, but is instead about what those groups need
to do if they want to achieve their goal. The author didn’t say that higher education benefits us all. They
said community groups should say that it does, in order to get funding. This is a subtle distinction, but
it’s important to notice these.
THE MOST AUTHORITATIVE GUIDE EVER WRITTEN ON GMAT READING COMPREHENSION
57. C
A historian tells us about an historical fact and then shares the view of an economist who offered an
explanation for that fact. This follows a very common pattern of "some people say something, but they
are wrong, and here's why," with the exception that the historian doesn't directly state that the
economist is wrong. Instead, he says that some evidence is required before that explanation can be
accepted. This is the main point of the argument, and our task is to find the answer that restates that
conclusion.
CONCLUSION: We shouldn’t accept the “thrift and hard work” hypothesis for the success of the
industrial revolution. We need historical evidence first.
REASONING: Successful explanations need to be based on facts.
ANALYSIS: Certain words indicate conclusions and premises. In the final sentence, “but” indicates that
the author disagrees with what was said before. “So” introduces their conclusion.
For instance: John says we should buy the car. But it’s too expensive, so we should look for a better deal.
But = contrasting evidence.
So = conclusion.
Answer choice (A): This is a true statement, but only restates the historical fact and not the author's
conclusion. The productivity growth of the economy is what we’re trying to explain. It’s not the
conclusion. The point of the argument is that we don’t yet know how to explain this growth.
Answer choice (B): Like answer A, this is true according to the stimulus, but still just the background
facts and not the conclusion of the argument.
This is just context. The fact that productivity growth raised living standards explains why we care
about the industrial revolution in the first place.
Answer choice (C): This is the correct answer choice. This answer correctly restates the conclusion
that more evidence is required before we can accept the position of the economist mentioned in the
passage.
Answer choice (D): At no point in the stimulus did the author claim that the economist was wrong. This
would have been a good answer if the stimulus had followed the usual pattern of a "some people say
something, but they are wrong" argument, but it did not. Instead, the structure was "some people say
something, but more evidence is needed."
The historian didn’t say that values were not the cause. The historian said that we don’t know if values
were the cause. It’s possible they were – we just lack evidence.
Answer choice (E): The historian does not claim that there was no change in values prior to the
Industrial Revolution, but only that more evidence was needed before one could make a causal claim
about such changes and growth in productivity.
This is just like D. The historian didn’t say there was no shift in values. They said we don’t know if there
was a shift in values. It’s possible that there was one.
THE MOST AUTHORITATIVE GUIDE EVER WRITTEN ON GMAT READING COMPREHENSION
58. C
CONCLUSION: The catering company shouldn’t raise their rates.
REASONING: The catering company’s mission is to provide low-cost gourmet catering.
ANALYSIS: There are two major clues that indicate the conclusion here:
• The word “should”. A sentence with the word “should” tends to be the conclusion.
• After all indicates that the sentence is evidence that supports the previous sentence.
Both of these tell us that the third sentence is the conclusion: the gourmet catering company shouldn’t
raise rates.
The sentence about the company training and hiring new staff isn’t structurally relevant to the
argument. It’s just context explaining why the catering company is raising rates.
A. This is just a fact. The client is arguing that the catering company should reconsider.
B. This is just the reason that the catering company wants to raise rates. The client’s conclusion is
that the catering company should reconsider that decision.
C. CORRECT. See the analysis above. You might have hesitated to choose this answer since it didn’t
mention reconsidering. But since the conclusion said “reconsider and not raise their rates.” It’s
implied that the author will only be happy if the reconsideration results in not changing rates.
D. This is just a fact supporting the conclusion. Because the company mission is to provide low-cost
catering, the client argues the company shouldn’t raise rates.
E. This is a fact supporting the conclusion that the catering company therefore shouldn’t raise its
rates.
59. C
Media darling Clemens was recently shown to be corrupt. This demonstrates that the media was too
deferential to public figures, which even the local newspaper’s editor admits.
"This demonstrates" is the key phrase here, indicating a conclusion preceded by premises. The first two
statements are evidence for the third sentence, so that's a conclusion. The admission by the editor is
further evidence of the deference afforded public figures, so the conclusion is that local media is too
deferential.
CONCLUSION: The Clemens scandal is an example of how the all, media is too nice to public figures.
REASONING: The media thought Clemens was honest. As such they didn’t bother investigating him. It
turns out that Clemens was dishonest.
ANALYSIS: The gist of the argument is that the media is too deferential to public figures like Clemens.
Clemens isn’t the point – he is just an example demonstrating this point about the media. Note that the
third sentence says “This [Clemens’ corruption] demonstrates”. Media failure is the real point of the
argument.
Ideally, you should be able to figure that out just from reading it. But if you’re still having trouble, I’ve
identified some words below that indicate conclusions and evidence.
“This demonstrates” indicates that the sentence is a conclusion and the previous sentence supports it.
“Even…. admitted” indicates that the sentence is evidence, supporting the conclusion in the previous
sentence. Here, “even” shows that someone we would expect to object (the newspaper editor) agrees
with the author.
(A) Background/counterpoint.
This fact supports the conclusion. Clemens was not honest, so the fact that the media portrayed him as
honest shows the media is too deferential.
THE MOST AUTHORITATIVE GUIDE EVER WRITTEN ON GMAT READING COMPREHENSION
60. B
(A) Equally Serious?? The passage only says Johnson's text is still guilty of its own forms of distortion
.... discard. The author never states that Johnson’s texts equally distort Dickinson’s work.
(B) Johnson’s use of the dash in his text of Dickinson’s poetry misleads readers about the poet’s
intentions. Exactly
(C) Because Dickinson never expected her poetry to be published, virtually any attempt at editing it
must run counter to her intentions.
The text is more focused on the limitations of Johnson’s texts than on Dickinson’s intentions or the
possibility to adequately edit his work.
That's going too far. We don't have evidence to believe Dickinson never expected her poetry to be
published. Irrelevant.
(D) Although Johnson’s attempt to produce a more faithful text of Dickinson’s poetry is well-meaning,
his study of the material lacks sufficient thoroughness.
Out of scope and irrelevant
The author does not say anything about the thoroughness of Johnson’s study of the material.
(E) Dickinson’s editors, including Johnson, have failed to deal adequately with the problem of
deciphering Dickinson’s handwritten manuscripts.
This shifts the focus from analyzing distortions created by Dickinson's editors to deciphering
Dickinson's handwritten manuscripts -- out of scope and irrelevant
The author says that editors often distorted Dickinson’s intentions. However, he does not say whether
these editors managed to decipher adequately Dickinson’s manuscripts or not.
Another reason to discard this option could be that both editors and Johnson had a problem to decipher
Dickinson’s punctuation, not manuscripts. Here, however, we could argue whether the concept
“punctuation” is part of a broader concept “manuscripts”.
(E) says problem of deciphering Dickinson's handwritten manuscripts. The passage sentence only says
“to standardize Dickinson’s often indecipherable handwritten punctuation is to render permanent”
etc... that doesn't mean these authors were tasked with dealing with the problem of Dickinson's
handwriting. Remember PROBLEM is the keyword – there was no such PROBLEM and hence the SHIFT
OF FOCUS OF E.
THE MOST AUTHORITATIVE GUIDE EVER WRITTEN ON GMAT READING COMPREHENSION
Passage 1
Immigrants’ adoption of English as their primary More direct efforts to force inclusion can be misguided.
language is one measure of assimilation into the larger For example, movements to declare English the official
United States society. Generally, languages define social language do not truly advance the cohesion of a
groups and provide justification for social structures. multicultural nation. They alienate the twenty million
Hence, a distinctive language sets a cultural group off people who do not speak English as their mother
from the dominant language group. Throughout United tongue. They are unnecessary since the public’s
States history this pattern has resulted in one business is already conducted largely in English.
consistent, unhappy consequence, discrimination Further, given the present state of understanding about
against members of the cultural minority. Language the effects of bilingual education on learning, it would
differences provide both a way to rationalize be unwise to require the universal use of English.
subordination and a ready means for achieving it. Finally, it is for parents and local communities to
choose the path they will follow, including how much of
Traditionally, English has replaced the native language their culture they want to maintain for their children.
of immigrant groups by the second or third generation.
Some characteristics of today’s Spanish-speaking 1. It can be inferred from the passage that one of
population, however, suggest the possibility of a the characteristics of immigrant groups to the
departure from this historical pattern. Many families United States has traditionally been that, after
retain ties in Latin America and move back and forth immigration, relatively few members of the
between their present and former communities. This group
“revolving door” phenomenon, along with the high A. became politically active in their new communities
probability of additional immigrants from the south, B. moved back and forth repeatedly between the
means that large Spanish-speaking communities are United States and their former communities
likely to exist in the United States for the indefinite C. used their native languages in their new
future. communities
D. suffered discrimination in their new communities
This expectation underlies the call for national support at the hands of the cultural majority
for bilingual education in Spanish-speaking E. sought assimilation into the dominant culture of
communities’ public schools. Bilingual education can the new communities they were entering
serve different purposes, however. In the 1960s, such
programs were established to facilitate the learning of 2. The passage suggests that one of the effects of
English so as to avoid disadvantaging children in their the debate over bilingual education is that it
other subjects because of their limited English. More has
recently, many advocates have viewed bilingual A. given the Hispanic community a new-found pride
education as a means to maintain children’s native in its culture
languages and cultures. The issue is important for B. hampered the education of Spanish-speaking
people with different political agendas, from students
absorption at one pole to separatism at the other. C. demonstrated the negative impact on imposing
English as the official United States language
To date, the evaluations of bilingual education’s impact D. provided a common banner under which the
on learning have been inconclusive. The issue of Spanish-speaking communities could rally
bilingual education has, nevertheless, served to unite E. polarized the opinions of local Spanish-speaking
the leadership of the nation’s Hispanic communities. community leaders
Grounded in concerns about status that are directly
traceable to the United States history of discrimination 3. The phrase “different political agendas” refers
against Hispanics, the demand for maintenance of the specifically to conflicting opinions regarding
Spanish language in the schools is an assertion of the the
worth of a people and their culture. If the United States A. means of legislating the assimilation of minorities
is truly a multicultural nation—that is, if it is one into United States society
culture reflecting the contributions of many—this B. methods of inducing Hispanics to adopt English as
demand should be seen as a demand not for separation their primary language
but for inclusion. C. means of achieving nondiscriminatory education
for Hispanics
D. official given responsibility for decisions regarding
bilingual education
E. extent to which Hispanics should blend into the
larger United States society
THE MOST AUTHORITATIVE GUIDE EVER WRITTEN ON GMAT READING COMPREHENSION
Immigrants’ adoption of English as their primary More direct efforts to force inclusion can be misguided.
language is one measure of assimilation into the larger For example, movements to declare English the official
United States society. Generally, languages define social language do not truly advance the cohesion of a
groups and provide justification for social structures. multicultural nation. They alienate the twenty million
Hence, a distinctive language sets a cultural group off people who do not speak English as their mother
from the dominant language group. Throughout United tongue. They are unnecessary since the public’s
States history this pattern has resulted in one business is already conducted largely in English.
consistent, unhappy consequence, discrimination Further, given the present state of understanding about
against members of the cultural minority. Language the effects of bilingual education on learning, it would
differences provide both a way to rationalize be unwise to require the universal use of English.
subordination and a ready means for achieving it. Finally, it is for parents and local communities to
choose the path they will follow, including how much of
Traditionally, English has replaced the native language their culture they want to maintain for their children.
of immigrant groups by the second or third generation.
Some characteristics of today’s Spanish-speaking 4. The author says that “It would be unwise to
population, however, suggest the possibility of a require the universal use of English.” One
departure from this historical pattern. Many families reason for this, according to the author, is that
retain ties in Latin America and move back and forth A. it is not clear yet whether requiring the universal
between their present and former communities. This use of English would promote or hinder the
“revolving door” phenomenon, along with the high education of children whose English is limited
probability of additional immigrants from the south, B. the nation’s Hispanic leaders have shown that
means that large Spanish-speaking communities are bilingual education is most effective when it
likely to exist in the United States for the indefinite includes the maintenance of the Spanish language
future. in the schools
C. requiring the universal use of English would
This expectation underlies the call for national support reduce the cohesion of the nation’s Hispanic
for bilingual education in Spanish-speaking communities and leadership
communities’ public schools. Bilingual education can D. the question of language in the schools should be
serve different purposes, however. In the 1960s, such answered by those who evaluate bilingual
programs were established to facilitate the learning of education, not by people with specific political
English so as to avoid disadvantaging children in their agendas
other subjects because of their limited English. More E. it has been shown that bilingual education is
recently, many advocates have viewed bilingual necessary to avoid disadvantaging in their general
education as a means to maintain children’s native learning children whose English is limited
languages and cultures. The issue is important for
people with different political agendas, from 5. In the last paragraph, the author of the passage
absorption at one pole to separatism at the other. is primarily concerned with discussing
A. reasons against enacting a measure that would
To date, the evaluations of bilingual education’s impact mandate the forced inclusion of immigrant groups
on learning have been inconclusive. The issue of within the dominant United culture
bilingual education has, nevertheless, served to unite B. the virtues and limitations of declaring English the
the leadership of the nation’s Hispanic communities. official language of the United States
Grounded in concerns about status that are directly C. the history of attitudes within the Hispanic
traceable to the United States history of discrimination community toward bilingual education in the
against Hispanics, the demand for maintenance of the United States
Spanish language in the schools is an assertion of the D. the importance for immigrant groups of
worth of a people and their culture. If the United States maintaining large segments of their culture to pass
is truly a multicultural nation—that is, if it is one on to their children
culture reflecting the contributions of many—this E. the difference in cultures between Hispanics and
demand should be seen as a demand not for separation other immigrant groups in the United States
but for inclusion.
THE MOST AUTHORITATIVE GUIDE EVER WRITTEN ON GMAT READING COMPREHENSION
Passage 2
Because the framers of the United States Constitution 6. The passage implies that which of the following
(written in 1787) believed that protecting property was a reason that the proportion of verdicts in
rights relating to inventions would encourage the new favor of patentees began to increase in the
nation's economic growth, they gave Congress—the 1830s?
national legislature—a constitutional mandate to grant A. Patent applications approved after 1836 were
patents for inventions. The resulting patent system has more likely to adhere closely to patent law.
served as a model for those in other nations. Recently, B. Patent laws enacted during the 1830s better
however, scholars have questioned whether the defined patent rights.
American system helped achieve the framers' goals. C. Judges became less prejudiced against patentees
These scholars have contended that from 1794 to during the 1830s.
roughly 1830, American inventors were unable to D. After 1836, litigated cases became less
enforce property rights because judges were representative of the population of patent
“antipatent” and routinely invalidated patents for disputes.
arbitrary reasons. This argument is based partly on E. The proportion of patent disputes brought to trial
examination of court decisions in cases where patent began to increase after 1836.
holders (“patentees”) brought suit alleging
infringement of their patent rights. In the 1820s, for 7. The passage implies that the scholars
instance, 75 percent of verdicts were decided against mentioned in the highlighted text would agree
the patentee. The proportion of verdicts for the with which of the following criticisms of the
patentee began to increase in the 1830s, suggesting to American patent system before 1830?
these scholars that judicial attitudes toward patent A. Its definition of property rights relating to
rights began shifting then. inventions was too vague to be useful.
B. Its criteria for the granting of patents were not
Not all patent disputes in the early nineteenth century clear.
were litigated, however, and litigated cases were not C. It made it excessively more difficult for inventors
drawn randomly from the population of disputes. to receive patents.
Therefore, the rate of verdicts in favor of patentees D. It led to excessive large number of patent-
cannot be used by itself to gauge changes in judicial infringement suits.
attitudes or enforceability of patent rights. If early E. It failed to encourage national economic growth.
judicial decisions were prejudiced against patentees,
one might expect that subsequent courts—allegedly 8. It can be inferred from the passage that the
more supportive of patent rights—would reject the frequency with which pre-1830 cases have
former legal precedents. But pre-1830 cases have been been cited in court decisions is an indication
cited as frequently as later decisions, and they continue that
to be cited today, suggesting that the early decisions, A. judicial support for patent rights was weaker in the
many of which clearly declared that patent rights were period before 1830
a just recompense for inventive ingenuity, provided a B. judicial support for patent rights did not increase
lasting foundation for patent law. The proportion of after 1830
judicial decisions in favor of patentees began to C. courts have not returned to judicial standards that
increase during the 1830s because of a change in the prevailed before 1830
underlying population of cases brought to trial. This D. the number of verdicts favoring patentees in
change was partly due to an 1836 revision to the patent patent-infringement suits increased after 1830
system: an examination procedure, still in use today, E. judicial bias against patentees didn’t persist after
was instituted in which each application is scrutinized 1830
for its adherence to patent law. Previously, patents
were automatically granted upon payment of a $30 fee.
THE MOST AUTHORITATIVE GUIDE EVER WRITTEN ON GMAT READING COMPREHENSION
Because the framers of the United States Constitution 9. It can be inferred from the passage that the
(written in 1787) believed that protecting property author and the scholars referred to in the
rights relating to inventions would encourage the new highlighted text disagree about which of the
nation's economic growth, they gave Congress—the following aspects of the patents defended in
national legislature—a constitutional mandate to grant patent-infringement suits before 1830?
patents for inventions. The resulting patent system has A. Whether the patents were granted for inventions
served as a model for those in other nations. Recently, that were genuinely useful
however, scholars have questioned whether the B. Whether the patents were actually relevant to the
American system helped achieve the framers' goals. growth of the United States economy
These scholars have contended that from 1794 to C. Whether the patents were particularly likely to be
roughly 1830, American inventors were unable to annulled by judges
enforce property rights because judges were D. Whether the patents were routinely invalidated for
“antipatent” and routinely invalidated patents for reasons that were arbitrary
arbitrary reasons. This argument is based partly on E. Whether the number of patents vindicated was
examination of court decisions in cases where patent significantly lower than the number in later suits
holders (“patentees”) brought suit alleging
infringement of their patent rights. In the 1820s, for 10. The author of the passage cites which of the
instance, 75 percent of verdicts were decided against following as evidence challenging the
the patentee. The proportion of verdicts for the argument referred to in the highlighted
patentee began to increase in the 1830s, suggesting to text?
these scholars that judicial attitudes toward patent A. The number of cases that were decided against
rights began shifting then. patentees in the 1820s
B. The total number of patent disputes that were
Not all patent disputes in the early nineteenth century litigated from 1794 to 1830
were litigated, however, and litigated cases were not C. The fact that later courts drew upon the legal
drawn randomly from the population of disputes. precedents set in pre-1830 patent cases
Therefore, the rate of verdicts in favor of patentees D. The fact that the number of judicial decisions
cannot be used by itself to gauge changes in judicial in favor of patentees began to increase during
attitudes or enforceability of patent rights. If early the 1830s
judicial decisions were prejudiced against patentees, E. The constitutional rationale for the 1836
one might expect that subsequent courts—allegedly revision of the patent system
more supportive of patent rights—would reject the
former legal precedents. But pre-1830 cases have been
cited as frequently as later decisions, and they continue
to be cited today, suggesting that the early decisions,
many of which clearly declared that patent rights were
a just recompense for inventive ingenuity, provided a
lasting foundation for patent law. The proportion of
judicial decisions in favor of patentees began to
increase during the 1830s because of a change in the
underlying population of cases brought to trial. This
change was partly due to an 1836 revision to the patent
system: an examination procedure, still in use today,
was instituted in which each application is scrutinized
for its adherence to patent law. Previously, patents
were automatically granted upon payment of a $30 fee.
THE MOST AUTHORITATIVE GUIDE EVER WRITTEN ON GMAT READING COMPREHENSION
Passage 3
Film scholars agree that Hollywood portrayals of 11. According to the passage, Apocalypse Now
America at war follow a cyclical pattern. During and Redux differed from Apocalypse Now in
immediately after a conflict, important films trumpet which of the following ways?
glory and sacrifice. Ten to fifteen years later, A. The added footage made it less appealing to a
questioning and sometimes pacifistic movies about the more culturally diverse audience.
conflict dominate. In the late 1960’s, “the raging bulls” B. The added footage made its portrayal of war
of Hollywood—the young trendsetters rising to less glorified and more ambiguous.
prominence—proclaimed this pattern obsolete. C. The added footage made its portrayal of war
However, the passage of time has demonstrated this less harsh and more glorified.
cultural pattern to be more resilient than it seemed in D. The added footage made it more similar in tone
those days of social change. to other war movies.
E. The removed footage made its portrayal of war
Throughout the majority of the last century, evidence less glorified and less appealing.
of the cyclical portrayal of war in film abounds. After
America declared war against Germany during World 12. The passage implies that the combat
War I, the still infant film industry glorified the fight depicted in All Quiet on the Western Front
against “the Hun.” By the early 1930’s, major releases least resembles the depiction of combat in
had changed their tone; for example, All Quiet on the which of the following?
Western Front put forth an anti-war message by A. Jarhead
displaying the horrors of combat. After World War II B. Apocalypse Now
began, the industry shifted gears. Suddenly, important C. The Bridge on the River Kwai
pictures again portrayed glories and courage without D. Platoon
the questioning or despair. For example, Guadalcanal E. Guadalcanal Diary
Diary, produced during the war, showed “the ultimate
sacrifice” as a noble and undoubted good. Once again, 13. In the second paragraph, the author implies
though, by 1957, films such as The Bridge on the River that “the Hun” refers to which of the
Kwai won awards for depicting the moral confusion of following?
war. A. The Huns
B. The Hungarians
Those who later declared this pattern dead based their C. The Austro-Hungarians
conviction on their hearts rather than their minds. D. The Germans
During the Vietnam War, the only major film about that E. The Russians
conflict was The Green Berets, starring John Wayne and
far closer in tone to Guadalcanal Diary than to The
Bridge on the River Kwai. Similarly, years went by
before more complex visions of war, such as
Apocalypse Now, and then Platoon, emerged.
Film scholars agree that Hollywood portrayals of 14. What is the function of the last paragraph of
America at war follow a cyclical pattern. During and the passage?
immediately after a conflict, important films trumpet A. It shows that, despite changes in the industry
glory and sacrifice. Ten to fifteen years later, and audience, the pattern discussed still exists.
questioning and sometimes pacifistic movies about the B. It points out that the film industry never
conflict dominate. In the late 1960’s, “the raging bulls” changes.
of Hollywood—the young trendsetters rising to C. It shows that changes in the film industry and
prominence—proclaimed this pattern obsolete. its audience have made the pattern previously
However, the passage of time has demonstrated this discussed obsolete.
cultural pattern to be more resilient than it seemed in D. It discusses how Jarhead and Apocalypse Now
those days of social change. Redux are fundamentally different from all the
war movies that preceded them.
Throughout the majority of the last century, evidence E. It demonstrates that war movies have changed
of the cyclical portrayal of war in film abounds. After to reflect the more culturally diverse audience.
America declared war against Germany during World
War I, the still infant film industry glorified the fight 15. Which one of the following does the author
against “the Hun.” By the early 1930’s, major releases believe is true about The Bridge on the River
had changed their tone; for example, All Quiet on the Kwai?
Western Front put forth an anti-war message by A. It deserved the awards that it won.
displaying the horrors of combat. After World War II B. It is a more intelligent and well-crafted movie
began, the industry shifted gears. Suddenly, important than The Green Berets.
pictures again portrayed glories and courage without C. It was the first movie to portray the moral
the questioning or despair. For example, Guadalcanal confusion of war.
Diary, produced during the war, showed “the ultimate D. Its portrayal of war is more ambivalent than
sacrifice” as a noble and undoubted good. Once again, that in Guadalcanal Diary.
though, by 1957, films such as The Bridge on the River E. It was more financially successful than any war
Kwai won awards for depicting the moral confusion of movie that came before it.
war.
Passage 4
Jacob Burckhardt's view that Renaissance European 16. The author of the passage discusses
women “stood on a footing of perfect equality” with Krontiris primarily to provide an example
Renaissance men has been repeatedly cited by feminist of a writer who
scholars as a prelude to their presentation of rich A. is highly critical of the writings of certain
historical evidence of women's inequality. In striking Renaissance women
contrast to Burckhardt, Joan Kelly in her famous 1977 B. supports Kelly's view of women's status
essay, “Did Women Have a Renaissance?” argued that during the Renaissance
the Renaissance was a period of economic and social C. has misinterpreted the works of certain
decline for women relative both to Renaissance men Renaissance women
and to medieval women. Recently, however, a D. has rejected the views of both Burckhardt and
significant trend among feminist scholars has entailed Kelly
a rejection of both Kelly's dark vision of the E. has studied Renaissance women in a wide
Renaissance and Burckhardt's rosy one. Many recent variety of social and religious contexts
works by these scholars stress the ways in which
differences among Renaissance women—especially in 17. According to the passage, feminist scholars
terms of social status and religion—work to complicate cite Burckhardt's view of Renaissance
the kinds of generalizations both Burckhardt and Kelly women primarily for which of the following
made on the basis of their observations about upper- reasons?
class Italian women. A. Burckhardt's view forms the basis for most
arguments refuting Kelly's point of view.
The trend is also evident, however, in works focusing B. Burckhardt's view has been discredited by
on those middle- and upper-class European women Kelly.
whose ability to write gives them disproportionate C. Burckhardt's view is one that many feminist
representation in the historical record. Such women scholars wish to refute.
were, simply by virtue of their literacy, members of a D. Burckhardt's work provides rich historical
tiny minority of the population, so it is risky to take evidence of inequality between Renaissance
their descriptions of their experiences as typical of women and men.
“female experience” in any general sense. Tina E. Burckhardt's work includes historical research
Krontiris, for example, in her fascinating study of six supporting the arguments of the feminist
Renaissance women writers, does tend at times to scholars.
conflate “women” and “women writers,” assuming that
women's gender, irrespective of other social 18. It can be inferred that both Burckhardt and
differences, including literacy, allows us to view Kelly have been criticized by the scholars
women as a homogeneous social group and make that mentioned in the highlighted text for which
group an object of analysis. Nonetheless, Krontiris of the following?
makes a significant contribution to the field and is A. Assuming that women writers of the
representative of those authors who offer what might Renaissance are representative of Renaissance
be called a cautiously optimistic assessment of women in general
Renaissance women's achievements, although she also B. Drawing conclusions that are based on the
stresses the social obstacles Renaissance women faced study of an atypical group of women
when they sought to raise their “oppositional voices.” C. Failing to describe clearly the relationship
Krontiris is concerned to show women intentionally between social status and literacy among
negotiating some power for themselves (at least in the Renaissance women
realm of public discourse) against potentially D. Failing to acknowledge the role played by
constraining ideologies, but in her sober and Renaissance women in opposing cultural
thoughtful concluding remarks, she suggests that such stereotypes
verbal opposition to cultural stereotypes was highly E. Failing to acknowledge the ways in which
circumscribed; women seldom attacked the basic social status affected the creative activities of
assumptions in the ideologies that oppressed them. Renaissance women
THE MOST AUTHORITATIVE GUIDE EVER WRITTEN ON GMAT READING COMPREHENSION
Jacob Burckhardt's view that Renaissance European 19. The author of the passage suggests that
women “stood on a footing of perfect equality” with Krontiris incorrectly assumes that
Renaissance men has been repeatedly cited by feminist A. social differences among Renaissance women
scholars as a prelude to their presentation of rich are less important than the fact that they were
historical evidence of women's inequality. In striking women
contrast to Burckhardt, Joan Kelly in her famous 1977 B. literacy among Renaissance women was more
essay, “Did Women Have a Renaissance?” argued that prevalent than most scholars today
the Renaissance was a period of economic and social acknowledge
decline for women relative both to Renaissance men C. during the Renaissance, women were able to
and to medieval women. Recently, however, a successfully oppose cultural stereotypes
significant trend among feminist scholars has entailed relating to gender
a rejection of both Kelly's dark vision of the D. Renaissance women did not face many difficult
Renaissance and Burckhardt's rosy one. Many recent social obstacles relating to their gender
works by these scholars stress the ways in which E. in order to attain power, Renaissance women
differences among Renaissance women—especially in attacked basic assumptions in the ideologies
terms of social status and religion—work to complicate that oppressed them
the kinds of generalizations both Burckhardt and Kelly
made on the basis of their observations about upper- 20. The last sentence in the passage serves
class Italian women. primarily to
A. suggest that Krontiris's work is not
The trend is also evident, however, in works focusing representative of recent trends among
on those middle- and upper-class European women feminist scholars
whose ability to write gives them disproportionate B. undermine the argument that literate women
representation in the historical record. Such women of the Renaissance sought to oppose social
were, simply by virtue of their literacy, members of a constraints imposed on them
tiny minority of the population, so it is risky to take C. show a way in which Krontiris's work
their descriptions of their experiences as typical of illustrates a “cautiously optimistic”
“female experience” in any general sense. Tina assessment of Renaissance women's
Krontiris, for example, in her fascinating study of six achievements
Renaissance women writers, does tend at times to D. summarize Krontiris's view of the effect of
conflate “women” and “women writers,” assuming that literacy on the lives of upper- and middle-class
women's gender, irrespective of other social Renaissance women
differences, including literacy, allows us to view E. illustrate the way in which Krontiris's study
women as a homogeneous social group and make that differs from the studies done by Burckhardt
group an object of analysis. Nonetheless, Krontiris and Kelly
makes a significant contribution to the field and is
representative of those authors who offer what might 21. The author of the passage implies that the
be called a cautiously optimistic assessment of women studied by Krontiris are unusual in
Renaissance women's achievements, although she also which of the following ways?
stresses the social obstacles Renaissance women faced A. They faced obstacles less formidable than
when they sought to raise their “oppositional voices.” those faced by other Renaissance women.
Krontiris is concerned to show women intentionally B. They have been seen by historians as more
negotiating some power for themselves (at least in the interesting than other Renaissance women.
realm of public discourse) against potentially C. They were more concerned about recording
constraining ideologies, but in her sober and history accurately than were other
thoughtful concluding remarks, she suggests that such Renaissance women.
verbal opposition to cultural stereotypes was highly D. Their perceptions are more likely to be
circumscribed; women seldom attacked the basic accessible to historians than are those of most
assumptions in the ideologies that oppressed them. other Renaissance women.
E. Their concerns are likely to be of greater
interest to feminist scholars than are the ideas
of most other Renaissance women.
THE MOST AUTHORITATIVE GUIDE EVER WRITTEN ON GMAT READING COMPREHENSION
Passage 5
When asteroids collide, some collisions cause an 22. The passage implies which of the following
asteroid to spin faster; others slow it down. If asteroids about the five asteroids mentioned in the
are all monoliths—single rocks—undergoing random highlighted text?
collisions, a graph of their rotation rates should show a A. Their rotation rates are approximately the
bell-shaped distribution with statistical “tails” of very same.
fast and very slow rotators. If asteroids are rubble B. They have undergone approximately the same
piles, however, the tail representing the very fast number of collisions.
rotators would be missing, because any loose C. They are monoliths.
aggregate spinning faster than once every few hours D. They are composed of fragments that have
(depending on the asteroid's bulk density) would fly escaped the gravity of larger asteroids.
apart. Researchers have discovered that all but five E. They were detected only recently.
observed asteroids obey a strict limit on rate of
rotation. The exceptions are all smaller than 200 23. The discovery of which of the following
meters in diameter, with an abrupt cutoff for asteroids would call into question the conclusion
larger than that. mentioned in the highlighted text?
A. An asteroid 100 meters in diameter rotating at
The evident conclusion—that asteroids larger than a rate of once per week
200 meters across are multicomponent structures or B. An asteroid 150 meters in diameter rotating at
rubble piles—agrees with recent computer modeling a rate of 20 times per hour
of collisions, which also finds a transition at that C. An asteroid 250 meters in diameter rotating at
diameter. A collision can blast a large asteroid to bits, a rate of once per week
but after the collision those bits will usually move D. An asteroid 500 meters in diameter rotating at
slower than their mutual escape velocity. Over several a rate of once per hour
hours, gravity will reassemble all but the fastest pieces E. An asteroid 1,000 meters in diameter rotating
into a rubble pile. Because collisions among asteroids at a rate of once every 24 hours
are relatively frequent, most large bodies have already
suffered this fate. Conversely, most small asteroids 24. The author of the passage mentions “escape
should be monolithic, because impact fragments easily velocity” in order to help explain which of
escape their feeble gravity. the following?
A. The tendency for asteroids to become smaller
rather than larger over time
B. The speed with which impact fragments
reassemble when they do not escape an
asteroid's gravitational attraction after a
collision
C. The frequency with which collisions among
asteroids occur
D. The rotation rates of asteroids smaller than
200 meters in diameter
E. The tendency for large asteroids to persist
after collisions
THE MOST AUTHORITATIVE GUIDE EVER WRITTEN ON GMAT READING COMPREHENSION
Passage 6
Jon Clark's study of the effect of the modernization of a 25. The information in the passage suggests
telephone exchange on exchange maintenance work that which of the following statements from
and workers is a solid contribution to a debate that hypothetical sociological studies of change
encompasses two lively issues in the history and in industry most clearly exemplifies the
sociology of technology: technological determinism social constructivists' version of
and social constructivism. technological determinism?
A. It is the available technology that determines
Clark makes the point that the characteristics of a workers' skills, rather than workers' skills
technology have a decisive influence on job skills and influencing the application of technology.
work organization. Put more strongly, technology can B. All progress in industrial technology grows out
be a primary determinant of social and managerial of a continuing negotiation between
organization. Clark believes this possibility has been technological possibility and human need.
obscured by the recent sociological fashion, C. Some organizational change is caused by
exemplified by Braverman's analysis, that emphasizes people; some is caused by computer chips.
the way machinery reflects social choices. For D. Most major technological advances in industry
Braverman, the shape of a technological system is have been generated through research and
subordinate to the manager's desire to wrest control of development.
the labor process from the workers. Technological E. Some industrial technology eliminates jobs,
change is construed as the outcome of negotiations but educated workers can create whole new
among interested parties who seek to incorporate their skills areas by the adaptation of the
own interests into the design and configuration of the technology.
machinery. This position represents the new
mainstream called social constructivism. 26. The information in the passage suggests
that Clark believes that which of the
The constructivists gain acceptance by following would be true if social
misrepresenting technological determinism: constructivism had not gained widespread
technological determinists are supposed to believe, for acceptance?
example, that machinery imposes appropriate forms of A. Businesses would be more likely to modernize
order on society. The alternative to constructivism, in without considering the social consequences
other words, is to view technology as existing outside of their actions.
society, capable of directly influencing skills and work B. There would be greater understanding of the
organization. role played by technology in producing social
change.
Clark refutes the extremes of the constructivists by C. Businesses would be less likely to understand
both theoretical and empirical arguments. the attitudes of employees affected by
Theoretically he defines “technology” in terms of modernization.
relationships between social and technical variables. D. Modernization would have occurred at a
Attempts to reduce the meaning of technology to cold, slower rate.
hard metal are bound to fail, for machinery is just scrap E. Technology would have played a greater part
unless it is organized functionally and supported by in determining the role of business in society.
appropriate systems of operation and maintenance. At
the empirical level Clark shows how a change at the
telephone exchange from maintenance-intensive
electromechanical switches to semi-electronic
switching systems altered work tasks, skills, training
opportunities, administration, and organization of
workers. Some changes Clark attributes to the
particular way management and labor unions
negotiated the introduction of the technology, whereas
others are seen as arising from the capabilities and
nature of the technology itself. Thus, Clark helps
answer the question: “When is social choice decisive
and when are the concrete characteristics of
technology more important?”
THE MOST AUTHORITATIVE GUIDE EVER WRITTEN ON GMAT READING COMPREHENSION
Jon Clark's study of the effect of the modernization of a 27. The author of the passage uses the
telephone exchange on exchange maintenance work expression “are supposed to” primarily in
and workers is a solid contribution to a debate that order to
encompasses two lively issues in the history and A. suggest that a contention made by
sociology of technology: technological determinism constructivists regarding determinists is
and social constructivism. inaccurate
B. define the generally accepted position of
Clark makes the point that the characteristics of a determinists regarding the implementation of
technology have a decisive influence on job skills and technology
work organization. Put more strongly, technology can C. engage in speculation about the motivation of
be a primary determinant of social and managerial determinists
organization. Clark believes this possibility has been D. lend support to a comment critical of the
obscured by the recent sociological fashion, position of determinists
exemplified by Braverman's analysis, that emphasizes E. contrast the historical position of determinists
the way machinery reflects social choices. For with their position regarding the exchange
Braverman, the shape of a technological system is modernization
subordinate to the manager's desire to wrest control of
the labor process from the workers. Technological 28. Which of the following statements about
change is construed as the outcome of negotiations Clark's study of the telephone exchange can
among interested parties who seek to incorporate their be inferred from information in the
own interests into the design and configuration of the passage?
machinery. This position represents the new A. Clark's reason for undertaking the study was
mainstream called social constructivism. to undermine Braverman's analysis of the
function of technology.
The constructivists gain acceptance by B. Clark's study suggests that the implementation
misrepresenting technological determinism: of technology should be discussed in the
technological determinists are supposed to believe, context of conflict between labor and
for example, that machinery imposes appropriate management.
forms of order on society. The alternative to C. Clark examined the impact of changes in the
constructivism, in other words, is to view technology technology of switching at the exchange in
as existing outside society, capable of directly terms of overall operations and organization.
influencing skills and work organization. D. Clark concluded that the implementation of
new switching technology was equally
Clark refutes the extremes of the constructivists by beneficial to management and labor.
both theoretical and empirical arguments. E. Clark's analysis of the change in switching
Theoretically he defines “technology” in terms of systems applies only narrowly to the situation
relationships between social and technical variables. at the particular exchange that he studied.
Attempts to reduce the meaning of technology to cold,
hard metal are bound to fail, for machinery is just scrap
unless it is organized functionally and supported by
appropriate systems of operation and maintenance. At
the empirical level Clark shows how a change at the
telephone exchange from maintenance-intensive
electromechanical switches to semi-electronic
switching systems altered work tasks, skills, training
opportunities, administration, and organization of
workers. Some changes Clark attributes to the
particular way management and labor unions
negotiated the introduction of the technology, whereas
others are seen as arising from the capabilities and
nature of the technology itself. Thus, Clark helps
answer the question: “When is social choice decisive
and when are the concrete characteristics of
technology more important?”
THE MOST AUTHORITATIVE GUIDE EVER WRITTEN ON GMAT READING COMPREHENSION
Passage 7
A small number of the forest species of lepidoptera 29. Which of the following, if true, would most
(moths and butterflies, which exist as caterpillars weaken the author's conclusion in the last
during most of their life cycle) exhibit regularly sentence of the first paragraph?
recurring patterns of population growth and decline— A. New research reveals that the number of
such fluctuations in population are known as species of birds and parasites that prey on
population cycles. Although many different variables lepidoptera has dropped significantly in recent
influence population levels, a regular pattern such as a years.
population cycle seems to imply a dominant, driving B. New experiments in which the habitats of
force. Identification of that driving force, however, has lepidoptera are altered in previously untried
proved surprisingly elusive despite considerable ways result in the shortening of lepidoptera
research. The common approach of studying causes of population cycles.
population cycles by measuring the mortality caused C. Recent experiments have revealed that the
by different agents, such as predatory birds or nuclear polyhedrosis virus is present in a
parasites, has been unproductive in the case of number of predators and parasites of
lepidoptera. Moreover, population ecologists' attempts lepidoptera.
to alter cycles by changing the caterpillars' habitat and D. Differences among the habitats of lepidoptera
by reducing caterpillar populations have not species make it difficult to assess the effects of
succeeded. In short, the evidence implies that these weather on lepidoptera population cycles.
insect populations, if not self-regulating, may at least E. Viral disease is typically observed in a large
be regulated by an agent more intimately connected proportion of the lepidoptera population.
with the insect than are predatory birds or parasites.
Passage 8
In Winters v. United States (1908), the Supreme Court 30. The passage suggests that, if the three
held that the right to use waters flowing through or criteria were the only criteria for
adjacent to the Fort Berthold Indian Reservation was establishing a reservation's water rights,
reserved to American Indians by the treaty which of the following would be true?
establishing the reservation. Although this treaty did A. The water rights of the inhabitants of the Fort
not mention water rights, the Court ruled that the Berthold Indian Reservation would not take
federal government, when it created the reservation, precedence over those of other citizens.
intended to deal fairly with American Indians by B. Reservations established before 1848 would
reserving for them the waters without which their be judged to have no water rights.
lands would have been useless. Later decisions, citing C. There would be no legal basis for the water
Winters, established that courts can find federal rights rights of the Rio Grande pueblos.
to reserve water for particular purposes if D. Reservations other than American Indian
reservations could not be created with
1. the land in question lies within an enclave under reserved water rights.
exclusive federal jurisdiction; E. Treaties establishing reservations would have
2. the land has been formally withdrawn from federal to mention water rights explicitly in order to
public lands—i.e., withdrawn from the stock of reserve water for a particular purpose.
federal lands available for private use under
federal land use laws—and set aside or reserved; 31. Which of the following most accurately
and summarizes the relationship between
3. the circumstances reveal the government intended Arizona v. California, as that decision is
to reserve water as well as land when establishing described in the passage, and the three
the reservation. criteria discussed?
A. Arizona v. California abolishes these criteria
Some American Indian tribes have also established and establishes a competing set of criteria for
water rights through the courts based on their applying the Winters doctrine.
traditional diversion and use of certain waters prior to B. Arizona v. California establishes that the
the United States' acquisition of sovereignty. For Winters doctrine applies to a broader range of
example, the Rio Grande pueblos already existed when situations than those defined by these criteria.
the United States acquired sovereignty over New C. Arizona v. California represents the sole
Mexico in 1848. Although they at that time became part example of an exception to the criteria as they
of the United States, the pueblo lands never formally were set forth in the Winters doctrine.
constituted a part of federal public lands; in any event, D. Arizona v. California does not refer to the
no treaty, statute, or executive order has ever Winters doctrine to justify water rights,
designated or withdrawn the pueblos from public whereas these criteria do rely on the Winters
lands as American Indian reservations. This fact, doctrine.
however, has not barred application of the Winters E. Arizona v. California applies the criteria
doctrine. What constitutes an American Indian derived from the Winters doctrine only to
reservation is a question of practice, not of legal federal lands other than American Indian
definition, and the pueblos have always been treated as reservations.
reservations by the United States. This pragmatic
approach is buttressed by Arizona v. California (1963),
wherein the Supreme Court indicated that the manner
in which any type of federal reservation is created does
not affect the application to it of the Winters doctrine.
Therefore, the reserved water rights of Pueblo Indians
have priority over other citizens' water rights as of
1848, the year in which pueblos must be considered to
have become reservations.
THE MOST AUTHORITATIVE GUIDE EVER WRITTEN ON GMAT READING COMPREHENSION
In Winters v. United States (1908), the Supreme Court 32. The “pragmatic approach” mentioned in
held that the right to use waters flowing through or the highlighted text of the passage is best
adjacent to the Fort Berthold Indian Reservation was defined as one that
reserved to American Indians by the treaty A. grants recognition to reservations that were
establishing the reservation. Although this treaty did never formally established but that have
not mention water rights, the Court ruled that the traditionally been treated as such
federal government, when it created the reservation, B. determines the water rights of all citizens in a
intended to deal fairly with American Indians by particular region by examining the actual
reserving for them the waters without which their history of water usage in that region
lands would have been useless. Later decisions, citing C. gives federal courts the right to reserve water
Winters, established that courts can find federal rights along with land even when it is clear that the
to reserve water for particular purposes if government originally intended to reserve
only the land
1. the land in question lies within an enclave under D. bases the decision to recognize the legal rights
exclusive federal jurisdiction; of a group on the practical effect such a
2. the land has been formally withdrawn from federal recognition is likely to have on other citizens
public lands—i.e., withdrawn from the stock of E. dictates that courts ignore precedents set by
federal lands available for private use under such cases as Winters v. United States in
federal land use laws—and set aside or reserved; deciding what water rights belong to reserved
and land
3. the circumstances reveal the government intended
to reserve water as well as land when establishing 33. It can be inferred from the passage that the
the reservation. Winters doctrine has been used to establish
which of the following?
Some American Indian tribes have also established A. A rule that the government may reserve water
water rights through the courts based on their only by explicit treaty or agreement
traditional diversion and use of certain waters prior to B. A legal distinction between federal lands
the United States' acquisition of sovereignty. For reserved for American Indians and federal
example, the Rio Grande pueblos already existed when lands reserved for other purposes
the United States acquired sovereignty over New C. Criteria governing when the federal
Mexico in 1848. Although they at that time became part government may set land aside for a particular
of the United States, the pueblo lands never formally purpose
constituted a part of federal public lands; in any event, D. The special status of American Indian tribes'
no treaty, statute, or executive order has ever rights to reserved land
designated or withdrawn the pueblos from public E. The federal right to reserve water implicitly as
lands as American Indian reservations. This fact, well as explicitly under certain conditions
however, has not barred application of the Winters
doctrine. What constitutes an American Indian 34. The author cites the fact that the Rio
reservation is a question of practice, not of legal Grande pueblos were never formally
definition, and the pueblos have always been treated as withdrawn from public lands primarily in
reservations by the United States. This pragmatic order to do which of the following?
approach is buttressed by Arizona v. California (1963), A. Suggest why it might have been argued that the
wherein the Supreme Court indicated that the manner Winters doctrine ought not to apply to pueblo
in which any type of federal reservation is created does lands
not affect the application to it of the Winters doctrine. B. Imply that the United States never really
Therefore, the reserved water rights of Pueblo Indians acquired sovereignty over pueblo lands
have priority over other citizens' water rights as of C. Argue that the pueblo lands ought still to be
1848, the year in which pueblos must be considered to considered part of federal public lands
have become reservations. D. Support the argument that the water rights of
citizens other than American Indians are
limited by the Winters doctrine
E. Suggest that federal courts cannot claim
jurisdiction over cases disputing the
traditional diversion and use of water by
Pueblo Indians
THE MOST AUTHORITATIVE GUIDE EVER WRITTEN ON GMAT READING COMPREHENSION
In Winters v. United States (1908), the Supreme Court 35. The passage suggests that the legal rights of
held that the right to use waters flowing through or citizens other than American Indians to the
adjacent to the Fort Berthold Indian Reservation was use of water flowing into the Rio Grande
reserved to American Indians by the treaty pueblos are
establishing the reservation. Although this treaty did A. guaranteed by the precedent set in Arizona v.
not mention water rights, the Court ruled that the California
federal government, when it created the reservation, B. abolished by the Winters doctrine
intended to deal fairly with American Indians by C. deferred to the Pueblo Indians whenever
reserving for them the waters without which their treaties explicitly require this
lands would have been useless. Later decisions, citing D. guaranteed by federal land-use laws
Winters, established that courts can find federal rights E. limited by the prior claims of the Pueblo
to reserve water for particular purposes if Indians
Passage 9
Antonia Castañeda has utilized scholarship from 37. Which of the following could best serve as
women's studies and Mexican-American history to an example of the kind of fictional plot
examine nineteenth-century literary portrayals of discussed by Antonia Castañeda?
Mexican women. As Castañeda notes, scholars of A. A land speculator of English ancestry weds the
women's history observe that in the United States, daughter of a Mexican vineyard owner after
male novelists of the period—during which, according the speculator has migrated to California to
to these scholars, women's traditional economic role in seek his fortune.
home-based agriculture was threatened by the B. A Californian woman of Hispanic ancestry
transition to a factory-based industrial economy— finds that her agricultural livelihood is
define women solely in their domestic roles of wife and threatened when her husband is forced to seek
mother. Castañeda finds that during the same period work in a textile mill.
that saw non-Hispanic women being economically C. A Mexican rancher who loses his land as a
displaced by industrialization, Hispanic law in result of the Mexican-American War migrates
territorial California protected the economic position to the northern United States and marries an
of “Californianas” (the Mexican women of the territory) immigrant schoolteacher.
by ensuring them property rights and inheritance D. A wealthy Californiana whose father has
rights equal to those of males. bequeathed her all his property contends with
avaricious relatives for her inheritance.
For Castañeda, the laws explain a stereotypical plot E. A poor married couple emigrate from French
created primarily by male, non-Hispanic novelists: the Canada and gradually become wealthy as
story of an ambitious non-Hispanic merchant or trader merchants in territorial California.
desirous of marrying an elite Californiana. These
novels' favorable portrayal of such women is 38. Which of the following, if true, would
noteworthy, since Mexican-American historians have provide the most support for Castañeda's
concluded that unflattering literary depictions of explanation of the “stereotypical plot”
Mexicans were vital in rallying the United States mentioned in the highlighted text?
public's support for the Mexican-American War A. Non-Hispanic traders found business more
(1846–1848). The importance of economic alliances profitable in California while it was a territory
forged through marriages with Californianas explains than when it became a state.
this apparent contradiction. Because of their real-life B. Very few marriages between Hispanic women
economic significance, the Californianas were and non-Hispanic men in nineteenth-century
portrayed more favorably than were others of the territorial California have actually been
same nationality. documented.
C. Records from the nineteenth century indicate
36. The “apparent contradiction” mentioned that some large and valuable properties were
refers to the discrepancy between the owned by elite Californianas in their own right.
A. legal status of Mexican women in territorial D. Unmarried non-Hispanic women in the
California and their status in the United States nineteenth-century United States were
B. unflattering depiction of Mexicans in novels sometimes able to control property in their
and the actual public sentiment about the own right.
Mexican-American War E. Most of the property in nineteenth-century
C. existence of many marriages between territorial California was controlled by
Californianas and non-Hispanic merchants and Hispanic men.
the strictures against them expressed in novels
D. literary depiction of elite Californianas and the
literary depiction of other Mexican individuals
E. novelistic portrayals of elite Californianas'
privileged lives and the actual circumstances
of those lives
THE MOST AUTHORITATIVE GUIDE EVER WRITTEN ON GMAT READING COMPREHENSION
Passage 10
Anthropologists once thought that the ancestors of 39. The passage suggests that proponents of
modern humans began to walk upright because it freed the theory mentioned in the highlighted
their hands to use stone tools, which they had begun to text assume that which of the following
make as the species evolved a brain of increased size steps in human evolution occurred most
and mental capacity. But discoveries of the three- recently?
million-year-old fossilized remains of our hominid A. Development of a nuclear family structure
ancestor Australopithecus have yielded substantial B. Transition from walking on all fours to walking
anatomical evidence that upright walking appeared upright
prior to the dramatic enlargement of the brain and the C. Dramatic enlargement of the brain
development of stone tools. D. Use of the hands to gather and carry food
E. Modification of propulsive muscles to provide
Walking on two legs in an upright posture (bipedal stability and control in locomotion
locomotion) is a less efficient proposition than walking
on all fours (quadrupedal locomotion) because several 40. According to the passage, the hominid
muscle groups that the quadruped uses for propulsion australopithecine most closely resembled a
must instead be adapted to provide the biped with modern human with respect to which of the
stability and control. The shape and configuration of following characteristics?
various bones must likewise be modified to allow the A. Brain size
muscles to perform these functions in upright walking. B. Tool-making ability
Reconstruction of the pelvis (hipbones) and femur C. Shape of the pelvis
(thighbone) of “Lucy,” a three-million-year-old D. Method of locomotion
skeleton that is the most complete fossilized skeleton E. Preference for certain foods
from the Australopithecine era, has shown that they
are much more like the corresponding bones of the 41. The passage suggests that, in comparison
modern human than like those of the most closely with the hominid australopithecines,
related living primate, the quadrupedal chimpanzee. modern humans are
Lucy's wide, shallow pelvis is actually better suited to A. less well adapted to large-group cooperation
bipedal walking than is the rounder, bowl-like pelvis of B. less well adapted to walking upright
the modern human, which evolved to form the larger C. more agile in running and climbing
birth canal needed to accommodate the head of a large- D. more well suited to a nuclear family structure
brained human infant. By contrast, the head of Lucy's E. more well suited to cooperative caring for
baby could have been no larger than that of a baby their offspring
chimpanzee.
42. The theory mentioned in the highlighted
If the small-brained australopithecines were not text suggests that which of the following
toolmakers, what evolutionary advantage did they gain was true for the hominid ancestors of
by walking upright? One theory is that bipedality modern humans before they made the
evolved in conjunction with the nuclear family: transition to walking upright?
monogamous parents cooperating to care for their A. Their brains were smaller than the brains of
offspring. Walking upright permitted the father to use present-day chimpanzees.
his hands to gather food and carry it to his mate from a B. They competed rather than cooperated in
distance, allowing the mother to devote more time and searching for food.
energy to nurturing and protecting their children. C. Their mating patterns and family structure
According to this view, the transition to bipedal were closer to those of present-day
walking may have occurred as long as ten million years chimpanzees than to those of modern humans.
ago, at the time of the earliest hominids, making it a D. Males played a more significant role in child
crucial initiating event in human evolution. rearing than they played after the transition to
walking upright.
E. Females' ability to nurture and protect their
offspring was limited by the need to find food
for themselves.
THE MOST AUTHORITATIVE GUIDE EVER WRITTEN ON GMAT READING COMPREHENSION
Passage 11
Recent feminist scholarship concerning the United 43. It can be inferred that the author of the
States in the 1920s challenges earlier interpretations passage disagrees with the “new
that assessed the 1920s in terms of the unkept scholarship” mentioned in the highlighted
“promises” of the women's suffrage movement. This text regarding the
new scholarship disputes the long-held view that A. degree to which the “promises” of the suffrage
because a women's voting bloc did not materialize movement remained unkept
after women gained the right to vote in 1920, suffrage B. degree to which suffrage for women improved
failed to produce long-term political gains for women. the morality of governance
These feminist scholars also challenge the old view that C. degree to which the 1920s represented a
pronounced suffrage a failure for not delivering on the period of decline for the feminist movement
promise that the women's vote would bring about D. degree of legislative success achieved by
moral, corruption-free governance. Asked whether feminist reformers during the 1920s
women's suffrage was a failure, these scholars cite the E. accuracy of the view that a women's voting
words of turn-of-the-century social reformer Jane bloc did not materialize once suffrage was
Addams, “Why don't you ask if suffrage in general is achieved
failing?”
44. It can be inferred from the passage that
In some ways, however, these scholars still present recent scholars cite the words of Jane
the 1920s as a period of decline. After suffrage, they Addams primarily in order to
argue, the feminist movement lost its cohesiveness, A. suggest that women's achievement of suffrage
and gender consciousness waned. After the mid-1920s, brought about changes in government that
few successes could be claimed by feminist reformers: were not taken into account by early
little could be seen in the way of legislative victories. interpretations
B. point out contradictions inherent in the goals
During this decade, however, there was intense of the women's suffrage movement
activism aimed at achieving increased autonomy for C. show why a women's voting bloc was not
women, broadening the spheres within which they formed when women won the right to vote
lived their daily lives. Women's organizations worked D. emphasize the place of social reform
to establish opportunities for women: they strove to movements in the struggle for suffrage for
secure for women the full entitlements of citizenship, women
including the right to hold office and the right to serve E. suggest that the old view of women's suffrage
on juries. was inappropriate
Passage 12
This passage is excerpted from material published in 46. According to the passage, the dynamical
1997. evidence referred to in the highlighted text
supports which of the following?
Is there a massive black hole at the center of our galaxy, A. Recent assumptions about the velocities of
the Milky Way? The evidence is inconclusive. Just as the stars
Sun's mass can be determined, given knowledge of B. Widely held predictions about the amount of
other variables, by the velocity at which its planets matter a black hole will engulf
orbit, the mass at the center of the Milky Way can be C. The existence of an extremely dense object at
revealed by the velocities of stars and gas orbiting the the center of the Milky Way
galactic center. This dynamical evidence, based on D. The contention that too much energy is coming
recently confirmed assumptions about the stars' from the mass at the Milky Way's galactic
velocities, argues for an extremely compact object with center for that mass to be a black hole
a mass two to three million times the mass of our Sun. E. The conclusion that a compact object of two to
Although according to current theory this makes the three million times the mass of our Sun is too
mass at the center of the galaxy too dense to be dense to be anything but a black hole
anything but a black hole, the relative lack of energy
radiating from the galactic center presents a serious 47. The “serious problem” referred to in the
problem. A black hole's gravity attracts surrounding highlighted text could be solved if which of
matter, which swirls around the black hole, emitting the following were true?
some energy as it is engulfed. Scientists believe that the A. Current assumptions about how much matter
amount of energy that escapes the black hole should be a black hole would engulf proved to be several
about 10 percent of the matter's rest energy (the thousand times too high.
energy equivalent of its mass according to the equation B. Current assumptions about how much matter
E=mc2). But when the energy coming from the galactic a black hole would engulf proved to be a few
center is compared to widely held predictions based on thousand times too low.
how much matter should be falling into a theoretical C. The object at the center of the Milky Way
central black hole, there is a discrepancy by a factor of turned out to be far denser than it is currently
a few thousand. estimated to be.
D. The object at the center of the Milky Way
turned out to be far more massive than it is
currently estimated to be.
E. Matter being engulfed by a black hole radiated
far more energy than is currently assumed.
Passage 13
Despite their many differences of temperament and of 49. According to the passage, the five writers
literary perspective, Emerson, Thoreau, Hawthorne, object to the scientific method primarily
Melville, and Whitman shared certain beliefs. Common because they think it
to all these writers is their humanistic perspective. Its A. is not the best way to obtain an understanding
basic premises are that humans are the spiritual center of the relationship between the individual and
of the universe and that in them alone is the clue to the cosmos
nature, history, and ultimately the cosmos. Without B. is so specialized that it leads to an
denying outright the existence of a deity, this understanding of separate parts of the
perspective explains humans and the world in terms of universe but not of the relationships among
humanity. those parts
C. cannot provide an adequate explanation of
This common perspective is almost always intuition and imagination
universalized. It emphasizes the human as universal, D. misleads people into believing they have an
freed from the accidents of time, space, birth, and understanding of truth, when they do not
talent. Thus, for Emerson, the “American Scholar” turns E. prevents people from recognizing the symbolic
out to be simply “Man Thinking,” while, for Whitman, nature of experience
the “Song of Myself” merges imperceptibly into a song
of all the “children of Adam,” where “every atom 50. The author quotes Whitman primarily in
belonging to me as good belongs to you.” order to
A. show that the poet does not agree with
Also common to all five writers is the belief that self- Emerson
realization depends on the harmonious reconciliation B. indicate the way the poet uses the humanist
of two universal psychological tendencies: first, the ideal to praise himself
self-asserting impulse of the individual to be C. suggest that the poet adapts the basic premises
responsible only to himself or herself, and second, the of humanism to his own individual outlook on
self-transcending impulse of the individual to know the world
and become one with that world. These conflicting D. illustrate a way the poet expresses the
impulses can be seen in the democratic ethic. relationship of the individual to the humanistic
Democracy advocates individualism, the preservation universe
of the individual's freedom and self-expression. But the E. demonstrate that the poet is concerned with
democratic self is torn between the duty to self, which the well-being of all humans
is implied by the concept of liberty, and the duty to
society, which is implied by the concepts of equality 51. It can be inferred that intuition is
and fraternity. important to the five writers primarily
because it provides them with
A third assumption common to the five writers is that A. information useful for understanding abstract
intuition and imagination offer a surer road to truth logic and scientific method
than does abstract logic or scientific method. It is B. the discipline needed in the search for truth
illustrated by their emphasis upon introspection— C. inspiration for their best writing
their belief that the clue to external nature is to be D. clues to the interpretation of symbolic
found in the inner world of individual psychology— experience
and by their interpretation of experience as, in essence, E. the means of resolving conflicts between the
symbolic. Both these stresses presume an organic self and the world
relationship between the self and the cosmos of which
only intuition and imagination can properly take
account. These writers' faith in the imagination and in
themselves led them to conceive of the writer as a seer.
THE MOST AUTHORITATIVE GUIDE EVER WRITTEN ON GMAT READING COMPREHENSION
Despite their many differences of temperament and of 52. The author discusses “the democratic
literary perspective, Emerson, Thoreau, Hawthorne, ethic” in order to
Melville, and Whitman shared certain beliefs. Common A. explain the relationship between external
to all these writers is their humanistic perspective. Its experience and inner imagination
basic premises are that humans are the spiritual center B. support the notion that the self contains two
of the universe and that in them alone is the clue to conflicting and irreconcilable factions
nature, history, and ultimately the cosmos. Without C. illustrate the relationship between the self's
denying outright the existence of a deity, this desire to be individual and its desire to merge
perspective explains humans and the world in terms of with all other selves
humanity. D. elaborate on the concept that the self
constantly desires to realize its potential
This common perspective is almost always E. give an example of the idea that, in order to be
universalized. It emphasizes the human as universal, happy, the self must reconcile its desires with
freed from the accidents of time, space, birth, and external reality
talent. Thus, for Emerson, the “American Scholar” turns
out to be simply “Man Thinking,” while, for Whitman, 53. It can be inferred that the idea of “an
the “Song of Myself” merges imperceptibly into a song organic relationship between the self and
of all the “children of Adam,” where “every atom the cosmos” is necessary to the thinking of
belonging to me as good belongs to you.” the five writers because such a relationship
A. enables them to assert the importance of the
Also common to all five writers is the belief that self- democratic ethic
realization depends on the harmonious reconciliation B. justifies their concept of the freedom of the
of two universal psychological tendencies: first, the individual
self-asserting impulse of the individual to be C. sustains their faith in the existence of a deity
responsible only to himself or herself, and second, the D. is the foundation of their humanistic view of
self-transcending impulse of the individual to know existence
and become one with that world. These conflicting E. is the basis for their claim that the writer is a
impulses can be seen in the democratic ethic. seer
Democracy advocates individualism, the preservation
of the individual's freedom and self-expression. But the
democratic self is torn between the duty to self, which
is implied by the concept of liberty, and the duty to
society, which is implied by the concepts of equality
and fraternity.
Passage 14
The final quarter of the nineteenth century marked a 54. According to information presented in the
turning point in the history of biology—biologists passage, which of the following is a true
became less interested in applying an ideal of historical statement about the methods of
explanation deductively to organic function and more explanation used by biologists and
interested in discerning the causes of vital processes historians in the nineteenth century?
through experimental manipulation. But it is A. Neither biologists nor historians were able to
impossible to discuss the history of biology in the develop methods of explanation that were
nineteenth century without emphasizing that those accepted by the majority of their colleagues.
areas of biology most in the public eye had depended B. The methods used by biologists to explain
on historical explanation. Wherever it was applied, phenomena changed dramatically, whereas
historical explanation was deemed causal explanation. the methods used by historians to explain
The biologist-as-historian and the general historian of events did not change as noticeably.
human events dealt with comparable phenomena and C. Biologists believed that they had refined the
assumed necessarily a common mode of explanation. methods of explanation used by historians.
D. Biologists' and historians' methods of
Nineteenth-century biologists found a historical explaining what they believed to be
explanation of organic function attractive partly comparable phenomena were similar.
because their observation of the formation of a new cell E. Although biologists and historians adopted
from a preexisting cell seemed to confirm a historical similar methods of explanation, the biologists
explanation of cell generation. The same direct were more apologetic about their use of these
observation of continuous stages of development was methods.
also possible when they examined the complex
sequence of events of embryogenesis. In both cases, the 55. Which of the following best summarizes the
observer received a concrete impression that the “turning point” mentioned in the
daughter cell was brought into being, or caused, by the highlighted text?
prior cell. The argument that these scientists employed A. The beginning of the conflict between
confuses temporal succession and causal explanation, proponents of the ideal of historical
of course, but such confusion is the heart of most explanation and the proponents of
historical explanation. experimentation
B. The substitution of historical explanation for
Not surprisingly, the evolutionary biologists of the causal explanation
nineteenth century encountered a particularly C. The shift from interest in historical
troublesome problem in their attempts to document explanation to interest in experimentation
historical explanation convincingly: the factual record D. The attention suddenly paid to problems of
of the history of life on earth (e.g., that provided by organic function
fossils) was incomplete. The temporal continuity of E. The growth of public awareness of the
living forms was convincing, but was an assumption controversies among biologists
that was difficult to uphold when one compared
species or organisms forming any two stages of the 56. The author implies that nineteenth-century
evolutionary record. Nineteenth-century biologists biologists who studied embryogenesis
recognized this problem and attempted to resolve it. believed that they
Their solution today appears to be only verbal, but was A. had discovered physical evidence that
then regarded as eminently causal. The fact of supported their use of historical explanation
evolution demanded some connection between all B. were the first biologists to call for systematic
reproducing individuals and the species that they experimentation on living organisms
compose, as well as between living species and their C. were able to use historical explanation more
extinct ancestors. Their solution, the concept of systematically than were biologists who did
heredity, seemed to fill in an admittedly deficient not study embryogenesis
historical record and seemed to complete the D. had inadvertently discovered an important
argument for a historical explanation of evolutionary part of the factual record of the history of living
events. organisms on earth
E. had avoided the logical fallacies that
characterize the reasoning of most nineteenth-
century biologists
THE MOST AUTHORITATIVE GUIDE EVER WRITTEN ON GMAT READING COMPREHENSION
The final quarter of the nineteenth century marked a 57. The passage would be most likely to appear
turning point in the history of biology—biologists in which of the following?
became less interested in applying an ideal of historical A. An essay investigating the methodology used
explanation deductively to organic function and more by historians of human events
interested in discerning the causes of vital processes B. A book outlining the history of biology in the
through experimental manipulation. But it is nineteenth century
impossible to discuss the history of biology in the C. A seminar paper on the development of
nineteenth century without emphasizing that those embryogenesis as a field of study in
areas of biology most in the public eye had depended nineteenth-century biology
on historical explanation. Wherever it was applied, D. A review of a book whose topic is the discovery
historical explanation was deemed causal explanation. of fossils in the nineteenth century
The biologist-as-historian and the general historian of E. A lecture whose subject is the limitations of
human events dealt with comparable phenomena and experimental investigation in modern biology
assumed necessarily a common mode of explanation.
Passage 15
Critics maintain that the fiction of Herman Melville 58. The author draws which of the following
(1819–1891) has limitations, such as its lack of conclusions about the fact that Melville's
inventive plots after Moby-Dick (1851) and its fiction often does not possess the qualities
occasionally inscrutable style. A more serious, yet of a Jamesian novel?
problematic, charge is that Melville is a deficient writer A. Literary critics should no longer use Jamesian
because he is not a practitioner of the “art of fiction,” as standards to judge the value of novels.
critics have conceived of this art since the late B. Literary critics who have praised Melville's
nineteenth-century essays and novels of Henry James. fiction at the expense of James's fiction should
Indeed, most twentieth-century commentators regard consider themselves justified.
Melville not as a novelist but as a writer of romance, C. Literary critics should no longer attempt to
since they believe that Melville's fiction lacks the place writers, including Melville and James, in
continuity that James viewed as essential to a novel: traditions or categories.
the continuity between what characters feel or think D. Melville and James should be viewed as
and what they do, and the continuity between different sorts of writers and one should not be
characters' fates and their pasts or original social regarded as inherently superior to the other.
classes. Critics argue that only Pierre (1852), because E. Melville and James nevertheless share
of its subject and its characters, is close to being a novel important similarities and these should not be
in the Jamesian sense. overlooked or slighted when literary critics
point out differences between the two writers.
However, although Melville is not a Jamesian novelist,
he is not therefore a deficient writer. A more 59. The author probably mentions Melville's
reasonable position is that Melville is a different kind Pierre to
of writer, who held, and should be judged by, A. refute those literary critics who have made
presuppositions about fiction that are quite different generalizations about the quality of Melville's
from James's. It is true that Melville wrote fiction
“romances”; however, these are not the escapist B. argue that the portrayal of characters is one of
fictions this word often implies, but fictions that Melville's more accomplished literary skills
range freely among very unusual or intense human C. give an example of a novel that was thought by
experiences. Melville portrayed such experiences James to resemble his own fiction
because he believed that these experiences best D. suggest that literary critics find few exceptions
enabled him to explore moral questions, an to what they believe is a characteristic of
exploration he assumed was the ultimate purpose Melville's fiction
of fiction. He was content to sacrifice continuity or E. reinforce the contention of literary critics
even credibility as long as he could establish a
significant moral situation. Thus, Melville's romances 60. Which of the following statements best
do not give the reader a full understanding of the describes the author's method of
complete feelings and thoughts that motivate actions argumentation in the highlighted text?
and events that shape fate. Rather, the romances leave A. The author describes an important standard of
unexplained the sequence of events and either simplify evaluation used by critics of Melville and then
or obscure motives. Again, such simplifications and attacks that standard.
obscurities exist in order to give prominence to the B. The author admits a contention put forward by
depiction of sharply delineated moral values, values critics of Melville but then makes a
derived from a character's purely personal sense of countercharge against those critics.
honor, rather than, as in a Jamesian novel, from the C. The author describes a charge advanced by
conventions of society. critics of Melville and then points out a logical
flaw in this charge.
D. The author provides evidence that seems to
support a position held by critics of Melville
but then demonstrates that the evidence
actually supports a diametrically opposed
position.
E. The author concedes an assertion made by
critics of Melville but then mitigates the weight
of the assertion by means of an explanation.
THE MOST AUTHORITATIVE GUIDE EVER WRITTEN ON GMAT READING COMPREHENSION
Critics maintain that the fiction of Herman Melville 61. Which of the following can logically be
(1819–1891) has limitations, such as its lack of inferred from the passage about the
inventive plots after Moby-Dick (1851) and its author's application of the term “romance”
occasionally inscrutable style. A more serious, yet to Melville's work?
problematic, charge is that Melville is a deficient writer A. The author uses the term in a broader way
because he is not a practitioner of the “art of fiction,” as than did Melville himself.
critics have conceived of this art since the late B. The author uses the term in a different way
nineteenth-century essays and novels of Henry James. than do many literary critics.
Indeed, most twentieth-century commentators regard C. The author uses the term in a more systematic
Melville not as a novelist but as a writer of romance, way than did James.
since they believe that Melville's fiction lacks the D. The author's use of the term is the same as the
continuity that James viewed as essential to a novel: term's usual meaning for twentieth-century
the continuity between what characters feel or think commentators.
and what they do, and the continuity between E. The author's use of the term is less
characters' fates and their pasts or original social controversial than is the use of the term
classes. Critics argue that only Pierre (1852), because “novel” by many commentators.
of its subject and its characters, is close to being a novel
in the Jamesian sense. 62. Which of the following can most logically be
inferred about the author's estimation of
However, although Melville is not a Jamesian novelist, the romantic and novelistic traditions of
he is not therefore a deficient writer. A more fiction?
reasonable position is that Melville is a different kind A. The romantic tradition should be considered at
of writer, who held, and should be judged by, least as valuable as the novelistic tradition in
presuppositions about fiction that are quite different the examination of human experience.
from James's. It is true that Melville wrote B. The romantic tradition should be considered
“romances”; however, these are not the escapist the more vital tradition primarily because
fictions this word often implies, but fictions that Melville is part of that tradition.
range freely among very unusual or intense human C. The romantic tradition should be considered
experiences. Melville portrayed such experiences the superior tradition because it is so
because he believed that these experiences best widespread.
enabled him to explore moral questions, an D. The romantic tradition has had as much
exploration he assumed was the ultimate purpose success in pleasing literary critics as has the
of fiction. He was content to sacrifice continuity or novelistic tradition.
even credibility as long as he could establish a E. The romantic and novelistic traditions have
significant moral situation. Thus, Melville's romances always made important contributions to
do not give the reader a full understanding of the literature, but their most important
complete feelings and thoughts that motivate actions contributions have been in the twentieth
and events that shape fate. Rather, the romances leave century.
unexplained the sequence of events and either simplify
or obscure motives. Again, such simplifications and 63. The author of the passage would be most
obscurities exist in order to give prominence to the likely to agree that a writer's fiction should
depiction of sharply delineated moral values, values be evaluated by which of the following
derived from a character's purely personal sense of criteria?
honor, rather than, as in a Jamesian novel, from the A. How consistently that fiction establishes
conventions of society. credibility with the reader
B. How skillfully that fiction supersedes the
presuppositions or conventions of a tradition
C. How completely that fiction satisfies the
standards of judgment held by most literary
critics
D. How well that fiction fulfills the premises
about fiction maintained by the writer of the
fiction
E. How well that fiction exhibits a continuity of
subject and style over the course of the writer's
career
THE MOST AUTHORITATIVE GUIDE EVER WRITTEN ON GMAT READING COMPREHENSION
Passage 16
Behind every book review there are two key figures: a 64. According to the passage, book review
book review editor and a reviewer. Editors decide editors pay attention to all of the following
whether a book is reviewed in their publication, when in deciding which books should be
the review appears, how long it is, and who writes the reviewed in their publications EXCEPT
review. A. news releases from publishers
B. sales figures compiled by bookstores
When many periodicals feature the same books, this C. the opinions of literary experts
does not prove that the editors of different periodicals D. the probability that the books will be
have not made individual decisions. Before publication, extensively advertised
editors receive news releases and printer's proofs of E. the likelihood that the books will be reviewed
certain books, signifying that the publishers will make in other publications
special efforts to promote these books. They will be
heavily advertised and probably be among the books 65. According to the passage, a major concern
that most bookstores order in quantity. Not having of the unenthusiastic book reviewers in the
such books reviewed might give the impression that highlighted text was
the editor was caught napping, whereas too many A. to ensure prompt payment for their work
reviews of books that readers will have trouble finding B. to influence public opinion of books
in stores would be inappropriate. Editors can risk C. to confirm the opinions of other reviewers
having a few of the less popular titles reviewed, but D. to promote new books by their favorite
they must consider what will be newsworthy, authors
advertised, and written about elsewhere. E. to have their reviews published in the
newspaper
If these were the only factors influencing editors, few
books that stand little chance of selling well would ever 66. The passage suggests which of the
be reviewed. But editors feel some concern about what following about book review readers?
might endure, and therefore listen to literary experts. A. They pay careful attention to reviewers' biases
A generation ago, a newspaper used a brilliant system as they read reviews.
of choosing which books to feature. The book review B. They disapprove of book review editors who
editor sent out a greater number of books than reviews try to influence what their reviewers write.
he actually intended to publish. If a review was C. They use book reviews in order to gauge
unenthusiastic, he reasoned that the book was not whether a book is likely to endure.
important enough to be discussed immediately, and if D. They expect to see timely reviews of widely
good reviews of enough other books came in, the publicized books in the periodicals they read.
unenthusiastic review might never be printed. The E. They are usually willing to search in several
unenthusiastic reviewers were paid promptly stores for a highly recommended book that is
anyway, but they learned that if they wanted their hard to find.
material to be printed, it was advisable to be kind.
67. Which of the following words, if substituted
Most editors print favorable and unfavorable reviews; for “brilliant” in the highlighted text, would
however, the content of the review may be influenced LEAST change the meaning of the sentence?
by the editor. Some editors would actually feel that A. showy
they had failed in their responsibility if they gave books B. articulate
by authors they admired to hostile critics or books by C. literate
authors they disapproved of to critics who might favor D. stingy
them. Editors usually can predict who would review a E. absurd
book enthusiastically and who would tear it to shreds.
THE MOST AUTHORITATIVE GUIDE EVER WRITTEN ON GMAT READING COMPREHENSION
Passage 17
There are two theories that have been used to explain 68. The author objects to the theory that all
ancient and modern tragedy. Neither quite explains the tragedy exhibits the workings of external
complexity of the tragic process or the tragic hero, but fate primarily because
each explains important elements of tragedy, and, A. fate in tragedies is not as important a cause of
because their conclusions are contradictory, they action as is the violation of a moral law
represent extreme views. The first theory states that B. fate in tragedies does not appear to be
all tragedy exhibits the workings of external fate. Of something that is external to the tragic hero
course, the overwhelming majority of tragedies do until after the tragic process has begun
leave us with a sense of the supremacy of impersonal C. the theory is based solely on an understanding
power and of the limitation of human effort. But this of ancient Greek tragedy
theory of tragedy is an oversimplification, primarily D. the theory does not seem to be a plausible
because it confuses the tragic condition with the tragic explanation of tragedy's ability to exhilarate an
process: the theory does not acknowledge that fate, in audience
a tragedy, normally becomes external to the hero only E. the theory does not seem applicable to the
after the tragic process has been set in motion. Fate, as large number of tragedies in which the hero
conceived in ancient Greek tragedy, is the internal overcomes fate
balancing condition of life. It appears as external only
after it has been violated, just as justice is an internal 69. Which of the following comparisons of the
quality of an honest person, but the external antagonist tragic with the ironic hero is best supported
of the criminal. Secondarily, this theory of tragedy does by information contained in the passage?
not distinguish tragedy from irony. Irony does not need A. A tragic hero's fate is an external condition, but
an exceptional central figure: as a rule, the more an ironic hero's fate is an internal one.
ignoble the hero the sharper the irony, when irony B. A tragic hero must be controlled by fate, but an
alone is the objective. It is heroism that creates the ironic hero cannot be.
splendor and exhilaration that is unique to tragedy. C. A tragic hero's moral flaw surprises the
The tragic hero normally has an extraordinary, often a audience, but an ironic hero's sin does not.
nearly divine, destiny almost within grasp, and the D. A tragic hero and an ironic hero cannot both be
glory of that original destiny never quite fades out of virtuous figures in the same tragedy.
the tragedy. E. A tragic hero is usually extraordinary, but an
ironic hero may be cowardly or even
The second theory of tragedy states that the act that villainous.
sets the tragic process in motion must be primarily a
violation of moral law, whether human or divine; in 70. The author contrasts an honest person and
short, that the tragic hero must have a flaw that has an a criminal primarily to
essential connection with sin. Again, it is true that the A. prove that fate cannot be external to the tragic
great majority of tragic heroes do possess hubris, or a hero
proud and passionate mind that seems to make the B. establish a criterion that allows a distinction to
hero's downfall morally explicable. But such hubris is be made between irony and tragedy
only the precipitating agent of catastrophe, just as in C. develop the distinction between the tragic
comedy the cause of the happy ending is usually some condition and the tragic process
act of humility, often performed by a noble character D. introduce the concept of sin as the cause of
who is meanly disguised. tragic action
E. argue that the theme of omnipotent external
fate is shared by comedy and tragedy
THE MOST AUTHORITATIVE GUIDE EVER WRITTEN ON GMAT READING COMPREHENSION
There are two theories that have been used to explain 71. The author suggests that the tragic hero's
ancient and modern tragedy. Neither quite explains the “original destiny never quite fades out of
complexity of the tragic process or the tragic hero, but the tragedy” primarily to
each explains important elements of tragedy, and, A. qualify the assertion that the theory of tragedy
because their conclusions are contradictory, they as a display of external fate is inconsistent
represent extreme views. The first theory states that B. introduce the discussion of the theory that
all tragedy exhibits the workings of external fate. Of tragedy is the consequence of individual sin
course, the overwhelming majority of tragedies do C. refute the theory that the tragic process is
leave us with a sense of the supremacy of impersonal more important than the tragic condition
power and of the limitation of human effort. But this D. support the claim that heroism creates the
theory of tragedy is an oversimplification, primarily splendor and exhilaration of tragedy
because it confuses the tragic condition with the tragic E. distinguish between fate as conceived in
process: the theory does not acknowledge that fate, in ancient Greek tragedy and fate in more recent
a tragedy, normally becomes external to the hero only tragedy
after the tragic process has been set in motion. Fate, as
conceived in ancient Greek tragedy, is the internal 72. In the author's opinion, an act of humility in
balancing condition of life. It appears as external only comedy is most analogous to
after it has been violated, just as justice is an internal A. a catastrophe in tragedy
quality of an honest person, but the external antagonist B. an ironic action in tragedy
of the criminal. Secondarily, this theory of tragedy does C. a tragic hero's pride and passion
not distinguish tragedy from irony. Irony does not need D. a tragic hero's aversion to sin
an exceptional central figure: as a rule, the more E. a tragic hero's pursuit of an unusual destiny
ignoble the hero the sharper the irony, when irony
alone is the objective. It is heroism that creates the
splendor and exhilaration that is unique to tragedy.
The tragic hero normally has an extraordinary, often a
nearly divine, destiny almost within grasp, and the
glory of that original destiny never quite fades out of
the tragedy.
Passage 18
Though most tennis players generally strive to strike 73. The author mentions “a penny that has
the ball on the racket's vibration node, more commonly been struck near its edge” in order to
known as the “sweet spot,” many players are unaware A. show how the center of mass causes the racket
of the existence of a second, lesser-known location on to spin
the racket face, the center of percussion, that will also B. argue that a penny spins in the exact way that
greatly diminish the strain on a player's arm when the a tennis racket spins
ball is struck. C. explain how translational motion works
D. provide an illustration of a concept
In order to understand the physics of this second sweet E. demonstrate that pennies and tennis rackets
spot, it is helpful to consider what would happen to a do not spin in the same way
tennis racket in the moments after impact with the ball
if the player's hand were to vanish at the moment of 74. According to the passage, all of the
impact. The impact of the ball would cause the racket following are true of the forces acting upon
to bounce backwards, experiencing a translational a tennis racket striking a ball EXCEPT
motion away from the ball. The tendency of this motion A. the only way to eliminate the jolt that
would be to jerk all parts of the racket, including the accompanies most strokes is to hit the ball on
end of its handle, backward, or away from the ball. the center of percussion
Unless the ball happened to hit the racket precisely at B. the impact of the ball striking the racket can
the racket's center of mass, the racket would strain a tennis player's arm
additionally experience a rotational motion around its C. there are at least two different forces acting
center of mass—much as a penny that has been upon the racket
struck near its edge will start to spin. Whenever the D. the end of the handle of the racket will jerk
ball hits the racket face, the effect of this rotational forward after striking the ball unless the ball
motion will be to jerk the end of the handle forward, strikes the racket's center of mass
towards the ball. Depending on where the ball strikes E. the racket will rebound after it strikes the ball
the racket face, one or the other of these motions will
predominate. 75. Which of the following can be inferred from
the passage?
However, there is one point of impact, known as the A. If a player holds the tennis racket anywhere
center of percussion, which causes neither motion to other than the end of the handle, the player will
predominate; if a ball were to strike this point, the experience a jolting sensation.
impact would not impart any motion to the end of the B. The primary sweet spot is more effective at
handle. The reason for this lack of motion is that the damping vibration than is the secondary sweet
force on the upper part of the hand would be equal and spot.
opposite to the force on the lower part of the hand, C. Striking a tennis ball at a spot other than the
resulting in no net force on the tennis players’ hand or center of percussion may result in a jarring
forearm. The center of percussion constitutes a second feeling.
sweet spot because a tennis player's wrist typically is D. Striking a tennis ball repeatedly at spots other
placed next to the end of the racket's handle. When the than a sweet spot leads to “tennis elbow.”
player strikes the ball at the center of percussion, her E. If a player lets go of the racket at the moment
wrist is jerked neither forward nor backward, and she of impact, the simultaneous forward and
experiences a relatively smooth, comfortable tennis backward impetus causes the racket to drop
stroke. straight to the ground.
Though most tennis players generally strive to strike 76. It can be inferred that a tennis ball that
the ball on the racket's vibration node, more commonly strikes a racket's center of percussion will
known as the “sweet spot,” many players are unaware do which of the following?
of the existence of a second, lesser-known location on I. Cause the racket to bounce backwards
the racket face, the center of percussion, that will also II. Not cause the wrist to jerk
greatly diminish the strain on a player's arm when the III. Allow for a cleaner stroke than a ball striking a
ball is struck. racket's primary sweet spot
Passage 19
It is common to assume that people who act morally 78. The author refers to a study by Khan and
are normally motivated to act so, while those who act Dhar in order to demonstrate that
immorally lack moral motivation. However, research A. individuals can perform moral actions without
carried out by Monin and Miller strongly points to the giving themselves moral license
existence of a moral licensing effect, according to which B. there is no correlation between individuals’
performance of morally commendable actions in turn giving themselves moral license and
licenses subjects to later engage in dubious behavior. individuals’ moral behavior
This work suggests that individuals are motivated to C. there is no relation between individuals’ moral
perform moral acts not by an innate goodness on their self-image and their moral standards
part, but by a desire to appear to be moral people, D. individuals appear to take any kind of moral
whether to themselves or others. Researchers continue behavior to count toward or against their
to disagree whether licensing is best understood as moral credits or credentials
involving moral credentials, in which subjects E. individuals can give themselves moral license
interpret their later, dubious activity in terms of the without performing any moral action
moral identity they have supposedly established for
themselves in prior acts, or as involving moral credits, 79. Which of the following results of a later
where subjects see themselves as having built up a study, if true, would weaken the conclusion
surplus that they can then “trade in” by acting selfishly. stated in the last sentence of the passage?
The research strongly suggests, however, that I. Subjects who, on a scale of generous to selfish,
individuals care about their moral self-image (how rated themselves as more generous were no
they think about themselves morally), and not merely more or less likely to act generously than
about what others think of them. Khan and Dhar, for subjects who rated themselves as more selfish.
example, found that subjects who were asked to II. Subjects who acted the most generously in a
merely imagine performing a generous act, such as spontaneous situation were largely unable to
tutoring a student, were later less likely to donate a identify clear reasons for their actions.
portion of their earnings to charity than those who III. Subjects who acted generously in the first
were asked to imagine engaging in a morally neutral phase of an experiment were less likely to act
activity; that is, those who simply imagined their own generously in the next phase of the
generosity then gave themselves license to be selfish. If experiment.
individuals are normally motivated to at least strive for
a moral self-image in this way, we should not be too A. I only
hasty to discount the common-sense assumption that B. II only
moral motivation is typically behind moral action. C. I and II only
D. I and III only
E. II and III only
THE MOST AUTHORITATIVE GUIDE EVER WRITTEN ON GMAT READING COMPREHENSION
Passage 20
Before the age of space exploration, the size and 80. The author mentioned "gravity map" in the
composition of the Moon's core were astronomical second paragraph in order to
mysteries. Astronomers assumed that the Moon's core A. illustrate the method used to correct the Lunar
was smaller than that of the Earth, in both relative and Prospector's velocity
absolute terms—the radius of the Earth's core is 55 B. confirm the results obtained by examining the
percent of the overall radius of the Earth and the core's Moon's magnetic field
mass is 32 percent of the Earth's overall mass—but C. detail a step in the investigation of the
they had no way to verify their assumption. However, composition of a specific orbiting body
data gathered by Lunar Prospector have now given D. detail a step in the investigation of the effects
astronomers the ability to determine that the Moon's of gravity on the Moon and the Earth
core accounts for 20 percent of the Moon's radius and E. explain a step in the process of confirming a
for a mere 2 percent of its overall mass. new hypothesis and discarding an older theory
Two sets of data have been analyzed. As part of the 81. If the wavelength of radio signals emitted
Doppler Gravity Experiment, scientists measured by a second Lunar Prospector changed only
minute, relatively rapid variations in the wavelength of in a slowly oscillating fashion, then which of
radio signals from Lunar Prospector as the craft moved the following could be most properly
towards or away from the Earth. Just as shifts in the inferred?
pitch of a siren can indicate the speed and direction of A. The velocity of the second spacecraft appears
travel of a police car, so did these radio-signal to be constant over time.
variations reveal even the slightest changes in the B. The size and mass of the Moon’s core may be
craft's velocity while the craft orbited the Moon. These more comparable to the size and mass of the
changes resulted from inconsistency in the Earth’s core than is currently believed.
gravitational pull of the Moon on the craft and C. The gravitational pull of the Moon seems to be
permitted the creation of a "gravity map" of both near more uniform than was measured by the first
and far sides of the Moon, highlighting new details of spacecraft.
the distribution of the Moon's internal mass. Through D. The Earth and the Moon seem to have
the analysis of lunar mass concentrations, scientists originated independently.
have been able to determine that the Moon has a small, E. It would be impossible to create a gravity map
metallic core, which, if composed mostly of iron, has a of the Moon.
radius of approximately 350 kilometers. The second
data set was gathered by capturing echoes of the faint 82. According to the passage, electrical
magnetic field generated by electrical currents within currents within the Moon’s core
the lunar core caused by the Moon’s monthly passage A. capture echoes of a faint magnetic field
through the tail of the Earth's magnetosphere. These B. are generated when the Moon moves through
magnetic data confirmed the results obtained through the Earth’s magnetosphere
examination of the gravity map. C. call the results of the radio signal analysis into
question
The size and composition of the Moon's core are not D. permit the creation of a gravity map
academic concerns; they have serious implications for corresponding to lunar mass concentrations
our understanding of the Moon's origins. For example, E. indicate the speed and direction of travel of the
if the Moon and Earth developed as distinct entities, the Moon
sizes of their cores should be more comparable. In
actuality, it seems that the Moon was once part of the
Earth and broke away at an early stage in the Earth's
evolution, perhaps as the result of a major asteroid
impact. The impact could have loosened iron that had
not already sunk to the core of the Earth, allowing it to
form the core around which the Moon eventually
accreted. Alternatively, according to fission theory, the
early Earth may have been spinning so rapidly that it
ejected a quantity of material by so-called centrifugal
force, material that later coalesced by mutual
gravitational attraction into the Moon.
THE MOST AUTHORITATIVE GUIDE EVER WRITTEN ON GMAT READING COMPREHENSION
Before the age of space exploration, the size and 83. According to the passage, scientists
composition of the Moon's core were astronomical employed one research method that
mysteries. Astronomers assumed that the Moon's core measured
was smaller than that of the Earth, in both relative and A. changes in a spacecraft’s velocity as it returned
absolute terms—the radius of the Earth's core is 55 to Earth
percent of the overall radius of the Earth and the core's B. changes in the gravitational pull on the Lunar
mass is 32 percent of the Earth's overall mass—but Prospector as it orbited the moon
they had no way to verify their assumption. However, C. changes in the moon’s gravity as it orbited the
data gathered by Lunar Prospector have now given earth
astronomers the ability to determine that the Moon's D. changes in a spacecraft’s radio signals as it
core accounts for 20 percent of the Moon's radius and changed position relative to the earth
for a mere 2 percent of its overall mass. E. changes in the moon’s gravitational pull over
time
Two sets of data have been analyzed. As part of the
Doppler Gravity Experiment, scientists measured 84. It can be inferred from the passage that:
minute, relatively rapid variations in the wavelength of A. the earth’s core contributes a higher
radio signals from Lunar Prospector as the craft moved proportion of both the earth’s total radius and
towards or away from the Earth. Just as shifts in the its total mass than the core of the moon
pitch of a siren can indicate the speed and direction of contributes to its own total radius and mass
travel of a police car, so did these radio-signal B. a comparison of the earth’s and moon’s cores
variations reveal even the slightest changes in the suggests that the earth did break away from
craft's velocity while the craft orbited the Moon. These the moon
changes resulted from inconsistency in the C. scientists will eventually be able to use the
gravitational pull of the Moon on the craft and gravity map to further the proof that a major
permitted the creation of a "gravity map" of both near asteroid impact caused the earth and moon to
and far sides of the Moon, highlighting new details of separate
the distribution of the Moon's internal mass. Through D. because the earth and moon were once joined,
the analysis of lunar mass concentrations, scientists it is probable that scientists will eventually
have been able to determine that the Moon has a small, discover fossils and other evidence on the
metallic core, which, if composed mostly of iron, has a moon that indicates that a major asteroid
radius of approximately 350 kilometers. The second impact caused the extinction of the dinosaurs
data set was gathered by capturing echoes of the faint E. the moon’s core has a higher ratio of iron in its
magnetic field generated by electrical currents within composition than does the earth’s
the lunar core caused by the Moon’s monthly passage
through the tail of the Earth's magnetosphere. These
magnetic data confirmed the results obtained through
examination of the gravity map.
Passage 21
The evolution of intelligence among early large mammals 85. It can be inferred from the passage that in
of the grasslands was due in great measure to the animals less intelligent than the mammals
interaction between two ecologically synchronized discussed in the passage
groups of these animals, the hunting carnivores and the A. past experience is less helpful in ensuring
herbivores that they hunted. The interaction resulting survival
from the differences between predator and prey led to a B. attention is more highly focused
general improvement in brain functions; however, certain C. muscular coordination is less highly developed
components of intelligence were improved far more than D. there is less need for competition among
others.
species
The kind of intelligence favored by the interplay of
E. environment is more important in establishing
increasingly smarter catchers and increasingly keener
escapers is defined by attention — that aspect of mind
the proper ratio of prey to predator
carrying consciousness forward from one moment to the
next. It ranges from a passive, free-floating awareness to 86. According to the passage, improvement in
a highly focused, active fixation. The range through these brain function among early large mammals
states is mediated by the arousal system, a network of resulted primarily from which of the
tracts converging from sensory systems to integrating following?
centers in the brain stem. From the more relaxed to the A. Interplay of predator and prey
more vigorous levels, sensitivity to novelty is increased. B. Persistence of free-floating awareness in
The organism is more awake, more vigilant; this animals of the grasslands
increased vigilance results in the apprehension of ever C. Gradual dominance of warm-blooded
more subtle signals as the organism becomes more mammals over cold-blooded reptiles
sensitive to its surroundings. The processes of arousal D. Interaction of early large mammals with less
and concentration give attention its direction. Arousal is intelligent species
at first general, with a flooding of impulses in the brain E. Improvement of the capacity for memory
stem; then gradually the activation is channeled. Thus, among herbivores and carnivores
begins concentration, the holding of consistent images.
One meaning of intelligence is the way in which these 87. According to the passage, as the process of
images and other alertly searched information are used in arousal in an organism continues, all of the
the context of previous experience. Consciousness links
following may occur EXCEPT
past attention to the present and permits the integration
A. the production of adrenaline
of details with perceived ends and purposes.
B. the production of norepinephrine
The elements of intelligence and consciousness come
together marvelously to produce different styles in C. a heightening of sensitivity to stimuli
predator and prey. Herbivores and carnivores develop D. an increase in selectivity with respect to
different kinds of attention related to escaping or chasing. stimuli
Although in both kinds of animal, arousal stimulates the E. an expansion of the range of states mediated
production of adrenaline and norepinephrine by the by the brain stem
adrenal glands, the effect in herbivores is primarily fear,
whereas in carnivores the effect is primarily aggression. 88. The sensitivity is most clearly an example
For both, arousal attunes the animal to what is ahead. of
Perhaps it does not experience forethought as we know it, A. “Free-floating awareness”
but the animal does experience something like it. The B. “Flooding of impulses in the brain stem”
predator is searchingly aggressive, inner-directed, tuned C. “The holding of consistent images”
by the nervous system and the adrenal hormones, but D. “Integration of details with perceived ends and
aware in a sense closer to human consciousness than, say, purposes”
a hungry lizard’s instinctive snap at a passing beetle. E. “Silk-thin veils of tranquility”
Using past events as a framework, the large mammal
predator is working out a relationship between
movement and food, sensitive to possibilities in cold trails
and distant sounds — and yesterday’s unforgotten
lessons. The herbivore prey is of a different mind. Its
mood of wariness rather than searching and its attitude
of general expectancy instead of anticipating are silk-thin
veils of tranquility over an explosive endocrine system.
THE MOST AUTHORITATIVE GUIDE EVER WRITTEN ON GMAT READING COMPREHENSION
The evolution of intelligence among early large 89. The author provides information that
mammals of the grasslands was due in great measure would answer which of the following
to the interaction between two ecologically questions?
synchronized groups of these animals, the hunting
carnivores and the herbivores that they hunted. The I. Why is an aroused herbivore usually
interaction resulting from the differences between fearful?
predator and prey led to a general improvement in II. What are some of the degrees of
brain functions; however, certain components of attention in large mammals?
intelligence were improved far more than others. III. What occurs when the stimulus that
The kind of intelligence favored by the interplay of causes arousal of a mammal is
increasingly smarter catchers and increasingly keener removed?
escapers is defined by attention — that aspect of mind
carrying consciousness forward from one moment to A. I only
the next. It ranges from a passive, free-floating B. III only
awareness to a highly focused, active fixation. The C. I and II only
range through these states is mediated by the arousal D. II and III only
system, a network of tracts converging from sensory E. I, II and III
systems to integrating centers in the brain stem. From
the more relaxed to the more vigorous levels,
sensitivity to novelty is increased. The organism is
more awake, more vigilant; this increased vigilance
results in the apprehension of ever more subtle signals
as the organism becomes more sensitive to its
surroundings. The processes of arousal and
concentration give attention its direction. Arousal is at
first general, with a flooding of impulses in the brain
stem; then gradually the activation is channeled. Thus,
begins concentration, the holding of consistent images.
One meaning of intelligence is the way in which these
images and other alertly searched information are
used in the context of previous experience.
Consciousness links past attention to the present and
permits the integration of details with perceived ends
and purposes.
The elements of intelligence and consciousness come
together marvelously to produce different styles in
predator and prey. Herbivores and carnivores develop
different kinds of attention related to escaping or
chasing. Although in both kinds of animal, arousal
stimulates the production of adrenaline and
norepinephrine by the adrenal glands, the effect in
herbivores is primarily fear, whereas in carnivores the
effect is primarily aggression. For both, arousal attunes
the animal to what is ahead. Perhaps it does not
experience forethought as we know it, but the animal
does experience something like it. The predator is
searchingly aggressive, inner-directed, tuned by the
nervous system and the adrenal hormones, but aware
in a sense closer to human consciousness than, say, a
hungry lizard’s instinctive snap at a passing beetle.
Using past events as a framework, the large mammal
predator is working out a relationship between
movement and food, sensitive to possibilities in cold
trails and distant sounds — and yesterday’s
unforgotten lessons. The herbivore prey is of a
different mind. Its mood of wariness rather than
searching and its attitude of general expectancy
instead of anticipating are silk-thin veils of tranquility
over an explosive endocrine system.
THE MOST AUTHORITATIVE GUIDE EVER WRITTEN ON GMAT READING COMPREHENSION
Passage 22
Commonplace items sometimes play surprising roles 90. The second paragraph performs which of
in world development. For example, though most the following functions in the passage?
people today associate nutmeg with simple baked A. It offers specific information to complete the
goods, this common spice once altered the course of logic of the author’s claims.
political history. For centuries, the nutmeg tree grew B. It summarizes and evaluates the evidence
only in the Banda Islands, a small chain in the given thus far.
southwest Pacific. Locals harvested the aromatic nuts C. It presents the author’s main point to explain a
of the tree and sold them to traders. Eventually these unique situation.
nuts, from which the spice is made, ended up as a D. It cites a particular case to demonstrate the
luxury item in the European market, via Venetian spice importance of historical change.
merchants. Eager to establish a monopoly over this E. It discusses the necessary outcome of the
valuable spice, the Dutch attacked the Bandas, author’s assertions.
subjugating the native people in a mostly successful
attempt to control the nutmeg trade. 91. Which of the following, if true, most
strengthens the claim that New Amsterdam
However, one island in the Banda chain remained in would have remained a Dutch possession if
the hands of the British and was the object of much not for the conflict over nutmeg?
conflict between the Netherlands and England. After A. Attempts to cultivate nutmeg trees outside of
many battles, the British offered to cede control of the the Banda Islands had failed.
island in exchange for New Amsterdam, a Dutch B. Few people lived in New Amsterdam before it
outpost on the east coast of North America. At the time, was ceded to the British.
the Dutch, inveterate traders, were more interested in C. The British controlled trade in other valuable
the spice trade than in the mercantile value of New spices, such as cloves.
Amsterdam and so accepted the offer. In 1667, the D. New Amsterdam served as a trading center for
Treaty of Breda gave the Dutch complete control of the furs exported to Europe.
Banda Islands, and thus of the nutmeg trade, and gave E. The Netherlands controlled no North
the British New Amsterdam, which they promptly American territories other than New
renamed New York. Amsterdam.
Passage 23
Nearly a century ago, biologists found that if they 94. It can be inferred from the passage that the
separated an invertebrate animal embryo into two morphogenetic determinants present in
parts at an early stage of its life, it would survive and the early embryo are
develop as two normal embryos. This led them to A. located in the nucleus of the embryo cells
believe that the cells in the early embryo are B. evenly distributed unless the embryo is not
undetermined in the sense that each cell has the developing normally
potential to develop in a variety of different ways. Later C. inactive until the embryo cells become
biologists found that the situation was not so simple. It irreversibly committed to their final function
matters in which plane the embryo is cut. If it is cut in D. identical to those that were already present in
a plane different from the one used by the early the unfertilized egg
investigators, it will not form two whole embryos. E. present in larger quantities than is necessary
for the development of a single individual
A debate arose over what exactly was happening.
Which embryo cells are determined, just when do they 95. It can be inferred from the passage that
become irreversibly committed to their fates, and what when biologists believed that the cells in
are the “morphogenetic determinants” that tell a cell the early embryo were undetermined, they
what to become? But the debate could not be resolved made which of the following mistakes?
because no one was able to ask the crucial questions in A. They did not attempt to replicate the original
a form in which they could be pursued productively. experiment of separating an embryo into two
Recent discoveries in molecular biology, however, parts.
have opened up prospects for a resolution of the B. They did not realize that there was a
debate. Now investigators think they know at least connection between the issue of cell
some of the molecules that act as morphogenetic determination and the outcome of the
determinants in early development. They have been separation experiment.
able to show that, in a sense, cell determination begins C. They assumed that the results of experiments
even before an egg is fertilized. on embryos did not depend on the particular
animal species used for such experiments.
Studying sea urchins, biologist Paul Gross found that an D. They assumed that it was crucial to perform
unfertilized egg contains substances that function as the separation experiment at an early stage in
morphogenetic determinants. They are located in the the embryo’s life.
cytoplasm of the egg cell; i.e., in that part of the cell’s E. They assumed that different ways of
protoplasm that lies outside of the nucleus. In the separating an embryo into two parts would be
unfertilized egg, the substances are inactive and are equivalent as far as the fate of the two parts
not distributed homogeneously. When the egg is was concerned.
fertilized, the substances become active and,
presumably, govern the behavior of the genes they 96. It can be inferred from the passage that the
interact with. Since the substances are unevenly initial production of histones after an egg is
distributed in the egg, when the fertilized egg divides, fertilized takes place
the resulting cells are different from the start and so A. in the cytoplasm
can be qualitatively different in their own gene activity. B. in the maternal genes
The substances that Gross studied are maternal C. throughout the protoplasm
messenger RNA’s—products of certain of the maternal D. in the beaded portions of the DNA strings
genes. He and other biologists studying a wide variety E. in certain sections of the cell nucleus
of organisms have found that these particular RNA’s
direct, in large part, the synthesis of histones, a class of 97. It can be inferred from the passage that
proteins that bind to DNA. Once synthesized, the which of the following is dependent on the
histones move into the cell nucleus, where section of fertilization of an egg?
DNA wrap around them to form a structure that A. Copying of maternal genes to produce
resembles beads, or knots, on a string. The beads are maternal messenger RNA’s
DNA segments wrapped around the histones; the B. Synthesis of proteins called histones
string is the intervening DNA. And it is the structure of C. Division of a cell into its nucleus and the
these beaded DNA strings that guide the fate of the cells cytoplasm
in which they are located. D. Determination of the egg cell’s potential for
division
E. Generation of all of a cell’s morphogenetic
determinants
THE MOST AUTHORITATIVE GUIDE EVER WRITTEN ON GMAT READING COMPREHENSION
Nearly a century ago, biologists found that if they 98. Which of the following circumstances is
separated an invertebrate animal embryo into two most comparable to the impasse biologists
parts at an early stage of its life, it would survive and encountered in trying to resolve the debate
develop as two normal embryos. This led them to about cell determination?
believe that the cells in the early embryo are A. The problems faced by a literary scholar who
undetermined in the sense that each cell has the wishes to use original source materials that are
potential to develop in a variety of different ways. Later written in an unfamiliar foreign language
biologists found that the situation was not so simple. It B. The situation of a mathematician who in
matters in which plane the embryo is cut. If it is cut in preparing a proof of a theorem for publication
a plane different from the one used by the early detects a reasoning error in the proof
investigators, it will not form two whole embryos. C. The difficulties of a space engineer who has to
design equipment to function in an
A debate arose over what exactly was happening. environment in which it cannot first be tested
Which embryo cells are determined, just when do they D. The predicament of a linguist trying to develop
become irreversibly committed to their fates, and what a theory of language acquisition when
are the “morphogenetic determinants” that tell a cell knowledge of the structure of language itself is
what to become? But the debate could not be resolved rudimentary at best
because no one was able to ask the crucial questions in E. The dilemma confronting a foundation when
a form in which they could be pursued productively. the funds available to it are sufficient to
Recent discoveries in molecular biology, however, support one of two equally deserving scientific
have opened up prospects for a resolution of the projects but not both
debate. Now investigators think they know at least
some of the molecules that act as morphogenetic
determinants in early development. They have been
able to show that, in a sense, cell determination begins
even before an egg is fertilized.
Passage 24
Many readers assume that, as a neoclassical literary critic, 99. The passage implies that the judging of
Samuel Johnson would normally prefer the abstract, the literary works according to preconceived
formal, and the regulated to the concrete, the natural, and rules
the spontaneous in a work of literature. Yet any close A. tends to lessen the effectiveness of much modern
reading of Johnson’s criticism shows that Johnson is not literary criticism
blind to the importance of the immediate, vivid, specific B. is the primary distinguishing mark of the
detail in literature; rather, he would underscore the need neoclassical critic
for the telling rather than the merely accidental detail. C. was the primary neoclassical technique against
which the Romantics rebelled
In other ways, too, Johnson’s critical method had much in D. is the underlying basis of much of Johnson’s
common with that of the Romantics, with whom Johnson critical work
and, indeed, the entire neoclassical tradition, are E. characterizes examples of bad neoclassical
generally supposed to be in conflict. Johnson was well criticism
aware, for example, of the sterility of literary criticism that
is legalistic or pedantic, as was the case with the worst 100. According to the author, Johnson’s defense of
products of the neoclassical school. His famous argument Shakespeare’s latest plays illustrates
against the slavish following of the “three unities” of Johnson’s reliance on which of the following
classical drama is a good example, as is his defense of the in his criticism?
supposedly illegitimate “tragicomic” mode of A. The sentiments endorsed by the Romantics
Shakespeare’s latest plays. Note, in particular, the basis of B. The criteria set forth by Wordsworth in his 1800
that defense: “That this is a practice contrary to the rules preface to the Lyrical Ballads
of criticism,” Johnson wrote, “will be readily allowed; but C. The precedents established by the Greek and
there is always an appeal from criticism to nature.” Roman playwrights of the Classical Age
D. The principles followed by the neoclassical
The sentiment thus expressed could easily be endorsed by school of criticism
any of the Romantics; the empiricism it exemplifies is vital E. His own experience and judgment
quality of Johnson’s criticism, as is the willingness to
jettison “laws” of criticism when to do so makes possible 101. According to the passage, Johnson’s views on
a more direct appeal to the emotions of the reader. the use of a special diction in the writing of
Addison’s Cato, highly praised in Johnson’s day for its poetry were
“correctness,” is damned with faint praise by Johnson: A. “modern” in their rejection of a clear-cut division
“Cato affords a splendid exhibition of artificial and between the diction of poetry and that of prose
fictitious manners, and delivers just and noble sentiments, B. “neoclassical” in their emphasis on the use of
in diction easy, elevated, and harmonious, but its hopes language that appeals directly to the emotions of
and fears communicate no vibration to the heart.” the reader
Wordsworth could hardly demur. C. “Romantic” in their defense of the idea that a
special diction for poetry could be stylistically
Even on the question of poetic diction, which, according to effective
the usual interpretation of Wordsworth’s 1800 preface to D. “modern” in their underlying concern for the
the Lyrical Ballads, was the central area of conflict impact of the literary work on the sensibility of
between Romantic and Augustan, Johnson’s views are the reader
surprisingly “modern.” In his Life of Dryden, he defends E. “neoclassical” in their emphasis on ease of
the use of a special diction in poetry, it is true; but his comprehension as a literary virtue
reasons are all-important. For Johnson, poetic diction
should serve the ends of direct emotional impact and ease 102. It can be inferred from the passage that in
of comprehension, not those of false profundity or addition to being a literary critic, Johnson was
grandiosity. “Words too familiar,” he wrote, “or too remote, also a
defeat the purpose of a poet. From those sounds which we A. surprisingly modern poet
hear on small or on coarse occasions, we do not easily B. poet in the Augustan mode
receive strong impressions, or delightful images; and C. dramatist
words to which we are nearly strangers, whenever they D. biographer
occur, draw that attention on themselves which they E. naturalist
should transmit to things.” If the poetic diction of the
neoclassical poets, at its worst, erects needless barriers
between reader and meaning, that envisioned by Johnson
would do just the opposite: it would put the reader in
closer contact with the “things” that are the poem’s
subject.
THE MOST AUTHORITATIVE GUIDE EVER WRITTEN ON GMAT READING COMPREHENSION
Passage 25
All of the cells in a particular plant start out with the 103. According to the passage, the five well-
same complement of genes. How then can these cells known plant hormones are not useful in
differentiate and form structures as different as roots, controlling the growth of crops because
stems, leaves, and fruits? The answer is that only a A. it is not known exactly what functions the
small subset of the genes in a particular kind of cell are hormones perform
expressed, or turned on, at a given time. This is B. each hormone has various effects on plants
accomplished by a complex system of chemical C. none of the hormones can function without the
messengers that in plants include hormones and other others
regulatory molecules. Five major hormones have been D. each hormone has different effects on different
identified: auxin, abscisic acid, cytokinin, ethylene, and kinds of plants
gibberellin. Studies of plants have now identified a new E. each hormone works on only a small subset of
class of regulatory molecules called oligosaccharins. a cell’s genes at any particular time
Unlike the oligosaccharins, the five well-known plant 104. The passage suggests that the place of
hormones are pleiotropic rather than specific; that is, hypothalamic hormones in the hormonal
each has more than one effect on the growth and hierarchies of animals is similar to the
development of plants. The five has so many place of which of the following in plants?
simultaneous effects that they are not very useful in A. Plant cell walls
artificially controlling the growth of crops. Auxin, for B. The complement of genes in each plant cell
instance, stimulates the rate of cell elongation, causes C. A subset of a plant cell’s gene complement
shoots to grow up and roots to grow down, and inhibits D. The five major hormones
the growth of lateral shoots. Auxin also causes the E. The oligosaccharins
plant to develop a vascular system, to form lateral
roots, and to produce ethylene. 105. The passage suggests that which of the
following is a function likely to be
The pleiotropy of the five well-studied plant hormones performed by an oligosaccharin?
is somewhat analogous to that of certain hormones in A. To stimulate a particular plant cell to become
animal. For example, hormones from the part of a plant’s root system
hypothalamus in the brain stimulate the anterior lobe B. To stimulate the walls of a particular cell to
of the pituitary gland to synthesize and release many produce other oligosaccharins
different hormones, one of which stimulates the C. To activate enzymes that release specific
release of hormones from the adrenal cortex. These chemical messengers from plant cell walls
hormones have specific effects on target organs all D. To duplicate the gene complement in a
over the body. One hormone stimulates the thyroid particular plant cell
gland, for example, another the ovarian follicle cells, E. To produce multiple effects on a particular
and so forth. In other words, there is a hierarchy of subsystem of plant cells
hormones. Such a hierarchy may also exist in plants.
Oligosaccharins are fragments of the cell wall released 106. The passage suggests that, unlike the
by enzymes: different enzymes release different pleiotropic hormones, oligosaccharins
oligosaccharins. There are indications that pleiotropic could be used effectively to
plant hormones may actually function by activating the A. trace the passage of chemicals through the
enzymes that release these other, more specific walls of cells
chemical messengers from the cell wall. B. pinpoint functions of other plant hormones
C. artificially control specific aspects of the
development of crops
D. alter the complement of genes in the cells of
plants
E. alter the effects of the five major hormones on
plant development
All RC Passages from GMAT Official Guide
2021
Scientists long believed that two nerve clusters in 503. The primary purpose of the passage is to
the human hypothalamus, called suprachiasmatic A. challenge recent findings that appear to
nuclei (SCNs), were what controlled our circadian contradict earlier findings
rhythms. Those rhythms are the biological cycles B. present two sides of an ongoing scientific
that recur approximately every 24 hours in debate
synchronization with the cycle of sunlight and C. report answers to several questions that
darkness caused by Earth's rotation. Studies have have long puzzled researchers
demonstrated that in some animals, the SCNs D. discuss evidence that has caused a long-
control daily fluctuations in blood pressure, body standing belief to be revised
temperature, activity level, and alertness, as well E. attempt to explain a commonly
as the nighttime release of the sleep-promoting misunderstood biological phenomenon
agent melatonin. Furthermore, cells in the human
retina dedicated to transmitting information 504. The passage mentions each of the following as
about light levels to the SCNs have recently been a function regulated by the SCNs in some
discovered. animals EXCEPT
A. activity level
Four critical genes governing circadian cycles B. blood pressure
have been found to be active in every tissue, C. alertness
however, not just the SCNs, of flies, mice, and D. vision
humans. In addition, when laboratory rats that E. temperature
usually ate at will were fed only once a day, peak
activity of a clock gene in their livers shifted by 12 505. The author of the passage would probably
hours, whereas the same clock gene in the SCNs agree with which of the following statements
remained synchronized with light cycles. While about the SCNs?
scientists do not dispute the role of the SCNs in A. The SCNs are found in other organs and
controlling core functions such as the regulation tissues of the body besides the
of body temperature and blood pressure, hypothalamus.
scientists now believe that circadian clocks in B. The SCNs play a critical but not exclusive
other organs and tissues may respond to external role in regulating circadian rhythms.
cues other than light—including temperature C. The SCNs control clock genes in a number
changes—that recur regularly every 24 hours. of tissues and organs throughout the body.
D. The SCNs are a less significant factor in
regulating blood pressure than scientists
once believed.
E. The SCNs are less strongly affected by
changes in light levels than they are by
other external cues.
In their study of whether offering a guarantee of 506. The primary purpose of the passage is to
service quality will encourage customers to visit a A. question the results of a study that
particular restaurant, Tucci and Talaga have examined the effect of service-quality
found that the effect of such guarantees is mixed. guarantees in the restaurant industry
For higher-priced restaurants, there is some B. discuss potential advantages and
evidence that offering a guarantee increases the disadvantages of service-quality
likelihood of customer selection, probably guarantees in the restaurant industry
reflecting the greater financial commitment C. examine the conventional wisdom
involved in choosing an expensive restaurant. For regarding the effect of service-quality
lower-priced restaurants, where one expects less guarantees in the restaurant industry
assiduous service, Tucci and Talaga found that a D. argue that only certain restaurants would
guarantee could actually have a negative effect: a benefit from the implementation of
potential customer might think that a restaurant service-quality guarantees
offering a guarantee is worried about its service. E. consider the impact that service-quality
Moreover, since customers understand a guarantees can have on the service
restaurant's product and know what to anticipate provided by a restaurant
in terms of service, they are empowered to
question its quality. This is not generally true in 507. It can be inferred that the author of the
the case of skilled activities such as electrical passage would agree with which of the
work, where, consequently, a guarantee might following statements about the appeal of
have greater customer appeal. service guarantees to customers?
A. Such guarantees are likely to be somewhat
For restaurants generally, the main benefit of a more appealing to customers of
service guarantee probably lies not so much in restaurants than to customers of other
customer appeal as in managing and motivating businesses.
staff. Staff members would know what service B. Such guarantees are likely to be more
standards are expected of them and also know appealing to customers who know what to
that the success of the business relies on their anticipate in terms of service.
adhering to those standards. Additionally, C. Such guarantees are likely to have less
guarantees provide some basis for defining the appeal in situations where customers are
skills needed for successful service in areas knowledgeable about a business's product
traditionally regarded as unskilled, such as or service.
waiting tables. D. In situations where a high level of financial
commitment is involved, a service
guarantee is not likely to be very
appealing.
E. In situations where customers expect a
high level of customer service, a service
guarantee is likely to make customers
think that a business is worried about its
service.
In their study of whether offering a guarantee of 508. According to the passage, Tucci and Talaga
service quality will encourage customers to visit a found that service guarantees, when offered
particular restaurant, Tucci and Talaga have by lower-priced restaurants, can have which
found that the effect of such guarantees is mixed. of the following effects?
For higher-priced restaurants, there is some A. Customers' developing unreasonably high
evidence that offering a guarantee increases the expectations regarding service
likelihood of customer selection, probably B. Customers' avoiding such restaurants
reflecting the greater financial commitment because they fear that the service
involved in choosing an expensive restaurant. For guarantee may not be fully honored
lower-priced restaurants, where one expects less C. Customers' interpreting the service
assiduous service, Tucci and Talaga found that a guarantee as a sign that management is
guarantee could actually have a negative effect: a not confident about the quality of its
potential customer might think that a restaurant service
offering a guarantee is worried about its service. D. A restaurant's becoming concerned that
Moreover, since customers understand a its service will not be assiduous enough to
restaurant's product and know what to anticipate satisfy customers
in terms of service, they are empowered to E. A restaurant's becoming concerned that
question its quality. This is not generally true in customers will be more emboldened to
the case of skilled activities such as electrical question the quality of the service they
work, where, consequently, a guarantee might receive
have greater customer appeal.
In Type 1 situations, there are many alternatives 553. Which of the following best describes the
and change is relatively easy. Open pursuit of relation of the second paragraph to the first?
alternatives—by frequent competitive bidding, if A. The second paragraph offers proof of an
possible—will likely yield the best results. In assertion made in the first paragraph.
Type 2 situations, where there are many B. The second paragraph provides an
alternatives but change is difficult—as for explanation for the occurrence of a situation
providers of employee health-care benefits—it is described in the first paragraph.
important to continuously test the market and C. The second paragraph discusses the
use the results to secure concessions from application of a strategy proposed in the first
existing suppliers. Alternatives provide a credible paragraph.
threat to suppliers, even if the ability to switch is D. The second paragraph examines the scope of
constrained. In Type 3 situations, there are few a problem presented in the first paragraph.
alternatives, but the ability to switch without E. The second paragraph discusses the
difficulty creates a threat that companies can use contradictions inherent in a relationship
to negotiate concessions from existing suppliers. described in the first paragraph.
In Type 4 situations, where there are few
alternatives and change is difficult, partnerships
may be unavoidable.
In corporate purchasing, competitive scrutiny is 554. It can be inferred that the author of the
typically limited to suppliers of items that are passage would be most likely to make which
directly related to end products. With “indirect” of the following recommendations to a
purchases (such as computers, advertising, and company purchasing health care benefits for
legal services), which are not directly related to its employees?
production, corporations often favor “supplier A. Devise strategies for circumventing the
partnerships” (arrangements in which the obstacles to replacing the current provider of
purchaser forgoes the right to pursue alternative health care benefits.
suppliers), which can inappropriately shelter B. Obtain health care benefits from a provider
suppliers from rigorous competitive scrutiny that that also provides other indirect products and
might afford the purchaser economic leverage. services.
There are two independent variables— C. Obtain bids from other providers of health
availability of alternatives and ease of changing care benefits in order to be in a position to
suppliers—that companies should use to evaluate negotiate a better deal with the current
the feasibility of subjecting suppliers of indirect provider.
purchases to competitive scrutiny. This can create D. Switch providers of health care benefits
four possible situations. whenever a different provider offers a more
competitive price.
In Type 1 situations, there are many alternatives E. Acknowledge the difficulties involved in
and change is relatively easy. Open pursuit of replacing the current provider of health care
alternatives—by frequent competitive bidding, if benefits and offer to form a partnership with
possible—will likely yield the best results. In the provider.
Type 2 situations, where there are many
alternatives but change is difficult—as for 555. Which of the following is one difference
providers of employee health-care benefits—it is between Type 2 situations and Type 4
important to continuously test the market and situations, as they are described in the
use the results to secure concessions from passage?
existing suppliers. Alternatives provide a credible A. The number of alternative suppliers available
threat to suppliers, even if the ability to switch is to the purchaser
constrained. In Type 3 situations, there are few B. The most effective approach for the purchaser
alternatives, but the ability to switch without to use in obtaining competitive bids from
difficulty creates a threat that companies can use potential suppliers
to negotiate concessions from existing suppliers. C. The degree of difficulty the purchaser
In Type 4 situations, where there are few encounters when changing suppliers
alternatives and change is difficult, partnerships D. The frequency with which each type of
may be unavoidable. situation occurs in a typical business
environment
E. The likelihood that any given purchase will be
an indirect purchase
In corporate purchasing, competitive scrutiny is 556. According to the passage, which of the
typically limited to suppliers of items that are following factors distinguishes an indirect
directly related to end products. With “indirect” purchase from other purchases?
purchases (such as computers, advertising, and A. The ability of the purchasing company to
legal services), which are not directly related to subject potential suppliers of the purchased
production, corporations often favor “supplier item to competitive scrutiny
partnerships” (arrangements in which the B. The number of suppliers of the purchased
purchaser forgoes the right to pursue alternative item available to the purchasing company
suppliers), which can inappropriately shelter C. The methods of negotiation that are available
suppliers from rigorous competitive scrutiny that to the purchasing company
might afford the purchaser economic leverage. D. The relationship of the purchased item to the
There are two independent variables— purchasing company's end product
availability of alternatives and ease of changing E. The degree of importance of the purchased
suppliers—that companies should use to evaluate item in the purchasing company's business
the feasibility of subjecting suppliers of indirect operations
purchases to competitive scrutiny. This can create
four possible situations.
The constructivists gain acceptance by 629. The information in the passage suggests that
misrepresenting technological determinism: Clark believes that which of the following
technological determinists are supposed to believe, would be true if social constructivism had not
for example, that machinery imposes appropriate gained widespread acceptance?
forms of order on society. The alternative to
A. Businesses would be more likely to modernize
constructivism, in other words, is to view technology
without considering the social consequences
as existing outside society, capable of directly
influencing skills and work organization. of their actions.
B. There would be greater understanding of the
Clark refutes the extremes of the constructivists by role played by technology in producing social
both theoretical and empirical arguments. change.
Theoretically he defines “technology” in terms of C. Businesses would be less likely to understand
relationships between social and technical variables. the attitudes of employees affected by
Attempts to reduce the meaning of technology to cold, modernization.
hard metal are bound to fail, for machinery is just D. Modernization would have occurred at a
scrap unless it is organized functionally and slower rate.
supported by appropriate systems of operation and
E. Technology would have played a greater part
maintenance. At the empirical level Clark shows how
in determining the role of business in society.
a change at the telephone exchange from
maintenance-intensive electromechanical switches to
semi-electronic switching systems altered work tasks,
skills, training opportunities, administration, and
organization of workers. Some changes Clark
attributes to the particular way management and
labor unions negotiated the introduction of the
technology, whereas others are seen as arising from
the capabilities and nature of the technology itself.
Thus Clark helps answer the question: “When is social
choice decisive and when are the concrete
characteristics of technology more important?”
Jon Clark's study of the effect of the modernization of 630. According to the passage, constructivists
a telephone exchange on exchange maintenance work employed which of the following to promote
and workers is a solid contribution to a debate that their argument?
encompasses two lively issues in the history and A. Empirical studies of business situations
sociology of technology: technological determinism
involving technological change
and social constructivism.
B. Citation of managers supportive of their
Clark makes the point that the characteristics of a position
technology have a decisive influence on job skills and C. Construction of hypothetical situations that
work organization. Put more strongly, technology can support their view
be a primary determinant of social and managerial D. Contrasts of their view with a misstatement of
organization. Clark believes this possibility has been an opposing view
obscured by the recent sociological fashion, E. Descriptions of the breadth of impact of
exemplified by Braverman's analysis, that emphasizes technological change
the way machinery reflects social choices. For
Braverman, the shape of a technological system is 631. The author of the passage uses the expression
subordinate to the manager's desire to wrest control
“are supposed to” primarily in order to
of the labor process from the workers. Technological
A. suggest that a contention made by
change is construed as the outcome of negotiations
among interested parties who seek to incorporate constructivists regarding determinists is
their own interests into the design and configuration inaccurate
of the machinery. This position represents the new B. define the generally accepted position of
mainstream called social constructivism. determinists regarding the implementation of
technology
The constructivists gain acceptance by C. engage in speculation about the motivation of
misrepresenting technological determinism: determinists
technological determinists are supposed to believe, D. lend support to a comment critical of the
for example, that machinery imposes appropriate position of determinists
forms of order on society. The alternative to
E. contrast the historical position of
constructivism, in other words, is to view technology
determinists with their position regarding the
as existing outside society, capable of directly
influencing skills and work organization. exchange modernization
Clark refutes the extremes of the constructivists by 632. Which of the following statements about
both theoretical and empirical arguments. Clark's study of the telephone exchange can be
Theoretically he defines “technology” in terms of inferred from information in the passage?
relationships between social and technical variables. A. Clark's reason for undertaking the study was
Attempts to reduce the meaning of technology to cold, to undermine Braverman's analysis of the
hard metal are bound to fail, for machinery is just function of technology.
scrap unless it is organized functionally and B. Clark's study suggests that the
supported by appropriate systems of operation and
implementation of technology should be
maintenance. At the empirical level Clark shows how
discussed in the context of conflict between
a change at the telephone exchange from
maintenance-intensive electromechanical switches to labor and management.
semi-electronic switching systems altered work tasks, C. Clark examined the impact of changes in the
skills, training opportunities, administration, and technology of switching at the exchange in
organization of workers. Some changes Clark terms of overall operations and organization.
attributes to the particular way management and D. Clark concluded that the implementation of
labor unions negotiated the introduction of the new switching technology was equally
technology, whereas others are seen as arising from beneficial to management and labor.
the capabilities and nature of the technology itself. E. Clark's analysis of the change in switching
Thus Clark helps answer the question: “When is social systems applies only narrowly to the situation
choice decisive and when are the concrete
at the particular exchange that he studied.
characteristics of technology more important?”
Because the framers of the United States 633. The passage implies that which of the
Constitution (written in 1787) believed that following was a reason that the proportion of
protecting property rights relating to inventions verdicts in favor of patentees began to
would encourage the new nation's economic increase in the 1830s?
growth, they gave Congress—the national A. Patent applications approved after 1836 were
legislature—a constitutional mandate to grant more likely to adhere closely to patent law.
patents for inventions. The resulting patent system B. Patent laws enacted during the 1830s better
has served as a model for those in other nations. defined patent rights.
Recently, however, scholars have questioned C. Judges became less prejudiced against
whether the American system helped achieve the patentees during the 1830s.
framers' goals. These scholars have contended that
D. After 1836, litigated cases became less
from 1794 to roughly 1830, American inventors
representative of the population of patent
were unable to enforce property rights because
disputes.
judges were “antipatent” and routinely invalidated
patents for arbitrary reasons. This argument is E. The proportion of patent disputes brought to
based partly on examination of court decisions in trial began to increase after 1836.
cases where patent holders (“patentees”) brought
suit alleging infringement of their patent rights. In 634. The passage implies that the scholars
the 1820s, for instance, 75 percent of verdicts were mentioned in the highlighted text would agree
decided against the patentee. The proportion of with which of the following criticisms of the
verdicts for the patentee began to increase in the American patent system before 1830?
1830s, suggesting to these scholars that judicial A. Its definition of property rights relating to
attitudes toward patent rights began shifting then. inventions was too vague to be useful.
B. Its criteria for the granting of patents were not
Not all patent disputes in the early nineteenth clear.
century were litigated, however, and litigated cases C. It made it excessively difficult for inventors to
were not drawn randomly from the population of receive patents.
disputes. Therefore, the rate of verdicts in favor of D. It led to excessive numbers of patent-
patentees cannot be used by itself to gauge changes infringement suits.
in judicial attitudes or enforceability of patent E. It failed to encourage national economic
rights. If early judicial decisions were prejudiced growth.
against patentees, one might expect that
subsequent courts—allegedly more supportive of 635. It can be inferred from the passage that the
patent rights—would reject the former legal
frequency with which pre-1830 cases have
precedents. But pre-1830 cases have been cited as
been cited in court decisions is an indication
frequently as later decisions, and they continue to
that
be cited today, suggesting that the early decisions,
A. judicial support for patent rights was
many of which clearly declared that patent rights
were a just recompense for inventive ingenuity, strongest in the period before 1830
provided a lasting foundation for patent law. The B. judicial support for patent rights did not
proportion of judicial decisions in favor of patentees increase after 1830
began to increase during the 1830s because of a C. courts have returned to judicial standards
change in the underlying population of cases that prevailed before 1830
brought to trial. This change was partly due to an D. verdicts favoring patentees in patent-
1836 revision to the patent system: an examination infringement suits did not increase after 1830
procedure, still in use today, was instituted in which E. judicial bias against patentees persisted after
each application is scrutinized for its adherence to 1830
patent law. Previously, patents were automatically
granted upon payment of a $30 fee.
Because the framers of the United States 636. It can be inferred from the passage that the
Constitution (written in 1787) believed that author and the scholars referred to in the
protecting property rights relating to inventions highlighted text disagree about which of the
would encourage the new nation's economic following aspects of the patents defended in
growth, they gave Congress—the national patent-infringement suits before 1830?
legislature—a constitutional mandate to grant A. Whether the patents were granted for
patents for inventions. The resulting patent system inventions that were genuinely useful
has served as a model for those in other nations. B. Whether the patents were actually relevant to
Recently, however, scholars have questioned the growth of the United States economy
whether the American system helped achieve the C. Whether the patents were particularly likely
framers' goals. These scholars have contended that
to be annulled by judges
from 1794 to roughly 1830, American inventors
D. Whether the patents were routinely
were unable to enforce property rights because
invalidated for reasons that were arbitrary
judges were “antipatent” and routinely invalidated
patents for arbitrary reasons. This argument is E. Whether the patents were vindicated at a
based partly on examination of court decisions in significantly lower rate than patents in later
cases where patent holders (“patentees”) brought suits
suit alleging infringement of their patent rights. In
the 1820s, for instance, 75 percent of verdicts were 637. The author of the passage cites which of the
decided against the patentee. The proportion of following as evidence challenging the
verdicts for the patentee began to increase in the argument referred to in the highlighted text?
1830s, suggesting to these scholars that judicial A. The proportion of cases that were decided
attitudes toward patent rights began shifting then. against patentees in the 1820s
B. The total number of patent disputes that were
Not all patent disputes in the early nineteenth litigated from 1794 to 1830
century were litigated, however, and litigated cases C. The fact that later courts drew upon the legal
were not drawn randomly from the population of precedents set in pre-1830 patent cases
disputes. Therefore, the rate of verdicts in favor of D. The fact that the proportion of judicial
patentees cannot be used by itself to gauge changes decisions in favor of patentees began to
in judicial attitudes or enforceability of patent increase during the 1830s
rights. If early judicial decisions were prejudiced E. The constitutional rationale for the 1836
against patentees, one might expect that revision of the patent system
subsequent courts—allegedly more supportive of
patent rights—would reject the former legal
precedents. But pre-1830 cases have been cited as
frequently as later decisions, and they continue to
be cited today, suggesting that the early decisions,
many of which clearly declared that patent rights
were a just recompense for inventive ingenuity,
provided a lasting foundation for patent law. The
proportion of judicial decisions in favor of patentees
began to increase during the 1830s because of a
change in the underlying population of cases
brought to trial. This change was partly due to an
1836 revision to the patent system: an examination
procedure, still in use today, was instituted in which
each application is scrutinized for its adherence to
patent law. Previously, patents were automatically
granted upon payment of a $30 fee.
Jacob Burckhardt's view that Renaissance European 638. The author of the passage discusses Krontiris
women “stood on a footing of perfect equality” with primarily to provide an example of a writer
Renaissance men has been repeatedly cited by who
feminist scholars as a prelude to their presentation A. is highly critical of the writings of certain
of rich historical evidence of women's inequality. In Renaissance women
striking contrast to Burckhardt, Joan Kelly in her B. supports Kelly's view of women's status
famous 1977 essay, “Did Women Have a during the Renaissance
Renaissance?” argued that the Renaissance was a C. has misinterpreted the works of certain
period of economic and social decline for women Renaissance women
relative both to Renaissance men and to medieval D. has rejected the views of both Burckhardt and
women. Recently, however, a significant trend
Kelly
among feminist scholars has entailed a rejection of
E. has studied Renaissance women in a wide
both Kelly's dark vision of the Renaissance and
variety of social and religious contexts
Burckhardt's rosy one. Many recent works by these
scholars stress the ways in which differences among
Renaissance women—especially in terms of social 639. According to the passage, Krontiris's work
status and religion—work to complicate the kinds differs from that of the scholars mentioned in
of generalizations both Burckhardt and Kelly made the highlighted text in which of the following
on the basis of their observations about upper-class ways?
Italian women. A. Krontiris's work stresses the achievements of
Renaissance women rather than the obstacles
The trend is also evident, however, in works to their success.
focusing on those middle- and upper-class B. Krontiris's work is based on a
European women whose ability to write gives them reinterpretation of the work of earlier
disproportionate representation in the historical scholars.
record. Such women were, simply by virtue of their C. Krontiris's views are at odds with those of
literacy, members of a tiny minority of the both Kelly and Burkhardt.
population, so it is risky to take their descriptions of D. Krontiris's work focuses on the place of
their experiences as typical of “female experience” women in Renaissance society.
in any general sense. Tina Krontiris, for example, in E. Krontiris's views are based exclusively on the
her fascinating study of six Renaissance women study of a privileged group of women.
writers, does tend at times to conflate “women” and
“women writers,” assuming that women's gender, 640. According to the passage, feminist scholars
irrespective of other social differences, including
cite Burckhardt's view of Renaissance women
literacy, allows us to view women as a
primarily for which of the following reasons?
homogeneous social group and make that group an
A. Burckhardt's view forms the basis for most
object of analysis. Nonetheless, Krontiris makes a
arguments refuting Kelly's point of view.
significant contribution to the field and is
representative of those authors who offer what B. Burckhardt's view has been discredited by
might be called a cautiously optimistic assessment Kelly.
of Renaissance women's achievements, although C. Burckhardt's view is one that many feminist
she also stresses the social obstacles Renaissance scholars wish to refute.
women faced when they sought to raise their D. Burckhardt's work provides rich historical
“oppositional voices.” Krontiris is concerned to evidence of inequality between Renaissance
show women intentionally negotiating some power women and men.
for themselves (at least in the realm of public E. Burckhardt's work includes historical
discourse) against potentially constraining research supporting the arguments of the
ideologies, but in her sober and thoughtful feminist scholars.
concluding remarks, she suggests that such verbal
opposition to cultural stereotypes was highly
circumscribed; women seldom attacked the basic
assumptions in the ideologies that oppressed them.
Jacob Burckhardt's view that Renaissance European 641. It can be inferred that both Burckhardt and
women “stood on a footing of perfect equality” with Kelly have been criticized by the scholars
Renaissance men has been repeatedly cited by mentioned in the highlighted text for which of
feminist scholars as a prelude to their presentation the following?
of rich historical evidence of women's inequality. In A. Assuming that women writers of the
striking contrast to Burckhardt, Joan Kelly in her Renaissance are representative of
famous 1977 essay, “Did Women Have a Renaissance women in general
Renaissance?” argued that the Renaissance was a B. Drawing conclusions that are based on the
period of economic and social decline for women study of an atypical group of women
relative both to Renaissance men and to medieval C. Failing to describe clearly the relationship
women. Recently, however, a significant trend
between social status and literacy among
among feminist scholars has entailed a rejection of
Renaissance women
both Kelly's dark vision of the Renaissance and
D. Failing to acknowledge the role played by
Burckhardt's rosy one. Many recent works by these
scholars stress the ways in which differences among Renaissance women in opposing cultural
Renaissance women—especially in terms of social stereotypes
status and religion—work to complicate the kinds E. Failing to acknowledge the ways in which
of generalizations both Burckhardt and Kelly made social status affected the creative activities of
on the basis of their observations about upper-class Renaissance women
Italian women.
642. The author of the passage suggests that
The trend is also evident, however, in works Krontiris incorrectly assumes that
focusing on those middle- and upper-class A. social differences among Renaissance women
European women whose ability to write gives them are less important than the fact that they were
disproportionate representation in the historical women
record. Such women were, simply by virtue of their B. literacy among Renaissance women was more
literacy, members of a tiny minority of the prevalent than most scholars today
population, so it is risky to take their descriptions of acknowledge
their experiences as typical of “female experience” C. during the Renaissance, women were able to
in any general sense. Tina Krontiris, for example, in successfully oppose cultural stereotypes
her fascinating study of six Renaissance women relating to gender
writers, does tend at times to conflate “women” and D. Renaissance women did not face many
“women writers,” assuming that women's gender, difficult social obstacles relating to their
irrespective of other social differences, including
gender
literacy, allows us to view women as a
E. in order to attain power, Renaissance women
homogeneous social group and make that group an
attacked basic assumptions in the ideologies
object of analysis. Nonetheless, Krontiris makes a
that oppressed them
significant contribution to the field and is
representative of those authors who offer what
might be called a cautiously optimistic assessment
of Renaissance women's achievements, although
she also stresses the social obstacles Renaissance
women faced when they sought to raise their
“oppositional voices.” Krontiris is concerned to
show women intentionally negotiating some power
for themselves (at least in the realm of public
discourse) against potentially constraining
ideologies, but in her sober and thoughtful
concluding remarks, she suggests that such verbal
opposition to cultural stereotypes was highly
circumscribed; women seldom attacked the basic
assumptions in the ideologies that oppressed them.
Jacob Burckhardt's view that Renaissance European 643. The last sentence in the passage serves
women “stood on a footing of perfect equality” with primarily to
Renaissance men has been repeatedly cited by A. suggest that Krontiris's work is not
feminist scholars as a prelude to their presentation representative of recent trends among
of rich historical evidence of women's inequality. In feminist scholars
striking contrast to Burckhardt, Joan Kelly in her B. undermine the argument that literate women
famous 1977 essay, “Did Women Have a of the Renaissance sought to oppose social
Renaissance?” argued that the Renaissance was a constraints imposed on them
period of economic and social decline for women C. show a way in which Krontiris's work
relative both to Renaissance men and to medieval illustrates a “cautiously optimistic”
women. Recently, however, a significant trend
assessment of Renaissance women's
among feminist scholars has entailed a rejection of
achievements
both Kelly's dark vision of the Renaissance and
D. summarize Krontiris's view of the effect of
Burckhardt's rosy one. Many recent works by these
scholars stress the ways in which differences among literacy on the lives of upper- and middle-
Renaissance women—especially in terms of social class Renaissance women
status and religion—work to complicate the kinds E. illustrate the way in which Krontiris's study
of generalizations both Burckhardt and Kelly made differs from the studies done by Burckhardt
on the basis of their observations about upper-class and Kelly
Italian women.
644. The author of the passage implies that the
The trend is also evident, however, in works women studied by Krontiris are unusual in
focusing on those middle- and upper-class which of the following ways?
European women whose ability to write gives them A. They faced obstacles less formidable than
disproportionate representation in the historical those faced by other Renaissance women.
record. Such women were, simply by virtue of their B. They have been seen by historians as more
literacy, members of a tiny minority of the interesting than other Renaissance women.
population, so it is risky to take their descriptions of C. They were more concerned about recording
their experiences as typical of “female experience” history accurately than were other
in any general sense. Tina Krontiris, for example, in Renaissance women.
her fascinating study of six Renaissance women D. Their perceptions are more likely to be
writers, does tend at times to conflate “women” and accessible to historians than are those of most
“women writers,” assuming that women's gender, other Renaissance women.
irrespective of other social differences, including
E. Their concerns are likely to be of greater
literacy, allows us to view women as a
interest to feminist scholars than are the ideas
homogeneous social group and make that group an
of most other Renaissance women.
object of analysis. Nonetheless, Krontiris makes a
significant contribution to the field and is
representative of those authors who offer what
might be called a cautiously optimistic assessment
of Renaissance women's achievements, although
she also stresses the social obstacles Renaissance
women faced when they sought to raise their
“oppositional voices.” Krontiris is concerned to
show women intentionally negotiating some power
for themselves (at least in the realm of public
discourse) against potentially constraining
ideologies, but in her sober and thoughtful
concluding remarks, she suggests that such verbal
opposition to cultural stereotypes was highly
circumscribed; women seldom attacked the basic
assumptions in the ideologies that oppressed them.
When asteroids collide, some collisions cause an 645. The passage implies which of the following
asteroid to spin faster; others slow it down. If about the five asteroids mentioned in the
asteroids are all monoliths—single rocks— highlighted text?
undergoing random collisions, a graph of their A. Their rotation rates are approximately the
rotation rates should show a bell-shaped same.
distribution with statistical “tails” of very fast and B. They have undergone approximately the same
very slow rotators. If asteroids are rubble piles, number of collisions.
however, the tail representing the very fast C. They are monoliths.
rotators would be missing, because any loose D. They are composed of fragments that have
aggregate spinning faster than once every few escaped the gravity of larger asteroids.
hours (depending on the asteroid's bulk density) E. They were detected only recently.
would fly apart. Researchers have discovered that
all but five observed asteroids obey a strict limit 646. The discovery of which of the following would
on rate of rotation. The exceptions are all smaller call into question the conclusion mentioned in
than 200 meters in diameter, with an abrupt the highlighted text?
cutoff for asteroids larger than that. A. An asteroid 100 meters in diameter rotating at
a rate of once per week
The evident conclusion—that asteroids larger B. An asteroid 150 meters in diameter rotating at
than 200 meters across are multicomponent a rate of 20 times per hour
structures or rubble piles—agrees with recent C. An asteroid 250 meters in diameter rotating at
computer modeling of collisions, which also finds a rate of once per week
a transition at that diameter. A collision can blast D. An asteroid 500 meters in diameter rotating at
a large asteroid to bits, but after the collision a rate of once per hour
those bits will usually move slower than their E. An asteroid 1,000 meters in diameter rotating
mutual escape velocity. Over several hours, at a rate of once every 24 hours
gravity will reassemble all but the fastest pieces
into a rubble pile. Because collisions among 647. According to the passage, which of the
asteroids are relatively frequent, most large following is a prediction that is based on the
bodies have already suffered this fate. Conversely, strength of the gravitational attraction of
most small asteroids should be monolithic, small asteroids?
because impact fragments easily escape their A. Small asteroids will be few in number.
feeble gravity. B. Small asteroids will be monoliths.
C. Small asteroids will collide with other
asteroids very rarely.
D. Most small asteroids will have very fast
rotation rates.
E. Almost no small asteroids will have very slow
rotation rates.
When asteroids collide, some collisions cause an 648. The author of the passage mentions “escape
asteroid to spin faster; others slow it down. If velocity” in order to help explain which of the
asteroids are all monoliths—single rocks— following?
undergoing random collisions, a graph of their A. The tendency for asteroids to become smaller
rotation rates should show a bell-shaped rather than larger over time
distribution with statistical “tails” of very fast and B. The speed with which impact fragments
very slow rotators. If asteroids are rubble piles, reassemble when they do not escape an
however, the tail representing the very fast asteroid's gravitational attraction after a
rotators would be missing, because any loose collision
aggregate spinning faster than once every few C. The frequency with which collisions among
hours (depending on the asteroid's bulk density) asteroids occur
would fly apart. Researchers have discovered that D. The rotation rates of asteroids smaller than
all but five observed asteroids obey a strict limit 200 meters in diameter
on rate of rotation. The exceptions are all smaller E. The tendency for large asteroids to persist
than 200 meters in diameter, with an abrupt after collisions
cutoff for asteroids larger than that.
Because technological change is likely to displace 10. According to the passage, small downward
some workers, it is a major factor in producing shifts in the demand for labor will not usually
structural unemployment. While technological cause unemployment because
advance almost invariably results in shifts in A. such shifts are frequently accompanied by
demands for different types of workers, it does upswings in the economy
not necessarily result in unemployment. B. such shifts usually occur slowly
Relatively small or gradual changes in demand C. workers can be encouraged to move to where
are likely to cause little unemployment. In the there are jobs
individual firm or even in the labor market as a D. normal attrition is often sufficient to reduce
whole, normal attrition may be sufficient to the size of the work force
reduce the size of the work force in the affected E. workers are usually flexible enough to learn
occupations. Relatively large or rapid changes, new skills and switch to new jobs
however, can cause serious problems. Workers
may lose their jobs and find themselves without
the skills necessary to obtain new jobs. Whether
this displacement leads to structural
unemployment depends on the amount of public
and private sector resources devoted to
retraining and placing those workers. Workers
can be encouraged to move where there are jobs,
to reeducate or retrain themselves, or to retire. In
addition, other factors affecting structural
unemployment, such as capital movement, can be
controlled.
Astronomers use an analogous procedure to 28. The primary purpose of the passage is to
“weigh” double-star systems. The speed with A. analyze a natural phenomenon in terms of its
which the two stars in a double-star system circle behavior under special conditions
one another depends on the gravitational force B. describe the steps by which a scientific
between them, which holds the system together. measurement is carried out
This attractive force, analogous to the tension in C. point out the conditions under which a
the string, is proportional to the stars' combined scientific procedure is most useful
mass, according to Newton's law of gravitation. D. contrast two different uses of a
By observing the time required for the stars to methodological approach in science
circle each other (the period) and measuring the E. explain a method by which scientists
distance between them, we can deduce the determine an unknown quantity
restraining force, and hence the masses.
Most pre-1990 literature on businesses' use of 29. The passage is primarily concerned with
information technology (IT)—defined as any form A. describing a resource and indicating various
of computer-based information system—focused methods used to study it
on spectacular IT successes and reflected a general B. presenting a theory and offering an opposing
optimism concerning IT's potential as a resource for point of view
creating competitive advantage. But toward the end C. providing an explanation for unexpected
of the 1980s, some economists spoke of a findings
“productivity paradox”: despite huge IT D. demonstrating why a particular theory is
investments, most notably in the service sectors, unfounded
productivity stagnated. In the retail industry, for E. resolving a disagreement regarding the uses
example, in which IT had been widely adopted
of a technology
during the 1980s, productivity (average output per
hour) rose at an average annual rate of 1.1 percent
30. The author of the passage discusses
between 1973 and 1989, compared with 2.4
percent in the preceding 25-year period. productivity in the retail industry in the first
Proponents of IT argued that it takes both time and paragraph primarily in order to
a critical mass of investment for IT to yield benefits, A. suggest a way in which IT can be used to
and some suggested that growth figures for the create a competitive advantage
1990s proved these benefits were finally being B. provide an illustration of the “productivity
realized. They also argued that measures of paradox”
productivity ignore what would have happened C. emphasize the practical value of the
without investments in IT—productivity gains introduction of IT
might have been even lower. There were even D. cite an industry in which productivity did not
claims that IT had improved the performance of the stagnate during the 1980s
service sector significantly, although E. counter the argument that IT could potentially
macroeconomic measures of productivity did not create competitive advantage
reflect the improvement.
31. The passage suggests that proponents of
But some observers questioned why, if IT had resource-based theory would be likely to
conferred economic value, it did not produce direct explain IT's inability to produce direct
competitive advantages for individual firms. competitive advantages for individual firms
Resource-based theory offers an answer, asserting by pointing out that
that, in general, firms gain competitive advantages A. IT is not a resource that is difficult to obtain
by accumulating resources that are economically
B. IT is not an economically valuable resource
valuable, relatively scarce, and not easily replicated.
C. IT is a complex, intangible resource
According to a recent study of retail firms, which
D. economic progress has resulted from IT only
confirmed that IT has become pervasive and
in the service sector
relatively easy to acquire, IT by itself appeared to
have conferred little advantage. In fact, though little E. changes brought about by IT cannot be
evidence of any direct effect was found, the frequent detected by macroeconomic measures
negative correlations between IT and performance
suggested that IT had probably weakened some 32. Which of the following best describes the
firms' competitive positions. However, firms' content of the first paragraph?
human resources, in and of themselves, did explain A. It presents two explanations for the success of
improved performance, and some firms gained IT- IT.
related advantages by merging IT with B. It provides evidence that decreases in
complementary resources, particularly human productivity will continue.
resources. The findings support the notion, founded C. It presents reasons for a decline in
in resource-based theory, that competitive productivity.
advantages do not arise from easily replicated D. It demonstrates the effect IT has had on
resources, no matter how impressive or productivity.
economically valuable they may be, but from E. It contrasts views concerning the degree of
complex, intangible resources. IT's success.
Most pre-1990 literature on businesses' use of 33. The passage suggests that the recent study of
information technology (IT)—defined as any form retail firms discussed in the second paragraph
of computer-based information system—focused supports which of the following conclusions
on spectacular IT successes and reflected a general regarding a firm's competitive advantage?
optimism concerning IT's potential as a resource for A. Human resources alone are more likely to
creating competitive advantage. But toward the end contribute to competitive advantage than is IT
of the 1980s, some economists spoke of a alone.
“productivity paradox”: despite huge IT B. Human resources combined with IT are more
investments, most notably in the service sectors, likely than human resources alone to have a
productivity stagnated. In the retail industry, for negative effect on competitive advantage.
example, in which IT had been widely adopted
C. Human resources combined with IT often
during the 1980s, productivity (average output per
have a negative effect on competitive
hour) rose at an average annual rate of 1.1 percent
advantage.
between 1973 and 1989, compared with 2.4
percent in the preceding 25-year period. D. IT by itself is much more likely to have a
Proponents of IT argued that it takes both time and positive effect than a negative effect on
a critical mass of investment for IT to yield benefits, competitive advantage.
and some suggested that growth figures for the E. The positive effect of IT on competitive
1990s proved these benefits were finally being advantage increases with time.
realized. They also argued that measures of
productivity ignore what would have happened 34. According to the passage, most pre-1990
without investments in IT—productivity gains literature on businesses' use of IT included
might have been even lower. There were even which of the following?
claims that IT had improved the performance of the A. Recommendations regarding effective ways
service sector significantly, although to use IT to gain competitive advantage
macroeconomic measures of productivity did not B. Explanations of the advantages and
reflect the improvement. disadvantages of adopting IT
C. Information about ways in which IT combined
But some observers questioned why, if IT had with human resources could be used to
conferred economic value, it did not produce direct increase competitive advantage
competitive advantages for individual firms. D. A warning regarding the negative effect on
Resource-based theory offers an answer, asserting competitive advantage that would occur if IT
that, in general, firms gain competitive advantages were not adopted
by accumulating resources that are economically
E. A belief in the likelihood of increased
valuable, relatively scarce, and not easily replicated.
competitive advantage for firms using IT
According to a recent study of retail firms, which
confirmed that IT has become pervasive and
relatively easy to acquire, IT by itself appeared to
have conferred little advantage. In fact, though little
evidence of any direct effect was found, the frequent
negative correlations between IT and performance
suggested that IT had probably weakened some
firms' competitive positions. However, firms'
human resources, in and of themselves, did explain
improved performance, and some firms gained IT-
related advantages by merging IT with
complementary resources, particularly human
resources. The findings support the notion, founded
in resource-based theory, that competitive
advantages do not arise from easily replicated
resources, no matter how impressive or
economically valuable they may be, but from
complex, intangible resources.
Most pre-1990 literature on businesses' use of 35. The author of the passage implies that toward
information technology (IT)—defined as any form the end of the 1980s, some economists
of computer-based information system—focused described which of the following as a
on spectacular IT successes and reflected a general “productivity paradox”?
optimism concerning IT's potential as a resource for A. Investments in IT would not result in
creating competitive advantage. But toward the end increases in productivity until the 1990s.
of the 1980s, some economists spoke of a B. Investments in IT did not lead to expected
“productivity paradox”: despite huge IT gains in productivity.
investments, most notably in the service sectors, C. Productivity in the retail industry rose less
productivity stagnated. In the retail industry, for rapidly than did productivity in other
example, in which IT had been widely adopted
industries.
during the 1980s, productivity (average output per
D. The gains in productivity due to the
hour) rose at an average annual rate of 1.1 percent
introduction of IT were not reflected in
between 1973 and 1989, compared with 2.4
percent in the preceding 25-year period. macroeconomic measures of productivity.
Proponents of IT argued that it takes both time and E. Most gains in productivity occurred in the
a critical mass of investment for IT to yield benefits, service sector and were therefore particularly
and some suggested that growth figures for the difficult to measure.
1990s proved these benefits were finally being
realized. They also argued that measures of 36. According to the passage, the recent study of
productivity ignore what would have happened retail firms discussed in the second paragraph
without investments in IT—productivity gains best supports which of the following
might have been even lower. There were even assessments of IT's potential?
claims that IT had improved the performance of the A. Even when IT gives a firm a temporary
service sector significantly, although competitive advantage, that firm is unlikely to
macroeconomic measures of productivity did not continue to achieve productivity gains.
reflect the improvement. B. The competitive advantages conferred by a
firm's introduction of IT are outweighed by
But some observers questioned why, if IT had IT's development costs.
conferred economic value, it did not produce direct C. A firm's introduction of IT is less likely to limit
competitive advantages for individual firms. its ability to achieve productivity gains than to
Resource-based theory offers an answer, asserting enhance that ability.
that, in general, firms gain competitive advantages D. Although IT by itself is unlikely to give a firm
by accumulating resources that are economically
a competitive advantage, IT combined with
valuable, relatively scarce, and not easily replicated.
other resources may do so.
According to a recent study of retail firms, which
E. Although IT by itself is unlikely to give a firm
confirmed that IT has become pervasive and
a competitive advantage, a firm that does not
relatively easy to acquire, IT by itself appeared to
have conferred little advantage. In fact, though little employ IT cannot achieve a competitive
evidence of any direct effect was found, the frequent advantage.
negative correlations between IT and performance
suggested that IT had probably weakened some
firms' competitive positions. However, firms'
human resources, in and of themselves, did explain
improved performance, and some firms gained IT-
related advantages by merging IT with
complementary resources, particularly human
resources. The findings support the notion, founded
in resource-based theory, that competitive
advantages do not arise from easily replicated
resources, no matter how impressive or
economically valuable they may be, but from
complex, intangible resources.
The dry mountain ranges of the western United 37. The passage is primarily concerned with
States contain rocks dating back 440 to 510 A. evaluating the evidence of a major geologic
million years, to the Ordovician period, and period and determining its duration
teeming with evidence of tropical marine life. This B. describing an evolutionary phenomenon and
rock record provides clues about one of the most speculating about its cause
significant radiations (periods when existing life- C. explaining the mechanisms through which
forms gave rise to variations that would marine life-forms evolved during a particular
eventually evolve into entirely new species) in the period
history of marine invertebrates. During this D. analyzing the impact on later life-forms of an
radiation the number of marine biological important evolutionary development
families increased greatly, and these families E. contrasting a period of evolutionary change
included species that would dominate the marine with other such periods
ecosystems of the area for the next 215 million
years. Although the radiation spanned tens of 38. Which of the following can be inferred from
millions of years, major changes in many species the passage regarding the geologic changes
occurred during a geologically short time span that occurred during the Ordovician period?
within the radiation and, furthermore, appear to A. They were more drastic than those associated
have occurred worldwide, suggesting that with other radiations.
external events were major factors in the B. They may have created conditions favorable
radiation. In fact, there is evidence of major to the evolution of many new life-forms.
ecological and geological changes during this C. They may have caused the extinction of many
period: the sea level dropped drastically and of the marine species living in shallow waters.
mountain ranges were formed. In this instance, D. They may have been a factor in the
rather than leading to large-scale extinctions, development of new species adapted to living
these kinds of environmental changes may have both on land and in water.
resulted in an enriched pattern of habitats and E. They hastened the formation of the extensive
nutrients, which in turn gave rise to the dry regions found in the western United
Ordovician radiation. However, the actual States.
relationship between these environmental
factors and the diversification of life-forms is not 39. Which of the following best describes the
yet fully understood. function of the last sentence of the passage?
A. It points out that the events described in the
passage may be atypical.
B. It alludes to the fact that there is disagreement
in the scientific community over the
importance of the Ordovician radiation.
C. It concludes that the evidence presented in
the passage is insufficient to support the
proposed hypothesis because it comes from a
limited geographic area.
D. It warns the reader against seeing a
connection between the biological and
geologic changes described in the passage.
E. It alerts the reader that current knowledge
cannot completely explain the relationship
suggested by the evidence presented in the
passage.
Seventeenth-century philosopher John Locke 40. The author of the passage is primarily
stated that as much as 99 percent of the value of concerned with
any useful product can be attributed to “the A. criticizing Locke's economic theories
effects of labor.” For Locke's intellectual heirs it B. discounting the contribution of labor in a
was only a short step to the “labor theory of modern economy
value,” whose formulators held that 100 percent C. questioning the validity of the labor theory of
of the value of any product is generated by labor value
(the human work needed to produce goods) and D. arguing for a more equitable distribution of
that therefore the employer who appropriates business profits
any part of the product's value as profit is E. contending that employers are
practicing theft. overcompensated for capital goods
Although human effort is required to produce 41. According to the author of the passage, which
goods for the consumer market, effort is also of the following is true of the distribution of
invested in making capital goods (tools, the income derived from the total output of
machines, etc.), which are used to facilitate the consumer goods in a modern economy?
production of consumer goods. In modern A. Workers receive a share of this income that is
economies about one-third of the total output of significantly smaller than the value of their
consumer goods is attributable to the use of labor as a contribution to total output.
capital goods. Approximately two-thirds of the B. Owners of capital goods receive a share of this
income derived from this total output is paid out income that is significantly greater than the
to workers as wages and salaries, the remaining contribution to total output attributable to the
third serving as compensation to the owners of use of capital goods.
the capital goods. Moreover, part of this C. Owners of capital goods receive a share of this
remaining third is received by workers who are income that is no greater than the proportion
shareholders, pension beneficiaries, and the like. of total output attributable to the use of capital
The labor theory of value systematically goods.
disregards the productive contribution of capital D. Owners of capital goods are not fully
goods—a failing for which Locke must bear part compensated for their investment because
of the blame. they pay out most of their share of this income
to workers as wages and benefits.
E. Workers receive a share of this income that is
greater than the value of their labor because
the labor theory of value overestimates their
contribution to total output.
Seventeenth-century philosopher John Locke 42. Which of the following statements, if true,
stated that as much as 99 percent of the value of would most effectively counter the author's
any useful product can be attributed to “the criticism of Locke at the end of the passage?
effects of labor.” For Locke's intellectual heirs it A. Locke was unfamiliar with the labor theory of
was only a short step to the “labor theory of value as it was formulated by his intellectual
value,” whose formulators held that 100 percent heirs.
of the value of any product is generated by labor B. In Locke's day, there was no possibility of
(the human work needed to produce goods) and ordinary workers becoming shareholders or
that therefore the employer who appropriates pension beneficiaries.
any part of the product's value as profit is C. During Locke's lifetime, capital goods did not
practicing theft. make a significant productive contribution to
the economy.
Although human effort is required to produce D. The precise statistical calculation of the
goods for the consumer market, effort is also productive contributions of labor and capital
invested in making capital goods (tools, goods is not possible without computers.
machines, etc.), which are used to facilitate the E. The terms “capital goods” and “consumer
production of consumer goods. In modern goods” were coined by modern economists
economies about one-third of the total output of and do not appear in Locke's writings.
consumer goods is attributable to the use of
capital goods. Approximately two-thirds of the 43. Which of the following best describes the
income derived from this total output is paid out organization of the passage?
to workers as wages and salaries, the remaining A. The author explores the origins of a theory
third serving as compensation to the owners of and explains why the theory never gained
the capital goods. Moreover, part of this widespread acceptance.
remaining third is received by workers who are B. The author introduces the premise of a theory,
shareholders, pension beneficiaries, and the like. evaluates the premise by relating it to
The labor theory of value systematically objective reality, then proposes a modification
disregards the productive contribution of capital of the theory.
goods—a failing for which Locke must bear part C. After quoting a well-known authority, the
of the blame. author describes the evolution of a theory,
then traces its modern form back to the
original quotation.
D. After citing a precursor of a theory, the author
outlines and refutes the theory, then links its
flaw to the precursor.
E. After tracing the roots of a theory, the author
attempts to undermine the theory by
discrediting its originator.
Seventeenth-century philosopher John Locke 44. Which of the following arguments would a
stated that as much as 99 percent of the value of proponent of the labor theory of value, as it is
any useful product can be attributed to “the presented in the first paragraph, be most
effects of labor.” For Locke's intellectual heirs it likely to use in response to the highlighted
was only a short step to the “labor theory of text?
value,” whose formulators held that 100 percent A. The productive contributions of workers and
of the value of any product is generated by labor capital goods cannot be compared because the
(the human work needed to produce goods) and productive life span of capital goods is longer
that therefore the employer who appropriates than that of workers.
any part of the product's value as profit is B. The author's analysis of the distribution of
practicing theft. income is misleading because only a small
percentage of workers are also shareholders.
Although human effort is required to produce C. Capital goods are valuable only insofar as they
goods for the consumer market, effort is also contribute directly to the production of
invested in making capital goods (tools, consumer goods.
machines, etc.), which are used to facilitate the D. The productive contribution of capital goods
production of consumer goods. In modern must be discounted because capital goods
economies about one-third of the total output of require maintenance.
consumer goods is attributable to the use of E. The productive contribution of capital goods
capital goods. Approximately two-thirds of the must be attributed to labor because capital
income derived from this total output is paid out goods are themselves products of labor.
to workers as wages and salaries, the remaining
third serving as compensation to the owners of 45. The author of the passage implies which of the
the capital goods. Moreover, part of this following regarding the formulators of the
remaining third is received by workers who are labor theory of value?
shareholders, pension beneficiaries, and the like. A. They came from a working-class background.
The labor theory of value systematically B. Their views were too radical to have popular
disregards the productive contribution of appeal.
capital goods—a failing for which Locke must C. At least one of them was a close contemporary
bear part of the blame. of Locke.
D. They were familiar with Locke's views on the
relationship between labor and the value of
products.
E. They underestimated the importance of
consumer goods in a modern economy.
Exactly when in the early modern era Native 46. The author of the passage draws conclusions
Americans began exchanging animal furs with about the fur trade in North America from all
Europeans for European-made goods is of the following sources EXCEPT
uncertain. What is fairly certain, even though they A. Cartier's accounts of trading with Native
left no written evidence of having done so, is that Americans
the first Europeans to conduct such trade during B. a seventeenth-century account of European
the modern period were fishing crews working settlements
the waters around Newfoundland. Archaeologists C. a sixteenth-century account written by a
had noticed that sixteenth-century Native sailing vessel captain
American sites were strewn with iron bolts and D. archaeological observations of sixteenth-
metal pins. Only later, upon reading Nicolas century Native American sites
Denys's 1672 account of seventeenth-century E. a sixteenth-century account of Native
European settlements in North America, did Americans in what is now New England
archaeologists realize that sixteenth-century
European fishing crews had dismantled and 47. The passage suggests that which of the
exchanged parts of their ships for furs. following is partially responsible for the
difficulty in establishing the precise date
By the time Europeans sailing the Atlantic coast of when the fur trade in North America began?
North America first documented the fur trade, it A. A lack of written accounts before that of
was apparently well underway. The first to record Nicolas Denys in 1672
such trade—the captain of a Portuguese vessel B. A lack of written documentation before 1501
sailing from Newfoundland in 1501—observed C. Ambiguities in the evidence from Native
that a Native American aboard the ship wore American sources
Venetian silver earrings. Another early chronicler D. Uncertainty about Native American trade
noted in 1524 that Native Americans living along networks
the coast of what is now New England had E. Uncertainty about the origin of artifacts
become selective about European trade goods: supposedly traded by European fishing crews
they accepted only knives, fishhooks, and sharp for furs
metal. By the time Cartier sailed the Saint
Lawrence River ten years later, Native Americans
had traded with Europeans for more than thirty
years, perhaps half a century.
Exactly when in the early modern era Native 48. Which of the following, if true, most
Americans began exchanging animal furs with strengthens the author's assertion in the first
Europeans for European-made goods is sentence of the second paragraph?
uncertain. What is fairly certain, even though they A. When Europeans retraced Cartier's voyage in
left no written evidence of having done so, is that the first years of the seventeenth century, they
the first Europeans to conduct such trade during frequently traded with Native Americans.
the modern period were fishing crews working B. Furs from beavers, which were plentiful in
the waters around Newfoundland. Archaeologists North America but nearly extinct in Europe,
had noticed that sixteenth-century Native became extremely fashionable in Europe in
American sites were strewn with iron bolts and the final decades of the sixteenth century.
metal pins. Only later, upon reading Nicolas C. Firing arms were rarely found on sixteenth-
Denys's 1672 account of seventeenth-century century Native American sites or on European
European settlements in North America, did lists of trading goods since such arms required
archaeologists realize that sixteenth-century frequent maintenance and repair.
European fishing crews had dismantled and D. Europeans and Native Americans had
exchanged parts of their ships for furs. established trade protocols, such as body
language assuring one another of their
By the time Europeans sailing the Atlantic coast of peaceful intentions, that antedate the earliest
North America first documented the fur trade, it records of trade.
was apparently well underway. The first to record E. During the first quarter of the sixteenth
such trade—the captain of a Portuguese vessel century, an Italian explorer recorded seeing
sailing from Newfoundland in 1501—observed many Native Americans with what appeared
that a Native American aboard the ship wore to be copper beads, though they may have
Venetian silver earrings. Another early chronicler been made of indigenous copper.
noted in 1524 that Native Americans living along
the coast of what is now New England had 49. Which of the following best describes the
become selective about European trade goods: primary function of the highlighted text?
they accepted only knives, fishhooks, and sharp A. It offers a reconsideration of a claim made in
metal. By the time Cartier sailed the Saint the preceding sentence.
Lawrence River ten years later, Native Americans B. It reveals how archaeologists arrived at an
had traded with Europeans for more than thirty interpretation of the evidence mentioned in
years, perhaps half a century. the preceding sentence.
C. It shows how scholars misinterpreted the
significance of certain evidence mentioned in
the preceding sentence.
D. It identifies one of the first significant
accounts of seventeenth-century European
settlements in North America.
E. It explains why Denys's account of
seventeenth-century European settlements is
thought to be significant.
Exactly when in the early modern era Native 50. It can be inferred from the passage that the
Americans began exchanging animal furs with author would agree with which of the
Europeans for European-made goods is following statements about the fur trade
uncertain. What is fairly certain, even though they between Native Americans and Europeans in
left no written evidence of having done so, is that the early modern era?
the first Europeans to conduct such trade during A. This trade may have begun as early as the
the modern period were fishing crews working 1480s.
the waters around Newfoundland. Archaeologists B. This trade probably did not continue much
had noticed that sixteenth-century Native beyond the 1530s.
American sites were strewn with iron bolts and C. This trade was most likely at its peak in the
metal pins. Only later, upon reading Nicolas mid-1520s.
Denys's 1672 account of seventeenth-century D. This trade probably did not begin prior to
European settlements in North America, did 1500.
archaeologists realize that sixteenth-century E. There is no written evidence of this trade
European fishing crews had dismantled and prior to the seventeenth century.
exchanged parts of their ships for furs.
51. Which of the following can be inferred from
By the time Europeans sailing the Atlantic coast of the passage about the Native Americans
North America first documented the fur trade, it mentioned in the highlighted text?
was apparently well underway. The first to record A. They had little use for decorative objects such
such trade—the captain of a Portuguese vessel as earrings.
sailing from Newfoundland in 1501—observed B. They became increasingly dependent on
that a Native American aboard the ship wore fishing between 1501 and 1524.
Venetian silver earrings. Another early chronicler C. By 1524, only certain groups of Europeans
noted in 1524 that Native Americans living along were willing to trade with them.
the coast of what is now New England had D. The selectivity of their trading choices made it
become selective about European trade goods: difficult for them to engage in widespread
they accepted only knives, fishhooks, and sharp trade with Europeans.
metal. By the time Cartier sailed the Saint E. The selectivity of their trading choices
Lawrence River ten years later, Native Americans indicates that they had been trading with
had traded with Europeans for more than thirty Europeans for a significant period of time
years, perhaps half a century. prior to 1524.
Opponents of these concessions from labor argue that 75. According to the author, “Sophisticated
such concessions do not save jobs, but either prolong the proponents” of concessions do which of the
agony of dying plants or finance the plant relocations following?
that employers had intended anyway. Companies make
investment decisions to fit their strategic plans and their
A. Support the traditional adversarialism
profit objectives, opponents point out, and labor costs characteristic of labor-management relations.
are usually just a small factor in the equation. Moreover, B. Emphasize the benefits unions can gain by
unrestrained by either loyalty to their work force or granting concessions.
political or legislative constraints on their mobility, the C. Focus on thorough analyses of current
companies eventually cut and run, concessions or no economic conditions.
concessions. D. Present management's reasons for
Wage-related concessions have come under particular
demanding concessions.
attack, since opponents believe that high union wages E. Explain domestic economic developments in
underlay much of the success of United States industry in terms of worldwide trends.
this century. They point out that a long-standing
principle, shared by both management and labor, has
been that workers should earn wages that give them the
income they need to buy what they make. Moreover, high
wages have given workers the buying power to propel
the economy forward. If proposals for pay cuts, two-tier
wage systems, and subminimum wages for young
workers continue to gain credence, opponents believe
the U.S. social structure will move toward that of a less-
developed nation: a small group of wealthy investors, a
sizable but still minority bloc of elite professionals and
highly skilled employees, and a huge mass of marginal
workers and unskilled laborers. Further, they argue that
if unions willingly engage in concession bargaining on
the false grounds that labor costs are the source of a
company's problems, unions will find themselves
competing with Third World pay levels—a competition
they cannot win.
Historians who study European women of the 76. According to the passage, much of the
Renaissance try to measure “independence,” research on Alessandra Strozzi done by the
“options,” and other indicators of the degree to historian mentioned in the second paragraph
which the expression of women's individuality supports which of the following conclusions?
was either permitted or suppressed. Influenced A. Alessandra used her position as her sons' sole
by Western individualism, these historians define guardian to further interests different from
a peculiar form of personhood: an innately those of her sons.
bounded unit, autonomous and standing apart B. Alessandra unwillingly sacrificed her own
from both nature and society. An anthropologist, interests in favor of those of her sons.
however, would contend that a person can be C. Alessandra's actions indicate that her
conceived in ways other than as an “individual.” motivations and intentions were those of an
In many societies a person's identity is not independent individual.
intrinsically unique and self-contained but D. Alessandra's social context encouraged her to
instead is defined within a complex web of social take independent action.
relationships. E. Alessandra regarded her sons' goals and
interests as her own.
In her study of the fifteenth-century Florentine
widow Alessandra Strozzi, a historian who 77. In the first paragraph, the author of the
specializes in European women of the passage mentions a contention that would be
Renaissance attributes individual intention and made by an anthropologist most likely in
authorship of actions to her subject. This order to
historian assumes that Alessandra had goals and A. present a theory that will be undermined in
interests different from those of her sons, yet the discussion of a historian's study later in
much of the historian's own research reveals that the passage
Alessandra acted primarily as a champion of her B. offer a perspective on the concept of
sons' interests, taking their goals as her own. personhood that can usefully be applied to the
Thus Alessandra conforms more closely to the study of women in Renaissance Europe
anthropologist's notion that personal motivation C. undermine the view that the individuality of
is embedded in a social context. Indeed, one could European women of the Renaissance was
argue that Alessandra did not distinguish her largely suppressed
personhood from that of her sons. In Renaissance D. argue that anthropologists have applied the
Europe the boundaries of the conceptual self Western concept of individualism in their
were not always firm and closed and did not research
necessarily coincide with the boundaries of the E. lay the groundwork for the conclusion that
bodily self. Alessandra's is a unique case among European
women of the Renaissance whose lives have
been studied by historians
Historians who study European women of the 78. The passage suggests that the historians
Renaissance try to measure “independence,” referred to in line 1 make which of the
“options,” and other indicators of the degree to following assumptions about Renaissance
which the expression of women's individuality Europe?
was either permitted or suppressed. Influenced A. That anthropologists overestimate the
by Western individualism, these historians define importance of the individual in Renaissance
a peculiar form of personhood: an innately European society
bounded unit, autonomous and standing apart B. That in Renaissance Europe, women were
from both nature and society. An anthropologist, typically allowed to express their
however, would contend that a person can be individuality
conceived in ways other than as an “individual.” C. That European women of the Renaissance had
In many societies a person's identity is not the possibility of acting independently of the
intrinsically unique and self-contained but social context in which they lived
instead is defined within a complex web of social D. That studying an individual such as
relationships. Alessandra is the best way to draw general
conclusions about the lives of women in
In her study of the fifteenth-century Florentine Renaissance Europe
widow Alessandra Strozzi, a historian who E. That people in Renaissance Europe had
specializes in European women of the greater personal autonomy than people do
Renaissance attributes individual intention and currently
authorship of actions to her subject. This
historian assumes that Alessandra had goals and 79. It can be inferred that the author of the
interests different from those of her sons, yet passage believes which of the following about
much of the historian's own research reveals that the study of Alessandra Strozzi done by the
Alessandra acted primarily as a champion of her historian mentioned in the second paragraph?
sons' interests, taking their goals as her own. A. Alessandra was atypical of her time and was
Thus Alessandra conforms more closely to the therefore an inappropriate choice for the
anthropologist's notion that personal motivation subject of the historian's research.
is embedded in a social context. Indeed, one could B. In order to bolster her thesis, the historian
argue that Alessandra did not distinguish her adopted the anthropological perspective on
personhood from that of her sons. In Renaissance personhood.
Europe the boundaries of the conceptual self C. The historian argues that the boundaries of
were not always firm and closed and did not the conceptual self were not always firm and
necessarily coincide with the boundaries of the closed in Renaissance Europe.
bodily self. D. In her study, the historian reverts to a
traditional approach that is out of step with
the work of other historians of Renaissance
Europe.
E. The interpretation of Alessandra's actions
that the historian puts forward is not
supported by much of the historian's
research.
Historians who study European women of the 80. The passage suggests that the historian
Renaissance try to measure “independence,” mentioned in the second paragraph would be
“options,” and other indicators of the degree to most likely to agree with which of the
which the expression of women's individuality following assertions regarding Alessandra
was either permitted or suppressed. Influenced Strozzi?
by Western individualism, these historians define A. Alessandra was able to act more
a peculiar form of personhood: an innately independently than most women of her time
bounded unit, autonomous and standing apart because she was a widow.
from both nature and society. An anthropologist, B. Alessandra was aware that her personal
however, would contend that a person can be motivation was embedded in a social context.
conceived in ways other than as an “individual.” C. Alessandra had goals and interests similar to
In many societies a person's identity is not those of many other widows in her society.
intrinsically unique and self-contained but D. Alessandra is an example of a Renaissance
instead is defined within a complex web of social woman who expressed her individuality
relationships. through independent action.
E. Alessandra was exceptional because she was
In her study of the fifteenth-century Florentine able to effect changes in the social constraints
widow Alessandra Strozzi, a historian who placed upon women in her society.
specializes in European women of the
Renaissance attributes individual intention and
authorship of actions to her subject. This
historian assumes that Alessandra had goals and
interests different from those of her sons, yet
much of the historian's own research reveals that
Alessandra acted primarily as a champion of her
sons' interests, taking their goals as her own.
Thus Alessandra conforms more closely to the
anthropologist's notion that personal motivation
is embedded in a social context. Indeed, one could
argue that Alessandra did not distinguish her
personhood from that of her sons. In Renaissance
Europe the boundaries of the conceptual self
were not always firm and closed and did not
necessarily coincide with the boundaries of the
bodily self.
In addition to conventional galaxies, the universe 81. According to the passage, conventional spiral
contains very dim galaxies that until recently galaxies differ from low-surface-brightness
went unnoticed by astronomers. Possibly as galaxies in which of the following ways?
numerous as conventional galaxies, these galaxies A. They have fewer stars than do low-surface-
have the same general shape and even the same brightness galaxies.
approximate number of stars as a common type B. They evolve more quickly than low-surface-
of conventional galaxy, the spiral, but tend to be brightness galaxies.
much larger. Because these galaxies' mass is C. They are more diffuse than low-surface-
spread out over larger areas, they have far fewer brightness galaxies.
stars per unit volume than do conventional D. They contain less helium than do low-surface-
galaxies. Apparently these low-surface- brightness galaxies.
brightness galaxies, as they are called, take much E. They are larger than low-surface-brightness
longer than conventional galaxies to condense galaxies.
their primordial gas and convert it to stars—that
is, they evolve much more slowly. 82. It can be inferred from the passage that which
of the following is an accurate physical
These galaxies may constitute an answer to the description of typical low-surface-brightness
long-standing puzzle of the missing baryonic galaxies?
mass in the universe. Baryons—subatomic A. They are large spiral galaxies containing
particles that are generally protons or neutrons— fewer stars than conventional galaxies.
are the source of stellar, and therefore galactic, B. They are compact but very dim spiral galaxies.
luminosity, and so their numbers can be C. They are diffuse spiral galaxies that occupy a
estimated based on how luminous galaxies are. large volume of space.
However, the amount of helium in the universe, as D. They are small, young spiral galaxies that
measured by spectroscopy, suggests that there contain a high proportion of primordial gas.
are far more baryons in the universe than E. They are large, dense spirals with low
estimates based on galactic luminosity indicate. luminosity.
Astronomers have long speculated that the
missing baryonic mass might eventually be 83. It can be inferred from the passage that the
discovered in intergalactic space or as some large “long-standing puzzle” refers to which of the
population of galaxies that are difficult to detect. following?
A. The difference between the rate at which
conventional galaxies evolve and the rate at
which low-surface-brightness galaxies evolve
B. The discrepancy between estimates of total
baryonic mass derived from measuring
helium and estimates based on measuring
galactic luminosity
C. The inconsistency between the observed
amount of helium in the universe and the
number of stars in typical low-surface-
brightness galaxies
D. Uncertainties regarding what proportion of
baryonic mass is contained in intergalactic
space and what proportion in conventional
galaxies
E. Difficulties involved in detecting very distant
galaxies and in investigating their luminosity
In addition to conventional galaxies, the universe 84. The author implies that low-surface-
contains very dim galaxies that until recently brightness galaxies could constitute an
went unnoticed by astronomers. Possibly as answer to the puzzle discussed in the second
numerous as conventional galaxies, these galaxies paragraph primarily because
have the same general shape and even the same A. they contain baryonic mass that was not taken
approximate number of stars as a common type into account by researchers using galactic
of conventional galaxy, the spiral, but tend to be luminosity to estimate the number of baryons
much larger. Because these galaxies' mass is in the universe
spread out over larger areas, they have far fewer B. they, like conventional galaxies that contain
stars per unit volume than do conventional many baryons, have evolved from massive,
galaxies. Apparently these low-surface- primordial gas clouds
brightness galaxies, as they are called, take much C. they may contain relatively more helium, and
longer than conventional galaxies to condense hence more baryons, than do galaxies whose
their primordial gas and convert it to stars—that helium content has been studied using
is, they evolve much more slowly. spectroscopy
D. they have recently been discovered to contain
These galaxies may constitute an answer to the more baryonic mass than scientists had
long-standing puzzle of the missing baryonic thought when low-surface-brightness
mass in the universe. Baryons—subatomic galaxies were first observed
particles that are generally protons or neutrons— E. they contain stars that are significantly more
are the source of stellar, and therefore galactic, luminous than would have been predicted on
luminosity, and so their numbers can be the basis of initial studies of luminosity in low-
estimated based on how luminous galaxies are. surface-brightness galaxies
However, the amount of helium in the universe, as
measured by spectroscopy, suggests that there 85. The author mentions the fact that baryons are
are far more baryons in the universe than the source of stars' luminosity primarily in
estimates based on galactic luminosity indicate. order to explain
Astronomers have long speculated that the A. how astronomers determine that some
missing baryonic mass might eventually be galaxies contain fewer stars per unit volume
discovered in intergalactic space or as some large than do others
population of galaxies that are difficult to detect. B. how astronomers are able to calculate the
total luminosity of a galaxy
C. why astronomers can use galactic luminosity
to estimate baryonic mass
D. why astronomers' estimates of baryonic mass
based on galactic luminosity are more reliable
than those based on spectroscopic studies of
helium
E. how astronomers know bright galaxies
contain more baryons than do dim galaxies
In addition to conventional galaxies, the universe 86. The author of the passage would be most
contains very dim galaxies that until recently likely to disagree with which of the following
went unnoticed by astronomers. Possibly as statements?
numerous as conventional galaxies, these galaxies A. Low-surface-brightness galaxies are more
have the same general shape and even the same difficult to detect than are conventional
approximate number of stars as a common type galaxies.
of conventional galaxy, the spiral, but tend to be B. Low-surface-brightness galaxies are often
much larger. Because these galaxies' mass is spiral in shape.
spread out over larger areas, they have far fewer C. Astronomers have advanced plausible ideas
stars per unit volume than do conventional about where missing baryonic mass might be
galaxies. Apparently these low-surface- found.
brightness galaxies, as they are called, take much D. Astronomers have devised a useful way of
longer than conventional galaxies to condense estimating the total baryonic mass in the
their primordial gas and convert it to stars—that universe.
is, they evolve much more slowly. E. Astronomers have discovered a substantial
amount of baryonic mass in intergalactic
These galaxies may constitute an answer to the space.
long-standing puzzle of the missing baryonic
mass in the universe. Baryons—subatomic 87. The primary purpose of the passage is to
particles that are generally protons or neutrons— A. describe a phenomenon and consider its
are the source of stellar, and therefore galactic, scientific significance
luminosity, and so their numbers can be B. contrast two phenomena and discuss a
estimated based on how luminous galaxies are. puzzling difference between them
However, the amount of helium in the universe, as C. identify a newly discovered phenomenon and
measured by spectroscopy, suggests that there explain its origins
are far more baryons in the universe than D. compare two classes of objects and discuss the
estimates based on galactic luminosity indicate. physical properties of each
Astronomers have long speculated that the E. discuss a discovery and point out its
missing baryonic mass might eventually be inconsistency with existing theory
discovered in intergalactic space or as some large
population of galaxies that are difficult to detect.
The fact that superior service can generate a 88. The primary purpose of the passage is to
competitive advantage for a company does not A. contrast possible outcomes of a type of
mean that every attempt at improving service will business investment
create such an advantage. Investments in service, B. suggest more careful evaluation of a type of
like those in production and distribution, must be business investment
balanced against other types of investments on C. illustrate various ways in which a type of
the basis of direct, tangible benefits such as cost business investment could fail to enhance
reduction and increased revenues. If a company is revenues
already effectively on a par with its competitors D. trace the general problems of a company to a
because it provides service that avoids a certain type of business investment
damaging reputation and keeps customers from E. criticize the way in which managers tend to
leaving at an unacceptable rate, then investment analyze the costs and benefits of business
in higher service levels may be wasted, since investments
service is a deciding factor for customers only in
extreme situations. 89. According to the passage, investments in
service are comparable to investments in
This truth was not apparent to managers of one production and distribution in terms of the
regional bank, which failed to improve its A. tangibility of the benefits that they tend to
competitive position despite its investment in confer
reducing the time a customer had to wait for a B. increased revenues that they ultimately
teller. The bank managers did not recognize the produce
level of customer inertia in the consumer banking C. basis on which they need to be weighed
industry that arises from the inconvenience of D. insufficient analysis that managers devote to
switching banks. Nor did they analyze their them
service improvement to determine whether it E. degree of competitive advantage that they are
would attract new customers by producing a new likely to provide
standard of service that would excite customers
or by proving difficult for competitors to copy. 90. The passage suggests which of the following
The only merit of the improvement was that it about service provided by the regional bank
could easily be described to customers. prior to its investment in enhancing that
service?
A. It enabled the bank to retain customers at an
acceptable rate.
B. It threatened to weaken the bank's
competitive position with respect to other
regional banks.
C. It had already been improved after having
caused damage to the bank's reputation in the
past.
D. It was slightly superior to that of the bank's
regional competitors.
E. It needed to be improved to attain parity with
the service provided by competing banks.
The fact that superior service can generate a 91. The passage suggests that bank managers
competitive advantage for a company does not failed to consider whether or not the service
mean that every attempt at improving service will improvement mentioned in the highlighted
create such an advantage. Investments in service, text
like those in production and distribution, must be A. was too complicated to be easily described to
balanced against other types of investments on prospective customers
the basis of direct, tangible benefits such as cost B. made a measurable change in the experiences
reduction and increased revenues. If a company is of customers in the bank's offices
already effectively on a par with its competitors C. could be sustained if the number of customers
because it provides service that avoids a increased significantly
damaging reputation and keeps customers from D. was an innovation that competing banks could
leaving at an unacceptable rate, then investment have imitated
in higher service levels may be wasted, since E. was adequate to bring the bank's general level
service is a deciding factor for customers only in of service to a level that was comparable with
extreme situations. that of its competitors
This truth was not apparent to managers of one 92. The discussion of the regional bank in the
regional bank, which failed to improve its second paragraph serves which of the
competitive position despite its investment in following functions within the passage as a
reducing the time a customer had to wait for a whole?
teller. The bank managers did not recognize the A. It describes an exceptional case in which
level of customer inertia in the consumer banking investment in service actually failed to
industry that arises from the inconvenience of produce a competitive advantage.
switching banks. Nor did they analyze their B. It illustrates the pitfalls of choosing to invest
service improvement to determine whether it in service at a time when investment is needed
would attract new customers by producing a new more urgently in another area.
standard of service that would excite customers C. It demonstrates the kind of analysis that
or by proving difficult for competitors to copy. managers apply when they choose one kind of
The only merit of the improvement was that it service investment over another.
could easily be described to customers. D. It supports the argument that investments in
certain aspects of service are more
advantageous than investments in other
aspects of service.
E. It provides an example of the point about
investment in service made in the first
paragraph.
In some ways, however, these scholars still 7. It can be inferred that the author of the
present the 1920s as a period of decline. After passage disagrees with the “new scholarship”
suffrage, they argue, the feminist movement lost mentioned in the highlighted text regarding
its cohesiveness, and gender consciousness the
waned. After the mid-1920s, few successes could A. degree to which the “promises” of the suffrage
be claimed by feminist reformers: little could be movement remained unkept
seen in the way of legislative victories. B. degree to which suffrage for women improved the
morality of governance
During this decade, however, there was intense C. degree to which the 1920s represented a period
activism aimed at achieving increased autonomy of decline for the feminist movement
for women, broadening the spheres within which D. degree of legislative success achieved by feminist
they lived their daily lives. Women's reformers during the 1920s
organizations worked to establish opportunities E. accuracy of the view that a women's voting bloc
for women: they strove to secure for women the did not materialize once suffrage was achieved
full entitlements of citizenship, including the right
to hold office and the right to serve on juries. 8. The purpose of the second paragraph of the
passage is to
A. suggest a reason why suffragist “promises” were
not kept
B. contrast suffragist “promises” with the reality of
the 1920s
C. deplore the lack of successful feminist reform in
the 1920s
D. explain a view held by feminist scholars
E. answer the question asked by Jane Addams
Recent feminist scholarship concerning the 9. It can be inferred from the passage that recent
United States in the 1920s challenges earlier scholars cite the words of Jane Addams
interpretations that assessed the 1920s in terms primarily in order to
of the unkept “promises” of the women's suffrage A. suggest that women's achievement of suffrage
movement. This new scholarship disputes the brought about changes in government that were
long-held view that because a women's voting not taken into account by early interpretations
bloc did not materialize after women gained the B. point out contradictions inherent in the goals of
right to vote in 1920, suffrage failed to produce the women's suffrage movement
long-term political gains for women. These C. show why a women's voting bloc was not formed
feminist scholars also challenge the old view that when women won the right to vote
pronounced suffrage a failure for not delivering D. emphasize the place of social reform movements
on the promise that the women's vote would in the struggle for suffrage for women
bring about moral, corruption-free governance. E. suggest that the old view of women's suffrage was
Asked whether women's suffrage was a failure, inappropriate
these scholars cite the words of turn-of-the-
century social reformer Jane Addams, “Why don't 10. It can be inferred that the analyses of the
you ask if suffrage in general is failing?” author of the passage and the scholars
mentioned in the highlighted text differ with
In some ways, however, these scholars still regard to which of the following?
present the 1920s as a period of decline. After A. The criteria they use to evaluate the success of the
suffrage, they argue, the feminist movement lost feminist movement during the 1920s
its cohesiveness, and gender consciousness B. Their interpretations of the “promises” of the
waned. After the mid-1920s, few successes could suffragist movement
be claimed by feminist reformers: little could be C. The suggestions they make for achieving feminist
seen in the way of legislative victories. goals
D. Their definitions of what constitutes a legislative
During this decade, however, there was intense victory
activism aimed at achieving increased autonomy E. Their estimations of the obstacles preventing
for women, broadening the spheres within which women's having achieved a voting bloc in the
they lived their daily lives. Women's 1920s
organizations worked to establish opportunities
for women: they strove to secure for women the 11. The “new scholarship” mentioned in the first
full entitlements of citizenship, including the right paragraph suggests which of the following
to hold office and the right to serve on juries. concerning the “promises” mentioned in the
highlighted text?
A. Failure to keep these promises is not a measure of
the success of the suffrage movement.
B. Failure to keep these promises caused the
feminist movement to lose cohesiveness during
the 1920s.
C. Failure to keep these promises led recent feminist
scholars to reevaluate the success of the suffrage
movement.
D. These promises included securing for women the
right to hold office and the right to serve on juries.
E. These promises were of little importance in
winning suffrage for women.
This passage is excerpted from material published 12. The primary purpose of the passage is to
in 1997. A. present several theories that could account for a
particular phenomenon
Is there a massive black hole at the center of our B. argue that a certain question needs to be
galaxy, the Milky Way? The evidence is reframed in light of new evidence
inconclusive. Just as the Sun's mass can be C. resolve an apparent inconsistency between two
determined, given knowledge of other variables, lines of evidence
by the velocity at which its planets orbit, the mass D. explain why a certain issue remains unresolved
at the center of the Milky Way can be revealed by E. present evidence that calls into question certain
the velocities of stars and gas orbiting the galactic assumptions of a current theory
center. This dynamical evidence, based on
recently confirmed assumptions about the stars' 13. According to the passage, the dynamical
velocities, argues for an extremely compact object evidence referred to in the highlighted text
with a mass two to three million times the mass supports which of the following?
of our Sun. Although according to current theory A. Recent assumptions about the velocities of stars
this makes the mass at the center of the galaxy too B. Widely held predictions about the amount of
dense to be anything but a black hole, the relative matter a black hole will engulf
lack of energy radiating from the galactic center C. The existence of an extremely dense object at the
presents a serious problem. A black hole's center of the Milky Way
gravity attracts surrounding matter, which swirls D. The contention that too much energy is coming
around the black hole, emitting some energy as it from the mass at the Milky Way's galactic center
is engulfed. Scientists believe that the amount of for that mass to be a black hole
energy that escapes the black hole should be E. The conclusion that a compact object of two to
about 10 percent of the matter's rest energy (the three million times the mass of our Sun is too
energy equivalent of its mass according to the dense to be anything but a black hole
equation E=mc2). But when the energy coming
from the galactic center is compared to widely 14. The “serious problem” referred to in the
held predictions based on how much matter highlighted text could be solved if which of the
should be falling into a theoretical central black following were true?
hole, there is a discrepancy by a factor of a few A. Current assumptions about how much matter a
thousand. black hole would engulf proved to be several
thousand times too high.
B. Current assumptions about how much matter a
black hole would engulf proved to be a few
thousand times too low.
C. The object at the center of the Milky Way turned
out to be far denser than it is currently estimated
to be.
D. The object at the center of the Milky Way turned
out to be far more massive than it is currently
estimated to be.
E. Matter being engulfed by a black hole radiated far
more energy than is currently assumed.
This passage is excerpted from material published 15. The “widely held predictions” mentioned in
in 1997. the highlighted text are predictions about the
A. compactness of objects whose mass is millions of
Is there a massive black hole at the center of our times the mass of our Sun
galaxy, the Milky Way? The evidence is B. velocities of stars orbiting the galactic center
inconclusive. Just as the Sun's mass can be C. amount of matter swirling around the object at
determined, given knowledge of other variables, the center of the Milky Way
by the velocity at which its planets orbit, the mass D. amount of matter falling into a theoretical central
at the center of the Milky Way can be revealed by black hole
the velocities of stars and gas orbiting the galactic E. amount of energy that should be coming from a
center. This dynamical evidence, based on black hole at the center of the Milky Way
recently confirmed assumptions about the stars'
velocities, argues for an extremely compact object
with a mass two to three million times the mass
of our Sun. Although according to current theory
this makes the mass at the center of the galaxy too
dense to be anything but a black hole, the relative
lack of energy radiating from the galactic center
presents a serious problem. A black hole's
gravity attracts surrounding matter, which swirls
around the black hole, emitting some energy as it
is engulfed. Scientists believe that the amount of
energy that escapes the black hole should be
about 10 percent of the matter's rest energy (the
energy equivalent of its mass according to the
equation E=mc2). But when the energy coming
from the galactic center is compared to widely
held predictions based on how much matter
should be falling into a theoretical central black
hole, there is a discrepancy by a factor of a few
thousand.
Despite their many differences of 16. The author's discussion of Emerson, Thoreau,
temperament and of literary perspective, Hawthorne, Melville, and Whitman is
Emerson, Thoreau, Hawthorne, Melville, and primarily concerned with explaining
Whitman shared certain beliefs. Common to all A. some of their beliefs about the difficulties
these writers is their humanistic perspective. Its involved in self-realization
basic premises are that humans are the spiritual B. some of their beliefs concerning the world and the
center of the universe and that in them alone is place that humanity occupies in the universal
the clue to nature, history, and ultimately the order
cosmos. Without denying outright the existence C. some of their beliefs concerning the relationship
of a deity, this perspective explains humans and between humanism and democracy
the world in terms of humanity. D. the way some of their beliefs are shaped by
This common perspective is almost always differences in temperament and literary outlook
universalized. It emphasizes the human as E. the effects of some of their beliefs on their
universal, freed from the accidents of time, space, writings
birth, and talent. Thus, for Emerson, the
“American Scholar” turns out to be simply “Man 17. According to the passage, the five writers
Thinking,” while, for Whitman, the “Song of object to the scientific method primarily
Myself” merges imperceptibly into a song of all because they think it
the “children of Adam,” where “every atom A. is not the best way to obtain an understanding of
belonging to me as good belongs to you.” the relationship between the individual and the
Also common to all five writers is the belief cosmos
that self-realization depends on the harmonious B. is so specialized that it leads to an understanding
reconciliation of two universal psychological of separate parts of the universe but not of the
tendencies: first, the self-asserting impulse of the relationships among those parts
individual to be responsible only to himself or C. cannot provide an adequate explanation of
herself, and second, the self-transcending impulse intuition and imagination
of the individual to know and become one with D. misleads people into believing they have an
that world. These conflicting impulses can be seen understanding of truth, when they do not
in the democratic ethic. Democracy advocates E. prevents people from recognizing the symbolic
individualism, the preservation of the individual's nature of experience
freedom and self-expression. But the democratic
self is torn between the duty to self, which is 18. The author quotes Whitman primarily in
implied by the concept of liberty, and the duty to order to
society, which is implied by the concepts of A. show that the poet does not agree with Emerson
equality and fraternity. B. indicate the way the poet uses the humanist ideal
A third assumption common to the five to praise himself
writers is that intuition and imagination offer a C. suggest that the poet adapts the basic premises of
surer road to truth than does abstract logic or humanism to his own individual outlook on the
scientific method. It is illustrated by their world
emphasis upon introspection—their belief that D. illustrate a way the poet expresses the
the clue to external nature is to be found in the relationship of the individual to the humanistic
inner world of individual psychology—and by universe
their interpretation of experience as, in essence, E. demonstrate that the poet is concerned with the
symbolic. Both these stresses presume an organic well-being of all humans
relationship between the self and the cosmos of
which only intuition and imagination can
properly take account. These writers' faith in the
imagination and in themselves led them to
conceive of the writer as a seer.
Despite their many differences of 19. It can be inferred that intuition is important to
temperament and of literary perspective, the five writers primarily because it provides
Emerson, Thoreau, Hawthorne, Melville, and them with
Whitman shared certain beliefs. Common to all A. information useful for understanding abstract
these writers is their humanistic perspective. Its logic and scientific method
basic premises are that humans are the spiritual B. the discipline needed in the search for truth
center of the universe and that in them alone is C. inspiration for their best writing
the clue to nature, history, and ultimately the D. clues to the interpretation of symbolic experience
cosmos. Without denying outright the existence E. the means of resolving conflicts between the self
of a deity, this perspective explains humans and and the world
the world in terms of humanity.
This common perspective is almost always 20. The author discusses “the democratic ethic” in
universalized. It emphasizes the human as order to
universal, freed from the accidents of time, space, A. explain the relationship between external
birth, and talent. Thus, for Emerson, the experience and inner imagination
“American Scholar” turns out to be simply “Man B. support the notion that the self contains two
Thinking,” while, for Whitman, the “Song of conflicting and irreconcilable factions
Myself” merges imperceptibly into a song of all C. illustrate the relationship between the self's
the “children of Adam,” where “every atom desire to be individual and its desire to merge
belonging to me as good belongs to you.” with all other selves
Also common to all five writers is the belief D. elaborate on the concept that the self constantly
that self-realization depends on the harmonious desires to realize its potential
reconciliation of two universal psychological E. give an example of the idea that, in order to be
tendencies: first, the self-asserting impulse of the happy, the self must reconcile its desires with
individual to be responsible only to himself or external reality
herself, and second, the self-transcending impulse
of the individual to know and become one with 21. It can be inferred that the idea of “an organic
that world. These conflicting impulses can be seen relationship between the self and the cosmos”
in the democratic ethic. Democracy advocates is necessary to the thinking of the five writers
individualism, the preservation of the individual's because such a relationship
freedom and self-expression. But the democratic A. enables them to assert the importance of the
self is torn between the duty to self, which is democratic ethic
implied by the concept of liberty, and the duty to B. justifies their concept of the freedom of the
society, which is implied by the concepts of individual
equality and fraternity. C. sustains their faith in the existence of a deity
A third assumption common to the five D. is the foundation of their humanistic view of
writers is that intuition and imagination offer a existence
surer road to truth than does abstract logic or E. is the basis for their claim that the writer is a seer
scientific method. It is illustrated by their
emphasis upon introspection—their belief that 22. The passage is most relevant to which of the
the clue to external nature is to be found in the following areas of study?
inner world of individual psychology—and by A. Aesthetics and logic
their interpretation of experience as, in essence, B. History and literature
symbolic. Both these stresses presume an C. Theology and sociology
organic relationship between the self and the D. Anthropology and political science
cosmos of which only intuition and imagination E. Linguistics and art
can properly take account. These writers' faith in
the imagination and in themselves led them to
conceive of the writer as a seer.
The final quarter of the nineteenth century marked 23. The primary purpose of the passage is to
a turning point in the history of biology—biologists A. compare the information about organic function
became less interested in applying an ideal of made available by historical explanation with that
historical explanation deductively to organic made available by the experimental investigation
function and more interested in discerning the of living organisms
causes of vital processes through experimental B. assess the influence that theories of history had
manipulation. But it is impossible to discuss the on developments in the field of biology in the
history of biology in the nineteenth century without nineteenth century
emphasizing that those areas of biology most in the C. discuss the importance of historical explanation
public eye had depended on historical explanation. in the thinking of nineteenth century biologists
Wherever it was applied, historical explanation was
D. contrast biologists' use of historical explanation
deemed causal explanation. The biologist-as-
during the early nineteenth century with its use
historian and the general historian of human events
during the final quarter of the nineteenth century
dealt with comparable phenomena and assumed
necessarily a common mode of explanation. E. evaluate the way in which the concept of heredity
altered the use of historical explanation by
Nineteenth-century biologists found a historical nineteenth-century biologists
explanation of organic function attractive partly
because their observation of the formation of a new 24. According to information presented in the
cell from a preexisting cell seemed to confirm a passage, which of the following is a true
historical explanation of cell generation. The same statement about the methods of explanation
direct observation of continuous stages of used by biologists and historians in the
development was also possible when they nineteenth century?
examined the complex sequence of events of A. Neither biologists nor historians were able to
embryogenesis. In both cases, the observer received develop methods of explanation that were
a concrete impression that the daughter cell was accepted by the majority of their colleagues.
brought into being, or caused, by the prior cell. The B. The methods used by biologists to explain
argument that these scientists employed confuses phenomena changed dramatically, whereas the
temporal succession and causal explanation, of methods used by historians to explain events did
course, but such confusion is the heart of most not change as noticeably.
historical explanation. C. Biologists believed that they had refined the
methods of explanation used by historians.
Not surprisingly, the evolutionary biologists of the D. Biologists' and historians' methods of explaining
nineteenth century encountered a particularly
what they believed to be comparable phenomena
troublesome problem in their attempts to
were similar.
document historical explanation convincingly: the
E. Although biologists and historians adopted
factual record of the history of life on earth (e.g., that
similar methods of explanation, the biologists
provided by fossils) was incomplete. The temporal
continuity of living forms was convincing, but was were more apologetic about their use of these
an assumption that was difficult to uphold when methods.
one compared species or organisms forming any
two stages of the evolutionary record. Nineteenth-
century biologists recognized this problem and
attempted to resolve it. Their solution today
appears to be only verbal, but was then regarded as
eminently causal. The fact of evolution demanded
some connection between all reproducing
individuals and the species that they compose, as
well as between living species and their extinct
ancestors. Their solution, the concept of heredity,
seemed to fill in an admittedly deficient historical
record and seemed to complete the argument for a
historical explanation of evolutionary events.
The final quarter of the nineteenth century marked 25. Which of the following best summarizes the
a turning point in the history of biology— “turning point” mentioned in the highlighted
biologists became less interested in applying an text?
ideal of historical explanation deductively to A. The beginning of the conflict between proponents
organic function and more interested in discerning of the ideal of historical explanation and the
the causes of vital processes through experimental proponents of experimentation
manipulation. But it is impossible to discuss the B. The substitution of historical explanation for
history of biology in the nineteenth century without causal explanation
emphasizing that those areas of biology most in the C. The shift from interest in historical explanation to
public eye had depended on historical explanation. interest in experimentation
Wherever it was applied, historical explanation was
D. The attention suddenly paid to problems of
deemed causal explanation. The biologist-as-
organic function
historian and the general historian of human events
E. The growth of public awareness of the
dealt with comparable phenomena and assumed
necessarily a common mode of explanation. controversies among biologists
Nineteenth-century biologists found a historical
explanation of organic function attractive partly 26. The author implies that nineteenth-century
because their observation of the formation of a new biologists who studied embryogenesis
cell from a preexisting cell seemed to confirm a believed that they
historical explanation of cell generation. The same A. had discovered physical evidence that supported
direct observation of continuous stages of their use of historical explanation
development was also possible when they B. were the first biologists to call for systematic
examined the complex sequence of events of experimentation on living organisms
embryogenesis. In both cases, the observer received C. were able to use historical explanation more
a concrete impression that the daughter cell was systematically than were biologists who did not
brought into being, or caused, by the prior cell. The study embryogenesis
argument that these scientists employed confuses D. had inadvertently discovered an important part
temporal succession and causal explanation, of of the factual record of the history of living
course, but such confusion is the heart of most organisms on earth
historical explanation. E. had avoided the logical fallacies that characterize
Not surprisingly, the evolutionary biologists of the the reasoning of most nineteenth-century
nineteenth century encountered a particularly biologists
troublesome problem in their attempts to
document historical explanation convincingly: the
27. The passage would be most likely to appear in
factual record of the history of life on earth (e.g., that
which of the following?
provided by fossils) was incomplete. The temporal
A. An essay investigating the methodology used by
continuity of living forms was convincing, but was
historians of human events
an assumption that was difficult to uphold when
one compared species or organisms forming any B. A book outlining the history of biology in the
two stages of the evolutionary record. Nineteenth- nineteenth century
century biologists recognized this problem and C. A seminar paper on the development of
attempted to resolve it. Their solution today embryogenesis as a field of study in nineteenth-
appears to be only verbal, but was then regarded as century biology
eminently causal. The fact of evolution demanded D. A review of a book whose topic is the discovery of
some connection between all reproducing fossils in the nineteenth century
individuals and the species that they compose, as E. A lecture whose subject is the limitations of
well as between living species and their extinct experimental investigation in modern biology
ancestors. Their solution, the concept of heredity,
seemed to fill in an admittedly deficient historical
record and seemed to complete the argument for a
historical explanation of evolutionary events.
Critics maintain that the fiction of Herman 28. The primary purpose of the passage is to
Melville (1819–1891) has limitations, such as its A. make a case for the importance of skillful
lack of inventive plots after Moby-Dick (1851) psychological motivation in well-written novels
and its occasionally inscrutable style. A more and romances
serious, yet problematic, charge is that Melville is B. contrast the romantic and novelistic traditions
a deficient writer because he is not a practitioner and assert the aesthetic superiority of the
of the “art of fiction,” as critics have conceived of romantic tradition
this art since the late nineteenth-century essays C. survey some of the responses to Melville's fiction
and novels of Henry James. Indeed, most put forward by James and twentieth-century
twentieth-century commentators regard Melville literary critics
not as a novelist but as a writer of romance, since D. argue that the charges made against Melville's
they believe that Melville's fiction lacks the fiction by literary critics are suspect and
continuity that James viewed as essential to a misleading
novel: the continuity between what characters E. note several accusations made against Melville's
feel or think and what they do, and the continuity fiction by literary critics and refute one of these
between characters' fates and their pasts or accusations
original social classes. Critics argue that only
Pierre (1852), because of its subject and its 29. The author draws which of the following
characters, is close to being a novel in the conclusions about the fact that Melville's
Jamesian sense. fiction often does not possess the qualities of
a Jamesian novel?
However, although Melville is not a Jamesian A. Literary critics should no longer use Jamesian
novelist, he is not therefore a deficient writer. A standards to judge the value of novels.
more reasonable position is that Melville is a B. Literary critics who have praised Melville's fiction
different kind of writer, who held, and should be at the expense of James's fiction should consider
judged by, presuppositions about fiction that are themselves justified.
quite different from James's. It is true that C. Literary critics should no longer attempt to place
Melville wrote “romances”; however, these are writers, including Melville and James, in
not the escapist fictions this word often implies, traditions or categories.
but fictions that range freely among very unusual D. Melville and James should be viewed as different
or intense human experiences. Melville portrayed sorts of writers and one should not be regarded
such experiences because he believed that these as inherently superior to the other.
experiences best enabled him to explore moral E. Melville and James nevertheless share important
questions, an exploration he assumed was the similarities and these should not be overlooked or
ultimate purpose of fiction. He was content to slighted when literary critics point out
sacrifice continuity or even credibility as long as differences between the two writers.
he could establish a significant moral situation.
Thus, Melville's romances do not give the reader 30. Which of the following would be the most
a full understanding of the complete feelings and appropriate title for the passage?
thoughts that motivate actions and events that A. Melville's Unique Contribution to Romantic
shape fate. Rather, the romances leave Fiction
unexplained the sequence of events and either B. Melville's Growing Reputation Among Twentieth-
simplify or obscure motives. Again, such Century Literary Critics
simplifications and obscurities exist in order to C. Melville and the Jamesian Standards of Fiction: A
give prominence to the depiction of sharply Reexamination
delineated moral values, values derived from a D. Romantic and Novelistic: The Shared
character's purely personal sense of honor, rather Assumptions of Two Traditions
than, as in a Jamesian novel, from the conventions E. The Art of Fiction: James's Influence on the
of society. Novelistic Tradition
Critics maintain that the fiction of Herman 31. The author probably mentions Melville's
Melville (1819–1891) has limitations, such as its Pierre to
lack of inventive plots after Moby-Dick (1851) A. refute those literary critics who have made
and its occasionally inscrutable style. A more generalizations about the quality of Melville's
serious, yet problematic, charge is that Melville is fiction
a deficient writer because he is not a practitioner B. argue that the portrayal of characters is one of
of the “art of fiction,” as critics have conceived of Melville's more accomplished literary skills
this art since the late nineteenth-century essays C. give an example of a novel that was thought by
and novels of Henry James. Indeed, most James to resemble his own fiction
twentieth-century commentators regard Melville D. suggest that literary critics find few exceptions to
not as a novelist but as a writer of romance, since what they believe is a characteristic of Melville's
they believe that Melville's fiction lacks the fiction
continuity that James viewed as essential to a E. reinforce the contention of literary critics
novel: the continuity between what characters
feel or think and what they do, and the continuity 32. Which of the following statements best
between characters' fates and their pasts or describes the author's method of
original social classes. Critics argue that only argumentation in the highlighted text?
Pierre (1852), because of its subject and its A. The author describes an important standard of
characters, is close to being a novel in the evaluation used by critics of Melville and then
Jamesian sense. attacks that standard.
B. The author admits a contention put forward by
However, although Melville is not a Jamesian critics of Melville but then makes a countercharge
novelist, he is not therefore a deficient writer. A against those critics.
more reasonable position is that Melville is a C. The author describes a charge advanced by critics
different kind of writer, who held, and should be of Melville and then points out a logical flaw in
judged by, presuppositions about fiction that are this charge.
quite different from James's. It is true that D. The author provides evidence that seems to
Melville wrote “romances”; however, these support a position held by critics of Melville but
are not the escapist fictions this word often then demonstrates that the evidence actually
implies, but fictions that range freely among supports a diametrically opposed position.
very unusual or intense human experiences. E. The author concedes an assertion made by critics
Melville portrayed such experiences because of Melville but then mitigates the weight of the
he believed that these experiences best assertion by means of an explanation.
enabled him to explore moral questions, an
exploration he assumed was the ultimate 33. Which of the following can logically be
purpose of fiction. He was content to sacrifice inferred from the passage about the author's
continuity or even credibility as long as he could application of the term “romance” to
establish a significant moral situation. Thus, Melville's work?
Melville's romances do not give the reader a full A. The author uses the term in a broader way than
understanding of the complete feelings and did Melville himself.
thoughts that motivate actions and events that B. The author uses the term in a different way than
shape fate. Rather, the romances leave do many literary critics.
unexplained the sequence of events and either C. The author uses the term in a more systematic
simplify or obscure motives. Again, such way than did James.
simplifications and obscurities exist in order to D. The author's use of the term is the same as the
give prominence to the depiction of sharply term's usual meaning for twentieth-century
delineated moral values, values derived from a commentators.
character's purely personal sense of honor, rather E. The author's use of the term is less controversial
than, as in a Jamesian novel, from the conventions than is the use of the term “novel” by many
of society. commentators.
Critics maintain that the fiction of Herman 34. Which of the following can most logically be
Melville (1819–1891) has limitations, such as its inferred about the author's estimation of the
lack of inventive plots after Moby-Dick (1851) romantic and novelistic traditions of fiction?
and its occasionally inscrutable style. A more A. The romantic tradition should be considered at
serious, yet problematic, charge is that Melville is least as valuable as the novelistic tradition in the
a deficient writer because he is not a practitioner examination of human experience.
of the “art of fiction,” as critics have conceived of B. The romantic tradition should be considered the
this art since the late nineteenth-century essays more vital tradition primarily because Melville is
and novels of Henry James. Indeed, most part of that tradition.
twentieth-century commentators regard Melville C. The romantic tradition should be considered the
not as a novelist but as a writer of romance, since superior tradition because it is so widespread.
they believe that Melville's fiction lacks the D. The romantic tradition has had as much success
continuity that James viewed as essential to a in pleasing literary critics as has the novelistic
novel: the continuity between what characters tradition.
feel or think and what they do, and the continuity E. The romantic and novelistic traditions have
between characters' fates and their pasts or always made important contributions to
original social classes. Critics argue that only literature, but their most important contributions
Pierre (1852), because of its subject and its have been in the twentieth century.
characters, is close to being a novel in the
Jamesian sense. 35. The author of the passage would be most
likely to agree that a writer's fiction should be
However, although Melville is not a Jamesian evaluated by which of the following criteria?
novelist, he is not therefore a deficient writer. A A. How consistently that fiction establishes
more reasonable position is that Melville is a credibility with the reader
different kind of writer, who held, and should be B. How skillfully that fiction supersedes the
judged by, presuppositions about fiction that are presuppositions or conventions of a tradition
quite different from James's. It is true that C. How completely that fiction satisfies the
Melville wrote “romances”; however, these are standards of judgment held by most literary
not the escapist fictions this word often implies, critics
but fictions that range freely among very unusual D. How well that fiction fulfills the premises about
or intense human experiences. Melville portrayed fiction maintained by the writer of the fiction
such experiences because he believed that these E. How well that fiction exhibits a continuity of
experiences best enabled him to explore moral subject and style over the course of the writer's
questions, an exploration he assumed was the career
ultimate purpose of fiction. He was content to
sacrifice continuity or even credibility as long as
he could establish a significant moral situation.
Thus, Melville's romances do not give the reader
a full understanding of the complete feelings and
thoughts that motivate actions and events that
shape fate. Rather, the romances leave
unexplained the sequence of events and either
simplify or obscure motives. Again, such
simplifications and obscurities exist in order to
give prominence to the depiction of sharply
delineated moral values, values derived from a
character's purely personal sense of honor, rather
than, as in a Jamesian novel, from the conventions
of society.
Behind every book review there are two key 36. According to the passage, book review editors
figures: a book review editor and a reviewer. pay attention to all of the following in deciding
Editors decide whether a book is reviewed in which books should be reviewed in their
their publication, when the review appears, how publications EXCEPT
long it is, and who writes the review. A. news releases from publishers
When many periodicals feature the same books, B. sales figures compiled by bookstores
this does not prove that the editors of different C. the opinions of literary experts
periodicals have not made individual decisions. D. the probability that the books will be extensively
Before publication, editors receive news releases advertised
and printer's proofs of certain books, signifying E. the likelihood that the books will be reviewed in
that the publishers will make special efforts to other publications
promote these books. They will be heavily
advertised and probably be among the books that 37. The main idea of the second paragraph is that
most bookstores order in quantity. Not having A. decisions made by book review editors are
such books reviewed might give the impression influenced by the business of selling books
that the editor was caught napping, whereas too B. book review editors must be familiar with all
many reviews of books that readers will have aspects of the book trade
trouble finding in stores would be inappropriate. C. advertising is the most important factor
Editors can risk having a few of the less popular influencing book sales
titles reviewed, but they must consider what will D. book reviews usually have no influence on what
be newsworthy, advertised, and written about books are ordered in quantity by stores
elsewhere. E. publishers deliberately try to influence the
decisions of book review editors
If these were the only factors influencing editors,
few books that stand little chance of selling well 38. According to the passage, a major concern of
would ever be reviewed. But editors feel some the unenthusiastic book reviewers in the
concern about what might endure, and therefore highlighted text was to
listen to literary experts. A generation ago, a A. ensure prompt payment for their work
newspaper used a brilliant system of choosing B. influence public opinion of books
which books to feature. The book review editor C. confirm the opinions of other reviewers
sent out a greater number of books than reviews D. promote new books by their favorite authors
he actually intended to publish. If a review was E. have their reviews published in the newspaper
unenthusiastic, he reasoned that the book was not
important enough to be discussed immediately, 39. The passage provides information to answer
and if good reviews of enough other books came which of the following questions?
in, the unenthusiastic review might never be A. Would most book review editors prefer to have
printed. The unenthusiastic reviewers were books reviewed without regard to the probable
paid promptly anyway, but they learned that if commercial success of the books?
they wanted their material to be printed, it was B. Are publishers' efforts to persuade bookstores to
advisable to be kind. order certain books generally effective?
C. On what basis do literary experts decide that a
Most editors print favorable and unfavorable book is or is not important?
reviews; however, the content of the review may D. What criteria are used to determine the length of
be influenced by the editor. Some editors would a particular book review?
actually feel that they had failed in their E. Have book review practices in general changed
responsibility if they gave books by authors they significantly since a generation ago?
admired to hostile critics or books by authors
they disapproved of to critics who might favor
them. Editors usually can predict who would
review a book enthusiastically and who would
tear it to shreds.
Behind every book review there are two key review a book enthusiastically and who would
figures: a book review editor and a reviewer. tear it to shreds.
Editors decide whether a book is reviewed in
their publication, when the review appears, how 40. The passage suggests which of the following
long it is, and who writes the review. about book review readers?
A. They pay careful attention to reviewers' biases as
When many periodicals feature the same books, they read reviews.
this does not prove that the editors of different B. They disapprove of book review editors who try
periodicals have not made individual decisions. to influence what their reviewers write.
Before publication, editors receive news releases C. They use book reviews in order to gauge whether
and printer's proofs of certain books, signifying a book is likely to endure.
that the publishers will make special efforts to D. They expect to see timely reviews of widely
promote these books. They will be heavily publicized books in the periodicals they read.
advertised and probably be among the books that E. They are usually willing to search in several
most bookstores order in quantity. Not having stores for a highly recommended book that is
such books reviewed might give the impression hard to find.
that the editor was caught napping, whereas too
many reviews of books that readers will have 41. Which of the following words, if substituted
trouble finding in stores would be inappropriate. for “brilliant” in the highlighted text, would
Editors can risk having a few of the less popular LEAST change the meaning of the sentence?
titles reviewed, but they must consider what will A. showy
be newsworthy, advertised, and written about B. articulate
elsewhere. C. literate
D. stingy
If these were the only factors influencing editors, E. absurd
few books that stand little chance of selling well
would ever be reviewed. But editors feel some 42. Which of the following is an assumption made
concern about what might endure, and therefore by the book review editor referred to in the
listen to literary experts. A generation ago, a highlighted text?
newspaper used a brilliant system of choosing A. A book of great worth will receive only good
which books to feature. The book review editor reviews.
sent out a greater number of books than reviews B. An important book will endure despite possible
he actually intended to publish. If a review was bad reviews.
unenthusiastic, he reasoned that the book was not C. Reviewers might hide their true opinions in order
important enough to be discussed immediately, to have their reviews published.
and if good reviews of enough other books came D. Book review editors should select reviewers
in, the unenthusiastic review might never be whose opinions can be guessed in advance.
printed. The unenthusiastic reviewers were paid E. Book review editors have an obligation to print
promptly anyway, but they learned that if they extensive reviews of apparently important books.
wanted their material to be printed, it was
advisable to be kind. 43. It can be inferred that, as a prospective book
buyer, the author of the passage would
Most editors print favorable and unfavorable generally respond to highly enthusiastic
reviews; however, the content of the review may reviews of new books with
be influenced by the editor. Some editors would A. resignation
actually feel that they had failed in their B. amusement
responsibility if they gave books by authors they C. condemnation
admired to hostile critics or books by authors D. skepticism
they disapproved of to critics who might favor E. disinterest
them. Editors usually can predict who would
There are two theories that have been used to 44. The primary purpose of the passage is to
explain ancient and modern tragedy. Neither A. compare and criticize two theories of tragedy
quite explains the complexity of the tragic process B. develop a new theory of tragedy
or the tragic hero, but each explains important C. summarize the thematic content of tragedy
elements of tragedy, and, because their D. reject one theory of tragedy and offer another
conclusions are contradictory, they represent theory in its place
extreme views. The first theory states that all E. distinguish between tragedy and irony
tragedy exhibits the workings of external fate. Of
course, the overwhelming majority of tragedies 45. The author states that the theories discussed
do leave us with a sense of the supremacy of in the passage “represent extreme views”
impersonal power and of the limitation of human because their conclusions are
effort. But this theory of tragedy is an A. unpopular
oversimplification, primarily because it confuses B. complex
the tragic condition with the tragic process: the C. paradoxical
theory does not acknowledge that fate, in a D. contradictory
tragedy, normally becomes external to the hero E. imaginative
only after the tragic process has been set in
motion. Fate, as conceived in ancient Greek 46. The author objects to the theory that all
tragedy, is the internal balancing condition of life. tragedy exhibits the workings of external fate
It appears as external only after it has been primarily because
violated, just as justice is an internal quality of an A. fate in tragedies is not as important a cause of
honest person, but the external antagonist of the action as is the violation of a moral law
criminal. Secondarily, this theory of tragedy does B. fate in tragedies does not appear to be something
not distinguish tragedy from irony. Irony does not that is external to the tragic hero until after the
need an exceptional central figure: as a rule, the tragic process has begun
more ignoble the hero the sharper the irony, C. the theory is based solely on an understanding of
when irony alone is the objective. It is heroism ancient Greek tragedy
that creates the splendor and exhilaration that is D. the theory does not seem to be a plausible
unique to tragedy. The tragic hero normally has explanation of tragedy's ability to exhilarate an
an extraordinary, often a nearly divine, destiny audience
almost within grasp, and the glory of that original E. the theory does not seem applicable to the large
destiny never quite fades out of the tragedy. number of tragedies in which the hero overcomes
fate
The second theory of tragedy states that the act
that sets the tragic process in motion must be 47. Which of the following comparisons of the
primarily a violation of moral law, whether tragic with the ironic hero is best supported
human or divine; in short, that the tragic hero by information contained in the passage?
must have a flaw that has an essential connection A. A tragic hero's fate is an external condition, but an
with sin. Again, it is true that the great majority of ironic hero's fate is an internal one.
tragic heroes do possess hubris, or a proud and B. A tragic hero must be controlled by fate, but an
passionate mind that seems to make the hero's ironic hero cannot be.
downfall morally explicable. But such hubris is C. A tragic hero's moral flaw surprises the audience,
only the precipitating agent of catastrophe, just as but an ironic hero's sin does not.
in comedy the cause of the happy ending is D. A tragic hero and an ironic hero cannot both be
usually some act of humility, often performed by virtuous figures in the same tragedy.
a noble character who is meanly disguised. E. A tragic hero is usually extraordinary, but an
ironic hero may be cowardly or even villainous.
There are two theories that have been used to 48. The author contrasts an honest person and a
explain ancient and modern tragedy. Neither criminal primarily to
quite explains the complexity of the tragic process A. prove that fate cannot be external to the tragic
or the tragic hero, but each explains important hero
elements of tragedy, and, because their B. establish a criterion that allows a distinction to be
conclusions are contradictory, they represent made between irony and tragedy
extreme views. The first theory states that all C. develop the distinction between the tragic
tragedy exhibits the workings of external fate. Of condition and the tragic process
course, the overwhelming majority of tragedies D. introduce the concept of sin as the cause of tragic
do leave us with a sense of the supremacy of action
impersonal power and of the limitation of human E. argue that the theme of omnipotent external fate
effort. But this theory of tragedy is an is shared by comedy and tragedy
oversimplification, primarily because it confuses
the tragic condition with the tragic process: the 49. The author suggests that the tragic hero's
theory does not acknowledge that fate, in a “original destiny never quite fades out of the
tragedy, normally becomes external to the hero tragedy” primarily to
only after the tragic process has been set in A. qualify the assertion that the theory of tragedy as
motion. Fate, as conceived in ancient Greek a display of external fate is inconsistent
tragedy, is the internal balancing condition of life. B. introduce the discussion of the theory that
It appears as external only after it has been tragedy is the consequence of individual sin
violated, just as justice is an internal quality of an C. refute the theory that the tragic process is more
honest person, but the external antagonist of the important than the tragic condition
criminal. Secondarily, this theory of tragedy does D. support the claim that heroism creates the
not distinguish tragedy from irony. Irony does not splendor and exhilaration of tragedy
need an exceptional central figure: as a rule, the E. distinguish between fate as conceived in ancient
more ignoble the hero the sharper the irony, Greek tragedy and fate in more recent tragedy
when irony alone is the objective. It is heroism
that creates the splendor and exhilaration that is 50. In the author's opinion, an act of humility in
unique to tragedy. The tragic hero normally has comedy is most analogous to
an extraordinary, often a nearly divine, destiny A. a catastrophe in tragedy
almost within grasp, and the glory of that original B. an ironic action in tragedy
destiny never quite fades out of the tragedy. C. a tragic hero's pride and passion
D. a tragic hero's aversion to sin
The second theory of tragedy states that the act E. a tragic hero's pursuit of an unusual destiny
that sets the tragic process in motion must be
primarily a violation of moral law, whether
human or divine; in short, that the tragic hero
must have a flaw that has an essential connection
with sin. Again, it is true that the great majority of
tragic heroes do possess hubris, or a proud and
passionate mind that seems to make the hero's
downfall morally explicable. But such hubris is
only the precipitating agent of catastrophe, just as
in comedy the cause of the happy ending is
usually some act of humility, often performed by
a noble character who is meanly disguised.